Download as pdf or txt
Download as pdf or txt
You are on page 1of 451

A Simplified approach to

Accounting Standards
With Ind AS and the Companies Act, 2013
CA/CMA Final
[May / June 2016 Examinations]
Sixth Edition

Prof. Israr Shaikh CA. Rajesh Makkar


Visiting Faculty of ICAI–WIRC B.Com. (H), F.C.A.
Mumbai Ex. Visiting Faculty of ICAI
Ex. Member–Financial Management
Committee of NIRC of ICAI

Carvinowledge
P R E S S
info@carvinowledge.in
www.carvinowledge.in
Carvinowledge
P R E S S

Accounting Standards
With Ind AS and the Companies Act, 2013, 6e

© Carvinowledge Press, 2016


All rights reserved. No part of this publication may be reproduced or transmitted, in any form or by any means, without permission.
Any person who commits any unauthorised act in relation to this publication may be liable to criminal prosecution and civil claims
for damages.

Sixth Edition 2016

Published by Eesha for Carvinowledge Press

Carvinowledge Press
B8/GM 4, DLF Colony, Dilshad Ext. II
Bhopura, Ghaziabad – 201005
Mobile: +91-9953922272, +91-9022413471
E-mail: info@carvinowledge.in
www.carvinowledge.in

This book is meant for educational and learning purposes. The author(s) of the book has/have taken all reasonable care to ensure that the contents of the book do
not violate any existing copyright or other intellectual property rights of any person in any manner whatsoever. In the event the author(s) has/have been unable to
track any source and if any copyright has been inadvertently infringed, please notify the publisher in writing for corrective action.
Dedication

This book belongs to my beloved brother whom I love the most


“Late Mr. Jamil Shaikh”
His almighty soul kept me working every time.
FOREWORD

I also embrace the efforts of publishers in putting their valuable time and enery

in publishing this book. I hope this book would serve and an asset for every

commerce Student and Professional.

My best wishes to them for all present and future endeavours.


EDITORIAL BOARD

Accounting Standards With Ind AS and the Companies Act, 2013 (Sixth Edition) has benefited from an
extensive development process. Over 30 faculty reviewers, students and industry professionals provided
feedback about the accuracy and relevance of the content as well as suggestions for its improvements. Though
we could not incorporate every one’s suggestion, we do acknowledge that their feedback was invaluable in
our attempt at creating the best possible Accounting Standards book.

CONSULTANT BOARD
The consultant board provided us with a detailed and critical analysis of each chapter and worked with us
throughout the development of the book. We would like to thank the following for their time and commitment:

Dr. Prashant Mishra Dr. Arvind K. Mishra CA. Rakesh Makkar


Indian Institute of Management JNU Chartered Accountant
Kolkata New Delhi New Delhi

Dr. Dhanesh Khatri Dr. Mahesh Gour CA Parimal Jha


Head, Department of Finance Faculty, IDT Chartered Accountant
Institute of Management Studies
BJS Rampuria Jain College
Bikaner

Eesha Narang Om S Trivedi CA Ashish Sodani


Hansraj College Author and Publishing Professional Chartered Accountant
New Delhi

SUPPLEMENT AUTHORS
This supplement includes introduction to the major concepts of each chapter, definitions and background for
each concept, applications of data and analytical problems that can be discussed in the class.

Amit Rajpurohit
Author and Publishing Professional

STUDENT REVIEWERS
We took the help of many students who class-tested the manuscript, evaluated it for clarity, and assessed
each feature. Their comments helped us to expand the book’s content, improved the pedagogical features,
and strengthened the assessment features. We are thankful to the following:

Mayank Malik Deepanshu Sharma Sanjhi Aggarwal Reena Gulati Prince Jain

Amit Goyal Shweta Garg Abhishek Kapuria Deepak Kumar Dwivedi

Shilpa Singhvi Bhavesh Vaghela Krishna Ravesh Akshay Garg


PREFACE

Welcome to this revised edition of Accounting Standards Simplified (With Ind AS and the Companies Act, 2013)!
It gives us immense pleasure and utmost satisfaction to present the sixth thoroughly revised edition of the
book on “Accounting Standards Simplified (With Ind AS and the Companies Act, 2013)!” Due to enactment
of the Companies Act, 2013 and introduction of Ind AS by ICAI, some additional needs regarding accounts have
come into place. These requirement have been incorporated in appropriate places of respective accounting
standards with a separate section on Ind AS. It is a comprehensive book explaining concepts, problems of
Accounting Standards and Ind AS in a lucid and informative manner. It has been developed exclusively for the
students of CA/CMA Final (New Course). The book will also be useful for the practitioners, professionals and
industry people. The book has been updated with provisions of New Schedule II, III, reference to IAS/IFRS
numbers, latest provisions on AS, MCA circulars.
Key features of the textbook:
◘◘ Comprehensive coverage of the syllabus of Accounting Standards prescribed by Institute of Chartered
Accountants of India relevant for May 2016 Examination.
◘◘ Ind AS covered with problems and solutions.
◘◘ Comparison between IFRS Vs Ind AS is being included wherever necessary.
◘◘ Detail Theory and Practice book on I – GAAP.
◘◘ Updated references of Revised Schedule II and III
◘◘ Extensive coverage of Forex AS-11, Group Consolidation, Financial Instruments AS-30
◘◘ MCA Circular para 46A explained with related problems and solutions
◘◘ Simplified presentation through charts and illustrations.
◘◘ Solved Past 27 exams’ paper with solution till November 2015.
◘◘ By exclusively discussing ‘How to read Ind AS’, the book effectively addresses the queries often raised by students
pertaining to problem solving.
◘◘ Each chapter begins with an overview to help students know, at a glance, what they would learn in the chapter.
◘◘ ‘Do You Know?’ incorporated in each chapter acts as “knowledge alert points”.
Valuable suggestions and constructive feedback from learners is welcome and would be gratefully
acknowledged.

Acknowledgements
We are grateful to our esteemed colleagues, friends and students who have contributed to this book by
advising us and by giving constructive feedback. Special thanks to Dr. Mahesh Gour and Amit Rajpurohit
who have encouraged me to write this book. This book would not have taken its present shape without the
continuous support and encouragement from CA Rakesh Makkar and the editorial and production team of
Carvinowledge Press. It has been a real pleasure working in coordination with their extremely professional
set up. We have immensely benefitted from referring to several books and publications. Thus, we owe an
enormous intellectual debt to all authors, publications, publishers and institutions whose work we have
drawn upon in developing this textbook.
We hope this book serves the purpose of its readers. Their valuable suggestions and constructive feedback
will be greatly acknowledged. Please feel free to e-mail your feedback, problems or suggestion to us on info@
carvinowledge.in or call us: +91-9953922272.
Authors
AN INTRODUCTION TO IND AS

ABBREVIATION STANDS FOR Numbers WHO ISSUED?


IFRS International Financial Reporting Standards 1-15 IASB
IAS International Accounting Standards 1-41 IASC
Ind AS Indian Accounting Standards 1-41, 101-115 ASB (Notified by CG)
Indian GAAPS Indian General Accepted A/C Principles 1-31 ASB (Notified by CG)

In the presentation of the Union Budget 2014–15, the Honorable Minister for Finance, Corporate Affairs and Information and
Broadcasting proposed the adoption of Ind AS. The Minister clarified that the respective regulators will separately notify the
date of implementation for banks and insurance companies. Also, standards for tax computation would be notified separately.
In accordance with the Budget statement, the MCA has notified Company (Indian Accounting Standard) Rules, 2015 vide its
G.S.R dated 16 February, 2015. Accordingly, it has notified 39 Ind AS and has laid down an Ind AS transition road map for
companies other than banking companies, insurance companies and non- banking finance companies.

Ind AS roadmap
Salient features:
◘◘ Early adoption permitted from 1 April 2015, with one year comparatives.
◘◘ Once adopted, cannot be revoked.
◘◘ Companies not covered by the roadmap to continue to apply existing standards.
◘◘ Phase I applicable from 1 April 2016 onward to:
ŠŠListed or unlisted companies whose net worth is >= INR 500 crores.
ŠŠHolding, subsidiaries, joint ventures or associates of these companies.
◘◘ Phase 2 is applicable from 1 April 2017 onward to:
ŠŠListed companies whose net worth is < INR 500 crores.
ŠŠUnlisted companies whose net worth is >= INR 250 crores but < INR 500 crores.
ŠŠHolding, subsidiaries, joint ventures or associates of these companies.
◘◘ Net worth for a company is to be calculated in accordance with its stand-alone financial statements as on 31 March
2014 or the first audited financial statements for accounting period which ends after that date. Accordingly, if any
company’s net worth is more than INR 500 crore as of March 31, 2015, then it will be covered in Phase 1 itself.
◘◘ Overseas subsidiary, associate, joint venture and other similar entity (ies) of an Indian company may prepare its stand-
alone financial statements in accordance with the requirements of the specific jurisdiction. However, for group reporting
purpose (s), it will have to report to its Indian parent under Ind AS to enable its parent to present CFS in accordance with Ind AS.
◘◘ As per exemption under Rule 5, Insurance companies, banking companies and non-banking Finance companies
are not required to apply Ind AS for preparing their financial statements Either voluntarily or mandatorily, as
specified in the roadmap (sub-rule (1) of rule 4).

How to read Ind AS? Let’s Guess the Mess!


(1)
CA Final Syllabus: (Excluding Financial Reporting other topics)
◘◘ Full Old AS 1-32
◘◘ Full Ind AS 1-41 plus 101-115
◘◘ Comparison between Ind AS and Indian GAAP
◘◘ Comparison between Ind AS and IAS / IFRS
Oh My GOD! What a voluminous syllabus?
Wait: CFS, Corporate Reconstruction, VAS, EVA, HRA, MF, NBFC, Valuation of Shares / Goodwill
(2) ICAI should have just introduced IFRS that’s all, replacing all these mess ups.
(3) PATTERN OF PAPER THAT IS: Type of Question expected to be asked in forthcoming May 2016 paper on Ind AS
/ Ratio of marks between AS Vs Ind AS Vs. IFRS Comparison Vs. Other FR Chapters? Just go for a blind date dude.
(4) Do you know Ind AS – 113, 114, 115 is not even adopted fully world - wide. But students are supposed to know at
that stage where they are unaware of Ind AS itself.
(5) I have authored Indian GAAP in greater details. All Ind AS has been dealt in details except after 110 – 115 which is
too advance.
(6) Students are requested to read Indian GAAP book and Ind AS book together by correlating. For example: If a
student is reading AS-12 (Government Grants) then immediately the student should refer Ind AS – 20 because
common points are not included in Ind AS. Only exclusive discussion is made in Ind AS. But Standards like Ind AS
-41 Agriculture is a stand - alone Standard where we don’t have any Indian GAAP.
(7) In my next editions I will try to cover more of Advanced Ind AS like Consolidation, Share Based Payments.
(8) My suggestion to students is that: Don’t refer Share Based Payment, CFS from Ind AS. This is because you have full
Accounting Chapters in your syllabus. Insurance Contracts Ind AS – 107 is undergoing revision by IASB, again skip that.

IMPORTANT ICAI ANNOUNCEMENTS FOR MAY 2016 EXAMINATION


Announcement regarding Non-application of AS 30, AS 31 and AS 32 in the Final Course Paper 1:
Financial Reporting - (23-11-2015)
The entire IAS 39 “Financial Instruments: Recognition and Measurement”, on which AS 30 “Financial Instruments:
Recognition and Measurement” was based, has been replaced by IFRS 9 “Financial Instruments”. Therefore, the
Government of India opted to notify Ind AS 109 “Financial Instruments” in correspondence to IFRS 9 and not
IAS 39. Also, AS 30, AS 31 and AS 32 on ‘Financial Instruments’ were earlier proposed to be made mandatory for
Level I entities only. However, after notification of Ind AS in February, 2015, these entities will be applying the
provisions stated in Ind AS 32, Ind AS 107 and Ind AS 109 and not AS 30, AS 31 and AS 32 for accounting of financial
instruments. Therefore, it is felt appropriate to make applicable Ind AS 32, Ind AS 107 and Ind AS 109 in place of
AS 30, AS 31 and AS 32 to the topic ‘Accounting for Financial Instruments’. Accordingly, it has been decided to make
Ind AS 32 “Financial Instruments: Presentation”, Ind AS 107 “Financial Instruments: Disclosures” and Ind AS 109
“Financial Instruments” applicable on the topic ‘Accounting for Financial Instruments’ instead of AS 30 “Financial
Instruments: Recognition and Measurement”, AS 31 “Financial Instruments: Presentation” and AS 32 “Financial
Instruments: Disclosures” from May, 2016 examinations for Paper 1: Financial Reporting at the Final level.
Further, it may also be noted that existing Accounting Standards as already given in the study material will continue
to be part of the syllabus along with the topic “Introduction of Indian Accounting Standards (Ind AS); Comparative
study of ASs vis-a-vis Ind ASs; Carve outs/ins in Ind ASs vis- à-vis International Financial Reporting Standards
(IFRSs)”, which has been recently included in the syllabus from May, 2016 examination and onward. Thus for the
remaining topics of the syllabus of Financial Reporting paper, existing Accounting Standards will still be applicable.

Clarification on Non-applicability of AS 30, 31 and 32 - (02-01-2016)


With reference to the “Announcement regarding Non-application of AS 30, AS 31 and AS 32 in the Final Course Paper 1: Financial
Reporting” dated 23.11.2015, it is clarified that students are not required to study the differences between Ind AS 32, 107 and 109
vis-à-vis AS 31, 32 and 30 respectively from May, 2016 examination.
Additional Secretary,
Board of Studies
CONTENTS

Foreword iv
Editorial Board v
Preface and Acknowledgements vi
An Introduction to Ind AS vii
Schedule -II: AS per the Companies Act, 2013 xii

Part I - Accounting Standards


Chapter 1. Accounting Standard Introduction and Framework 1
Chapter 2. AS-1: Disclosure of Accounting Policies 8
Chapter 3. AS-2 (Revised): Valuation of Inventories 11
Chapter 4. AS-3 (Revised): Cash Flow Statement 24
Chapter 5. AS-4: Events Occuring After the Balance Sheet 34
Chapter 6. AS-5 (Revised): Net Profits or Loss for the Period, Prior Period Items and
Changes in Accounting Policies 40
Chapter 7. AS-6: Depreciation Accounting 47
Chapter 8. AS-7 (Revised): Construction Contracts 55
Chapter 9. AS-9: Revenue Recognition 69
Chapter 10. AS-10: Accounting for Fixed Assets 79
Chapter 11. AS-11 (Revised): The Effect of Changes in Foreign Exchange Rates 91
Chapter 12. AS-12: Government Grants 101
Chapter 13. AS-13: Investment Accounts 107
Chapter 14. AS-14: Amalgamation and Absorption 117
Chapter 15. AS-15 (Revised): Employees Benefits 121
Chapter 16. AS-16: Borrowing Costs 137
Chapter 17. AS-17: Segmental Reporting 150
Chapter 18. AS-18: Related Party Disclosures 166
x Accounting Standards

Chapter 19. AS-19: Leases 173


Chapter 20. AS-20: Earning per Share 193
Chapter 21. AS-22: Taxes on Income 208
Chapter 22. AS-24: Discontinuing Operations 226
Chapter 23. AS-25: Interim Financial Reporting 232
Chapter 24. AS-26: Intangible Assets 243
Chapter 25. AS-28: Impairment of Assets 255
Chapter 26. AS-29: Provisions, Contingent Liabilities and Contingent Assets 273
Chapter 27. AS-21: Consolidated Financial Statements (Holding Company Vs Subsidiary) 283
Chapter 28. AS-23: Accounting for Investment in Associates 289
Chapter 29. AS-27: Financial Interest in Joint Ventures 300
Chapter 30. Comprehensive Problems on CFS (AS-21 + AS-23 + AS-27) Group Consolidation 210
Chapter 31. AS-30, 31, 32 Financial Instruments Corresponding to IAS-32, 39, IFRS-7 322

Extra Practical Problems from Past Year Papers and RTPs of Financial Reporting and Audit 362
Solved Question Paper—CA Final-May 2013 to November 2015 402

Part II - Ind AS
T R E ND A NA LYSIS OF P REVIOUS Y EAR EXAMINATI ON S

Accounting Standards: Chapter-Wise Analysis from November 2008 – November 2015


N 08 J 09 N 09 M 10 N 10 M 11 N 11 M 12 N 12 M 13 N 13 M 14 N 14 M 15 N 15
AS-2 5 5 10 9 5 9
AS-3 4
AS-4 5 19 10
AS-5 10 5 5 4 4 4
AS-6 4 4
AS-7 5 5 5
AS-9 5 5 5 5 5 4 9
AS-10 5 9 9 5 5
AS-11 5 4
AS-12 9 5 4 4
AS-13 4 5 4
AS-15 4 5 10 11 9 5 5
AS-16 5 5 9 4
AS-17 5 5
AS-18 4 4
AS-19 5 4 11 5 5 4 4
AS-20 5 4 4 4 5 5
AS-21
AS-23 5 8 5 4 4
AS-27
AS-22 4 5 5 10 5 6 5 4 5
AS-24 4 4
AS-25 5 5 10 5 5
AS-26 5 4 8
9
AS-28 4 5 4 5 5 5 5
AS-29 5 5 4 4 10 3 4 4
AS-30 4 8 17 20 5 12 8 4 10 8 8 4
Misc. 12 4 12 5 8
Sch. VI
4 - 5 4 5 8 4
or III
Total 42 46 54 42 84 40 51 48 56 57 68 41 60 60 47

Average 53.5 marks


Note:
(i) The above scanner of AS includes Financial Reporting and Audit both.
(ii) It covers only the current syllabus (November 2008 – November 2015).
(iii) AS-21/23/27 means only Accounting Standard related questions and not CFS.
(iv) AS-30 includes 30/31/32.
(v) In November 2010 AS was asked for 84 marks!
SCHEDULE II: DEPRECIATION
[AS PER THE COMPANIES ACT, 2013]

Key highlights:
◘◘ Instead of % of depreciation now it is useful life of the asset. Remembering rate is difficult but life is
easy. Example: 9 years is easy to recall but 11.11% is tough!
◘◘ Maximum cap on salvage value.
◘◘ Other than SLM, WDV even units of production can be applied.
◘◘ Component accounting is now mandatory.
◘◘ Schedule II is almost adhering to the principles of IFRS – IAS : 16 (Thanks Central Government to keep
policies akin to global guidelines)
PART A : Deals with main provisions relating to depreciation of tangible and intangible assets.
PART B : Deals with provisions relating to depreciation of assets covered by the act of the parliament.
PART C : Its provides the list of the useful life of an asset. It also provides some relevant notes related to the
calculation of depreciation.

DO YOU KNOW
(i) AS-6 states that depreciated should be calculated as per the rates given under special act / statute?
Example: Electricity Act, Companies Act 1956. But now Schedule II overrides all the acts / statutes.

(ii) Useful life is inversely proportion to Rate of depreciation. Higher the rate lower the useful life.

PART A of Schedule II
TANGIBLE FIXED ASSETS
Depreciation as per Schedule II is the systematic allocation of depreciation over the useful life. It means
depreciation is to be allocated as per the method chosen and up to its useful life.
Method permitted can be SLM, WDV or even production / unit method.
Depreciation amount will be (Cost – residual value). Can we say that Depreciation as per Schedule II is the
systematic allocation of depreciation amount over the useful life.
Useful life: Schedule II does not provides rates but has provided the maximum useful life. The useful life
given in PART ‘C’ is the maximum life (minimum rates of depreciation) .
Residual Value: The New Schedule related to depreciation casts a cap on the RV to be 5% on original cost.
Different Useful life and Residual Value: The useful life of an asset shall not be longer than the useful life
specified in Part ‘C’ and the residual value of an asset shall not be more than 5% of the original cost of the
asset:
Provided that where a company uses a useful life or residual value of the asset which is different from the
above limits, justification for the difference shall be disclosed in its financial statement.;
 xiii

INTANGIBLE FIXED ASSETS:


Schedule II covers only one type of intangible asset for amortisation i.e. Toll rights. All other intangible assets
are to be covered by AS-26. Can we conclude that amortization of Patents, Copy - writes, Licenses; Brand etc
are still all covered by AS-26 except “Service Concessions arrangements”.

PART ‘B’
The useful life or residual value of any specific asset, as notified for accounting purposes by a Regulatory
Authority constituted under an Act of Parliament or by the Central Government shall be applied in
calculating the depreciation to be provided for such asset irrespective of the requirements of this Schedule II.
Example: Depreciation rates for Electricity Supply Company has to follow the old Schedule.

PART ‘C’:
Here, useful life of assets is given (students need not remember the same).
NESD: means No extra shift depreciation.

Other important notes:


1) Asset costing ` 5000 or less:
There is no specific requirement to charge 100% depreciation on assets whose actual cost does not
exceed `5,000, so any assets irrespective of value should be categorized based on Schedule II and
depreciated based on Useful life of assets. However, Para 12 of Accounting Standard 6 says that In
respect of depreciable assets which do not have material value, depreciation is often allocated fully
in the accounting period in which they are acquired. This means irrespective of the cost price of the
asset schedule II wants the asset to be depreciated as per the useful life.
2) Additions and Dispositions:
Where during any financial year, any addition has been made or sold, discarded, demolished or
destroyed, the depreciation on such assets shall be calculated on pro rata basis.
3) Only one Intangible assets covered:
All Intangible Assets are covered by AS-26 except Toll road under BOT system. Amortization in such
case is based on Revenue basis.
4) Depreciation for double, triple shift assets:
In Schedule XIV of Companies Act, 1956 for the assets used in double shift or triple shift, shift wise
different rates are prescribed, whereas in Schedule II of Companies Act, 2013 it specifies that if an
asset is used for any time during the year for double shift, the depreciation will increase by 50% for
that period and in case of the triple shift the depreciation shall be calculated on the basis of 100% for
that period. Increase in depreciation means decrease in life.
The above provisions are not applicable to NESD assets.
5) Component Accounting - Note No 4 of Schedule II : (supported by AS-10 as well as IAS – 16)
(a) Useful life specified in the Schedule II to the Companies Act is for whole of the asset. Where
cost of a part of the asset is significant to the total cost of the asset and useful life of that part
is different from the useful life of the remaining asset, useful life of that significant part will be
determined separately
(b) This paragraph talks about the ‘component approach’, which is in line with international
practices. It requires companies to separately depreciate parts of an asset that are significant
and have a useful life different from the useful life of the asset as a whole
xiv Accounting Standards

(c) The component approach is already allowed in current Accounting Standard 10 – “Accounting
for Fixed Assets” in Paragraphs 8.3, but it seems to be a choice of matter. Now it seems as per the
note 4 it is mandatory in Companies Act, 2013.
(d) Previously companies use to charge component replacement to profit / loss account. But now it
will be separately capitalised.
Example: Building and its Elevator can be treated as a separate component. Also Aircraft body
and its engine can be treated as 2 separate assets.
(d) Effect of Schedule II on Revaluation of assets:
Schedule II has no effect on Revaluation provisions. Lets see what it was:
Whenever an asset is upward revalued the depreciation on the excess portion is charged to the
Revaluation Reserves Account.
(e) Transitional Effect of Schedule II – Very Important provision at the date of applicability of
Schedule II of Companies Act, 2013
From the date of the Companies Act coming into effect, the carrying amount of the asset as on
that date:
(i) Shall be depreciated over the remaining useful life of the asset according to the Act .This
means shifting from the old provisions to the new one the remaining carrying amount is to
be just spread over the remaining useful as per the new provisions on a systematic basis.
(ii) After retaining the residual value, shall be recognized in the opening balance of retained
earnings where the remaining useful life of an asset is nil. It means if the remaining
useful life as per the new provisions is Nil, then the balance in the asset after keeping aside
5% on original cost is adjusted in the opening reserves (free reserves).
(f) Disclosures in Schedule II
The following information shall also be disclosed in the accounts, namely:
(i) Depreciation methods used; and
(ii) The useful lives of the assets for computing depreciation, if they are different from the life
specified in the Schedule
(g) Amortization of Service Concession Arrangement:
In certain countries construction or maintenance of infrastructure for public services (roads,
bridges, water distribution facilities) is contracted out to private-sector operators. The private
sector operators are usually paid for their services over the term of the arrangements. Such
arrangements are often described as “build-operate-transfer” (“BOT”) or a “public-to-private
service concession arrangement.”
Consideration given by the grantor to the operator: If the operator renders construction or
upgrade services, the consideration received or receivable by the operator shall be recognized
at its fair value. The consideration may be treated as an intangible asset.
Recognizing an “intangible asset”—the operator shall recognize an intangible asset to the extent
that it receives a right or a licence to charge users of the public service.
CHAPTER 1
Accounting Standard
Introduction and Framework

Chapter Outline
vv Accounting Standard vv Applicability of AS
vv AS and Auditors vv Companies Accounting Standard
vv Is AS Mandatory Rules (CASR) (2006)
vv Procedure of Issuing AS vv NACAS (National Advisory Committee for
Accounting Standard) or NFRA (National
vv Advantage and Significance of AS
Financial Reporting Standards)

Accounting Standard
Accounting Standards are written documents or policy documents prepared by an expert body or government
body or any other regulatory body for true disclosure and presentation of financial statements. It covers the
aspects of recognition, measurement, treatment, presentation and the disclosures of accounting transactions.
In India AS are issued by the Institute of Chartered Accountants of India (ICAI). The work for preparation and
integration is entrusted by ICAI to ASB.

AS and Auditors
Auditors of every company are required to report under section 143(3)(e) of the Companies Act, 2013,
whether the Profit and Loss Account and Balance sheet are as per AS or not. Statutory auditors are required to
make qualifications in their audit report. While discharging the attest functions, it is the duty of the auditors
to report whether there is compliance of AS by the company. Compliance of AS is the prime duty of the
management and an Auditor is only required to report whether mandotory AS are complied or not.

Is AS Mandatory
Yes, they are mandatory. Following are the contentions to support our argument:
(i) Auditors of every company are required to report under section 143(3)(e) of the Companies Act,
2013 whether the Profit and Loss Account and Balance sheet are as per AS or not. Statutory auditors
are required to make qualifications in their audit report for non-compliance of accounting standards.
(ii) As per Section 134(5)(a) of the Companies Act, 2013 Directors under the ‘Directors’ Responsibility
Statement’ have to report whether or not the company has followed the applicable AS in preparing
its Financial Statements.
(iii) SEBI has made mandatory for all the listed companies to follow AS.
(iv) Financial Statements cannot be described as complying with the AS unless they comply with all the
applicable AS.
2 Accounting Standards

Is AS Mandatory to Co-operative Societies and Charitable Institutions?


No, because it does not run on profit basis. Such societies and clubs are not answerable to shareholders and
even such institutions are floated for charity and social purposes. Yes, of course audit is necessary but AS may
not be applied. But if the society runs on profit basis then AS has to be applied. Even if suppose a part of the
activities of such societies or institutions is profit oriented then AS has to be applied in totality.
E.g: suppose a NGO receives donations and utilise it for the welfare of orphanage community then AS need
not be applied.

Procedure of Issuing AS
(i) The AS body forms a study group known as Accounting Standard Board (ASB) to handle problems
arising on any accounting transaction.
(ii) The study group comprises of experts from ICAI, SEBI, ICSI, and RBI etc.
(iii) ASB shall determine broad areas in which AS need to be formulated and the priority in regard to the
selection thereof.
(iv) The ASB thus formed will hold meetings and receives comments from the members.
(v) On the basis of the work of the Study Groups and the dialogues with the organizations referred
to above, an exposure draft copy of standard is made by the ASB and issued for comments to the
Institute.
(vi) It is finally submitted to NFRA (government body) who assist the government in laying and
formulation of AS.
(vii) ICAI may modify/amend the draft and issues a final draft in the form of Accounting Standard.
(viii) The final AS will then be explained to the professionals (including general public) through journals,
new letters and conducting seminars.

Advantage and Significance of AS


(i) AS makes the Financial Statements more clear and informative.
(ii) Shareholders can rely on such Financial Statements because it shows more transparency + true and
fair view.
(iii) Wise decisions can be made by the investors by comparing Financial Statements of two or more
companies.
(iv) It harmonises the diverse accounting policies and practices adopted in India.
(v) A Company marching towards AS can attract more lenders and investors.
(vi) It reduces the Cost of Capital and uncertainities.
(vii) There are certain areas where important information is not required by law to be disclosed, standards
may call for such disclosures.

Though Accounting Standard has Received Much Importance Worldwide Now, But How it is
Criticized?
(i) Interpretation of AS is a hard nut to crack.
(ii) Many Private Limited Companies are not following AS because they say they are not answerable to
Shareholders.
(iii) Many Chartered Accountants have misinterpreted AS and made it a mess.
(iv) Many issues in AS are based on actual facts and circumstances of the cases which have led to loopholes
and manipulation.
(v) Many CA’s are following accounting practices and policies as per their whims and fancies.
(vi) AS cannot override the statutes and laws.
Accounting Standard Introduction and Framework 3

Applicability of AS
There are 2 classifications made by two different acts/statute.
(I) BY COMPANIES ACCOUNTING STANDARD RULES 2006 (CASR) – SMCs and Non SMCs.
(II) BY ICAI – (i) Level I, (ii) Level II, (iii) Level III Enterprises

Classification made by CASR 2006 or Central Government or MCA


Companies are classified into only two Levels, viz,
(i) Non-SMC companies; and
(ii) Small and medium companies, for brevity referred to as SMCs.
Small and Medium Companies (SMCs): Small and Medium Companies (SMCs) has been defined in rule
2(f): As per the rule, company which satisfies all the following five conditions as at the end of the accounting
period shall be called SMC:
(a) the equity debt securities of the company are not listed or are not in the process of listing of any stock
exchange, whether in India or outside India,
(b) the company is not a bank or financial institution or insurance company,
(c) the company’s turnover (excluding other income) does not exceed `50 crores in the immediately
preceding accounting year,
(d) the company does not have borrowing (including public deposits) exceeding `10 crores at any time
during the immediately preceding accounting year; and
(e) the company is not a holding company or subsidiary of a non-SMC.
Exemptions/relaxations to SMCs: The SMCs are given the following relaxation in complying the notified
accounting standards-
◘◘ SMCs need not prepare cash flow statement as per AS-3 and need not disclose the segment reporting
as per AS-17.
◘◘ The SMCs have been given following relaxation as regards AS-15 “Employee Benefits”:
ww SMCs need not comply paras 11 to 16 of AS-15 to the extent they deal with recognition and
measurement of short-term accumulated compensating absences.
ww Discounting the amount payable after 12 months of balance sheet as regards defined contribu-
tion plans and termination benefits.
ww Recognition, measurement and disclosure principles in respect of defined benefit plans and
other long-term employee benefits plan under para 119 – 123 ans 129 - 131. However such
enterprises should provide and disclose the accrued liability in respect of defined benefit plan
and other long-term employee benefit plan as per actuarial valuation based on projected unit
credit method and discount rate based on yield on Government bonds.
◘◘ SMCs need not disclose diluted EPS as per AS-20 “Earning Per Share”.
◘◘ SMCs need not comply with disclosure requirements regarding operating leases of sub-paras
(b) and (d) of para 46 and sub-paras (a), (b) and (e) of para 2S of AS-19 “Leases” and sub-paras
(a) and (f) of para 37 and sub-paras (c), (e) and (f) of para 22 of AS-19 regarding disclosure for finance
lease by the lessor and lessee - respectively.
◘◘ Value in use has been differently defined for SMCs which provides for just calculating value in use
based on a reasonable estimate of future cash flows rather than present value technique.
◘◘ SMCs are exempt from disclosure requirements of paras 66 and 67 of AS-29 regarding provisions and
its descriptions.
◘◘ AS-18 “Related Party Disclosures” will now apply to all companies including SMCs and as no exemp-
tions /relaxations has been given by Companies (Accounting Standards) Rules, 2006.
4 Accounting Standards

Classification by ICAI (applicable only to non-corporate entities now)


(a) Enterprises falling under Level I is not a SME - Small and Medium Enterprises.
(b) Enterprises under Level II and III categories are referred to as SMEs.

Applicability of Accounting Standards to Non-Corporate Entities


For the purpose of applicability of accounting standards entities are classified into three categories by the
ICAI, however this classification is not applicable to companies covered by classification made by Companies
(Accounting Standards) Rules, 2006.
Level I Entities: Non-corporate entities which fall in anyone or more of the following categories, at the end
of the relevant accounting period, are classified as Level I entities:
(i) Entities whose equity or debt securities are listed or are in the process of listing on any stock
exchange, whether in India or outside India.
(ii) Banks (including co-operative banks), financial institutions or entities carrying on insurance
business.
(iii) All commercial, industrial and business reporting entities, whose turnover (excluding other income)
exceeds rupees fifty crore in the immediately preceding accounting year.
(iv) All commercial, industrial and business reporting entities having borrowings (including public
deposits) in excess of rupees ten crore at any time during the immediately preceding accounting
year.
(v) Holding and subsidiary entities of anyone of the above.
Level II Entities (SMEs): Non-corporate entities which are not Level I entities but fall in anyone or more of the
following categories are classified as Level II entities:
(i) All commercial, industrial and business reporting entities, whose turnover (excluding other income)
exceeds rupees 1 crore but does not exceed rupees fifty crore in the immediately preceding accounting
year.
(ii) All commercial, industrial and business reporting entities having borrowings (including public
deposits) in excess of rupees 1 crore but not in excess of rupees 10 crore at any time during the
immediately preceding accounting year.
(iii) Holding and subsidiary entities of anyone of the above.
Level III Entities (SMEs): Non-corporate entities which are not covered under Level I and Level II are
considered as Level III entities.
(i) Applicability of Accounting Standard to Level I - All the 29 Accounting Standards are fully applicable
to Level I entities.
(ii) Applicability of Accounting Standard to Levels II and III entities (SME) - For the purpose of applicability
of accounting standard to LevelII enterprises the case can be divided into three categories:
Accounting standards fully applicable.
Accounting standards applicable but relaxation from certain disclosure requirements.
Accounting standards not applicable.
Accounting Standards fully applicable: AS-1, AS-2, AS-4, AS-5, AS-6, AS-7, AS-8, AS-9, AS-10,
AS-11, AS-12, AS-13, AS-14, AS-15, AS-16, AS-22, AS-26 and AS-28.
Accounting Standards applicable but relaxation from certain disclosures requirements: AS-19,
AS-20 and AS-29.
Accounting Standards not applicable: AS-3, AS-17, AS-18 and AS-24. AS-21, AS-23, AS-25 and AS-27
are not applicable because of existing regulation in India.
Accounting Standard Introduction and Framework 5

Note: Consequent upon the issue of Companies (Accounting Standards) Rules, 2006, the applicability of the
Accounting Standards as announced by the Institute of Chartered Accountants mentioned above is only for
the entities other than companies. For the companies the applicability of Accounting Standards is as per
Companies (Accounting Standards) Rules, 2006.

Companies Accounting Standard Rules (CASR) (2006)


◘◘ CASR are provided in section 133 of Companies Act.
◘◘ Central Govt. has wide powers to insert, frame, amend CASR.
◘◘ CASR notifies AS 1-29 (not 30,31,32) till now.
◘◘ Exemptions and relaxations for SMCs are provided in CASR.
◘◘ Except a few all the ASI are now incorporated in CASR.
◘◘ Consolidated Financial Statements are still optional under CASR.
◘◘ CASR (now 2009) is yet to notify the IND AS which will be applicable in India.

NACAS (National Advisory Committee for Accounting Standard)/NFRA (National


Financial Reporting Authority)
Under section 210A of Companies 1956 the CG by notification has constituted a committee to advise the
Central Government on the formulation and laying down of accounting standard policies and accounting
standards. NACAS has notified till now 1-29 AS. Members of NACAS are deemed to be government employees.
Under the new companies act the same authority is known as NFRA under Sec. 132.

PROBLEMS AND SOLUTIONS

Problem 1: Comment whether the following Companies can be classified as a Small and Medium Sized
Company (SMC) per the Companies (AS) Rules, 2006.
(a) A Private Limited, a Subsidiary of a Multinational Company listed on London Stock Exchange. It has
a turnover of `12 Crores and borrowings of ` 5 Crores.
(b) B Private Limited has turnover of ` 45 Crores, Other Income of `7 Crores and Bank Borrowings of
` 9 Crores.
(c) C Limited has appointed Merchant Bankers to prepare a Red-Herring Prospectus for the purpose of
filing the same with SEBI. (CA Final Audit Nov. 2008)
Solution: As per the Companies (Accounting Standards) Rules 2006, “Small and Medium Sized Company”
(SMC) means/a company :
(i) Whose equity or debt securities are not listed or are not in the process of listing on any stock exchange,
whether in India or outside India; (ii) Which is not a bank, financial institution or an insurance
company; (iii) Whose turnover (excluding other income) does not exceed rupee; fifty crore in the
immediately preceding accounting year, (iv) Which does not have borrowings (including public
deposits) in excess of rupees ten crore at any time during the immediately preceding accounting
year; and (v) which is not a holding or subsidiary company of a company which is not a small and
medium-sized company.
(a) Since A Pvt. Ltd. is a subsidiary of another Company which is listed, on London Stock Exchange
(and is therefore not a SMC), A Pvt. Ltd. cannot be a SMC. The turnover and borrowings are not
relevant in this case.
6 Accounting Standards

(b) As per the definition of SMC, point (iii), a company will be a SMC if it’s turnover does not exceed
` 50 crores or borrowings do not exceed `10 crore. For calculating this turnover, other income
is not to be included. Since B Pvt. Ltd. has a turnover of ` 45 crores and borrowing; of ` 9 crores,
it will satisfy the definition and can be classified as SMC.
(c) As per the definition of SMC, point (i) a company will be a SMC if it is not listed or in the process
of listing. Since C Pvt. Ltd. has appointed merchant bankers to prepare a Red Herring Prospectus
for the purpose of filling the same with SEBI, it is in the process of listing on a Stock Exchange.
Hence C Ltd. cannot be classified as a SMC.
Problem 2: A Company which satisfies the conditions of a Small and Medium sized Company (SMC) as per
Companies (Accounting Standards) Rules, 2006 has represented that it does not require to give disclosures
required by AS-3 “Cash Flow Statements” and AS-18 “Related Party Disclosures” in its Financial Statements.
Comment.
Solution: AS-3 is not mandatory for SMCs, but there is no exemption/from AS-18 disclosure requirement
the company should ensure compliance with AS-18 and also state that it has availed the exemption given to
SMCs in respect of AS-3.
P roblem 3: XYZ Ltd. with a turnover of `35 lakhs and borrowings of `10 lakhs during any time in the
previous year, wants to avail the exemptions available in adoption of Accounting Standards applicable to
Companies for the year ended 31.3.08. Advice the Management the exemptions that are available as per
Companies (AS) Rules, 2006.
Solution: XYZ company is a SMC company as defined in AS rules no 2 (f) of the companies act.
The exemptions applicable for Small and Medium sized company (SMC) are as follows:
(a) Need not disclose certain information and this fact shall be conveyed by way of a note to financial
statement.
(b) If XYZ becomes a non-SMC, then the previous year’s information need not be disclosed, but the
information of this fact shall be disclosed by way of a note.
(c) In case XYZ opts not to avail exemptions applicable to SMC for any or some of the ASs, it shall disclose
the AS in respect of which it has availed the exemption.
(d) In case XYZ desires to disclose the information not required to be disclosed pursuant to the exemptions
availed, it shall disclose that information in compliance with the relevant accounting standard.
(e) XYZ may opt for availing certain exemptions from compliance with the requirements prescribed in an
Accounting Standard. Provided that such a partial exemption and disclosure shall not be permitted
to mislead any person.
If XYZ is a partnership firm, the applicability of this Act cannot be extended at all. The question of
application of AS to such firms will not arise and therefore there is no room for exemptions as well as
additional exemptions.
Accounting Standard Introduction and Framework 7

List of Accounting Standard Interpretation (ASI)

ASI No. Related AS Topic


1 16 Interpretation of the term ‘substantial period’
2 2,10 Treatment of machinery spares
3 22 Computation of deferred tax during tax holiday u/s.80-IA and 80IB
4 22 Computation of deferred tax in respect of losses under the head Capital Gains
5 22 Computation of deferred tax during tax holiday u/s. 10A and 10B
6 22 Computation of current and deferred tax subject to MAT u/s. 15JB
7 22 Disclosure of DTA/DTL
8 21,23,27 Interpretation of the term ‘near future’
9 22 Interpretation of the term ‘virtual certainty’
10 16 Computation of exchange difference to be treated as borrowing cost
11 22 Computation of deferred tax in case of Amalgamations
12 20 Applicability of AS-20 to unlisted companies
13 18 Aggregation of related party disclosures
14 9 Manner of disclosure of excise duty
15 21 Notes to the Consolidated Financial Statements (CFS)
16 23 Treatment in CFS: Dividend proposed by an associate
17 23 Treatment in CFS: Changes in equity not included in Profit and Loss A/c.
18 23 Consideration of potential equity to ascertain whether the investee is an associate
19 18 Interpretation of the term ‘intermediary’
20 17 Disclosure of segment information in certain cases (Revised)
21 18 Non-executive directors, whether related parties
22 17 Interest expenses; whether to treat as segment expenses
23 18 Remuneration paid to key management personnel; whether related party transaction
24 21 Subsidiaries having two parents
25 21 Shares held as stock-in-trade
26 21 Consolidation of current and deferred tax
27 25 Applicability of AS 25
28 21,27 Disclosure of post-acquisition reserves in Consolidated Financial Statements
29 7 Turnover in case of contractors
30 29 Onerous Contract
CHAPTER 2
AS-1: Disclosure of
Accounting Policies

Chapter Outline
vv Accounting Policies vv Selection of AP or Principles of AP or Points
vv Requirements of Accounting Policies – AS - 1 to be Considered While Selecting AP
vv Fundamental Accounting Assumptions vv Are Changes in Accounting Policies Allowed?

Accounting Policies
Accounting Policies are rules/principles and methods to adopt such principles in financial accounts for true
and fair recording, and presentation of financial statements.
As per IAS 8, Accounting Policies are the specific principles, bases, conventions, rules and practices applied
by an entity in preparing and presenting the Financial Statement.

Example: A Company may have an option of charging depreciation as per Straight Line Method, Written
Down Value, Sinking Fund method. If among the methods Straight Line Method is chosen, then we say
company’s policy to charge depreciation is Straight Line Method. Accounting Policies are part of Financial
Statements (FS).
A company following any Accounting Policies has to choose Accounting Policies according to the nature
of the business. Choosing of a correct Accounting Policy is very important because it affects Profit and Loss
Account, Balance sheet etc. As different companies operate under different circumstances, Accounting
Policies has to be applied accordingly.
Thus Accounting Policies mean:
◘◘ Specific Principles (Example: Value of stock at Cost/MV whichever is less)
◘◘ Methods to apply such Principles (Example: Cost formulae FIFO, WAC).

Requirements of Accounting Policies – AS - 1


(i) Disclose all the significant Accounting Policies adopted in the preparation of financial accounts.
(ii) The financial statements should disclose true and fair view of the financial accounts the year.
(iii) Proper disclosure of Accounting Policies would facilitate more meaningful information to the users.
(iv) The following are to be considered for choosing any standard: Materiality, Substance over form,
Prudence.
(v) In the absence of disclosure and information provided by the company it is assumed that the basic
principle of Going Concern, Accrual System and Consistency is followed.
AS-1: Disclosure of Accounting Policies 9

Fundamental Accounting Assumptions


Before a shareholder refers to any financial statements he assumes the following:
(i) Going Concern: Going concern assumption means a company has no intention to wind-up the business
in the next few years, hence every shareholder assumes a company runs its business continuously. If in any
year the company has an intention to wind up its business then the company has to inform the investors in
its Financial Statement regarding closure of business in near future. The definition does not specify what is
near future (foreseeable future). SA570 issued by ICAI defines foreseeable future as a period not exceeding
one year after the balance sheet date.
(ii) Consistency: It means following the same policies every year. According to SA710 Comparatives,
consistency is a quality of the relationship between two accounting numbers. Consistency is a prerequisite
for true comparability of financial statements.
(iii) Accrual: It is a mercantile system. Accrual of expenses and sale of goods or rendering of services though
on credit is important, receipt and payment is immaterial. It is assumed in case of Revenues and Costs that :
(a) They are recorded on the basis of dues and accruals and not on the basis of receipts and payments.
(b) They are recorded for a particular financial year.
(c) Matching concept is adopted.
If a company is not following any of the above assumptions, it has to give a note in Financial Statement that in
future or at present the Company is not following particular fundamental accounting assumption.

Selection of Accounting Policies or Principles of Accounting Policies or Points to


be Considered While Selecting Accounting Policies
There is no single list of Accounting Policies applicable to all the companies. Before selecting any Accounting
Policy, a company has to consider the following factors:
(i) Prudence: Future is uncertain; hence decisions are to be made wisely. As per Prudence concept loss is to
be recognized (recorded) even if it is not sure. But profit though very sure should not be recognized. Profits
are to be recorded only when it is realized. In short; unrealised losses plus realised losses both are to be
recorded but in case of profits only realized profits are to be accounted. No Castles in the air.

Example: Investment are valued at cost or market value whichever is less. However, the exercise of
prudence does not allow:
◘◘ The creation of hidden reserves.
◘◘ Understatement of assets or income.
◘◘ Overstatement of liabilities.

(ii) Substance over form: One should see the economic reality of a transaction and not the legal form. Legal
form is to be ignored and the substance is to be considered.

Example: By legal form lessor is the owner during the lease period but actually speaking lessee bears the
risks and rewards relating to the asset. So, one should see the economic reality of the transaction.
Hence lessee is the owner who uses the asset and not the lessor.
(iii) Materiality: If the decision of the investors changes by assessing the transaction then the transaction is
material. In other words, financial statements should disclose all facts, which may influence the mind of the
shareholders. SA – 320 on Materiality has explained in detail the concept of Materiality.
10 Accounting Standards

Changes in Accounting Policies Allowed?


One cannot change Accounting Policies regularly. If a company wants to change Accounting Policies once
followed then it has to satisfy the following conditions:
(i) Such change should be allowed by Statute. For example: Before AS – 2 was revised many companies
were following LIFO method of Stock Valuation. But, AS –2 (Revised) has specifically excluded LIFO
method. Thus, all the companies henceforth have to adopt FIFO, WAC method but not LIFO.
(ii) If change in accounting policies present more true picture of financial statements then such change
is allowed.
(iii) A note should be given by the company in its annual accounts regarding the change. Also the amount
of difference is to be disclosed after all the change have been incorporated in the financial statements.
CHAPTER 3
AS-2 (Revised):
Valuation of Inventories

Chapter Outline
vv Objective and Scope of AS-2 vv Joint Product and By-products
vv Meaning of Inventory vv Cost Formulae
vv Valuation of Inventory vv Net Realizable Value
vv Absorption Costing – For Fixed Overheads vv Disclosures
vv Highlights of Cenvat Credit and
Excise Duty (Guidance Note)

Objective and Scope of AS-2


Objective: Inventory is said to be “bread and butter” for every business concern.
The reasons emphasized for valuing stock includes:
◘◘ Stock constitutes one of the major parts of the total assets employed.
◘◘ It determines the amount of profit by valuing closing stock.
◘◘ It even shows the financial position of a company, which reflects in the Balance Sheet.
◘◘ Inventories are subjected to obsolescence, spoilage etc.
Scope: AS-2 excludes within its scope the following:
◘◘ WIP arising out of construction contracts. (See AS-7)
◘◘ WIP arising out of service providers. Ex: Service Contract of a CA Firm
◘◘ Financial instruments like Shares, Debentures etc. (See AS-13)
◘◘ Livestock, agricultural and forest products and mineral oil and gases to the extent they are measured at net
realizable value in accordance with well-established practices in these industries.

Meaning of Inventory
Inventories Consists of the Following:
◘◘ Goods purchased and held for resale → (Merchandise purchased by retailers).
◘◘ Held for sale in the ordinary course of business → (Finished goods).
◘◘ In the process of production → (WIP).
◘◘ Materials and supplies awaiting production → (Raw-materials, stores, loose tools, consumables).
12 Accounting Standards

DO YOU KNOW
AS-2 (Rev.) is not applicable to WIP of Service Companies, but any unconsumed material attracts AS-2.
Ex: Services of Saloon/Beauty Parlour is Service Industry but materials kept for use is inventory.

Note: Whether spare parts used in machinery comes under AS-2?


Actually speaking machinery spares are capitalized under AS-10. Such spare parts are specific in nature and
in irregular use. But if the spare parts are used very commonly then they come under AS-2. Such commonly
used spare parts are passed through revenue account in the year of its consumption.

Valuation of Inventory (on the B/S Date)


Inventories are Valued at (on the B/S Date) Lower of Cost or Net Realizable Value
Cost of Inventory: It includes-
◘◘ Cost of purchase
◘◘ Cost of conversion
◘◘ Other costs
Cost of Purchase: It includes-
◘◘ Purchase price
◘◘ Duties and taxes (except those which is refundable from the tax departments)
◘◘ Freight inwards
◘◘ Any other expenditure directly attributable to the acquisition
Less:
◘◘ Trade discounts and not Cash discount
◘◘ Rebates, Duty Drawbacks or any other similar item.
◘◘ Duties and taxes recovered from taxing authorities
Cost of Conversion: The principle behind the inclusion of cost is that all the cost incurred for bringing the
inventory to its present condition and location should form part of cost of stock.
Cost of Conversion includes → Costs incurred directly towards production
(Direct Material + Direct Labour + Direct Expenses + Systematic allocation of Fixed and Variable overheads
incurred in converting materials into finished products)
Variable production overheads: These are those indirect costs that vary according to the production
volume (like indirect materials, indirect labour).
Allocation of Variable production overheads: It is based on actual units produced.
Fixed production overheads: These are costs that remain constant irrespective of level of production (like
depreciation, salary/rent etc. of factory management)
Allocation of Fixed production overheads: As per pre-revised AS-2 and accordingly the companies were
following allocation of fixed production overhead based on marginal costing or treating fixed production
overhead as period costs and fixed production overhead were debited to Profit and Loss Account. But now as
per AS-2(revised) fixed production overheads are allocated based on Normal Capacity Levels.
What is Normal Capacity levels?
Normal Capacity Level (NCL) is the level of production expected to achieve on an average over a number
of periods under normal circumstances or seasons, taking into account the loss of capacity due to planned
maintenance.
AS-2 (Revised): Valuation of Inventories 13

Absorption Costing – For Fixed Overheads


If Actual production > Normal If Actual production < Normal
FPOH Absorption 100% Actual units x Recovery rate
Inefficiency debit to P/L Nil FPOH incurred – FPOH absorbed
Valuation of CS Based on FPOH/Actual units Based on FPOH/Normal units
Factory overheads such as factory managers salary, cost of stationery (for preparing job cards, time
sheets etc.) incurred in factory are cost incurred for production activities. But administration expenses like
printing stationery used by offices, rent of the office has to be expensed because such expenses are not clearly
related/incurred in effecting the change in the present condition and location of stock. Also Transportation,
unloading expenses incurred in moving the stock from factory to the sellers stock yard before sale, should
be construed as the cost bringing the inventory to its present condition and location and should form part
of cost of production. But the expenses incurred from the stock yard to the customer location are selling and
distribution expenses.

Highlights of Cenvat Credit and Excise Duty (Guidance Note)


◘◘ As per AS-2 CENVAT credit should not form part of cost of raw materials (as given by guidance note and
supported by New Schedule III). It is to be reduced from Costs of materials. CENVAT credit (inputs) a/c
is required to be incorporated on the asset side. Whenever excise duty liability is calculated CENVAT
credit can be utilized.
◘◘ But excise duty payable on Finished goods should be provided and forms part of cost of production.
It means excise duty payable on finished goods is a direct production costs. A company has to provide
for the Excise Duty on finished goods once they are manufactured.
◘◘ Excise duty payable on finished goods should be included in the valuation of finished goods as per
AS-2, though excise duty is paid only on the clearance of goods.
◘◘ The liability for excise duty is accrued once the goods are manufactured.
◘◘ No excise duty is chargeable on goods meant for export sales.

Guidance Note vs Tax Filing Return


The above ‘Guidance Note’ is applicable for AS-2 but for filing tax returns Gross approach has to be followed
i.e. raw materials should be inclusive of input duty (Section 145A).

DO YOU KNOW
Income Tax follows gross approach and AS-2 follows net approach, but the overall impact of CENVAT/EXCISE
remains the same.

Joint Product and By-products


During the process of raw-materials more than one product may be produced in addition to the main products.
It may be joint products or by-products.
Allocation for Joint products: When two or more products are simultaneously produced with a common
set of inputs each product is of significant value then such products are called joint products. Ex. In case of
Meat processing joint products are meat, hides, bones. If the costs of conversion are not identifiable then
allocation will be based on rational and consistent basis.
14 Accounting Standards

Allocation as per physical units method or relative sales method at the separation point is the most
acceptable basis for apportionment of joint costs.
Allocation for by-products: By-products incidentally results from the common process. The value of such
by-products or waste or scrap is very insignificant. Thus, such by – products or waste etc. are valued at Net
Realizable Value (NRV) and this NRV is deducted from the costs of main product.

Formulae
Common cost for (product A+ product B + by-product X) XXX
Less: NRV of byproducts/waste/scrap (XXX)
Net joint cost to be apportioned between products A and B XXX

Any Other Cost


Any other cost includes expenses incurred in bringing the inventories to its present location and condition.
Example loading and unloading charges, carriage inwards, special design cost etc.

Exclusions from Cost of Inventories


In determining the cost of inventories certain costs are excluded which are expensed in the year it is incurred.
Following are the list of elements to be excluded from cost:
(i) Abnormal amount of wasted materials, labour or other production costs.
(ii) Interest and storage cost (see detailed explanation below).
(iii) Administrative overheads (only that part of expenses, which do not contribute to bringing the
inventories to its present location and condition).
(iv) Selling and distribution overheads (advertisements, after sales services etc.).

Note: Why Interest and storage cost not included in valuation of stock?
As per AS-2 interest and storage costs are not to be included in the cost of the inventories. The reason being
such costs is not incurred for the bringing of inventories to its present location and condition. However,
if one refers to AS-16 (Borrowing Costs) few exceptions are given. AS-16 states that if an asset takes a
very long time to get manufactured then in such case costs of storage etc. is to be included in the costs of
inventories because such long period is necessary for the intended use or sale of the asset.
Normal/Abnormal Losses: In determining the cost of inventories a company may incur some normal or
abnormal losses. Accounting treatment for such losses is as follows:
(i) Normal loss: Normal loss always forms part of cost of production. It means good units will have to
bear the cost of normal loss.
(ii) Abnormal loss: Abnormal loss is an expense which should be charged to Profit and Loss Account. It
means abnormal losses have to be written off in Profit and Loss Account.

Cost Formulae
An enterprise should use different cost formulae according to the nature of product and its operations scale.
Following are some of the cost formulae as permitted by AS-2.
(i) Specific identification method: In this method specific costs are assigned to specific items of
inventories. Costs can be properly appropriated to the finished products. Specific identification
method can be followed only when the items are not ordinarily interchangeable or in case of goods
or services produced and segregated for specific project.
AS-2 (Revised): Valuation of Inventories 15

Example: CNP Ltd. is a consumer durable company and has different products. As the products are
specifically identifiable and not interchangeable in terms of use/size/brands/value etc we have to
assign costs to each product separately.
(ii) FIFO method: We are very conversant with FIFO method, as we have done in our earlier class. This
method is used only when goods are ordinarily interchangeable. Here we assume that the items,
which were purchased, first will be consumed or sold first. This means the items remaining in stock
will be nothing but latest purchases. Thus one can say to the items of materials that “you are in
queue”.
(iii) Weighted Average method: Under this method total cost is divided by total units with every fresh
purchases or periodically say month or weekly etc. This method very well takes care of changes in
prices.
Following technique is to be used only when it is impossible to calculate cost by the above three
methods. But the following techniques are to be used if it approximates the actual cost.
(iv) Standard Costing: Standard Costing (SC) is a pre-determined cost which takes into account normal
level of consumption of materials, labour and indirect services. It is used widely in industries
having non-repetitive process like manufacturer of automobile, boilers, heavy electrical equipment.
Standard costing is to be updated and revised periodically.
(v) Retail Method: Retail or Adjusted selling price method is used in the following circumstances:
(1) Large number of innumerable different items in the stock (Example: Cutlery Retail Shop)
(2) Unit cost of such items is very low.
(3) Detailed records of such stock is not kept.
(4) All the items having similar margins.
(5) It is impossible to use any other method.
Procedure used to compute cost:
(1) Determine normal retail (selling) price of the year-end stock.
(2) Find the Gross margin
(3) Cost of end inventory = Normal retail price – Gross margin.

SPECIAL HIGHLIGHTS OF COST FORMULAE TO BE USED


(a) AS-2 favours FIFO or Weighted average cost method.
(b) Specific identification method is to be used only when goods are not normally similar/interchangeable.
(c) HIFO/LIFO is scrapped.
(d) Standard costing/Retail inventory method is to be adopted as a matter of convenience if the results approx.
the actual cost.

Net Realizable Value


Valuation of Finished Goods (FG), Work-in-progress (WIP) and Intermediate Products
Net Realizable Value (NRV) is
Estimated Selling Price in the ordinary course of business xxxx
Less: Estimated costs of completion –xxx
Less: Estimated costs necessary to make the sale –xxx
NET REALIZABLE VALUE xxxx
16 Accounting Standards

Note: The above formula for NRV is applicable to WIP (intermediate products). For finished goods we should
not consider the cost to complete the goods.
Other points for NRV:
(1) NRV should be based on most reliable evidence at the time of valuation.
(2) NRV is to be estimated at every balance sheet date.
(3) If NRV > Cost, then stock is to be valued at Cost.
(4) If NRV < Cost, then stock is to be valued at NRV. (In short stock is to be valued at Cost or NRV
whichever is less).

Why one should go for NRV on balance sheet date?


It’s like a person should get itself checked by a Doctor periodically so that he may not be suffering from any
diseases. Its rightly said “A stitch in time saves nine”. Similarly a company should assess NRV of stock the
reason being the stock may suffer from the following diseases:
(1) Stock may have gone out of fashion or become obsolete, so even cost may not be recoverable.
(2) Selling prices of stock have come down.
(3) Estimated costs of completion have increased.
(4) Estimated costs necessary to make the sale have increased.

Purpose for which Stock is kept


If the inventories are kept to satisfy firm sale or service contracts then following are the various situations as
tabulated below :

Situation NRV is based on


(1) Closing Stock = Contracted Value NRV is based on Contract Price
(2) Closing Stock > Contracted Value NRV of contracted value is based on Contract Price and
NRV of excess quantity on Selling Price.
(3) Closing Stock < Contracted Value NRV is based on Contract Price. Provide for Contingent loss
as per AS-29 on difference.
Contracted value means goods kept for specific contracts and not to be sold in the market.

Valuation of Raw Materials


One should not use the formulae of NRV here. Following is the valuation principle of RM: If both the conditions
are satisfied then RM is valued at Replacement Cost, else Historical Cost:
(i) finished goods is expected to be sold below cost,
(ii) the price of the raw material is on decline.

Disclosure in the Financial Statement


(i) Accounting policy used including the cost formulae.
(ii) Carrying amount of inventories viz., WIP, Raw-materials etc.
AS-2 (Revised): Valuation of Inventories 17

PROBLEMS AND SOLUTIONS


Problem 1: Anil Pharma Ltd. ordered 16,000 kg of certain material at `160 per unit. The purchase price
includes excise duty `10 per kg in respect of which full CENVAT credit is admissible. Freight incurred
amounted to `1,40,160. Normal transit loss is 2%. The company actually received 15,500 kg and consumed
13,600 kg of material. Compute cost of inventory under AS-2 and amount of abnormal loss.
(CA Final May 2011)(Marks 4)
Solution: Normal cost per kg of RM
`
Purchase price (16000 × 160) 2560000
(–) cenvat credit (16000 x 10) – 160000
(+) Freight 140160
Total Cost of RM 2540160
Normal cost of RM = 2540160/15680 (normal units) 162/-

Allocation of material cost Kgs Per kg `


Material consumed trf to trading a/c 13600 162 2203200
Closing stock of RM trf to B/S 1900 162 307800
Abnormal loss trf to P& L a/c 180 162 29160
Total Cost of RM 15680 2540160

Problem 2: X Limited is engaged in manufacture of Cement. In the P/L account for the year ended
31/3/2007, it discloses its revenue from sales transactions (turnover) net of excise duty. The excise duty
collected and paid on sales transaction and that related to difference between Closing and Opening stock is,
however disclosed in the “Notes to Accounts”. (Audit CA Final)
Solution: As per Accounting Standard Interpretation – ASI 14 ‘Disclosure of Revenue from Sales Transactions’
(Revised) issued by ICAI in Aug 2006, turnover should be disclosed as follows on the face of the statement of
Profit & Loss:
Turnover (Gross) XX
Less : Excise duty XX
Turnover (Net) XX
Further, the excise duty shown above should be the total duty except the duty related to the difference between
closing stock & opening stock which should be recognized separately in the statement of profit and loss, with an
explanatory note in the notes to accounts to explain the nature of the two amounts of excise duty. Even as per Revised
Schedule VI Excise duty should be deducted from Sales and Services.
Since the company, X Ltd has not followed the above ASI 14, the auditor would have to qualify his report
accordingly.
Problem 3: In a manufacturing process of Vijoy Limited, one by-product BP emerges besides two main
products MP1 and MP2 apart from scrap. Details of cost of production process is here under:

Item Unit Amount Output (unit) Closing stock as on


31-3-2012
Raw Material 15000 1,60,000 MP1 – 6250 800
Wages -- 82,000 MP2 – 5000 200
Fixed Overhead -- 58,000 BP – 1600 --
Variable Overhead -- 40,000 -- --
18 Accounting Standards

Average market price of MP1 and MP2 is `80 per unit and `50 per unit respectively, by-product is sold
@ `25 per unit. There is a profit of `5,000 on sale of by-product after incurring separate processing charges
of `4000 and packing charges of `6,000. `6,000 was realized from sale of scrap. Calculate the value of Closing
Stock of MP1 and MP2 as on 31-3-2012. (CA Final Nov 2012)
Solution: As per para 10 of AS-2 (Rev), allocation as per physical units method or relative sales method at the
separation point is the most acceptable basis for apportionment of joint costs. The value of such by-products or
waste or scrap is very insignificant. Thus, such by – products or waste etc. are valued at Net Realizable Value (NRV)
and this NRV is deducted from the costs of main product.
Total joint cost = ` 1,60,000 + ` 8,2000 + ` 58,000 + ` 40,000 = ` 3,40,000
NRV of By–product = Sale Price ` 40,000 (1,600 × 25) – Further Cost ` 10,000 = ` 30,000
NRV of By–product and Scrap = ` 30,000 + ` 6,000 = ` 36,000
Net Joint Cost = ` 3,40,000 – ` 36,000 = ` 3,04,000.
Production Quantity of MP1 and MP2 is 6,250 : 5,000
Apportionment of Joint Cost based on sale value (80 × 6,250) : (50 × 5,000) or 2:1 = 2,02,667 : 1,01,333 =
` 3,04,000.
Average Joint Cost per unit = ` 32.43 for M1 and ` 20.27 for M2.
Closing Stock valuation = 800units x 32.43 = ` 25,944 and 200 units × 20.27 = ` 4,054
Problem 4:
Particulars 2005 2006
Actual units produced (Kgs.) 40,000 50,000
Normal production at normal capacity (Kgs.) 48,000 48,000
Fixed production overheads (`) 60,000 60,000
Closing Stock (Kgs.) 3,600 4,000
Cost of Closing Stock per unit excluding FPOH (`) @7.2/- @7.5/-
You are required to: (i) Value Closing Stock. (ii) Find the amount debited to P/L A/C.
Solution: Let Actual production be (AP) and Normal production be (NP).
First lets find FPOH allocation per unit = 60,000/48,000 = ` 1.25/-.
For 2005: It is a Case of AP < NP. As per AS-2 if AP < NP (40,000 < 48,000) then FPOH remains same
(i.e. 1.25/-). FPOH is recovered only 40000 × 1.25 = 50,000. Unallocated FPOH will be written-off = 60000
incurred – 50000 = 10,000
Hence, Value of Closing Stock = 3,600 units × (7.20+1.25) = ` 30,420.
Amount written off to Profit & loss account = ` 10,000(Unabsorbed).
For 2006: It is a AP > NP case. As per AS-2 if AP > NP (50,000 > 48,000) then FPOH per unit allocation should
not be 1.25/-. It should be reduced to @1.20/ otherwise inventory will be valued at above cost. FPOH are fully
absorbed.
* 1.20/- = FPOH (60,000)/AP (50,000).
Hence, Value of Closing Stock = 4,000 units × (7.50+1.20) = ` 34,800.
Amount written off to Profit & loss account = Nil.
Problem 5: Cuba Compu Techniques Pvt. Ltd. manufactures computers, during the year ended 31st March
2005 the company manufactures 550 computers. It adopts the policy of valuing finished stock of goods at a
standard cost of `1,80,000 per computer. Details of cost is as follows:
Materials & stores `400 lakhs, Direct labour `250 lakhs, VPOH `150 lakhs, FPOH `290 lakhs. FPOH
includes interest and storage cost of `100 lakhs. Compute value of stock. (C.A. Final May 2002)
AS-2 (Revised): Valuation of Inventories 19

Solution: The company follows Standard Costing policy of valuing stock, but as per AS-2 valuation will be
done as per absorption costing. Valuation as per absorption costing is worked out as follows:
Materials ` 400 lakhs + Direct labour ` 250 lakhs + VPOH ` 150 lakhs + FPOH ` *190 lakhs.
= ` 990 lakhs. * FPOH is excluding interest and storage cost see para 3.1(b).
Production of goods (computers) = 550 computers (assuming to be normal production)
Cost per computer (by absorption costing) = ` 990 lakhs/550 = ` 1.80 → as per AS-2.
Cost per computer (by standard costing) = ` 1.80 lakhs.
The company follows Standard costing which is used only if the result approximates the actual cost. And
incidentally cost per computer as per absorption costing is same as standard costing.
Problem 6: Carvi & Sons operates a retail shop. The store provides you the following information for valuing
closing stock as on 31.03.2013:

Particulars Units Cost price of the Retail price of the


goods goods
(`) (`)
Opening inventory 2,59,000 10,50,222 12,64,748
Goods purchased 12,00,000 61,20,000 78,05,923
Goods sold (Sale Value) 8,30,456 -- 55,81,454

Solution:
Particulars Units Cost price of the Retail price of the
goods (`) goods (`)
Opening inventory 2,59,000 10,50,222 12,64,748
Goods purchased 12,00,000 61,20,000 78,05,923
Total Goods 14,59,000 71,70,222 90,70,671
Less: Actual Sales -8,30,456 -- -55,81,454
Closing Stock 6,28,544 ? 34,89,217
Cost to Retail ratio = 71,70,222 ÷ 90,70,671 = 79.048%.
Applying Cost/Retail ratio to Stock at retail price = 79.048% × 34,89,217 = ` 27,58,156.
Value of Closing Stock of Carvi & Sons = ` 27,58,156.
Problem 7: In all the cases assume Cost of inventories = ` 50/- and Future estimated costs = ` 2
Closing Stock Contract Stock Contract Price General Selling price
(1) 10000 units 8000 units 55/- 56/-
(2) 10000 units 8000 units 49/- 53/-
(3) 10000 units 14000 units 49/- 54/-
(4) 10000 units 14000 units 49/- 50/-

Solution: Basic principle is Cost or NRV whichever is less:


(1) NRV for 8,000 units will be based on Contract Price = (55 – 2) = ` 53 and 2,000 will be on GSP =
(56 – 2) = ` 54. Hence value of stock = (8,000 × 50 + 2,000 × 50) = ` 5,00,000.
(2) NRV for 8,000 units will be based on Contract Price = (49 – 2) = ` 47 and 2,000 will be on GSP =
(53 – 2) = ` 51. Hence value of stock = (8,000 × 47 + 2,000 × 50) = ` 4,76,000.
(3) NRV for 10,000 units will be based on Contract Price = (49 – 2) = ` 47 and provision for onerous
contract (refer AS-29) should be made for 4,000 units × 3 = ` 12,000. Hence value of stock =
(10,000 × 47) = ` 4,70,000.
20 Accounting Standards

(4) NRV for 10,000 units will be based on Contract Price = (49 – 2) = ` 47. Hence value of stock =
(10,000 × 47) = ` 4,70,000. Provision for onerous contract (refer AS-29) should be made for 4,000
units × 1 = ` 4,000. The loss on supply is calculated based on difference between GSP and Contract
Price. As Cost of manufacturing future stock is high therefore the company will buy stock at ` 50 and
supply it to the customer. (Students are required to refer AS-29 onerous contract)
Problem 8: State with reference to accounting standard, how will you value the inventories in the following cases:
(i) Raw material was purchased at `100 per kilo. Price of raw material is on the decline. The finished
goods in which the raw material is incorporated is expected to be sold at below cost. 10,000 kgs. of
raw material is on stock at the year end. Replacement cost is `80 per kg.
(ii) In a production process, normal waste is 5% of input. 5,000 MT of input were put in process resulting
normal wastage of 300 MT. Cost per MT of input `1,000. The entire quantity of waste in on stock at
the year end.
(iii) Per kg. of finished goods consisted of:
Material cost ` 100 per kg.
Direct labour cost ` 20 per kg.
Direct variable production overhead ` 10 per kg
Fixed production charges for the year on normal capacity of one lacs kgs. is ` 10 lacs. 2,000 kgs.of finished
goods are on stock at the year end. (C.A. Final Nov. 2000)
Solution:
(i) As per para 24 of AS-2 (Revised) on Valuation of Inventories, materials and other supplies held for
use in the production of inventories are not written down below cost if the finished product in which
they will be incorporated are expected to be sold at or above cost. However, when there has been a
decline in the price of materials and it is estimated that the cost of the finished products will exceed
net realizable value, the materials are written down to net realizable value. In such circumstances,
the replacement cost of the materials maybe the best available measure of their net realizable value.
Hence, in the given case, the stock of 10,000 kgs of raw material will be valued at ` 80 per kg.
(ii) As per para 13 of AS-2 (Revised), abnormal amounts of waste materials, are recognized as expenses
in the period in which they are incurred.
In this case, normal waste is 250 MT and abnormal waste is 50 MT.

Particulars MT Rs.
Input cost 5000 50,00,000
Less: Normal loss (250) -
Normal Cost of Raw – Material 4750 50,00,000
Cost p.u. 1052.63
Cost of abnormal loss (50) 52,632
Cost of inventories 4700 49,47,368

(iii) In accordance with paras 8 and 9 of AS-2 (Revised), the costs of conversion includes a systematic
allocation of fixed and variable production overheads that are incurred in converting materials into
finished goods. The allocation of fixed production overheads for the purpose of their inclusion in the
costs of conversion is based on the normal capacity of the production facilities.
AS-2 (Revised): Valuation of Inventories 21

The cost of pre Kg. of finished goods can be computed as follows:


`
Material cost 100
Direct labor cost 20
Direct variable production overhead 10
 `10,00,00 
Fixed production overheads =  10
 1,00,000 

140
Thus the value of 2,000 Kgs. of finished goods on stock at the yearend will be `2,80,000 (2,000 Kgs.* `140)
Problem 9: CNP Ltd. is manufacturing goods for local sale and exports. As on 31st March, 2013, it has the
following finished stocks in the factory warehouse:
(i) Goods meant for local sale `100 lakhs (cost ` 75 lakhs).
(ii) Goods meant for exports ` 50 lakhs (cost ` 20 lakhs).
Excise duty is payable at the rate of 16%. The company’s Managing Director says that excise duty is
payable only on clearance of goods and hence is not a cost. Please advise CNP using guidance note, if
any issued on this, including valuation of stock. (C.A. Final May 2003/2011/2015)
Solution: Guidance Note on Accounting Treatment for Excise Duty states that excise duty is a duty on
manufacture or production of excisable goods in India.
Central Excise Rules, 1944 provide that “excise duty shall be collected at the time of removal of goods
from factory premises or from approved place of storage.” Therefore, the levy of excise duty remains upon the
manufacture or production alone. Only the collection part of it is shifted to the stage of removal.
Further, paragraph 23(i) of the Guidance Note makes it clear that excise duty should be considered as
a manufacturing expense and like other manufacturing expenses be considered as an element of cost for
inventory valuation.
Therefore, in the given case of CNP Ltd., the Managing Director’s contention that “excise duty is payable
only on clearance of goods and hence is not a cost is incorrect. Excise duty on the goods meant for local sales
should be provided for at the rate of 16% on the selling price. Excise duty on goods meant for exports, should
not be provided for.
P roblem 10: The management of the a company tells you that the WIP is not valued since it is difficult to
ascertain the same, in view of the multiple processes involved. (C.A. Final May 2001)
Solution: Inventories includes WIP as well as per AS–2. Inventories includes (RM+WIP+FG). Even Guidance
Notes provides the need for calculating stage of completion. After all Stock is to be valued at lesser of Cost or
NRV. Companies do take help of experts for technical valuation. Hence vain is mandatory for WIP otherwise
the audit reports are qualified.
P roblem 11: A private limited company manufacturing fancy terry towels had valued its closing stock of
inventories of finished goods at the realisable value, inclusive of profit and the export cash incentives. Firm
contracts had been received and goods were packed for export, but the ownership in these goods had not
been transferred to the foreign buyers. Comment on the valuation of the stock by the company.
(C. A. Final Nov. 2006)
Solution: Accounting Standard-2 “Valuation of Inventories” states that inventories should be valued at
lower of historical cost and net realisable value. AS 9 on “Revenue Recognition” states, “at certain stages in
specific industries, such as when agricultural crops have been harvested or mineral ores have been extracted,
performance may be substantially complete prior to the execution of the transaction generating revenue. In
such cases, when sale is assured under forward contract or a government guarantee or when market exists
and there is a negligible risk of failure to sell, the goods invoiced are often valued at Net-realisable value.”
Terry Towels do not fall in the category of agricultural crops or mineral ores. Accordingly, taking into
account the facts stated, the closing stock of finished goods (Fancy terry towel) should have been valued at
cost and not at net realisable value (including profits). Further, export incentives are recorded only in the
year the export sale takes place. Therefore, the policy adopted by the company for valuing its closing stock of
inventories of finished goods is not correct.
22 Accounting Standards

Problem 12: Night Ltd. sells beer to customers. Some of the customers consume the beer in the bars run by
Night Limited. While leaving the bars, the Consumers leave the empty bottles in the bars and the Company
takes possession of these empty bottles. The Company has laid down a detailed internal record procedure
for accounting for these empty bottles which are sold by the Company by calling for tenders. Keeping this in
view:
(a) Decide whether the Stock of Empty Bottles is an Asset of the Company.
(b) If so, whether the Stock of Empty Bottles existing as on the date of Balance Sheet is to be considered as
Inventories of the Company and valued as per AS–2, or to be treated as scrap and shown at realizable
value with corresponding credit to ‘Other Income’? (CA Final Nov 2010)
Solution: (i) Irrespective of the fact whether the asset is tangible or intangible one has to get the benefit from
it. Night Ltd sells these bottles by calling tenders (earns revenue). Therefore Night Ltd has to record the stock
of empty bottles as assets. (ii) Normally scrap materials/waste are never valued in detail i.e. they are just
valued @NRV and reduced from cost of main products or it may be even shown as a separate income in the
profit/loss account. But Night Ltd has a detailed inventory checking and controlling system. Hence inventory
of empty bottles are to be valued as per AS-2 and not to be treated as scrap.
Problem 13: Calculate the value of raw materials and closing stock based on the following information:
Raw material X
Closing balance 500 Units
`per unit
Cost price including excise duty 200
Excise duty (Cenvat credit is receivable on the excise duty paid) 10
Freight inward 20
Unloading charges 10
Replacement cost 150
Finished goods Y
Closing Balance 1200 units
`per unit
Material consumed 220
Direct labour 60
Direct overhead 40
Total fixed overhead for the year was `2,00,000 on normal capacity of 20,000 units.
Calculate the value of the closing stock, when
(i) Net Realizable Value of the Finished Goods Y is `400.
(ii) Net Realizable Value of the Finished Goods Y is `300.
Solution:
Statement showing valuation of Raw Material and Finished Goods at cost
Raw Material X `
Cost Price 200
Less: CENVAT credit (10)
Add: Freight Inward 190
Unloading charges 20
Cost 10
220
AS-2 (Revised): Valuation of Inventories 23

Raw Material X `
Materials consumed 220
Direct labour 60
Direct overhead 40
Fixed overheads (2,00,000/20,000) 10
Cost 330

(i) When Net Realisable Value (NRV) of the Finished Goods Y is `400
NRV is greater than the cost of Finished Goods Y i.e. `330
Hence, Raw Material and Finished Goods will be valued at cost
Accordingly, value of closing stock will be:

Qty Rate Amount (`)


Raw Material X 500 220 1,10,000
Finished Goods Y 1,200 330 3,96,000
Total cost of closing stock 5,06,000

(ii) When Net Realisable Value of the Finished Goods Y is `300


NRV is less than the cost of Finished Goods Y i.e. `330
Hence, Raw Material is to be valued at replacement cost and Finished Goods are to be valued at NRV.
Accordingly, value of closing stock will be:

Qty Rate Amount (`)


Raw Material X 500 150 75,000
Finished Goods Y 1,200 300 3,60,000
Total cost of closing stock 4,35,000

Note: It has been assumed that Raw Material X is used for production of Finished Goods Y.
CHAPTER 4
AS-3 (Revised):
Cash Flow Statement

Chapter Outline
vv Introduction vv Reporting Cash Flows on Net Basis
vv Cash and Cash Equivalents vv Foreign Currency Cash Flows
vv Classification of Cash Flows vv Extraordinary Items
vv Presentation of Cash Flows from vv Taxes on Income
Operating Activities vv Investments in Subsidiaries,
vv Presentation of Cash Flows from Associates and Joint Ventures
Investing and Financing Activities

Introduction
AS-3, Cash Flow Statements recommends that an enterprise should prepare a cash flow statement and
present it in addition to the balance sheet and profit and loss account. The cash flow statement assists a
reader in assessing the ability of an enterprise to generate sufficient cash flows, for its different needs. A
cash flow statement also assists the reader in comparing the operating performance of different enterprises
because it is free from the effects of using different accounting policies for the same transactions (selection
and adoption of accounting policies is inherent in accrual-based financial statements, i.e., balance sheet and
profit and loss account).

Cash and Cash Equivalents


The cash flow statement shows the inflows and outflows of cash and cash equivalents.
Cash comprises cash in hand and demand deposits with banks.
Cash Equivalents are defined as “Short term, highly liquid investments that are readily convertible into
known amounts of cash and which are subject to an insignificant risk of changes in value”. Thus, an investment
is cash equivalent only if it is readily convertible to a known amount of cash, is subject to an insignificant risk
of changes in value, and has a short maturity of, say. three months or less from the date of acquisition.
Examples of Cash equivalents: An example of an investment which can be included as cash equivalent
would be preference shares acquired shortly before their specified, redemption date (provided there is
only an insignificant risk of failure of the company to repay the amount at maturity). Other examples are:
Commercial papers, Certificate of deposits, Treasury bills.
AS-3 (Revised): Cash Flow Statement 25

Classification of Cash Flows


The standard recommends that the cash flow statement should report the cash flows during the period
classified by operating, investing and financing activities. The objective is to enable the reader to assess the
impact of each type of cash flows separately.
(i) Operating Activities: Operating activities are “the principal revenue-producing activities of the
enterprise and other activities that are not investing or financing activities.”
Examples of cash flows from operating activities –
ww cash receipts from the sale of goods/rendering of services
ww royalties, fees and commissions, and
ww cash payments to suppliers for goods and services and to employees
ww Cash payments or refunds of income taxes are also part of cash flows from operating activities
unless they can be specifically identified with financing and investing activities, e.g. tax on gain
on sale of a long-term investment.
(ii) Investing Activities: Investing activities are “the acquisition and disposal of long-term assets and
other investments not included in cash equivalents. “Examples of cash flows arising from investing
activities are cash payments/receipts arising from. acquisition or construction/ disposal of fixed
assets (including intangibles), interest and dividend income received etc.
(iii) Financing Activities: Financing activities are activities that result in changes in the size and composition
of the owners’ capital (incl. preference share capital in the case of a company) and borrowings of the
enterprise. “Examples of cash flows arising from financing activities are cash proceeds from issuing shares,
loans, notes, bonds; and other short or long-term borrowings, and cash repayments of amounts borrowed,
payment of interest on borrowing and dividends on shares.

Presentation of Cash Flows from Operating Activities


Cash flows from operating activities should be presented in the cash flow statement using either direct
method or indirect method.
Direct Method: Under this method, major classes of gross cash receipts and gross cash payments are
disclosed. These may be worked out either from the accounting records of the enterprise i.e., by analyzing
accounts relating to cash, bank (only those bank balances which are available on demand and short term
liquid Investments qualifying as cash equivalents); or
Indirect Method: Under the indirect method, the net cash flow from operating activities is determined by
adjusting net profits or loss for the effects of:
(a) Changes during the period in inventories and operating receivables and payables;
(b) Non-cash items such as depreciation, provisions, deferred taxes, and unrealized foreign exchange
gains and losses; and
(c) All other items for which the cash effects are investing or financing cash flows.

Presentation of Cash Flows from Investing and Financing Activities


Cash flows arising from investing activities should be reported separately. Similarly, cash flows from financing
activities are required to be classified separately. Under each, major categories of gross cash receipts and
cash payments should be shown separately. Thus, payments for purchase of fixed assets cannot be shown
after deducting the proceeds from sale of such assets.
26 Accounting Standards

Reporting Cash Flows on Net Basis


Cash flows arising from the following operating, investing or financing activities may be reported on net
basis:
(a) Cash receipts and payments on behalf of customers.
Example: Cash flows by a bank against acceptances and repayment of demand deposits.
(b) Cash receipts and payments for items in which the turnover is quick, the amounts are large,
and the maturities are short.
Example: Purchase and sale of investments by an investment company.
Net basis of cash flows are allowed in the following three circumstances:
(i) Cash receipts and payments for the acceptance and repayment of deposits with a fixed maturity
date;
(ii) The placement of deposits with and withdrawal of deposits from other financial enterprises; and
(iii) Cash advances and loans made to customers and the repayment of those advances and loans.

Foreign Currency Cash Flows


The foreign currency cash flows are recorded in reporting currency using exchange rate prevailing on date
of cash flow. Realised gains/losses in respect of cash flows are shown as a part of reconciliation of changes in
cash and cash equivalents during the year. Unrealised gains/losses in respect of cash flows are not cash flows
(they are to be eliminated from net profit).

Extraordinary Items
The cash flows associated with extraordinary items are disclosed separately as arising from operating,
investing or financing activities in the cash flow statement, to enable users to understand their nature and
effect on the present and future cash flows of the enterprise. These disclosures are in addition to the separate
disclosures of the nature and amount of extraordinary items required by Accounting Standard (AS)-5, Net
Profit or Loss for the Period, Prior Period Items and Changes in Accounting Policies.

Taxes on Income
Taxes on income arise on transactions that give rise to cash flows that are classified as operating, investing or
financing activities in a cash flow statement. While tax expense may be readily identifiable with investing or
financing activities, the related tax cash flows are often impracticable to identify and may arise in a different
period from the cash flows of the underlying transactions. Therefore, taxes paid are usually classified as
cash flows from operating activities. However, when it is practicable to identify the tax cash flow with an
individual transaction that gives rise to cash flows that are classified as investing or financing activities,
the tax cash flow is classified as an investing or financing activity as appropriate.

Investments in Subsidiaries, Associates and Joint Ventures


The aggregate cash flows arising from acquisitions and from disposals of subsidiaries or other business units
should be presented separately and classified as investing activities.
AS-3 (Revised): Cash Flow Statement 27

Format for Direct Method of Cash Flow Statement


Cash from Operating Activities: (CFOA)
Receipts from Debtors –––
(–) Payments made to Suppliers of Goods (– – –)
(–) Payments made to Expenses Creditors (– – –)
xxx
Cash from Operations:
Less: Income Tax paid (after adjusting Refund) (– – –)
+ Extra-ordinary activities –––
xxx
Cash from Operating Activities: (A)
Cash from Investing Activities: (CFIA)
(–) Purchase of Fixed Assets (– – –)
Sale of Fixed Assets –––
(–) Purchase of Investments (– – –)
Sale of Investments –––
(–) Investments in Joint Venture (– – –)
Interest received on Investments –––
Dividend received –––
xxx
Cash from Investing Activities: (B)
Cash from Financing Activities: (CFFA)
Issue of Share Capital (Face Value + Premium) –––
Receipt of Calls –––
(–) Redemption of Capital (FV + POR) (– – –)
(–) Buy Back of Shares (including Premium paid) (– – –)
(–) Debentures redeemed/Loan repaid (– – –)
(–) Interest paid on Debentures/Loan (– – –)
(–) Dividend paid (– – –)
Cash from Financing Activities: (C) xxx
Net ↑ or ↓ in Cash & Cash Equivalents (A+B+C) xxx
Add: Opening balance of Cash & Cash Equivalents xxx
Closing balance of Cash & Cash Equivalents xxx

Format for Indirect Method of Cash Flow Statement


Cash from Operating Activities:
Net Profit before Taxes and Extraordinary items: –––
+/- Adjustments (Eliminate all the Non – Cash & Non – Operating Cash Flows) –––
Add back all the Expenditure side of items: (Elimination)
  •  + Depreciation –––
  •  + Goodwill w-off –––
28 Accounting Standards

  •  + Preliminary expenses w-off –––


  •  + Interest provided –––
  •  + Foreign Currency loss –––
  •  + Loss on Sale of Asset/Investment –––
Less all the Income side of items: (Elimination)
  •  (–) Dividend Income (– – –)
  •  (–) Interest income (– – –)
  •  (–) Profit on Sale of Asset/Investment (– – –)
xxx
Cash from Operations before Working Capital changes
+/– Changes in Working Capital:
(–) increase in Current Assets (– – –)
+ decrease in Current Assets –––
+ increase in Current Liabilities –––
(–) decrease in Current Liabilities (– – –)
Cash from Operations xxx
Less: Income Tax paid (after adjusting Refund) (– – –)
Extra-ordinary activities –––
Cash from Operating Activities: (A) xxx xxx

Note: Remember that CFIA and CFFA remains the same for both Direct and Indirect Method.

PROBLEMS AND SOLUTIONS

Problem 1: Following is the Trading Profit and loss Account of New India Ltd. for the year ended 31.03.2005.
Particulars ` Particulars `
To Opening Stock 400,000 By Sales 4,032,000
To Purchases 2,802,000 By Closing Stock 520,000
To Gross Profit 1,350,000
4,552,000 4,552,000
To Operating Expenses 6,70,000 By Gross Profit b/d 1,350,000
To Interest expense 15,000 By Compensation for loss of goodwill 100,000
To Depreciation 2,70,000 By Interest recd. from customers 5,000
To Taxation 2,50,000 from investments (40% Short term) 30,000
To Net Profit 3,70,000 By Dividend received 40,000
By Profit on sale of fixed assets 50,000
1,575,000 1,575,000
AS-3 (Revised): Cash Flow Statement 29

Profit and Loss Appropriation Account


Particulars ` Particulars `
To Transfer to Reserve 100,000 By Balance b/d 590,000
To Dividend Proposed 120,000 By Net Profit b/d 370,000
To Balance c/f. 740,000
960,000 960,000

Following is the comparative Balance Sheet of New India Limited:

Liabilities 31.03.2004 31.03.2005 Assets 31.03.2004 31.03.2005


Share Capital (` 10) 700,000 900,000 Fixed Assets 1,000,000 800,000
Reserves 100,000 200,000 (-) PFD (400,000) (320,000)
600,000 480,000
Profit & Loss balance 590,000 740,000 Long-term Investments 300,000 400,000
8% Convertible Debentures 100,000 50,000 Short-term Investments 22,000 40,000
S. Creditors 120,000 150,000 Debtors 365,000 501,000
Creditors for expenditure 22,000 31,000 Stock 400,000 520,000
Proposed Dividend 60,000 120,000 Cash with Bank 170,000 495,000
Taxation provision 165,000 250,000 Share issue expenses – 5,000
1,857,000 2,441,000 1,857,000 2,441,000

During the year company purchased fixed asset costing ` 6,00,000.


Prepare Cash Flow Statement (Revised) as per AS - 3 both Direct and Indirect Method.
Solution:
Cash Flow Statement (Direct Method)
Cash from Operating Activity (CFOA)
Collections from customers 3,89,6000
Payments to suppliers –2,77,2000
Payments to expenses creditors -6,61,000
Interest recd from customers 5,000
Interest on Short Term Investment 12,000

Cash from Operations 4,80,000


Less: Tax paid –1,65,000
+ Extraordinary activity
Compensation received 1,00,000
Cash from Operating Activities (A) 4,15,000
Cash from Investing Activity (CFIA)
Purchase of Fixed Assets (FA) –6,00,000
Sale of Fixed Assets (FA) 5,00,000
Purchase of investments –1,00,000
30 Accounting Standards

Dividend received 40,000


Interest from Long term Investment 18,000
Cash from Investing Activities (B) –1,42,000
Cash from Financing Activity (CFFA)
Issue of shares (Net of Expenditures) 1,95,000
Redemption of Debenture –50,000
Dividend paid –60,000
Interest paid on Debenture –15,000
Cash from Financing Activities (C) 70,000
Net Increase in Cash and Cash Equivalents (A + B + C) 3,43,000
+ Opening bal of Cash and Cash Equivalents 1,92,000
Closing bal of Cash and Cash Equivalents 5,35,000

Cash Flow Statement (Indirect Method)


CFOA
Net profit before taxes
+/- Adj for Non Cash/Non Operating items 6,20,000
Interest expenses 15,000
Depreciation 2,70,000
Compensation of Goodwill –1,00,000
Interest from Long-term Investments –18,000
Dividend received –40,000
Profit on sale of assets –50,000 77,000
+/- Adj for Working Capital changes
Increase in Debtors –1,36,000
Increase in Stock –1,20,000
Increase in Sundry Creditors – goods 30,000
Increase in Sundry Creditors – expenses 9,000 –2,17,000
Cash from Operations 4,80,000

Less: Tax paid –1,65,000


+ Extraordinary activity
Compensation received 1,00,000
Cash from Operating Activities (A) 4,15,000

Problem 2: Arrange and redraft the following Cash Flow Statement in proper order keeping in mind the
requirements of AS-3:
` (in lacs) ` (in lacs)
Net Profit 60,000
Add: Sale of Investments 70,000
Depreciation on Assets 11,000
AS-3 (Revised): Cash Flow Statement 31

Issue of Preference Shares 9,000


Loan raised 4,500
Decrease in Stock 12,000
1,66,500
Less: Purchase of Fixed Assets 65,000
Decrease in Creditors 6,000
Increase in Debtors 8,000
Exchange gain 8,000
Profit on sale of investments 12,000
Redemption of Debenture 5,700
Dividend paid 1,400
Interest paid 945 (1,07,045)
59,455
Add: Opening cash and cash equivalent 12,341
Closing cash and cash equivalent 71,796
(C.A. Final Nov 2007) (6 marks)
Solution:
Cash Flow Statement
Cash Flows from Operating Activities (CFOA) (` in lacs)
Net profit 60,000
Less: Exchange gain (8,000)
Less: Profit on sale of investments (12,000)
40,000
Add: Depreciation on assets 11,000
Change in current assets and current liabilities 51,000
(–) Increase in debtors (8,000)
(+) Decrease in stock 12,000
(–) Decrease in creditors (6,000) (2,000)
Net cash from operating activities 49,000
Cash Flows from Investing Activities (CFIA)
Sale of investments 70,000
Purchase of fixed assets (65,000)
Net cash from Investing activities 5,000
Cash Flows from Financing Activities (CFFA)
Issue of preference shares 9,000
Loan raised 4,500
Redemption of Debentures (5,700)
Interest paid (945)
Dividend paid (1,400)
Net cash from financing activities 5,455
Net increase in cash and cash equivalents 59,455
Add: Opening Cash and Cash Equivalents 12,341
Closing Cash and Cash Equivalents 71,796
32 Accounting Standards

Problem 3: Explain the impact of the items given below on Cash flow:
(i) Sales made to a foreign party worth $10,000 when exchange rate was 1$ = ` 40. Out of this $8,000 were
collected when exchange rate was 1$ = ` 42. The year – end exchange rate is ` 43/$.
(ii) $1,00,000 is kept in foreign bank a/c rate as on 1/4/2011 exchange rate was 1$ = ` 40. The year –
end exchange rate is ` 43/$. Assume there is no change in the bank balance throughout the year.
Solution: As per para 25 exchange gains/losses should be shown as a part of reconciliation. Foreign CF are
recorded at prevailing rate on the date of cash flow.
(i) Debtors (10000 × 40) ...Dr. 4,00,000
To Sales 4,00,000
Bank (8000 × 42) ...Dr. 3,36,000
To Debtors 3,20,000
To Exchange gain (AS-11) 16,000 (P/L)
Debtors (2,000 × 3) ...Dr. 6,000
To Exchange gain (AS-11) 6,000 (P/L)
Total Profit and Loss credit due to forex gain is 22,000 out of which 16000 is realized and 6000
unrealized. But unrealized gain can be ignored because 6000 will increase Net Profit on one side and
Debtors on other side.
(ii) Increase in foreign bank balance is an unrealized gain ($1,00,000 x 3) = ` 3,00,000. First effect will
be to eliminate ` 3,00,000 from Net Profit and then show ` 3,00,000 as an addition to the opening
balance of cash and cash equivalents.
Problem 4: Bellhop LLC submits the following information pertaining to year 2011. Using the data, you are
required to find the ending cash and Bank balances given an opening figure thereof was` 1.55 million.
(` millions)
Additional shares issued 6.50
CAPEX (Capital expenditure) 9.90
Proceeds from Assets sold 1.60
Dividends declared 0.50
Loss from disposal of Assets (1.20)
Net Income 3.30
Increase in Accounts Receivable 1.50
Redemption of 4.5% debentures 2.50
Depreciation & Amortization 0.75
(CA Final May 2012 Marks 4)
Solution:
Cash Flow Statement (Indirect Method)
(` In Millions)

Net profit before taxes 3.30


+/- Adj for Non Cash/Non Operating items
Depreciation 0.75
Loss on sale 1.20
+/- Adj for Working Capital changes
Increase in Debtors –1.50 0.45
Cash from Operations 3.75
AS-3 (Revised): Cash Flow Statement 33

Cash from investing activities


Cap expenses –9.9
Sale of Fixed Assets 1.60
Cash from Investing activities –8.3
Cash from Financing Activity
Issue of shares 6.5
Dividend –0.5
Redp of debn –2.5
Cash from Financing Activities 3.50
Total Cash and Cash Equivalents –1.05
Opening Balance 1.55
Closing Balance 0.50

Problem 5: Money Ltd., a non financial company has the following entries in its Bank Account. It has sought
your advice on the treatment of the same for preparing Cash Flow Statement.
(i) Loans and Advances given to the following and interest earned on them:
(1) to suppliers
(2) to employees
(3) to its subsidiaries companies
(ii) Investment made in subsidiary Smart Ltd. and dividend received
(iii) Dividend paid for the year
(iv) TDS on interest income earned on investments made
(v) TDS on interest earned on advance given to suppliers
(vi) Insurance claim received against loss of fixed asset by fire
Discuss in the context of AS 3 Cash Flow Statement.
Solution: Treatment as per AS 3 ‘Cash Flow Statement’
(i) Loans and advances given and interest earned
(1) to suppliers Cash flows from operating activities
(2) to employees Cash flows from operating activities
(3) to its subsidiary companies Cash flows from investing activities
(ii) Investment made in subsidiary company and dividend received Cash flows from investing activities
(iii) Dividend paid for the year Cash flows from financing activities
(iv) TDS on interest income earned on investments made Cash flows from investing activities
(v) TDS on interest earned on advance given to suppliers Cash flows from operating activities
(vi) Insurance claim received against loss of fixed asset by fire Extraordinary item to be shown
under a separate heading as ‘Cash
inflow from operating activities’.
CHAPTER 5
AS-4: Events Occuring After
the Balance Sheet

Chapter Outline
vv Introduction vv Exceptions to General Rule
vv Events Occurring after the Balance Sheet Date
i) Adjusting events
ii) Non-adjusting events

Events Occurring after the Balance Sheet Date


Many events occurs between the period from the end of the financial year till the date of approval of accounts.
These are significant events can be favourable or unfavourable to a company. The birth of such events takes
place between BALANCE SHEET DATE and ACCOUNTS APPROVAL DATE. These significant events between
the Balance Sheet date and accounts approval date are known as Events occurring after the Balance Sheet
date. Adjustments are required to be made for those events which were outstanding on the balance sheet
date and which mature after the Balance Sheet date. Two types of events can be identified:
(i) Adjusting events: Providing further evidence of conditions that existed on the Balance Sheet date.
(ii) Non-adjusting events: Events arising from the conditions that arose after the Balance Sheet date.

SIGNIFICANT EVENTS

Events whose circumstances Events whose circumstances does not


exists on the Balance Sheet (B/S) date exists on the Balance Sheet (B/S) date
Or
Totally new events
FEATURES as PER AS-4:
◘◘ Significant events
◘◘ Favorable or unfavorable
◘◘ Such events crystallize between the close of financial year and accounts approval date.
◘◘ Conditions for such event should prevail on the Balance Sheet date.
◘◘ Such events then are known as adjusting events and are to be recorded for the previous year.
AS-4: Events Occuring After the Balance Sheet 35

DO YOU KNOW
Events are different as compared to transactions.

Exceptions to General Rule


(i) Going concern assumption becomes invalid: It is a general rule that totally new events should not
be recorded on the balance sheet date. But even if this incident took place after the balance sheet
date (totally new events) we have to record such event if such event disturbs the going concern
assumption.
Example: Destruction of major plant. One has to record this event because it disturbs our fundamental
accounting assumption “GOING CONCERN”
(ii) Proposed Dividend: A company may not have any idea on balance sheet date of declaring Dividend.
But after the year ends it may make up a mind to declare Dividend. Though the circumstances do not
exist on the balance sheet date one has to record such event because of Company Act, 1956. Under
the New Schedule III dividends are required to be disclosed in the notes. But as AS-4 is superior
to provision of Schedule III, Dividend proposed by a company should be recorded by the declaring
company for the previous year under audit. (refer special problems).

Summary of Significant Events


(i) Providing further evidence of conditions that existed (i) Events arising from the conditions that arose after the
on the Balance Sheet date. Balance Sheet date.
(ii) Events which circumstances exists on the balance sheet (ii) Events which circumstances does not exists on the
date. balance sheet date.
(iii) Events matures after Balance Sheet date. (iii) Totally new events
(iv) Known as Adjusting Events. (iv) Known as Non - Adjusting Events
(v) Record such event for the financial year under process. (v) Do not record only disclose in Directors Report (if
material).
(vi) Example: Debtors may be doubtful on the Balance (vi) Example: Debtors may be good on the Balance Sheet
Sheet date and declared insolvent after the Balance date. Due to fire in the factory they become insolvent
Sheet date. after the Balance Sheet date.

Some more notes and explanation regarding Adjusting and Non – Adjusting Events:
Example: Adjusting Events
Patly Pourri Limited whose accounting year ends on 31.03.2009, agreed in principle to sale plot of land on
20.03.2009 for ` 5 crores. Upto 31.03.2009 the company has not recorded the event. But on 24.04.2009 the
company received the valuation report and immediately signed the agreement for sale of land. The Financial
Statement was approved on 16.06.2009.
Analysis of Adjusting Events:
◘◘ Subsequent Event: Sale of Land.
◘◘ Subsequent Event (period): 1.4.2009 – 16.6.2009.
◘◘ Conditions existed on the Balance Sheet date: Yes, Condition, which led to the sale of Land existed on
the Balance Sheet
◘◘ Subsequent Event provides further evidence: Evidence of signing the agreement.
◘◘ Effect: Record the Sale of Land.
36 Accounting Standards

Example: Non-Adjusting Events


Market Value of investments held by Bharti Communications Limited in VP Ltd. declines by 10% after the
year end but before Financial Statement were approved.
Analysis of Non-Adjusting Events:
◘◘ Subsequent Event: Decline of Market Value of investments.
◘◘ Conditions existed on the Balance Sheet date: No, only circumstances changed after the Balance Sheet
date
◘◘ Effect: Do not record the decline of investments.

Other examples of adjusting and non-adjusting events


Adjusting Events:
(i) Subsequent determination of selling proceeds of a sale of asset which was sold before Balance Sheet
date.
(ii) Amount received from insurance company on claim pending on the Balance Sheet date.
Non-Adjusting Events: Changes in forex rates, Mergers and Acquisitions, Allotment for shares and debentures.

PROBLEMS AND SOLUTIONS

Problem 1: Explain as to how you would treat the following events occurring after the balance sheet date
in the financial statements:
a) A major fire has damaged the assets in the factory on April 2, two days after the closure of accounts.
The loss is estimated at ` 20 crores out of which ` 12 crores would be recoverable from the insurers.
b) The dispute for bonus to employees was before the arbitrator and he gave the award in favour of
workmen for ` 3 lacs.
c) A contract for civil construction was performed during the accounting year. The client has gone on
appeal for damages of low quality works and was awarded an amount of ` 1.5 lacs.
Solution:
(a) AS 4 provides that a deterioration in operating and financial position, or unusual changes affecting
the existence of the enterprise after the Balance Sheet, date may indicate a need to consider whether
it is proper to use the fundamental accounting assumption of going concern in the preparation of the
financial statements.
Taking note of the facts of the case, it is to be decided whether a net loss of ` 8 crores (over all loss
` 20 crores, of which ` 12 crores recoverable from insurers) affects the sub-stratum of the enterprise,
and whether the going concern assumption is appropriate.
In the present case it appears from the phrase “the factory” that the operations are carried out in
a “single factory”. Since insurance claim will not be sufficient to recommence operations, the going
concern assumption is likely to get affected. If this were the case, adjustments to the financial
statements would be necessary.
Nevertheless, If the size, nature and complexity of operations of the enterprise is such that this
loss will not materially affect the substratum of the enterprise, a disclosure In Directors’ report is
appropriate.
(b) Condition obtained on the Balance Sheet date relates to a dispute, and about which additional
evidence has emerged after Balance Sheet date. Accordingly, it is necessary to make adjustments, by
way of a provision for ` 3 lacs towards bonus payment for the year just ended.
(c) The award of ` 1.5 lacs by the arbitrator is an adjusting event within the meaning of AS-4. Accordingly,
a provision of ` 1.5 lacs ought to be made.
AS-4: Events Occuring After the Balance Sheet 37

Problem 2: For the 2 companies whose FY ended on 31st Dec. 2012, the financial statements were approved
by their respective directors on 25th March 2013.
During 2013, the following material events took place:
a. Keswani Ltd. sold a property which was included in the balance sheet at ` 2,00,000 and for which
contracts had been exchanged on 15th Dec. 2012. The sale was completed on 10th Feb. 2013 at a
price of ` 4,00,000. (CA Final, Nov 2010)
b. On 2nd March 2013, the mail order activities of S Ltd. (a retail trading group) were shut down with
closure costs amounting to ` 75 lakhs.
You are required to state, with reasons, how each of the above items numbered (a) and (b) should be dealt
with in the financial statements for the year ended 31st Dec. 2012.
Solution:
(a) Keswani Ltd.: The sale of property should be treated as an adjusting event since contracts had been
exchanged prior to the year-end. The effect of the sale would be reflected in the financial statements
to 31.12.2012 including the profit on sale to be reflecting for the year end..
(b) S Ltd.: A closure not anticipated at the year-end would be treated as a non-adjusting event. Disclosure
would be required since non-disclosure would affect a user’s ability to reach a proper understanding
of the financial statements.
P roblem 3: The accounts of АВС Ltd., for the year ended 31st March. 2015, was approved by the BOD of
the Co. on 19th May 2015. The Directors recommended a Dividend @ 10%. However, the Directors feel that
this need not be disclosed in the account for the year ended 31st March 2015 since it does not provide any
additional Information to the condition prevailing on the date of Balance Sheet i.e. on 31st March 2015. Do
you agree with the BOD of ABC Ltd.? (Final C.A. Audit Nov. 2012)
Solution: If an item is statutorily required to be shown in the financial statements, assets and liabilities are
adjusted for such events occurring after the balance sheet dale. For example, proposed dividend is shown in
the profit and loss account and balance sheet although such an item is approved only in the Annual General
Meeting. Under the New Schedule III dividends are required to be disclosed in the notes. But as AS-4 is superior
to any provision to Schedule III hence at present Dividend proposed by a company should be recorded by the
declaring company for the previous year under audit.
P roblem 4: A Limited Company closed its accounting year on 30.6.2012 and the accounts for that period
were considered and approved by the board of directors on 20th August, 2012. The company was engaged in
laying pipeline for an oil company, deep beneath the earth. While doing the boring work on 1.9.2012, it had
met a rocky surface for which it was estimated that there would be an extra cost to the tune of ` 80 lakhs. You
are required to state with reasons, how the event will be dealt with in the financial statements for the year
ended 30.6.2012. (Final C.A. May 1999)
Solution: Para 3.2 of AS-4 (Revised) on Contingencies and Events occurring after the Balance Sheet Date
defines ‘events occurring after the balance sheet date’ as ‘significant events, both favourable and unfavourable,
that occur between the balance sheet date and the date on which financial statements are approved by the
Board of Directors in the case of a company. The given case is discussed in the light of the above mentioned
definition and requirements given in Paras 13-15 of the said AS-4 (Revised).
In this case the incidence, which was expected to push up cost, became evident after the date of approval
of the accounts. It was not an ‘event occurring after the balance sheet date’ at the all. For events occring….AS-4
the event should take place between year ending till accounts approval date.
P roblem 5: While preparing its final accounts for the year ended 31st March, 2003 a company made a
provision for bad debts @ 5% of its total debtors. In the last week of February, 2003 a debtor for ` 2 lakhs had
suffered heavy loss due to an earthquake; the loss was not covered by any insurance policy. In April, 2003 the
debtor became a bankrupt. Can the company provide for the full loss arising out of insolvency of the debtor
in the final accounts for the year ended 31st March, 2003?
(Final C.A. Nov. 1993, Nov. 1995, Nov. 2003, June 2009)
38 Accounting Standards

Solution: As per paras 8.2 and 13 of Accounting Standard-4 on Contingencies and Events Occurring after
the Balance Sheet Date, Assets and Liabilities should be adjusted for events occurring after the balance sheet
date that provide additional evidence to assist estimation of amounts relating to conditions existing at the
balance sheet date.
So full provision for bad debt amounting to ` 2 lakhs should be made to cover the loss arising due to the
insolvency in the Final Accounts for the year ended 31st March, 2003. It is because earthquake took place
before the balance sheet date.
Had the earthquake taken place after 31st March, 2003, then mere disclosure required as per para 15,
would have been sufficient.
P roblem 6: VV Ltd. had announced a voluntary retirement plan for its employees on January 1, 2000.
The scheme is scheduled to close on June 30, 2000. The scheme envisaged an initial lump sum payment of
maximum of ` 2 lakhs and monthly payments over the balance period of service of employees coming under
the plan. 200 employees opted for the scheme as on March31, 2000. The total lump sum payment for these
employees would be ` 250 lakhs and the aggregate of future payments to them would amount to ` 1,500
lakhs. However no payment had been made to the employees under the scheme up to March31, 2000. Nor the
company made any provision in its accounts towards any liability under the scheme.
(Final Audit C.A. May 2000)
Solution: Accounting Standard (AS)-4 (Revised) on ‹Contingencies and Events Occurring After the Balance
Sheet Date›, states that events occurring after the balance sheet date are those significant events, both
favourable and unfavourable, that occur between the balance sheet date and the date on which the financial
statements are approved by the Board of Directors in the case of a company, and,by the corresponding
approving authority in the case of the any other entity.
Two types of events can be identified as:
(a) those which provide further evidence of conditions that existed at the balance sheet date; and
(b) those which are indicative of conditions that arose subsequent to the balance sheet date.
It further states that assets and liabilities should be adjusted for events occurring after the balance sheet
date that provide additional evidence to assist the estimation of amounts relating to conditions existing at
the balance sheet date or that indicate that the fundamental accounting assumption of going concern (i.e., the
continuance of existence or substratum of the enterprise) is not appropriate.
As per facts of the case, a condition existed on the balance sheet date (31st March, 2000) regarding the
liability towards the Voluntary Retirement Plan (VRP) since the management started the VRP in the month
of January, 2000 and 200 employees opted for the VRP as on March 31, 2000. Since it was probable that the
future events will confirm that a liability has been incurred on the balance sheet date and that the amount
could be estimated on reasonable basis, a provision for payments under the VRP would be required to be
made for an appropriate amount for the aforesaid number of employees.
P roblem 7: Subsequent to the balance sheet date State Bank of India have granted a remission of liability to
KF Limited (a sick Co.) of ` 40,00,000 under a BIFR package. If the conditions of remission were existing on
the Balance Sheet date should both the Banks record the remissions of liability?
Solution: Yes, the remission of liabilities to a sick company under BIFR package, should be recognized on the
Balance Sheet Date as the conditions typically existed on the balance sheet date. The sick company would
have been registered with the BIFR, incapable of fulfilling its repayment obligations, negotiations between
the sick company, financial institution and the Board would have been on going on the Balance Sheet date.
P roblem 8: Aaksha Limited made investments of ` 20,00,000 in the month of April 2007 in the acquisition
of another company the negotiations for which had started during the financial year end 31st March, 2007.
Advise on accounting for the year ended 31st March, 2007. (CA Final Nov 2009)
Solution: If investments were purchased before March and payments made after March then it is a purchase
of investments. In the present case the investments are purchased in April. This means the investments in
the books of investor company get registered in the month of April only. The negotiations do not entitle the
investor the shareholder before March. Therefore it is not the event occurring after the balance sheet date.
AS-4: Events Occuring After the Balance Sheet 39

Problem 9: After the close of the accounting year of XYZ Co. Ltd., there was a severe earthquake. As a result
of which a portion of the building and the rolling mill was destroyed. The extent of the damage was beyond
repair. The company’s resources were inadequate to find the replacement of the assets so destroyed.
(C.A. Final May 1995)
Solution: The event is a non-adjusting event since it occurred after the year-end and does not concern
conditions existing at the year-end. However, it is necessary to consider the validity of the going concern
assumption having regard to the extent of insurance cover. If the going concern basis is still valid, full
disclosure of the event should be provided.
Problem 10: A company deals in petroleum products. The government fixes the sale price of petrol.
After the Balance Sheet date, but before the finalization of the companies accounts, the government
unexpectedly increased the price retrospectively. Can the company record for the additional revenue at the
close of the year? Discuss? (C.A. Final)
Solution: Unexpected increase in the selling price of petrol by the govt after the Balance Sheet date cannot be
adjusted at the Balance Sheet date since it does not represent a condition on the Balance Sheet date (as very
much clear from the wording ‘unexpected rise in prices’). The event is a non-adjusting even since it occurred
after the year-end and does not concern conditions existing at the year-end. However, disclosure may be
made if the amount is material. The increase in the revenue due to the increase in Sales Price should be
recorded in the subsequent year under the head exceptional item.
Problem 11: Neel Limited has its Corporate Office in Mumbai, and sells its products to Stockists all over
India. On 31st March 2013, the Company wants to recognize receipt of cheques bearing date 31st March
2013 or before, as “Cheques in Hand”, by reducing “Trade Receivables”. The “Cheques in Hand” is shown
in the Balance Sheet as an item of Cash and Cash Equivalents. All Cheques are presented to the Bank in the
month of April 2013 and are also realized in the same month in normal course after deposit in the bank. State
with reasons, whether each of the following is an Adjusting Event, and how this fact is to be disclosed by the
Company, with reference to the relevant Accounting Standard.
(i) Cheques collected by the Marketing Personnel of the Company from the Stockists on or before 31st
March 2013. (ii) Cheques sent by the Stockists through courier on or before 31st March 2013.
Solution: Cheques collected by the Marketing Personnel of the Company from the Stockists on or before 31st
March Marketing Personnel, being employees of the Company, have collected the amount before 31st March,
and it constitutes a “receipt” by the Company, before the B/s date. Subsequent realization in April provides
additional evidence of the matter, and hence the Company can consider this as an “Adjusting Event” and
account for such items as “Cheques in Hand”. As per AS-4 it is to be recorded as an adjusting event.
Cheques sent by the Stockists by through Courier on or before 31st March, the Company has not “received”
the Cheques before 31st March, and hence, mere sending of the cheques by the Stockists through Courier
does not pertain to conditions prevailing on the B/s date. This amount should not be adjusted in the accounts.
AS-5 (Revised):
CHAPTER 6
Net Profits or Loss for the Period,
Prior Period Items and Changes in
Accounting Policies

Chapter Outline
vv Objective vv Changes in Accounting Policies
vv Components of Net Profit vv Comparison between Prior Period Items,
vv Prior Period Items (PPI) Changes in Accounting Estimates and
Changes in Accounting Policies
vv Accounting Estimates (AE)

Objective
The objective of this standard is to prescribe the classification and disclosures of various components of
Net profit. By adopting this standard, the whole company will have uniformity in accounts.

Components of Net Profit


Profit and loss account is one of the important part of the whole financial statements. Users of financial
statements analyse income statement for various reasons. Major decisions of the shareholders depend on the
income statement. Also disclosing only the Net Profit will not suffice. Hence proper disclosure and classification
of Net Profit is of utmost importance. All the items of income and expenditure which are recognized in a period
should be included in the profit/loss statement unless an accounting standard requires or permits otherwise.
For example: Forex difference in case of non–integral operations should be transferred to Foreign exchange
translation reserve account. The main objective of AS-5 is to disclose the total composition of profit/loss.

Profit or Loss due to Ordinary Activities


Ordinary activities are activities which are part of business activities. All the items (income/expenses) are
generally ordinary items. Ordinary activities are normally undertaken as a part of business and incidental to
the main business. All Operating items are ordinary items.

Profit/Loss from Exceptional Items


It is basically an ordinary item but sometimes it is necessary to disclose some items separately within
ordinary due to its nature/size of amount, such items are exceptional items. Again AS-5 (Revised) wants
every company to disclose exceptional items separately while drawing income statement. The nature and the
amount is to be disclosed separately so that its effect in the Profit and Loss Account can be perceived.
AS-5 (Revised): Net Profits or Loss for the Period, Prior Period Items and Changes in Accounting Policies 41

Example: Profits and loss on sale of assets, Litigation settlement, writing down stock at NRV, Reversal of
provisions, restructuring expenses, litigation settlements, embezzlement of cash, profit/loss on sale of
long-term investments.

DO YOU KNOW
All ordinary items may not be exceptional, but all exceptional items are ordinary items.

Note: As per the New Schedule III Profit/loss on sale of long or current investments should be disclosed in the
notes under the head “Other income” or “Other expenses” based on profit/loss. As per AS-5 profit or loss on
sale of long-term investments should be disclosed as exceptional items. Its only a difference of presentation,
hence we should follow New Schedule III.

Profit Loss due to Extra-ordinary Activities


Extra-ordinary activities are the activities which are distinct from ordinary. It can be income or expenses.
They are not expected to occur regularly.
The nature and the amount is to be disclosed separately so that its effect in the Profit and Loss Account
can be perceived.

Example: Attachment of a property by the government, losses due to earthquake, Government refund as
per AS-12, two exceptions referred to in AS-12 for grants.

Prior Period Items (PPI)


It is an expense/income recorded in the current year for the omission, errors, and mistake that occurred in
some past years. Whatever omission or mistake has occurred in the previous year requires rectification in
the current year. PPI existing in the financial statement is the curse to the company. It has to be disclosed
separately so that the effect of such item on the profit and loss of current year can be properly perceived.
Errors may be in the form of mathematical mistakes, oversight, omission, wrong interpretation. More number
of PPI means more inefficiency of internal control, internal check/audit department. The way of disclosing
PPI may be different for different companies. Some companies show PPI after the determination of NP/Loss
for the year. Some companies may show PPI under parenthesis.

Examples of PPI: Casting Error, Capital expenditure treated as revenue, Finance lease treated as operating
lease, Omitting any particular income or expenditure in some past year, Applying incorrect rate of Depreciation
etc.

Note: As per the New Schedule III disclosure of PPI is not mentioned hence we should include it in the other
income/expenses, with some details in the footnote about the nature of PPI.

Accounting Estimates (AE)


Due to inherent uncertainties in the business some items (income/expenses) which cannot be exactly
determined requires estimation. Such provisions are accounted based on estimation and known as Accounting
Estimates. The estimation of such amount requires judgment. The amount of provision required for the year
can be obtained from various experts like Lawyers, Engineers, valuers etc. Assessment of estimates is to be
done on every Balance Sheet date. On every Balance sheet date if amount provided previously and an amount
42 Accounting Standards

as assessed found to be different, then such difference is due to change in accounting estimates. Even the
developments in such estimates year to year have to be monitored. It has to be updated with the experience
and new information collected. Changes in Accounting Estimates are also to be disclosed separately. The use
of reasonable estimates is an essential part of preparation of financial statements and does not undermine
their reliability.

Examples of AE:
i) Useful life of assets (of course it relates to depreciation)
ii) Provision for bad debts
iii) Income Tax provision.
iv) Provision for inventory obsolescence.

Disclosures: Amount and nature of change. If the quantification of amount is not possible then fact of such
change in accounting estimate is to be disclosed.

Changes in Accounting Policies


One cannot change APs regularly. If a company wants to change AP once followed then it has to satisfy the
following conditions.
(i) Such change should be allowed by Statute. For example: Before AS-2 was revised many companies
were following LIFO method of Stock Valuation. But AS-2 (Revised) has specifically excluded LIFO
method. Thus, all the companies henceforth have to adopt FIFO, WAC method but not LIFO.
(ii) If change in accounting policies present more true and fair picture of financial statements then before
then such change is allowed.
(iii) A note should be given by the company in its annual accounts regarding the change. Also the amount
of difference is to be disclosed after all the change has affected the financial statements.
Changes in accounting policies may not significant effect on the profit/loss statement but will have
significant effect in the later years.

Examples of AP: Change in the method of Depreciation, Change in the method of Stock Valuation, Change in
the approach of accounting for Government Grants, Revenue recognition policy etc.

To sum up, the following are to be shown separately including their (nature and amounts)
(1) Ordinary items (only exceptional items),
(2) Extra - ordinary items,
(3) Prior period items,
(4) Changes in Accounting Estimates,
(5) Changes in Accounting Policies.

Comparison between Prior Period Items, Changes in Accounting Estimates and


Changes in Accounting Policies
Whether an item (income/expense) is exactly Prior period items, Changes in Accounting Estimates, Changes
in Accounting Policies is to be seen properly. It depends upon the actual facts and circumstances of the case
+ Judgment.
AS-5 (Revised): Net Profits or Loss for the Period, Prior Period Items and Changes in Accounting Policies 43

Prior period items Changes in Accounting Estimates Changes in Accounting Policies


Point 1: Definition
1) It is an expense/income of the current 1) There are some items which are 1) These are those incomes or exps.
year for the omission, errors, and not exactly determined but can which are determined by the policy
mistake occurred in some past years. only be estimated. adopted by the company.
Point 2: Purpose/Reason behind it
2) Frauds, errors, omissions give rise to 2) No proper estimation made due 2) Either change in AP mandated by
Prior Period items. to insufficient information. statute or for more true and fair
disclosure of Accounts.
Point 3: Persons Affecting the Change
3) It is the staff/accountant/department 3) Change in Accounting estimates 3) Persons affecting the change are
concerned. may be due to wrong judgment Statute, Law, Industries etc.
by the experts like lawyers,
actuary etc.
Point 4: What it Shows
4) It shows the efficiency of the accounts 4)
It shows the expertise and 4) It shows how much the accounting
and other departments. knowledge of the consultants practices and policies are strong
and advisors of the company. and percolated in the company.
Point 5: Examples
5) Sales to a party not recorded in 5) Provision for B/D, Provision for 5) Change in the method of
previous year now recorded. tax. Depreciation.

DO YOU KNOW
Occasionally it may be difficult to distinguish between changes in measurement bases (i.e., Accounting Policies)
and Changes in Estimate. In such cases, the change is treated as a change in estimate.

PROBLEMS AND SOLUTIONS

Problem 1: Revision of Wages from 1st January 2011 would costs additional liability of ` 2,50,000 per
annum. Advise about the treatment in the final accounts for the year ending 31.03.2013.
(C.A. Final Nov. 1993, May 1995, May 2002)
Solution: Additional liability for salaries of ` 2,50,000 should be included in the salaries of the current year
i.e. 2012-2013. It may be noted that additional salaries are an expense arising from the ordinary activities
of the enterprise. Although infrequent in occurrence, such an expense does not qualify as an extra-ordinary
item (Para 6.2 of AS-5). One can show the item as excpetional but not extraordinary.
Problem 2: It was found that value of stock costing ` 50,000 had been included twice in the stock sheet as
on 31st March 2010. Advise about the treatment in the final accounts for the year ending 31.03.2011.
(C.A. Final Nov. 1993, May 1995, May 1999, Nov. 2003)
Solution: Rectification of error in stock valuation is a Prior Period Item vide para 3.1 of AS-5. It is necessary
to deduct ` 50,000 from opening stock in the Profit and Loss Account for the year ended on 31st March, 2011.
It is also necessary to charge ` 50,000 as prior period adjustment in the Profit and Loss Account below the
line. It may be mentioned that para 9 of AS-5 requires separate disclosure of Prior Period Items.
44 Accounting Standards

Problem 3: How would you deal with the following in the annual accounts for the year ending 31/03/2005
of ABC Ltd.:
i) The company has to pay delayed cotton clearing charges, over and above the negotiated price, for
taking late delivery of cotton from the suppliers godown. Upto the last year the company use to add
such charges in closing stock valuation. But for the current year the company wants write off the
charges against revenue as the charges are in the nature of interest.
(C.A. Final May 1996, May 2002, Nov 2012)
ii) Fuel surcharge is billed by State Electricity Board at provisional rates. The Final bill for fuel surcharge
for ` 5,30,000 for the period October 2000 – September 2004, has been received and paid in February
2005. (C.A. Final May 1996)
iii) There was a major theft of stores valued at ` 10,00,000 last year which was identified in the current
year. (C.A. Final May 1998)
iv) During the year 2004 – 05 stock was written down to its NRV. (C.A. Final May 2002)
v) ABC Ltd. was making provision for non-moving stocks based on no issues for the last 12 months upto
31.3.2005.
The company wants to provide during the year ending 31.3.2006 based on technical evaluation:

Total value of stock ` 100 lakhs


Provision required based on 12 months issue ` 3.5 lakhs
Provision required based on technical evaluation ` 2.5 lakhs
Does this amount to change in Accounting Policy? Can the company change the method of provision?
(C.A Final. May 2003, Nov 2010, Nov 2012, May 2015)
Solution:
i) a) Para 11 of AS-5 states that a change in an accounting policy should be made only if the adoption of
a different accounting policy is required by statute or for compliance with an accounting standard or if
it is considered that the change would result in a more appropriate preparation or presentation of the
financial statements of an enterprise. Therefore, the changein the method of stock valuation is justified
in view of the fact that the change is in line with the recommendations of AS-2 and would result in more
appropriate preparation of the financial statements. As per AS-2, this accounting policy adopted for
valuation of inventories including the cost formulae used should be disclosed in the financial statements.
Also, appropriate disclosure of the change and the amount by which any item in the financial
statements is affected by such change is necessary as per AS-1, AS-2, and AS-5.
Therefore, the undermentioned note should be given in the Annual Accounts.
“In compliance with the Accounting Standards issued by the ICAI, delayed cotton clearing charges
which are in the nature of interest have been excluded from the valuation of closing stock unlike
preceding years. Had the company continued the accounting practice followed earlier, the value of
closing stock as well as profit before tax for the year would have been higher by ` ………………..”.
ii) There are 2 possibilities in this case (i) fuel surcharge is recorded at provisional rates, (ii) fuel
surcharge is not recorded. If fuel surcharge is recorded then the revised amount paid in 2004-05 is
change in a/c estimate. If fuel charges were not recorded previously, then it is a prior period item. This
is a material item and has to be disclosed separately. Also the payments are not for extra-ordinary
activity because payment towards fuel charges is a part of business operations.
iii) AS-5 : Disclosure as a separate item. This is a prior period item. Loss of theft should have been
recorded in the last year.
iv) As per AS-5 (Revised), it is an exceptional item.
v) The decision of making provision for non-moving stocks on the basis of technical evaluation does not
amount to change in accounting policy. Accounting policy of a company may require that provision
AS-5 (Revised): Net Profits or Loss for the Period, Prior Period Items and Changes in Accounting Policies 45

for non-moving stocks should be made. The method of estimating the amount of provision may be
changed in case a more prudent estimate can be made.
In the given case, considering the total value of stock, the change in the amount of required provision
of non-moving stock from ` 3.5 lakhs to ` 2.5 lakhs is also not material. The disclosure can be made
for such change in the following lines by way of notes to the accounts in the annual accounts of ABC
Ltd. for the year 2002-03:
“The company has provided for non-moving stocks on the basis of technical evaluation unlike
preceding years. Had the same method been followed as in the previous year, the profit for the year
and the corresponding effect on the year end net assets would have been higher by ` 1 lakh”.
P roblem 4: In the context of relevant Accounting Standards, give your comments on the following matters
for the financial year ending on 31.3.2002:
While preparing its final accounts for the year ended 31st March, 2002 Rainbow Limited created a provision
for Bad and Doubtful debts are 2% on trade debtors. A few weeks later the company found that payments
from some of the major debtors were not forthcoming Consequently the company decided to increase the
provision by 10% on the debtors as on 31st March, 2002 as the accounts were still open awaiting approval of
the Board of Directors. Is this to be considered as an extra ordinary item or prior period item?
(C.A. Final Nov. 2002)
Solution: It is an adjusting item as per AS-4 (Rev). The revision in doubtful debts improves the position of
financial statements where information is collected after the balance sheet date. It is not a change in accounting
estimates because the provision has not elapsed year. Also it is not prior period item (no error took place in the
preparation of financial statmeents) or even extraordinary item (debtors are operating assets).
P roblem 5: As an auditor state your views on the following situations:
(a) Y Ltd. provided ` 25 lacs for inventory obsolescence in 1998-1999. In the subsequent year, it was
determined that 50% of such stocks was usable. The company wants to adjust the same, through
prior-period adjustment account, as the provision was made in the earlier years.
(C.A. Final Nov. 2007)
(b) In the previous year ‘Y’ Ltd. has made a provision of 10% of the contract value on ongoing projects.
The actual loss on completion of the contract in the subsequent year was 11%. The management
adjusted the difference in the previous year’s account. (C. A. Final May 1997)
(c) A suit for damages of ` 1 lakh for breach of contract of sale (breach occurred in 1995) was decreed
in favour of May Ltd. in March, 1997. The company has included the amount in its turnover for the
financial year 1997. (C. A. Final May 1998)
Solution:
(a) As per AS-5 on “Net Profit or Loss for the Period, Prior Period Items and Changes in Accounting
Policies”, prior period items are income or expenses which arise in the current period as a result
of errors or omissions in the preparation of the financial statements of one or more prior periods.
The write-back of provision made in respect of inventories in the earlier year does not constitute
prior period adjustment since it neither constitutes error nor omission but it merely involves making
estimates based on prevailing circumstances when financial statement were being prepared. It is a
mere estimate process involving judgment based on the latest information available. An estimate may
have to be revised if changes occur regarding the circumstances on which the estimate was based,
or as a result of new information, more experience or subsequent developments. The revision of
the estimate, by its nature, does not bring the adjustment within the definitions of an extraordinary
item or a prior period item. In this case, T Ltd. provided ` 25 lakhs for inventory obsolescence in
1998-1999. In the subsequent year due to changed circumstances, it was determined that 50% of such
stock was usable. Revision of such an estimate does not bring the resulting amount of ` 12.5 lakhs within
the definition either of a prior period item or of an extra-ordinary item. The amount, however, involved
is material and requires separate disclosure to understand the financial position and performance of an
enterprise. Accordingly, the accounting treatment followed by the company is not proper.
(b) The provision of loss of 10% of the contract value by Y Ltd. in previous year is in the nature of
accounting estimate since due to uncertainties inherent in any business activity, it is difficult to
measure such item in a precise manner. Accordingly, adjusting the difference in the previous years’
46 Accounting Standards

account is not correct. AS-5 on Net Profit or loss for the Period, Prior Period Items and Changes in
Accounting Policies, states that the effect of a change in an accounting estimate should be included
in determination of net profit or loss in the period of the change, if the change affects the period
only. Thus, the management should adjust the difference in the current period only. Alternatively, the
auditor should quality his report.
(c) Amount should not be included in turnover. It is an ordinary activity but is to be disclosed separately
as per AS -5 as exceptional item. The expression “turnover” normally implies the amount for which
the sales are effected or services rendered by an enterprise. Thus, apparently as per this definition,
the amount received on account of breach of contract of sales would not form part of the turnover.
Now question may arise whether such amount is to be disclosed either as a prior period item or
extraordinary item as a separate item in the financial statements. As per the AS-5 on Net Profit or
Loss for the “Period, Prior Period Items and Changes in Accounting Policies”, it is not a prior period
item as the suit was decreed only during the current financial year. Securing damages for breach of
contract cannot be treated as an extraordinary item also since it forms part of ordinary activities of
the business. The separate disclosure of such an item should be called for as per AS-5 in case having
regard to nature, size or incidence of the item concerned and that the separate disclosure is relevant
to explain the performance of the enterprise for the period. Thus, having regard to the materiality
of the amount received it may be considered whether the same may be disclosed separately the
profit and loss account. Alternatively, a separate disclosure may be made in the notes to financial
statements. If the company does not comply with the AS the auditor may qualify his report.
P roblem 6: XYZ Ltd., as part of overall cost cutting measure announced voluntary retirement scheme (VRS)
to its employees, to reduce the employee strength. During the first half year ended 30.9.2002 the company
paid a compensation of ` 72 lakhs to those who availed the scheme. The Chief Accountant has reflected this
payment as part of regular salaries and wages paid by the company. Is this correct? (CA Final May 2003)
Solution:
Accounting Treatment of Payment on account of VRS: As per AS 5, “Net Profit or Loss for the Period, Prior Period
items and Changes in Accounting Policies” the payment made to its employees on account of VRS as an overall cost
cutting measure would fall in the ambit of ordinary activities connected with the business of the enterprise. AS-5
requires that when items of income and expense within profit or loss from ordinary activities are of such size,
nature or incidence that their disclosure is relevant to explain the performance of the enterprise for the period,
the nature and amount of such items should be disclosed separately. In fact an activity like VRS can very well be
treated as restructuring exercise carried by the enterprise. Though this is not an extraordinary item, AS 5 requires
that items of income and expense which are not extraordinary items, the nature and amount of such items may be
relevant to users of financial statements in understanding the financial position and performance of an enterprise
and in making projections about financial position and performance. Disclosure of such information is sometimes
made in the notes to the financial statements. Considering the above, the compensation of ` 72 Lakhs paid towards
VRS availed by employees should be shown separately in the profit and loss account of XYZ Ltd., so that the effect
of it on the operating results of the Company during the previous year can be perceived. Therefore, clubbing of `
72 lakhs with the regular salaries and wages of the company by the Chief Accountant is not appropriate and, thus,
the separate disclosure is necessary.
P roblem 7: During the course of the last three years, a company owning and operating Helicopters lost
four Helicopters. The company’s Accountant felt that after the crash, the maintenance provision created in
respect of the respective helicopters was no longer required, and proposed to write back to the Profit and
Loss account as a prior period item.
Is the Company’s proposed accounting treatment correct? Discuss. (C.A. Final May 2007)
Solution: It is not a PPI because it is not a mistake/omission of past year. It is a normal operating item and
should be credited to P/L account. But the helicopter provision write back is a special operating item. It
requires a separate disclosure (exceptional item).
P roblem 8: Z Ltd. sustained loss during the year due an enemy combat.
(C.A. Final May 1998 Audit)
Solution: Extraordinary activity. Similar to loss due to an earthquake. Situations/events on which
management has no control. (Please define Extra-ordinary activities)
CHAPTER 7
AS-6:
Depreciation Accounting

Chapter Outline
vv Depreciation vv The Effects of Change in Historical Cost and
vv Depreciable Assets Estimated Life of the Asset on Depreciation
vv Exclusions from AS-6 vv Second Hand Asset/Acquired in
Amalgamation Vs Depreciation
vv How to Calculate Amount of Depreciation
vv Disclosures in AS-6
vv Methods of Depreciation

Depreciation
It is the wear and tear of asset – Economics. In accounting, it is defined as a distribution of total cost over the
useful life of the asset. Depreciation is nothing but amortization of cost of assets over its useful life.

DO YOU KNOW
Under IAS, Fixed assets and Depreciation is combined in a single standard IAS-16.

Depreciable Assets
To bring an asset under depreciation chargeable asset, one has to satisfy all the conditions given below:
Condition 1: Assets should have been held for production of goods and services and not for sale.
Condition 2: Assets should be of limited life.
Condition 3: Assets used for more than 1 year- it means assets used for less than 1 year- should be charged
to revenue.
If any one of the above definitions is not satisfies it is not a depreciable fixed asset. Example: Land has all
the features of a depreciable fixed asset except condition 2, hence, Land is not depreciated.

Exclusions from AS-6


(a) Forest plantations, (d) Livestock,
(b) Mineral and Natural Gas asset, (e) Goodwill,
(c) R and D expenditure, (g) Land (being life unlimited).
48 Accounting Standards

How to Calculate Amount of Depreciation

Amount of Depreciation Depends on

Scrap Value Historical Cost Estimated Life*

(Refer AS – 10) It is recoverable amount at the Life is estimated in the beginning.


end, If RV is insignificant then
scrap value is to be taken at Nil.

*Estimated life can be either useful life (physical life) or economic life (number of productions one can reap from the assets)

SCHEDULE II: DEPRECIATION [AS PER THE COMPANIES ACT, 2013]

Key highlights:
◘◘ Instead of % of depreciation now it is useful life of the asset. Remembering rate is difficult but life is
easy. Example: 9 years is easy to recall but 11.11% is tough!
◘◘ Maximum cap on salvage value.
◘◘ Other than SLM, WDV even units of production can be applied.
◘◘ Component accounting is now mandatory.
◘◘ Schedule II is almost adhering to the principles of IFRS – IAS : 16 (Thanks Central Government to keep
policies akin to global guidelines)
PART A : Deals with main provisions relating to depreciation of tangible and intangible assets.
PART B : Deals with provisions relating to depreciation of assets covered by the act of the parliament.
PART C : Its provides the list of the useful life of an asset. It also provides some relevant notes related to the
calculation of depreciation.

DO YOU KNOW
(i) AS-6 states that depreciated should be calculated as per the rates given under special act / statute?
Example: Electricity Act, Companies Act 1956. But now Schedule II overrides all the acts / statutes.
(ii) Useful life is inversely proportion to Rate of depreciation. Higher the rate lower the useful life.

PART A of Schedule II
TANGIBLE FIXED ASSETS
Depreciation as per Schedule II is the systematic allocation of depreciation over the useful life. It means
depreciation is to be allocated as per the method chosen and up to its useful life.
Method permitted can be SLM, WDV or even production / unit method.
AS-6: Depreciation Accounting 49

Depreciation amount will be (Cost – residual value). Can we say that Depreciation as per Schedule II is the
systematic allocation of depreciation amount over the useful life.
Useful life: Schedule II does not provides rates but has provided the maximum useful life. The useful life
given in PART ‘C’ is the maximum life (minimum rates of depreciation) .
Residual Value: The New Schedule related to depreciation casts a cap on the RV to be 5% on original cost.
Different Useful life and Residual Value: The useful life of an asset shall not be longer than the useful life
specified in Part ‘C’ and the residual value of an asset shall not be more than 5% of the original cost of the
asset:
Provided that where a company uses a useful life or residual value of the asset which is different from the
above limits, justification for the difference shall be disclosed in its financial statement.;

INTANGIBLE FIXED ASSETS:


Schedule II covers only one type of intangible asset for amortisation i.e. Toll rights. All other intangible assets
are to be covered by AS-26. Can we conclude that amortization of Patents, Copy - writes, Licenses; Brand etc
are still all covered by AS-26 except “Service Concessions arrangements”.

PART ‘B’
The useful life or residual value of any specific asset, as notified for accounting purposes by a Regulatory
Authority constituted under an Act of Parliament or by the Central Government shall be applied in
calculating the depreciation to be provided for such asset irrespective of the requirements of this Schedule II.
Example: Depreciation rates for Electricity Supply Company has to follow the old Schedule.

PART ‘C’
Here, useful life of assets is given (students need not remember the same).
NESD: means No extra shift depreciation.

Other important notes:


1) Asset costing ` 5000 or less:
There is no specific requirement to charge 100% depreciation on assets whose actual cost does not
exceed `5,000, so any assets irrespective of value should be categorized based on Schedule II and
depreciated based on Useful life of assets. However, Para 12 of Accounting Standard 6 says that In
respect of depreciable assets which do not have material value, depreciation is often allocated fully
in the accounting period in which they are acquired. This means irrespective of the cost price of the
asset schedule II wants the asset to be depreciated as per the useful life.
2) Additions and Dispositions:
Where during any financial year, any addition has been made or sold, discarded, demolished or
destroyed, the depreciation on such assets shall be calculated on pro rata basis.
3) Only one Intangible assets covered:
All Intangible Assets are covered by AS-26 except Toll road under BOT system. Amortization in such
case is based on Revenue basis.
4) Depreciation for double, triple shift assets:
In Schedule XIV of Companies Act, 1956 for the assets used in double shift or triple shift, shift wise
different rates are prescribed, whereas in Schedule II of Companies Act, 2013 it specifies that if an
asset is used for any time during the year for double shift, the depreciation will increase by 50% for
that period and in case of the triple shift the depreciation shall be calculated on the basis of 100% for
that period. Increase in depreciation means decrease in life.
50 Accounting Standards

The above provisions are not applicable to NESD assets.


5) Component Accounting - Note No 4 of Schedule II : (supported by AS-10 as well as IAS – 16)
(a) Useful life specified in the Schedule II to the Companies Act is for whole of the asset. Where
cost of a part of the asset is significant to the total cost of the asset and useful life of that part
is different from the useful life of the remaining asset, useful life of that significant part will be
determined separately
(b) This paragraph talks about the ‘component approach’, which is in line with international
practices. It requires companies to separately depreciate parts of an asset that are significant
and have a useful life different from the useful life of the asset as a whole
(c) The component approach is already allowed in current Accounting Standard 10 – “Accounting
for Fixed Assets” in Paragraphs 8.3, but it seems to be a choice of matter. Now it seems as per the
note 4 it is mandatory in Companies Act, 2013.
(d) Previously companies use to charge component replacement to profit / loss account. But now it
will be separately capitalised.
Example: Building and its Elevator can be treated as a separate component. Also Aircraft body
and its engine can be treated as 2 separate assets.
(d) Effect of Schedule II on Revaluation of assets:
Schedule II has no effect on Revaluation provisions. Lets see what it was:
Whenever an asset is upward revalued the depreciation on the excess portion is charged to the
Revaluation Reserves Account.
(e) Transitional Effect of Schedule II – Very Important provision at the date of applicability of
Schedule II of Companies Act, 2013
From the date of the Companies Act coming into effect, the carrying amount of the asset as on
that date:
(i) Shall be depreciated over the remaining useful life of the asset according to the Act .This
means shifting from the old provisions to the new one the remaining carrying amount is to
be just spread over the remaining useful as per the new provisions on a systematic basis.
(ii) After retaining the residual value, shall be recognized in the opening balance of retained
earnings where the remaining useful life of an asset is nil. It means if the remaining
useful life as per the new provisions is Nil, then the balance in the asset after keeping aside
5% on original cost is adjusted in the opening reserves (free reserves).
(f) Disclosures in Schedule II
The following information shall also be disclosed in the accounts, namely:
(i) Depreciation methods used; and
(ii) The useful lives of the assets for computing depreciation, if they are different from the life
specified in the Schedule
(g) Amortization of Service Concession Arrangement:
In certain countries construction or maintenance of infrastructure for public services (roads,
bridges, water distribution facilities) is contracted out to private-sector operators. The private
sector operators are usually paid for their services over the term of the arrangements. Such
arrangements are often described as “build-operate-transfer” (“BOT”) or a “public-to-private
service concession arrangement.”
Consideration given by the grantor to the operator: If the operator renders construction or
upgrade services, the consideration received or receivable by the operator shall be recognized
at its fair value. The consideration may be treated as an intangible asset.
Recognizing an “intangible asset”—the operator shall recognize an intangible asset to the extent
that it receives a right or a licence to charge users of the public service.
AS-6: Depreciation Accounting 51

Disclosures in AS-6
◘◘ Total cost of each class of asset.
◘◘ Total depreciation on each class of asset.
◘◘ Accumulated depreciation.
◘◘ Method of depreciation.
◘◘ A change in method of depreciation.
◘◘ Effect of the revaluation of the fixed asset on the amount of depreciation.

PROBLEMS AND SOLUTIONS

Problem 1: As an auditor, state your views on the following situation: Y Ltd. Purchased an existing bottling
unit. The method of charging depreciation of machinery of the acquired unit was different from that followed
by the company in its other units. The company wants to continue to charge depreciation for the acquired
unit, in the method followed earlier by them and which was not consistent with their own method.
(CA Final Audit, Nov. 1997, Nov. 2013)
Solution: If a company has two class of assets and method of depreciation varies in the two cases then in
such case different method of depreciation is allowed. But such different method is required to be followed
consistently every year. Hence the view of the Chief Accountant that the company cannot follow SLM method
of depreciation at the newly acquired unit is not correct and the company can follow the SLM method of
depreciation at the newly acquired unit.
Problem 2: The company procured ocean going dredgers which were cleared by the Customs without levy
of customs duty in the year 2010. However, in 2013, the Customs authorities alleged willful misstatement
and suppression of facts. The company disputed the allegations but believes that the liability will fructify.
Therefore, the amount of custom duty sought to be levied on the respective dredgers has been capitalized and
a provision for custom duty was made in the books. Advise your client on how to charge depreciation on the
addition to fixed asset in 2013 caused by capitalization of the customs duty.
Solution: Any change in the historical cost of an asset like price fluctuation, govt. grant etc. should be given
prospective effect. But Custom duty is a case of asset escaping duty. This means the asset initially would
have been more by customs duty. Such additions are to be given retrospective effect. Also such additions will
attract AS-5 as the proportionate depreciation relating to the customs duty will be charged to P/L as a prior
period item.
Problem 3: An item of plant was purchased on 01.04.2006 for ` 2,00,000. The diminishing balance rate
applicable to the plant was 15%. The written down value of the plant as on 31.03.2008 was ` 1,44,500.
On 01.04.2008 the enterprise decided to change the method from diminishing balance to straight line. The
enterprise decided to write off the book value of ` 1,44,500 over the remaining useful life of plant i.e. 15 years
(Out of the total useful life of 17 years, 2 years have already elapsed)
Comment whether the accounting treatment is correct. If not, give the correct accounting treatment with
reason.
Solution: The change in the method of depreciation should be given retrospective effect (AS-5), Change in
accounting policy. The company is required to depreciate as per SLM ` 11,765 (Cost/17) every year. Also the
change in the method of depreciate is to be given retrospective effect. The Opening reserves as on 1/4/2008
is to be adjusted by ` 31,970 (55,500 Old method – 23,530 New method). AS-5 (Rev) requires separate
disclosure viz change in a/c policy.
52 Accounting Standards

Problem 4: A company has acquired a Plant and Machinery at ` 7.50 lakhs on installment basis from a
dealer (the cash down price is ` 7 lakhs). During the year, company paid ` 50,000 as down payment and ` 2.5
lakhs as the installment for the year. The company provided the depreciation @ 15% on ` 3 lakhs. Comment,
whether the depreciation on ` 3 lakhs is correct? If not on what value depreciation should be charged?
Solution: It is a case of Hire purchase covered by AS-19. The asset belongs to the company who acquired it
on deferred basis. The depreciation will be always charged on the total cash price of the asset i.e. ` 7,00,000.
Depreciation will neither be charged on the total installment value i.e. ` 7,50,000, nor the amount paid during
the year i.e. ` 3,00,000. Therefore depreciation for the year = 15% on ` 7,00,000 = ` 1,05,000.
Problem 5: Cost of the asset = 5,00,000, Scrap value ` 25,000, Estimated life = 4 years. Compute Depreciation
rate as per SLM and WDV method?
Solution:
Depreciation as per SLM = (500000 – 25000)/4 = 118750
Depreciation rate SLM = 118750/500000 = 23.75%
Depreciation as per WDV
25000
Depreciation % = 1 – 4 = 52.71%
500000
Problem 6: Calculation of rate:
If the useful life is 10 years calculate the rate of depreciation assuming SLM and WDV both? Consider RV as
2% of cost?
Solution:
Assume cost Rs 100 and RV is 2. As per Schedule II: 98 (100-2) is known as depreciable amount.
SLM % = 98 / 10 = Rs 9.8 p.a. = 9.8 / 100 = 9.8% p.a.
WDV% = 1-(Salvage / Cost)1/n = 1 – (2/100)1/10 = 32.36% p.a
Problem 7: Calculation of rate:
If the useful life is 6 years, Cost of the asset is Rs 4,00,000, Calculate the rate of depreciation assuming SLM
and WDV both? Consider RV as 10% of cost?
Solution:
Assuming that the company provides sufficient justification for 10% RV, the rates are calculated as follows:
(as per Schedule II the maximum RV cannot exceed 5% of the cost)
SLM
Annual dep = 360000 / 6 = 60000.
Dep % = 60000 / 400000 = 15%
WDV
Dep % = 1-(Salvage / Cost)1/n = 1 – (40000/400000)^1/6 = 31.86% p.a
Problem 8: Transitional provisions:
Edition Limited has purchased an office equipment of Rs 5,00,000 on 1/4/2012. Depreciation as per Schedule
XIV was 16.33% p.a. SLM. You are required to calculate the depreciation for the year 2014-15 by both SLM
and WDV if the useful life of the asset under Schedule II is 5 years. Assume residual value as per Schedule II.
Solution:
Balance carrying amount of the asset as on 1/4/2014 = 500000 – 500000 x 16.33% x 2 = Rs 3,36,700
Balance life left out as per Schedule II = 5-2 = 3 years.
Systematic allocation over 3 years.
Depreciation for the year 2014-15:
SLM = (336700 – 5% x 336700) / 3 = ` 1,06,622
AS-6: Depreciation Accounting 53

WDV % = 1 – (16835 / 336700)^1/3 = 36.85%


Depreciation = 336700 x 36.85% = ` 1,24,074
Problem 9: Transitional provisions:
Edition Limited has purchased an office equipment of Rs 5,00,000 on 1/4/2010. Depreciation as per Schedule
XIV was 16.33% p.a. SLM. You are required to calculate the depreciation for the year 2014-15 by SLM method
if the useful life of the asset under Schedule II is 4 years. Assume residual value as per Schedule II.
Solution:
Balance carrying amount of the asset as on 1/4/2014 =500000 – 500000 x 16.33% x 4 = Rs 1,73,400
Balance life left out as per Schedule II = 4-4 = 0..
Depreciation for the year 2014-15 = Nil
As the entire asset is written off as per Schedule II, the balance carrying amount should be written
off to opening reserves subject to salvage.
Accounting Entry:
General Reserves Dr Rs 1,48,400
To Office Equipment Rs 1,48,400 (173400 – 500000 x 5%)
If the company has overcharged depreciation as per Schedule XIV then the overcharged amount should not
be written back to profit / loss account. Ex: Balance of the asset on 1/4/2014 is Rs 45000 and salvage value
Rs 50000. The remaining life of the asset under the new provisions is 2 years. Here the salvage value itself
is Rs 45000 which is to be kept aside. Also Rs 5000 excess depreciation already charged should be ignored.
Problem 10: Sale of asset already written off:
Suppose in the year 2015-16 the above asset was sold for ` 18,000. Pass the journal.
Solution:
Accounting Entry:
Cash Dr Rs 18,000
Loss on sale (bal fig) Rs 7,000
To Office Equipment Rs 25,000 (500000 x 5%)
Problem 11: Pro rata depreciation:
On 15/07/2014 Adam Ltd acquired an asset for Rs 20,00,000 with a useful life 15 years and 5% salvage
value. You are required to calculate the depreciation for the year 2014-15 by WDV method.
Solution:
Depreciation % = 1 – (100000 / 2000000)1/15 = 18.10%
No of days asset is used = 259 days out of total days 365
Depreciation = 2000000 x 18.10% x 259/365 = ` 2,56,871.
Problem 12: Double and triple shift:
Welcome Limited has purchased Machinery for civil construction for Rs 60 lakhs. The asset is not a NESD
asset. The estimated life is 15 years. And the company follows SLM method of depreciation. You are required
to calculate the depreciation for the year 2014-15 assuming the asset is used for (a) Single shift, (b) Double
shift, ( c ) Triple shift. Assume residual value as per Schedule II.
Solution:
First for Single shift:
SLM
Annual depreciation = 60lakhs x 95%/ 15 = 3.8 lakhs p.a.
Depreciation % = 3.8 / 60 = 6.33%
54 Accounting Standards

Double shift:
Depreciation % = 6.33% x 1.5 = 9.50%. Depreciation = 60 lakhs x 9.5% = ` 5.7 lakhs
Triples shift:
Depreciation % = 6.33% x 2 = 12.67%. Depreciation = 60 lakhs x 12.67% = ` 7.60 lakhs
Problem 13: Reverse calculation:
Vakoba Limited has purchased an equipment whose depreciation rate is 11.875% p.a. SLM. Calculate the
useful life. Assume cost Rs 200000 and RV 5%.
Solution:
Depreciation = 200000 x 11.875% = Rs 23750 p.a.
Depreciation = (Cost – SV) / Useful life : 23750 = (200000 – 10000) / n
n = 8 years.
Vakoba Limited has purchased an equipment whose depreciation rate is 45.06% p.a. by WDV.
Calculate the useful life. Assume cost Rs 200000 and RV 5%.
Solution:
Depreciation % = 1 – (10000 / 200000)1/n = 45.06%s
0.05 1/n = 1 - 0.4506 = 0.54938
0.05 = 0.54938 n ..................(multiplying both the sides by the power of 1/n)
Referring the compounding table n = 5 years.
Problem 14: Revenue based amortisation:
Buddhi Developers entered into a contract with the government to construct a highway under the BOT
system. Buddhi Dev construct the infrastructure for 2 years and will acquire the rights to collect toll charges
from the vehicles running on the highway. Construction costs estimated including profits is Rs 5000 lakhs
You are required to calculate the annual amortization:
Solution:
Suppose the service construction period is 6 years. Estimated revenue is given:
# is actual revenue (` in lakhs)
Year Year 1 Year 2 Year 3 Year 4 Year 5 Year 6
1 #1000
2 1000 #1050
3 1100 1050 #1100
4 1100 1050 1100 #1105
5 1100 1050 1100 1105 #1110
6 1100 1050 1100 1100 1110 #1120

Amortization for the first year = (5000 x 1000 / 6400) = ` 781.25 lakhs
Amortization for the second year = (5000 x 2050 / 6250) = ` 1640.
But 781.25 is already recorded. So in the second year amortization will be = 1650-781.25 = `
858.75lakhs.
Amortization for the third year = (5000 x 3150 / 6450) = ` 2442.
But 1640 is already recorded. So in the third year amortization will be
= 2442 – 1640 = ` 802lakhs. Similarly we calculate for other years.
CHAPTER 8
AS-7 (Revised):
Construction Contracts

Chapter Outline
vv Objective vv Contract Costs
vv Meaning and Scope vv Recognition of Revenues and Costs
vv Types of Construction Contracts vv Accounting for Expected Loss
vv Contract Revenues vv Disclosures as per AS-7 (Revised)

Objective
In case of accounting for inventories are concerned purchasing raw – materials, manufacturing them and
then ultimately selling the finished products requires approximately 2 months to 1 year. This means accrual
of cost and revenues are well covered within one financial year. But a contract generally commences in one
year and ends in different financial year. The execution of work is spread over two or more financial year.
Here unlike inventories (AS-2) allocation of costs and revenue becomes difficult. Also long range contracts
carries with it risks and uncertainties. Many a times a contractor works on many contracts in a particular
financial year. Therefore the primary issue in studying AS- 7 is to know the proper allocation of costs and
revenue for a particular financial year. How a contractor should recognize profit/loss for one financial year
for each contract out of so many contracts is guided by AS-7 (Revised)?

DO YOU KNOW
AS -7 (R) is an exception to the general rule set out in AS-2 which does not permit recognition of profit on
inventories.

Meaning and Scope


AS -7 (Revised) is mandatory from 01.04.2003. AS- 7 is applicable to the Contractors only. Contractor is that
party who performs the contract. He has a right to receive progress payments. Contractee has the right to get
the contract work completed.
A construction contract is a contract specifically negotiated for the construction of a single asset like dam,
bridge, pipeline or combination of assets that are closely interrelated or interdependent in terms of its design,
technology or function or their purpose or use like manufacturing various parts of a huge plant. The above
definition covers all direct construction contracts. AS- 7 (R) well within its ambit covers contracts which are
56 Accounting Standards

not construction contracts but are related to construction contracts. Following are the contracts which are
also covered by AS-7(Revised):
i) Contracts involving rendering of services or which are assisting or related to a construction contract.
Example: Services rendered by an architect.
ii) Contracts for destruction of assets like demolition of buildings.
iii) Restoration of assets or restoration of environment. Example: leveling of land or removal of garbage.
Illustration: A contract for building a super market is given to X Ltd. but before building the structure, it
requires land leveling and removal of debris etc. which was handed over to Y Ltd. Here there are two separate
contracts one construction of super market and second leveling of land etc.

Indirect Contracts are also Covered


Contracts relating to rendering services that are not related to construction of asset, for example: providing
strategic services to an enterprise would not attract the applicability of AS-7. AS-7 will be applicable to that
contract which is directly responsible for creating an asset. Services rendered for maintenance activities not
involving construction of asset will be covered by AS-9. Where the existing depth of sea is increased thereby
resulting in creation of enhanced facilities at port (capital dredging activities) then AS-7 will be applicable.
Also AS-7 will be applicable to a gold mine concern having expertise in bringing mines into production stage,
contracts of lining, equipping on commercial basis and handing over to the clients.

Types of Construction Contracts


(i) Fixed price contracts, (ii) Costs plus contracts, (iii) Hybrid contracts.
i) Fixed price contracts: A Fixed price contract is a contract where the price of the contract is fixed in
advance at the time of entering into the agreement. Such types of contracts carry less uncertainty.
In some cases the price may be fixed based on per unit of output produced. Example: Price may be
fixed based on per square feet of the building to be constructed. Fixed price contracts also includes
escalation clause for reimbursement of expenses in case of rise in the price of materials, labour etc.
ii) Costs plus contracts: A Costs plus contract is a contract where not only the costs incurred are
reimbursed but also a fixed fee is paid or a margin of profit is paid which is the profit on the contract.
Such types of contracts are entered when the contract costs with reasonable accuracy is not possible
due to uncertainty. Costs which are reimbursed should be identified and well defined.
Hence, Total Contract Price as per costs plus contracts = (Costs reimbursed + Fix Fee or % of
Costs reimbursed).
iii) Hybrid contracts: It contains the features of both i.e. fixed price contract as well as Costs plus
contract.

Combining and Segmenting Contracts


Combining of contracts means consolidating various works/contracts into a single contract and Segmenting
of contracts means splitting up of a single contract into several contracts.
First of all, AS-7 applies to each contract separately and not to each work as such. Therefore, a proper
distinction is to be made for combining various contracts or segmenting a single contract. For understanding
this, Accounting Standard has given three different situations:
Situation A: A single contract may give so many works to be performed or a single contract may contain
combination of various assets to be constructed. Then each of the work performed/construction of asset is to
be taken as a separate contract only when:
◘◘ Each work performed/construction of asset is a separate proposal.
◘◘ Each work performed/construction of asset is subject to separate negotiation.
AS-7 (Revised): Construction Contracts 57

◘◘ Contractor may accept or reject a particular work to be performed/asset to be constructed (Even


freedom is vested in the hands of contractee also).
◘◘ Costs and revenues of each work performed/completion of asset can be ascertained separately.
Situation B: A contract may initially stipulates for construction of an additional asset or an additional asset
may be added at the time of performing original contract. Whatever may be the case whether one should treat
that additional asset as a separate contract or is to be included within the original contract. The additional
asset is to be treated as a separate contract only when:
◘◘ The additional asset differs significantly from the assets covered by the original contract.
◘◘ The additional asset is a separate proposal and is subject to separate negotiation.
◘◘ The contract price of the additional asset is independent of the price of original contract.
Situation C: Sometimes a group of contracts may be treated as a single contract. It does not matter whether
the group of contracts have been received from a single customer or received from several customers. A
group of contracts may be treated as a single contract when:
◘◘ A group of contracts is assigned as a single package.
◘◘ All the contracts put together forms part of a single project giving an overall profit margin.
◘◘ All the contracts individually are interdependent on each other.
◘◘ All such contracts can be performed simultaneously (concurrently) or consequently (one after the
other).
Note: Situation A and B relate to segmenting contracts and situation C relates to combining contracts.

DO YOU KNOW
We are not going by the mere form of the contracts but we are going by the real substance in the contract/
contracts for segmenting contracts and combining contracts ……….Principle of Substance over form.

Illustration: Universal Builders Inc. is well known for its expertise in building flyovers and maintaining these
structures. Impressed with Universal’s track record, the local municipal authorities have invited them to
submit a tender for a two-year contract to build a super flyover in the heart of the city (the largest in the
region) and another tender for maintenance of the flyover for 10 years after completion of the construction.
Required: Evaluate whether these two contracts should be segmented or combined into one contract.
Solution: The two contracts should be combined and treated as a single contract because
◘◘ The two contracts are very closely related to each other and, in fact, are part of a single contract with
an overall profit margin.
◘◘ The contracts have been negotiated as a single package.
◘◘ The contracts are performed in a continuous sequence.

Contract Revenues
It is the Gross Revenue Income accounted for construction contracts. Following are the elements of Contract
Revenues:

Elements of Revenue Points to be considered/Conditions


Initial contract price – As agreed at the inception of the Consideration may be received or receivable.
contract.
Revenue from Variation in price – May be due to change • Variation in the price should be accepted by the
in the scope of work or change in the specification/design customer.
of the asset. • Amount should be measured reliably.
58 Accounting Standards

A Claim – It is the amount, which the contractor seeks to • Negotiations should have been reached reasonably at
collect from the contractee or any third party. Such claim an advanced stage.
was not included in the original contract price. • Claim should be accepted by the customer/third.
Example: Price Escalations for materials, labour has to • Amount should be measured reliably.
be reimbursed to the contractor by the contractee, cost
overruns because of clear site is not provided to the
contractor.
Incentives – It is extra amount payable for meeting the set • Performance of the contract should have been reached
out standards or exceeding the standards. reasonably at an advanced stage.
Example: Early completion of the contract can attract • It is probable that the contractor will meet the set out
incentives to the contractor. standards or may exceed the standards.
• Amount should be measured reliably.
Penalties – Penalties levied by the contractee reduces • Reduction of the contract revenue.
the contract revenue. Penalties may be levied for late
completion of the contract.

Other Points to be Considered for Contract Revenue


◘◘ A contract price once fixed can be increased or decreased subsequently.
◘◘ Contract revenues once estimated has to be revised and updated at regular intervals.
◘◘ Wherever possible uncertainties has to be resolved surrounding the contract revenue.

Contract Costs
Following are the elements of Contract Costs:

Elements of Costs Examples


Prime costs: These are the direct costs which are charged Direct materials, stores, labour costs, supervision costs,
to the contract specifically. depreciation on construction assets, freight and taxes,
Less: Incidental income – It is the income from sale of Consultancy fees, hire charges, third party claims.
assets used in the contract. Such incidental income has to Sale of materials and plant
be deducted from the above direct costs. Sometimes interest earned on advances received from
contractee.
Allocated Contract Cost – These are the costs that are allocated Insurance incurred for all the contracts, borrowing
to various contracts. Such costs are allocated on a systematic costs After satisfying conditions of AS-16, Construction
and rational basis. The allocation is based on normal capacity overheads or any other expenses which can be allocated.
levels. Cost driver can be prime costs or contract revenue.
Costs specifically chargeable to a customer – These are Some administrative expenses.
the indirect costs whose reimbursement is mentioned in
the terms of the contract includes administrative costs,
development costs.
Costs incurred for securing a contract – Such costs Tender fees.
should be reimbursed and separately identified. If such
costs are already expensed when the contract was not in
place then such costs cannot be recognised when contract
is secured.

Exclusions from Contract Costs: Following are to be excluded from contract costs:
i. Administrative expenses not reimbursed
ii. Selling expenses (excluded even if they are reimbursed)
iii. R and D costs
iv. Abnormal losses on contract. Such costs have to be expensed in General Profit and Loss account.
AS-7 (Revised): Construction Contracts 59

Recognition of Revenues and Costs


Once we know the various elements of costs and revenues then comes the recognition of the revenues and
costs. As far as costs plus contracts are concerned determination of cost and revenue is very simple. What
poses the problem is the recognition of costs and revenues in case of fixed price contracts. Following are the
principles for the recognition of revenues and costs with various possibilities:

Principles of Revenue and Costs Recognition


Possibility 1: When the outcome of the contract can be reliably measured:
When the outcome of the contract is reliably measured recognition of the revenues and costs again depend
on whether the contract is fixed price contract or Costs plus contract.
For Costs plus Contracts: In case of Costs plus contracts the outcome of the contract is reliably measured
when the following conditions are satisfied:
a) It is probable that the economic benefits from the contract will flow to the enterprise.
b) Contract costs whether or not specifically reimbursed are clearly identified and estimated reliably.
For Fixed price Contracts: In case of Fixed price contracts the outcome of the contract is reliably measured
when the following conditions are satisfied:
a) Total contract revenue can be measured reliably.
b) It is probable that the economic benefits from the contract will flow to the enterprise.
c) contract costs incurred upto date plus future estimated costs to complete the contract is measured
reliably.
d) Contract costs attributable to the contract is identified and estimated reliably and can be compared
with the prior estimates.
If any cost is incurred which relates to future activity then as on the year end such costs has to be added
to the WIP and it cannot be expensed. For example: Materials bought but not in use, which relates to future
activity, has to be carry forward in WIP rather than being expensed.

What is Stage of Completion?


Stage of completion measures the work performed on a contract. It is also known as percentage completion
method, which works on the accrual system of accounting. Stage of completion is very useful for recognising
revenue and costs for construction contracts. Billing pattern would not reflect the revenue recognition. The
Stage of completion can be calculated in variety of ways:
Method 1: Physical proportion method: Here stage of completion is determined by analysis of the finished
product. Number of units completed is analysed and calculated to get the stage of completion. For example:
If as building structure contains 5 Floors and ground floor and if the contractor has completed the physical
work of ground floor and first floor then stage of completion here is 33.33%.
Method 2: Surveys of work performed method: Under physical proportion method technical analysis of
the work was absent hence this method is not much reliable. Stage of completion can also be determined by
acquiring a certificate from an architect/engineer who makes a technical analysis of the work performed. He
can properly value the Work completed and the work incomplete.
Method 3: Costs to costs method: This method is more reliable as compared to the above two methods and
hence accountants are more comfortable to use this method. This method works on the principle that costs
incurred to date reflects the stage of completion.
Costs incurred to date
Formula for calculating Stage of completion = × 100
Estimated total cos ts
60 Accounting Standards

DO YOU KNOW
Percentage completion method is applied on a cumulative basis year after year. Also percentage completion
method works on the accrual basis of accounting….Fundamental Accounting Assumption.

Steps to solve the problem on recognition of revenue and costs: Costs to costs basis.
Steps Particulars Amount
Step 1: Compute actual costs incurred upto date = (Actual costs incurred – Closing stock of materials xxx
unused)
Step 2: Ascertain estimated future costs to complete the contract xxx
Step 3: Ascertain Total Estimated Costs (Step 1 + Step 2) xxx
Costs incurred to date
Step 4: Determine the stage of completion =
Estimated total costs xxx
Step 5: Ascertain total contract revenue = xxx
Total Contract price x % of Completion + Escalation claims
Step 6: Ascertain contract revenue for the year = (Total contract revenue – Revenue already recognised
in previous years) xxx
Step 7: Ascertain contract costs for the year = (Contract costs incurred upto date – Costs already
recognised in previous years) xxx
Step 8: Profit for the year (Contract profit) = (Step 6 – Step 7) xxx

DO YOU KNOW
Stage of completion method (%) is calculated without taking into effect the inflation. In other words Stage of
completion method (%) is calculated based on original estimates.
Some important points:
yy Estimated costs to complete require a proper adjustment to be made for expected increase/decrease in
prices or changes in foreign exchange rates. Adjustment should also be made for any delay that may arise
out of labour problems. Estimated costs have to be updated quarterly in case of listed companies.
yy Revenue is not based on progress payments or advances received.
yy Any contract costs which are not utilized at the balance sheet date (closing stock of materials) should not
be expensed but shown as an asset. It is also known as contract costs for future activity (WIP).

Possibility 2: When the outcome of the contract cannot be reliably measured:


When the outcome of the contract cannot be reliably measured recognition of the revenues and costs is
accounted as follows:
◘◘ revenue should be recognised only to the extent of costs incurred, and of which, recovery is certain.
◘◘ costs should be expensed in the year in which which costs are incurred.

Accounting for Expected Loss


(Very important para asked 5 times in CA Final)
When it is probable that the total contract costs exceeds contract revenue the difference of expected loss has
to be charged to Profit and Loss Account. 100% expected loss has to be accounted for. Such expected loss has
to be expensed irrespective of:
◘◘ Progress payments received.
◘◘ Percentage (%) of work completed.
AS-7 (Revised): Construction Contracts 61

◘◘ Profits on other contracts.


Expected loss to be expensed = (Cost incurred to date + Estimated cost to complete) – Total Revenue

DO YOU KNOW
AS -7 (R) is one of the example of Onerous Contract covered defined by ASI-30.

Disclosures as per AS-7 (Revised)


◘◘ The amount of revenue recognised during the year.
◘◘ Aggregate of all costs incurred + profit recognised – losses recognised upto reported date.
◘◘ Advance received.
◘◘ Gross dues to customers (show as Liability).
◘◘ Gross dues from customers (show as Asset).
(ii) In addition to the above an enterprise should disclose:
ww Retentions
ww Method of calculating % completion of work
ww Stage of completion till date
(iii) AS-29 Disclosures:
ww Contingencies relating to claims, guarantees etc. as required by AS-29.
ww Indian GAAPS relating to AS-7 (Rev) are in complete harmony with IAS-11. IFRIC (interpreta-
tion committee) brought out by IASB provides for issues on service concession arrangements
referred to as BOT (Build – Operate – Transfer). Here the asset is first constructed, then operated
for some years and then transferred.

PROBLEMS AND SOLUTIONS

Problem 1: Comment on the following whether they are covered under AS-7 (Revised) with reasons:
(Similar Question asked in Audit Nov 2010)
Case I: AB construction company constructs 18 residential flats during the accounting year 2004 – 2005. Out
of 18 flats 4 flats are kept for entertainment purposes and for serving the guests and the balance are sold to
the Companies Directors for accommodation.
Case II: CD construction company constructs a big MALL SHOPPING CENTRE for selling the shops and
commercial complexes. The MALL is constructed on their own to sell and not under any contract with any
contractee.
Solution:
Case I: Flats which are kept for the guests etc. is Fixed Assets for the company. As such it is not a sale
transaction at all. And even the balance Flats sold to the Companies Directors will not be covered by AS-7 but
it will be income accounted as per AS-9 (Other Income). Even there is no contractor - contractee relationship.
Hence the transactions will not be covered by AS-7.
Case II: Constructing the MALL and then selling the shops and premises on their own account, and not under
a contract does not attract AS-7. Shops and premises unsold or in process should be shown as closing stock
as per AS-2. Even the materials and equipment is to be shown under as closing stock. When the shops and
premises are sold at that point of time it will be treated as sale as per AS-9. It is a case of Real estate developers.
62 Accounting Standards

Problem 2: Determine whether the contracts are segmenting contracts or combining contracts from the
following situations given below:
i) Mehta Constructions has 4 contracts for construction and installation of ATM booths for Central
Bank. Each contract is a separate proposal. The contract price per contract ` 32 lakhs.
ii) Construction of super bazar for AN Ltd. was divided into the following stages: Stage1 – Leveling the
ground, Stage2 – Construction of super bazaar, Stage3 – Canteen attached to the super bazaar. All the
contracts are assigned as a package.
Solution:
i) A single contract may give so many works to be performed or a single contract may contain
combination of various assets to be constructed. Here a single gives 4 works to be performed. Then
each of the work performed is to be taken as a separate contract because :
ww each work performed/construction of asset is a separate proposal.
ww each work performed/construction of asset is subject to separate negotiation.
ww Contractor may accept or reject a particular work to be performed/asset to be constructed.
ww Costs &revenues of each work performed/completion of asset can be ascertained separately.
ii) Sometimes, a group of contracts may be treated as a single contract. The whole contract has been
given, as a package containing various works to be performed. Therefore, the group of contracts may
be treated as a single contract because :
ww group of contracts is, assigned as a single package.
ww all the contracts put together forms part of a single project giving an overall profit margin.
ww all the contracts individually are interdependent on each other.
ww all such contracts can be performed simultaneously (concurrently) or consequently (one after
the other). Hence construction of the total super bazaar including canteen etc. is a single contract.
Problem 3: Western Developers undertook a contract for improvement and development of chains of
godowns for government. Information about the project is given below: Year I: Basic initial agreed price
` 500 lakhs, construction costs incurred ` 130 lakhs, expenses not related to contract ` 2 lakhs, Prudent
estimates for the expenditure is ` 270 lakhs. Year II: Claim accepted by contractee (all conditions are fulfilled)
` 10 lakhs, incentives ` 10 lakhs (not yet certain), construction costs already spent upto date ` 200 lakhs,
costs yet to be incurred ` 205 lakhs. Contractee has given the contractor advances at the beginning of the
contract of ` 60 lakhs. It will be adjusted equally over 4 years against progress billings. Cash payments
made by contractee to Western Developers is: 95 lakhs (yr1), 45 lakhs (yr2) and remaining in 4th year of
completion after adjusting advances. The Progress billings are 120 lakhs and 200 lakhs (cumulative) in the
first and second year respectively.
Draw P/L, Contract A/C and balance sheet for 2 years only (in extract form).
Solution:
Steps Particulars Year I Year II
Step 1 Costs to date 130 200
Step 2 Future Costs 270 205
Step 3 Total Costs 400 405
Step 4 % of Completion 32.50% 49.38%
Step 5 Contract Price × % 162.50 (500 × 32.5%) 215.84 (510 × 49.38%)
Step б Revenue for the year 162.50 89.34 (215.84 – 162.5)
Step 7 Cost for the year 130 70
Step 8 Profit/Loss 32.50 19.34
AS-7 (Revised): Construction Contracts 63

Contract Account
Yr1 Yr2 Yr1 Yr2
To Contracts exps 130 70 By Contract revenue 162.50 89.34
To P/L 32.50 19.34

General P/L
To Expenses 2 – By Contract P/L b/d 32.50 19.34
To Net P/L c/f 30.50 19.34

Balance Sheet (extracts)


Liabilities Yr 1 Yr 2 Assets Yr 1 Yr 2
Share Capital — Gross amt due from customers (WIP)# 42.50 51.84
Reserves (Net) 30.50 49.84 Net debtors ($) 10 30
Gross amt due to customers (WIP) — — Cash bal * 23 —
Advances 45 30
Bank o/d * 2
75.50 81.84 75.50 81.84
# 42.50 = aggregate of costs incurred 130 + profits/losses recognised 32.50 Less: Progress billings 120.
# 15.84 = 200 + 32.50 + 19.34 Less 200.
$ Net debtors = Progress billings – Advance adj – cash recd = 10 = 120 – 15 – 95; 30 = 200 – 30 – (95 + 45)
* cash bal/bank o/d = advances recd + fresh cash from contractee – Expenses = 23 = 60 + 95 – 130 – 2; (2) = opening balance
+ fresh cash recd – Expenses = 23 + 45 – 70.

Problem 4: Pakshatru engineering Company, undertook a contract for construction of a small cannel.
Following are the expenses incurred and other details for the year 2005 provided to you:

Particulars ` Other details:


Opening Work in progress (excl. materials) 65,00,000 1) Materials on site totalled ` 2,75,000.
Opening Stock of Materials on site 2,00,000 2) 50% of the administrative expenses is identified and
Materials purchased 14,00,000 reimbursed by the contractee.
Overheads 10,50,000 3) The supervisor has devoted ½ of his time for this
Direct Labour 12,00,000 contract and balance ½ on another contract.
Sale of materials 25,000
Depreciation 7,00,000
Administrative Expenses 9,00,000
Research & development expenses 50,000
Cost of Materials lost by theft 4,000
Supervisors Salary 84,000

Compute Cost of the contract for the year 2005 and also state the amount transferred to General profit and
loss account.
64 Accounting Standards

Solution:
Particulars ` Reason for inclusion
Opening Work in progress 65,00,000 Opening Cost b/d.
Materials Consumed (See WN) 12,96,000 Direct Cost
Overheads 10,50,000 Direct Cost
Direct Labour 12,00,000 Direct Cost
Depreciation 7,00,000 Direct Cost
Administrative Expenses 4,50,000 Allocated and identified expenses
Supervisors Salary 42,000 Allocated and identified expenses
Total 1,12,38,000
* Materials Consumed = 2,00,000 + 14,00,000 – 25,000 – 4000 – 2,75,000 = ` 12,96,000

Costs transferred to General/Profit and Loss Account


Particulars `
Administrative Expenses 4,50,000
Cost of Materials lost by theft 4,000
Research and development expenses 50,000
Total 5,04,000

Problem 5: Ummagudu Limited provides the following details related to a construction of project:
2013-14 2014-15 2015-16
Cost incurred till date 20,00,000 27,00,000 57,00,000
Estimated cost on prudent basis 30,00,000 27,00,000 -
Revenue
Initial agreed price 66,00,000 - -
Claims - 4,00,000 -
Incentives - - 2,00,000
Progress billings 22,00,000 39,00,000 70,00,000
Cash receipts 11,00,000 30,00,000 68,00,000

Adjustments: Claims raised were immediately accepted by the contractee. The probability of receiving the
incentives was very low as the work was not completed on time.
Calculate Profit / Loss every year. Also list the disclosures as per AS-7 (Rev)
Solution:
Profit / loss:
2013-14 2014-15 2015-16
Cost incurred till date.............a 20,00,000 27,00,000 57,00,000
Estimated cost on prudent basis......b 30,00,000 27,00,000 -
Total Costs = ...................a+b = c 50,00,000 54,00,000 57,00,000
Stage of Completion % = a / c 40% 50% 100%
Revenue 66,00,000 70,00,000 70,00,000
Cumulative Revenue = 26,40,000 35,00,000 70,00,000
Less: Revenue already recognised Nil (26,40,000) (35,00,000)
AS-7 (Revised): Construction Contracts 65

Revenue for the year 26,40,000 8,60,000 35,00,000


Costs for the year 20,00,000 7,00,000 30,00,000
Profit for the year 6,40,000 1,60,000 5,00,000

Gross amount due to / from customers:


Aggregate of costs incurred plus profits recognised less 26,40,000 35,00,000 70,00,000
losses recognised
Less: Progress Billings (22,00,000) (39,00,000) (70,00,000)
Gross amount due 4,40,000 (4,00,000) Nil

Retention:
Progress billings 22,00,000 39,00,000 70,00,000
Less: Cash received (11,00,000) (30,00,000) (68,00,000)
Net Retentions 11,00,000 9,00,000 2,00,000

Disclosures as per AS-7


Contract Revenue 26,40,000 8,60,000 35,00,000
Contract Costs 20,00,000 7,00,000 30,00,000
Gross amount due to / from customers 4,40,000 (4,00,000) Nil
Retention 11,00,000 9,00,000 2,00,000
Advances received - - -

Problem 6: On 01.12.2005 Vishwakarma Construction Co. Ltd. undertook a contract to construct a building
for ` 85,00,000. On 31.03.2006 the company found that it had already spent ` 64,99,000 on the construction.
Prudent estimates of additional cost for completion was for ` 32,01,000. What is the additional provision
for foreseeble loss, which must be made in the final accounts for the year ended 31.03.2006 as per AS-7
(Revised)? Also draw extract of P/L?
(C. A. Final Nov. 2003, May 2010, May 2011) (Advanced Accounts Marks 5)
Solution: When it is probable that the total contract costs exceeds contract revenue the difference of expected
loss has to be charged to P/L account. 100% expected loss has to be accounted for. Such expected loss has to
be expensed irrespective of:
◘◘ progress payments received, % of work completed, profits on other contracts.
Expected loss to be expensed = (Cost incurred to date + Estimated cost to complete) – Total Revenue
= (64,99,000 + 32,01,000) – 85,00,000 = ` 12,00,000.
Hence Vishwakamia Construction Co. Ltd. should expense loss of ` 12,00,000 for the year 31.03.2006. Out
of which Current Year loss = 8.04 lakhs and Provision = 3.96 lakhs. (See note below)
Cost to date 6499000
+ Estimated costs + 3201000
Total Costs 9700000
Stage of completion 67%
Total Contract Revenue 8500000
Contract Revenue for the year 5696000
Contract Costs for the year (6499000)
Loss = 804000
66 Accounting Standards

Problem 7: Gas Ltd. has won a contract to construct and operate a gas pipeline. Gas Ltd. will construct the
pipeline and the associated infrastructure necessary to operate it. It will then operate it for 20 years. Gas Ltd.
will receive fess under the contract over 20 years. It will receive reimbursement of construction costs over
the five years following construction plus annual fee over 20 year period. Consequently, the profit that Gas
Ltd. will earn for the construction of pipeline is included in the annual fees it will receive. How should Gas ltd.
recognize the fees it will receive?
Solution: Gas Ltd has to split up the income received into two: (i) Construction of pipe line and (ii) Operation
of pipe line. The income earned from Construction of pipe line will be accounted as per AS-7 and the income
earned from Operation of pipe line will be accounted as per AS-9 over the period of 20 years.
Problem 8: Your client a contractor has just entered into a contract with the local municipal body for
building a flyover. As per the contract terms, your client will receive an additional ` 5 million if the flyover is
constructed within a period of two years. Your client wants to recognize this revenue since in the past it has
been able to meet these targets very easily. Advise your client keeping the revised AS 7 in mind.
(CA Final May 2007 Advanced Accounts)
Solution: According to para 14 of AS 7 (Revised) ‘Construction Contracts’, incentive payments are additional
amounts payable to the contractor if specified performance standards are met or exceeded. For example, a
contract may allow for an incentive payment to the contractor for early completion of the contract. Incentive
payments are included in contract revenue when: (i) the contract is sufficiently advanced that it is probable
that the specified performance standards will be met or exceeded; and (ii) the amount of the incentive
payment can be measured reliably. In the given problem, the contract has not even begun and hence the
contractor (Mr. X) should not recognize any revenue of this contract.
P roblem 9: Regal reconstruction Co undertook a contract on 1/4/2009. The contract revenue was decided
as reimbursable costs plus 20%. Costs incurred during the year 2009-10 was ` 3,25,000, of which ` 3,00,000
was reimbursable. Show P/L A/C.
Solution:
Contract Account for 2009-10
Particular ` Particular `
To Contract exp 3,25,000 By Contract Revenue 3,60,000
To Contract profit 35,000

Problem 10: Following is the information relating to Contract AQSAA Limited for 2008 – 2009 which is the
first year of contract:
Particulars ` in lacs
Costs to date 910
Estimated future Costs yet to be incurred 545
Total Contract Price 1,700
Progress billings 757
Progress payments recd 700

Draw extract of contract P & L a/c and also draw summarized balance sheet as per AS-7(Revised).
Solution: First check TC > TR if no else proceed further
Particulars ` lakhs
Costs to date (WC + W UC) 910
Future Costs 545
Total Costs 1455
Percentage (%) of Completion 62.5%
AS-7 (Revised): Construction Contracts 67

Contract Price x % (1700 x 62.5%) 1063


Costs incurred (910)
Profit for the year 153
Contract P/L A/C
To Contract Costs 910 By Contract Revenue 1063
To Net Profit 153

Balance Sheet as on 31.3.2009


Liabilities Assets  
Net profit 153 Due from customer – WIP 306
Cash receivable (Net) 57

Due from customer = 1063 (cost + Profit) – 757 (progress bill) = 306
Net customer = Progress billing – Cash recd – Advance adj = 757 – 700 = 57
Problem 11: PRZ &Sons Ltd are Heavy Engineering contractors specializing in construction of dams. From
the records of the company, the following data is available pertaining to year ended 31st March, 2012. Using
this data and applying the relevant Accounting Standard you are required to;
(i) Compute the amount of profit/loss for year ended 31st March, 2012.
(ii) Arrive at the contract work in progress as at the end of financial year 2011-12.
(iii) Determine the amount of revenue to be recognized out of the total contract value.
(iv) Work out the amount due from/to customers as at year end.
(v) List down relevant disclosures with figures as per relevant Accounting Standard.
(` Crore)
Total Contact Price 2,400
Work Certified 1,250
Work pending certification 250
Estimated further cost to completion 1,750
Stage wise payments received 1,100
Progress payments in pipe line 300
(CA Final May 2012)
Solution: First check TC > TR (Total Cost > Total Revenue)
Particulars (` in cr)
Costs to date (WC + W UC) (a) 1500
Future Costs (b) 1750
Total Costs (c) 3250
Total Revenue 2400
Total loss to be recognized as per para 35 (850)
Let’s compute current year loss
Stage of Completion 46.15%
Contract Revenue = (2400 × 46.15%) 1107.6
- Costs incurred –1500
Loss for the year –392.4
68 Accounting Standards

Answers:
i) P/L for the year -392.4
ii) WIP (WC+WUC) 1500
iii) Contract revenue 1107.6
iv) Gross amt due to/from customer
Aggregate of costs incurred 1500
Less: Loss recognized –850
WIP (total) = 650
Less: Progress billings 1400
Amt due to customer = (liability) 750
Disclosures as per AS-7 para 41
Contract revenue recognized 1107.6
Contract costs incurred and recognized Profit and Loss 650
Net Debtors 300
Gross amount due to customers for contract work: (liability) 750
CHAPTER 9
AS-9: Revenue Recognition

Chapter Outline
vv Objective and Scope vv Revenue Recognition in Case of Sales of Goods
vv Revenue Recognition in case of vv Principles of Revenue Recognition in
Interest/Dividend/Royalty Case of Rendering of Services
vv Effect of Uncertainties on Revenue Recognition

Objective and Scope


The standard includes income from ordinary activities from sales of goods, services rendered, employment
of resources by company yielding dividend, interest and royalty.
Revenue does not include: Revaluation in the value of assets, unrealized gains from current assets,
realized or unrealized gains from foreign exchange transactions, realized gains from retirement of a liability
at less than carrying amount.

Applicability
Revenue recognition (AS-9) is not applicable to the following areas (as they are covered by another accounting
standard):
◘◘ Revenue from Construction contracts,
◘◘ Hire purchase
◘◘ Leasing
◘◘ Government grants and subsidies
◘◘ Insurance Contract
◘◘ Realized and unrealized gains on sale of fixed assets

Definition of Revenue
It is the gross inflow of cash or any other consideration to be received or receivables in the ordinary course
of business in the form of:
(i) Sales of goods.
(ii) Services rendered.
(iii) Employment of resources by company yielding dividend, interest etc.
Only gross inflow is revenue. Savings in cost is not revenue.
70 Accounting Standards

Example: Trade Discounts are savings in costs and cannot be disclosed separately. Such discounts have to be
deducted from the purchases. For a Commission agent revenue is the commission on the sales and not the
gross inflow.

Common Principles of Revenue Recognition or Principles of Revenue Recognition or


Effect of Uncertainties on Revenue Recognition: (Nov 2012 Audit)
(i)
Revenue should be measured.
(ii)
The amount to be received must be certain.
(iii)
No uncertainty should exist in the revenue.
(iv)
If the uncertainty is temporary then recognize the revenue.
If the uncertainty is permanent then, postpone the revenue. Example: It may be uncertain that a
(v)
foreign governmental authority will grant permission to remit the consideration from a sale in a
foreign country. In such case revenue is not recognized. When the permission is granted, the
uncertainty is removed and then the revenue can be recognized.
(vi) If the revenue is recognized and in future suppose uncertainty exist for ultimate collection, then
instead of debiting sales make the provision for loss in the form of bad - debts.
(vii) When the ability to access the ultimate collection with reasonable certainty is lacking at the time
of raising any claim. For example: For Escalation price, Export in incentives, revenue recognition is
postponed to the extent of uncertainty involved.
(viii) Sometimes the uncertainty may be limited to determination of exact amount of the revenue. This
type of uncertainty should not result in non – recognition of entire amount of revenue. In such case
minimum receivable revenue should be recognized. Example: In case of fertilizer companies revenue
is recognized relating to subsidy on estimated basis.

Revenue Recognition in Case of Sales of Goods


Special conditions:
(i) The seller should transfer significant risk and rewards to the buyer.
SPECIAL DISCUSSION ON
(ii) Ownership in the goods should be transferred. SALES RETURN
(iii) Willing, buyer and willing seller. PLEASE REFER SOLUTION
(iv) Consideration should be received or receivable. OF PROBLEM 5
Transfer of risks and rewards – prerequisite to the sale of goods:
One of the most important condition for sale is that there should be a transfer of substantial risks and
rewards related to the goods and the seller should not have any effective control related to the ownership of
goods. In most of the cases transfer of goods should coincide by the transfer of risks and rewards related to
the goods. In some cases where transfer of risks and rewards related to the goods do not coincide with the
transfer of goods then sale is to be assumed when the transfer of risks and rewards takes place.
If suppose lorry receipt is in the name of the seller and endorsed to the buyer and the seller can cancel
it any time during transit then the seller still retains effective control on the goods. In this case transfer of
ownership takes place when the buyer actually receives and approves the goods However it is also possible
that the sale agreement specifies that the risks and rewards is transferred when the seller hands over the
goods to the transporter. In this case sales takes place when the goods are handed over to the transporter
though the seller may retain the lorry receipt (LR) as a measure of security for non – payment by the buyer.

Special Cases in Sales of Goods


Case 1: Goods sold subject to installation and inspection: Sale is to be recognized when the installation
is completed and the buyer accepts the goods. But if the approval is expired and buyers do not reply within
AS-9: Revenue Recognition 71

that period then the seller can assume the sales. Sometimes installation process may be very simple, in that
case revenue can be recognized when goods are delivered to the buyers place (ex: unpacking a television and
connecting power).
Case 2: Consignment Sale: Date of transfer of stock is not important but sale made by agent is important.
Case 3: Warranty Sales: In case of warrantee sales, the revenue is to be recognized after making warrantee
provisions. Such provision is based on the past experience of trend of cost of returns and repairs.
Case 4: Guarantee Sales: In case of unlimited right of return (money back if not completely satisfied) the
revenue is to be recognized after making a suitable provision for returns based on past experience.
Case5: Cash on delivery: Revenue is recognized when the seller or his agent receives cash. Ex: A Grocery
delivers goods at the door step of the buyer against cash.
Case 6: Special order shipment: These are fixed readymade contracts in which revenue is to be recognized
on the date of goods are manufactured and ready for delivery of goods and not on the date of goods reached
the buyers place.
Case 7: Subscription for Publications: In case of publication one has to see whether the publication is
repetitive or of different sale value. If it is repetitive then recognize the sale as per SLM basis. And if it is
fluctuating in nature then recognize on sale value method.
Case 8: Delivery delayed at buyer’s request: Revenue is to be recognised if all the conditions are satisfied:
◘◘ Goods are delayed at the buyers request and not at the seller’s request.
◘◘ Goods are ready for delivery.
◘◘ Buyers accepts the billing and title of goods is transferred by the seller to the buyer.
Case 9: Repo Agreements: Repo agreements are agreements where the seller repurchases the goods at
a later date. Such agreements are not covered by AS – 9 because it is a finance transaction (Substance over
form). Hence the sales realisation is to be treated as an advance (liability).
Case 10: Installment sales: Only sales part is to be recognized as revenue on the date of sale and not the
interest. Interest is to be spread over the unpaid balance in the ratio of unpaid balance.
Case 11: Goods are delivered only when final payment is received: Revenue is not to be recognized
until the final payment is received. But if a significant portion of deposit is received then revenue may be
recognized based on past experience.

Principles of Revenue Recognition in Case of Rendering of Services:


Major part of the revenue is even generated from Services rendered. The Standard prescribes two methods :
Proportionate Completion Method (PCM) Completed Contract Method (CCM)
(1) In this revenue is recognized as and when services are (1) In this revenue is recognized after completing the
rendered. Revenue is recognized proportionately by whole project/act.
reference to the performance of each act.
(2) Number of acts are repetitive or in more frequency. (2) Number of acts may be one or hardly 2-3.
(3) It is based purely on accrual concept. (3) It is against accrual concept.
(4) ICAI favours such method and even internationally (4) ICAI has omitted this method of accounting in
accepted. some cases.
(5) The revenue recognized under this method would be (5) In this revenue is recognized on the completion of
determined on the basis of contract value, associated the final act.
costs, number of acts.
(6) Example: Constructions projects, Tuition Fees. (6) Example: Concerts, banquets, Media commission
72 Accounting Standards

DO YOU KNOW
If consultancy services like project manager services not related to construction contracts then they are
covered by AS-9, but completely devoted to the projects are covered by AS-7.

Special Cases on Rendering of Services


Case 1: Media Advertisement Commission (CCM): In case of advertisement revenue, recognize the
revenue only when the related add is displayed in front of the public, irrespective of the advance received.
Case 2: Advertisement Commission (PCM): This is a commission in relation to the production of
advertisement. Revenue will be recognized based on stage of completion.
Case 3: Financial Services (CCM as well as PCM): In case of financial services determine the certainty of
payments & the nature of transaction. For loan arrangement process recognize revenue on receipt of fees
and in case of loan management recognize revenue during the life of the loan. For ex: - Loan processing fees
can be recognized immediately. Where the service is provided on a continuous basis then fees may even be
spread over the life of the loan.
Case 4: Insurance Commission (CCM): It is to be recognized on the effective date of commencement or
renewal of policies.
Case 5: Admission fees (CCM): Admission fees are to be recognized when the whole act is completed. (CCM).
The act may comprise of any artistic act or any type of special event.
Case 6: Installation fee (CCM): It is to be recognized when the whole installation is completed and the
customer has accepted the installation.
Case 7: Tuition fees (PCM): It is to be recognized over the period of instruction.
Case 8: Membership fees or entrance fees (CCM): It is to be recognized when membership fees is received.
The entrance fees is to be capitalized and added to the corpus.
Case 9: Membership fees for services rendered (PCM): Fees received for services rendered during the
year should be allocated on a suitable rational basis.
Case 10: Commission income (Principal agency relationship) (CCM): An agent has to recognize the
commission received on sales and not the proceeds of sales turnover as Revenue. Similarly commission on sale of
tickets is revenue and not sale of tickets for an agent.
Case 11: Freight on incomplete voyage (PCM as well as CCM): It is recognised when the voyage is
completed. Even one can record revenue (freight) on pro–rata basis for voyage-in-progress also as on the
balance sheet date.
Case 12: Revenue from internet (PCM as well as CCM): It is recognised based on the usage time utilised
on the internet.

Principles of Revenue Recognition in Case of Companies Employing Resources


In this case, the company may have some idle assets like cash, vacant land etc. Such assets are employed to
earn rent, royalty, interest or dividend.
Case 1-Interest: It is based on time proportion. It is simply the rate of interest applied on the outstanding
balance of the principal amount.
Case 2-Royalty: It is the charges for use of assets. Recognition of revenue is based on agreement entered
into. But one has to see the substance of the transaction for recognizing revenue. Ex: Suppose Dr. Thomas
wrote a book Harry Putter and gave the publishing rights to Cawasji Publishers. They agreed a royalty of
` 60 per book. If on March ending 200 books are sold. Dr. Thomas can record a Royalty income of 60 × 200 =
AS-9: Revenue Recognition 73

` 12,000 as revenue. But Royalty income is subject to a condition then such condition should get fulfilled to
recognise revenue.
Case 3 - Dividend: When the owners right to receive dividend is established (AGM held is a enough evidence
for recognizing revenue). If the right to receive dividend is established subsequent to the balance sheet
date the dividend income is recognized in the subsequent year. In case of interim dividend actual receipt of
dividend is the date of recognition of revenue.

Disclosures in Revenue Recognition


◘◘ Postponement of Revenue.
◘◘ Sales should be disclosed in the form of : Gross Sales – Excise Duty = Net Sales (even mandated by New
Schedule VI, also refer ASI-14).

Few Important Points in AS-9


◘◘ If the interest income is ballooned then as per the substance interest should be accounted on effective
rate of interest basis rather then as per the loan agreement.
◘◘ Whenever any revenue is received for manufacturing complex plant or equipment then it is to be
accounted as per AS-7 and not as per AS-9.
◘◘ Insurance claims are not covered by AS-9 because they are themselves not exactly measurable.
Insurance claims can be recognized when the amount thereof can be measured reliably and ultimate
collection is reasonably certain.
◘◘ Cash rebate is to be recognized when cash rebate is received and not when sales are made.
◘◘ Export incentives should not be added to the turnover but shown as an operating income.
It is to be recognized when exports are made and the exporter becomes entitled to such benefit.

PROBLEMS AND SOLUTIONS

Problem 1: Total sales of a company included a sum of `4,00,000 representing royalty receivable for the
supply of know – how. The amount is payable in dollars. However, exchange permission denied the remittance
of currencies due to some problems among inter – countries. Comment.
Solution: The basic condition for RR is that no uncertainty should prevail at the time of recognising the
revenue. In the current case ` 4,00,000 which is a royalty income is received in $. The nature of uncertainty
has to be studied and based on the fact the revenue should be recorded. In the present case there is a problem
in the inter – countries and not inter-companies hence the nature of the problem has to be studied.
In the present case the amount of ` 4,00,000 is already included in the revenue. This means when the
revenue accrued was recorded there was no uncertainty. A provision now is to be made for covering the
uncertainty.
Problem 2: M Ltd. deals in manufacture parts for Steel Plants. It quotes prices in various tenders issued
by Steel plant companies. As per terms of contract, full price of machinery is not released by the steel plants
companies, but 10% is retained. The company accounts for 90% of the invoice value as revenue and balance
10% on receipt basis. Advise.
(Final C.A. May 2002) (5 Marks)
Solution: AS-9 on ‘Revenue Recognition’, states that revenue from sale of goods should be recognised when
the seller of goods has transferred to the buyer the property in the goods for a price or all significant risks
74 Accounting Standards

and rewards of ownership have been transferred to the buyer and the seller retains no effective control of
the goods transferred to a degree usually associated with ownership and no significant uncertainty exists
regarding the amount of consideration. In the case of M Ltd., the goods as well as the risks and rewards of
ownership have been transferred to the steel plants. The invoice raised by M Ltd. is for the full price, but 10%
less is received as the same is kept as ‘Retention Money’. In this case, therefore, revenue has to be recognised
at the full invoice price, i.e., 100% has to be accounted as Sales Income. Depending on the past experience of
recovering the balance 10% from the steel plants, M Ltd., can, however, make a provision for sales income
which is not likely to realised. In the absence of the above, the auditor will have to qualify his report.
Problem 3: Advise to Pentagon
st
Ltd. about the treatment of the following in the final statement of account
for the year ended on 31 March, 2003.
A claim lodged with the Railways in March 2000 for loss of goods of ` 2,00,000 had been passed for
payment in March 2003 for `1,50,000. No entry was passed in the books of the company, when the claim was
lodged. (Final C.A. Nov. 1993) (2 Marks)
Solution: The company has not recorded the revenue in March 2000 when the loss took place because the
claim was not certain to receive. The company was correct by not recording the claim. Now in March 2003 the
company has received the claim (ultimate collection is certain) and revenue/claim can be recorded.
P roblem 4: Due to price revision with effect from 1st October 2005 the company has to receive ` 4,75,000
from its customers in respect of sales made in 2005-06. Give the treatment in the final accounts for the year
March 2006.
(Final C.A. Nov. 1993/May 1995) (2/4 Marks respectively)
*
Solution: If the claim is certain to receive the company can record the income.
Problem 5: A Ltd. is a manufacturer of readymade garments. It sells its products to franchisees located across the
country. Readymade garment industry is subject to change in trends of fashion and as such, some of the goods are
returned and A Ltd. accepts them back as sales returns. On the basis of past trends such returns are estimated to be
at 20% of the sales of any year. For the financial year 2011-12. A Ltd. had accounted for the actual sales return made
upto 31st March 2012 but has not reversed the possible expected return that are likely to happen after 31st March
2012, in respect of the sale made for the FY 11-12. Mr. X the auditor of A Ltd wants this to be considered in the accounts
for the year ended on 31st March 2012 but the company is of the opinion that although there is a probability of some
goods being returned by the franchisees, there is no significant uncertainty regarding the amount of consideration
that will be derived from the sale of goods, since the goods are not in the possession of the company and risk and
rewards of ownership still lie with the franchisees and the company cannot record sales returns in its books of
account in respect of goods that are likely to be received after the date of balance sheet. Comment.
(CA Final Audit May 2012) (Marks 5)
Solution: The basic issue raised in the query relates to accounting for sales returns that occur after the
date of balance sheet in a situation where franchisees have a right to return the goods. The committee also
presumes that franchisee is neither acting as an agent of the company nor as a consignee as covered under
paragraph A(2)(c) of the Appendix to AS 9. Further, it is presumed that the company has complied with
revenue recognition criteria as per AS 9.
There is a right of return by the franchisees and such right would give rise to a present obligation on the
company. The committee is of the view that obligation in respect of sales return can be estimated reliably
on the basis of past experience and other relevant factors such as fashion trend etc. Therefore, considering
the definition of provision as per Accounting Standard 29‐ ‘Provisions, Contingent Liabilities and Contingent
Asset’ (AS 29), provision in respect of sales returns should be recognised in the present case.
The provision should be measured at the best estimate of loss expected to be incurred in respect of expected
sales returns including incremental selling cost that would be necessary to resell the goods returned. Further,
provision should be reviewed at each balance sheet date and if necessary, should be adjusted to reflect current

*
Write some points on uncertainty of Revenue or Just refer Problem 3.
AS-9: Revenue Recognition 75

best estimate (paragraph 52 of AS 29). Committee is of the view that necessary adjustments should be made
in the amount of provisions for actual sales returns that occur between the balance sheet date and the date
of approval of financial statements.........AS-4.
The present policy of the company regarding recognition of sales return after the date of balance sheet
in the books of account only upon receipt of goods from the franchisees is not correct. The company should
recognize a provision in respect of sales returns at the best estimate of the loss expected to be incurred by
the company in respect of such returns including any incremental cost that would be necessary to resell
the goods expected to be returned. Necessary adjustment to the provisions should be made for actual sales
returns. Also provisions should be reviewed at each balance sheet date and if necessary should be adjusted
to reflect the current best estimates.
Problem 6: The BOD decided on 31/03/2006, to increase the SP of certain items retrospectively from
Jan 1 2006. In view of this price revision, with effect from 1/1/2006, the company has to receive `15,00,000 from
its customers, in respect of the sales made between the 3 months. Give the treatment in the final accounts for the
year March 2006. Final C.A. May 1996) (3 Marks)
*
Solution: If the claim is certain to receive the company can record the income.
Problem 7: Y Limited used certain resources of X Limited in return, X company Limited received
` 10,00,000 and `15,00,000 as interest and royalties respectively from Y Limited during the year 2005 – 06.
On what basis these revenues can be recognised by X Limited.
(Final C.A. May 1999) (3 Marks)
Solution: The income received by X Limited is covered by Employment of Resources yielding Royalty, Dividend
and Interest.
Case 1: Interest : It is based on time proportion. It is simply the rate of interest applied on the outstanding
balance of the principal amount. But as per AS-30 now interest is accounted for as per effective cost method.
Case 2: Royalty : It is the charges for use of assets. Recognition of revenue is based on agreement entered into.
Problem 8: TVSM company has taken a Transit Insurance Policy. Suddenly in the year 2012-13 the
percentage of accident has gone up to 7% and the company wants to recognise insurance claim as revenue in
2012-13 in accordance with relevant Accounting Standards. Do you agree?
(Final C.A. May 2003) (4 Marks)
Solution: AS 9 on Revenue Recognition defines revenue as ‘gross inflow of cash, receivables or other
consideration arising in the course of the ordinary activities of the enterprise from the sale of goods, from
the rendering of services and from the use by others of enterprise resources yielding interest, royalties and
dividends.
To recognise revenue AS 9 requires that revenue arises from ordinary activities and that it is measurable
and there should be no uncertainty. As per para 9.2 of the Standard, where the ability to assess the ultimate
collection with reasonable certainty is lacking at the time of raising any claim, revenue recognition is postponed
to the extent of uncertainty involved. In such cases, it may be appropriate to recognise revenue only when it is
reasonably certain that the ultimate collection will be made.
In the given case, TVSM company wants to suddenly recognise Insurance claim because it has increased
over the previous year. But, there are uncertainties involved in the settlement of the claim. Also, the claim
does not seem to be in the course of ordinary activity of the company.
Hence, TVSM company is not advised to recognise the Insurance claim as revenue.
Problem 9: SCL Ltd. Company sells agriculture products to dealers. One of the condition of sale is that
interest is payable at the rate of 2% p.m., for delayed payments. Percentage of interest recovery is only 10%
on such overdue outstanding due to various reasons. During the year 2002-2003 the company wants to
recognise the entire interest receivable. Do you agree? (Final C.A. May 2003) (4 Marks)

*
Write some points on uncertainty of Revenue or Just refer Problem 3.
76 Accounting Standards

Solution: As per para 9.2 of AS 9 on Revenue Recognition, where the ability to assess the ultimate
collection with reasonable certainty is lacking at the time of raising any claim, e.g. for escalation of price,
export incentives, interest etc, revenue recognition is postponed to the extent of uncertainty involved.
In such cases, it may be appropriate to recognise revenue only when it is reasonably certain that the ultimate
collection will be made. Where there is no uncertainty as to ultimate collection, revenue is recognised at the
time of sale or rendering of service even though payments are made by instalments.
Thus, SCL Ltd. cannot recognise the interest amount unless the company actually receives it. Hence, the
company is advised to recognise interest receivable only on receipt basis.
Problem 10: Victory Ltd. purchased goods on credit from Lucky Ltd. for ` 250 crores for export. The export
order was cancelled. Victory Limited decided to sell the same goods in the local market with a price discount.
Lucky Limited was requested to offer a price discount of 15%. The chief Accountant of Lucky Ltd. wants to
adjust the sales figure to the extent of the discount requested by Victory Ltd. Discuss whether this treatment
is justified. (Final C.A. Audit May 2000, FR May 2006, May 2015)
Solution: Lucky Ltd. had sold goods to Victory Ltd on credit worth for `250 crores and the sale was completed
in all respects. Victory Ltd s decision to sell the same in the domestic market at a discount does not affect
the amount recorded as sales by Lucky Ltd. The price discount of 15% offered by Lucky Ltd. after request of
Victory Ltd. was not in the nature of a discount given during the ordinary course of trade because otherwise
the same would have been given at the time of sale itself. Now, as far Lucky Ltd is concerned, there appears
to be an uncertainty relating to the collectability of the debt, which has arisen subsequent to the time of
sale therefore, it would be appropriate to make a separate provision to reflect the uncertainty relating to
collectability rather than to adjust the amount of revenue originally recorded. Therefore, such discount
should be written off to the profit and loss account and not shown as deduction from the sales figure.
Problem 11: Decide when and how much revenue can be recognised in the following situations.
1. AMC sells goods to XYZ every month. In May 2013, when AMC was about to deliver the monthly
consignment of goods, XYZ informs that Quick and Fast who provide delivery services to XYZ ceases
business. XYZ requests AMC to store the goods, at his risk, until he can find alternative transport.
2. Sabu Ltd. sold accounting software to Babu Ltd. via internet for ` 55,000. It was agreed that Babu Ltd.
can make payment on receipt of the software.
3. Solomon Cables gets a contract from a local telephone exchange for installing a telephone network in
10 identical buildings.
4. In September, Chinki decorators sell tickets for an exhibition for ` 7,00,000. The exhibition will take
place in December.
5. Ferrari Drive is a car club. To become a member of this club, a client has to pay ` 2,000 registration
fees and 3,000 annual membership fees. How should F Drive recognise its revenue?
Solution:
1. The goods are not physically delivered but the risk and returns in the goods are transferred by AMC
to XYZ. Hence AMC can record the revenue for the month.
2. Revenue can be recorded only on the receipt of `55,000 cash.
3. Solomon Cables can record 10% of revenue with every completion of work (it is a case of rendering
of services)
4. The revenue will be recorded in the month of December when the exhibition takes place. Entry to be
recorded in the month of September:
Cash ...Dr 7,00,000
To Advance revenue 7,00,000
Entry in December:
Advance revenue ...Dr 7,00,000
To Sales 7,00,000
AS-9: Revenue Recognition 77

5. Registration fees of `2,000 will be added in the corpus (Capital a/c.) and 3,000 annual membership
fees will be amortised equally over each month as revenue.
Problem 12: Sale of goods costing ` 54,000 with a profit margin of 10% on selling price is included in the
inventory as delivery of goods was postponed at buyer’s request.
(Final C.A. Nov. 1999, Nov. 2001) (5 Marks)
Solution: AS-9 on “Revenue Recognition”, states that, “a key criterion for determining when to recognise
revenue from a transaction involving the sale of goods in that the seller has transferred the property in the
goods to the buyer for a consideration”. The transfer of property in goods, in most cases, results in or coincides
with the transfer of significant risks and rewards of ownership to the buyer. Accordingly, the sale is complete
in the given instance and, thus, the inclusion of the goods sold in the inventory because of the deferment of
delivery of goods at buyer’s request is not in accordance with AS-9. Thus, ` 54,000 should be excluded from
inventory, and the amount of `59,400 should be included in sales with corresponding increase in Debtor’s
balances.
Problem 13: Faaltoo Limited entered into an agreement with Paltoo Limited for sale of goods costing
` 4,00,000 at (Cost + 20%). The sale transaction took place on 15th February, 2005. On the same day Faaltoo
Limited entered into another agreement with Paaltoo Limited for repurchasing the same goods at ` 5,40,000
on 15th August 2005. Pass entries in the books of both the companies for the year 31st March 2005.
(CA Final Nov. 2009, Nov. 2013) (5 Marks)
Solution: Please refer November 2013 FR Paper Question 3.
Problem 14: Excel Limited is a manufacturing company in consumer durable goods with an annual turnover
of ` 1,000 lakhs. The company receives orders from its commission agents all over the country, but goods are
dispatched directly to the customers. The documents including the transport bills are sent through the bank
for collection. At the end of 5th year, it is found that documents covering despatch of goods worth `20 lakhs
were still lying with the banks not cleared by the customers even though the normal collection period of
20 days from the date of dispatch has expired. Should revenue be recognized? (CA Final)
Solution: Revenue can be recognized if the seller of the goods has transferred to the buyer all the significant
risks and rewards attached to the ownership. Seller should not have any effective control over the goods. Also
there should not be any uncertainty for the ultimate collection.
In the given case since transport bills were sent through the bank for collection, it may be said that the
seller has retained effective control over the ownership. Further since the documents were not cleared by the
customers even after the expiry of the normal period of collection, there is an uncertainty in the realization
of sale proceeds. Hence, revenue should not be recognized.
Problem 15: Satyam Computechnique Limited appointed some franchisees to conduct training.
The institute earns franchisee fees of ` 7 crores in exchange of its brand name and technical assistance.
The company wants to recognize full franchisee fees upfront?
Solution: The franchisee fee is in the nature of royalty in exchange of a right to use an asset (brand) over
defined period of time. Such fees should be recognized on time proportion basis. However recognizing entire
fees upfront is unreasonable.
Problem 16: Media advertisers obtained rights for one day world cup cricket tournament to be held in
May/June 2009 for ` 250 lakhs. By 31/3/2009 they have paid ` 150 lakhs to secure these advertisement
rights. The balance of ` 100 lakhs was paid in April 2009. By 31/3/2009 they procured advertisement for
70% of the available for 350 lakhs. The advertisers paid 60% of the amount by that date. The balance 40%
was received in April 2009. Advertisement for the balance 30% time was procured in April 2009 for `150
lakhs. The advertisers paid the full amount while booking the advertisement. 25% of the advertisement time
is expected to available in May 2009 and the balance 75% in June 2009.
You are asked to pass journals for the month of: March, April, May and June 2009.
78 Accounting Standards

Solution:
31.3 Adv right bought Dr. 250 31.5 P/L A/c. Dr. 62.5
To Cash 150 To Adv right bought 62.5
To payable 100 (Expenses recognized)
(adv right bought) 250 x 25%
31.3 Cash Dr. 210 31.5 Adv right sold Dr. 125
Receivable Dr. 140 To P/L A/c. 125
To Adv right sold 350 (Income recognized)
(Collection of contract receipts) (350 +150) 25%
30.4 Payable 100 30.6 P/L A/c. Dr. 187.5
To Cash 100 To Adv right bought 187.5
(Balance paid) (Expenses recognized)
250 - 62.5
30.4 Cash Dr. 150 30.6 Adv right sold Dr. 375
To Adv. right sold 150 To P/L 375
(balance collections) (Income recognized)
500 - 125
30.4 Cash Dr. 140
To receivable 140
(Remaining balance rights
collected)

Problem 17: Gold Eagle Ltd, has been a successful jewellers for the past 100 years and sales are against
cash only. The Company diversified into apparels. A young senior executive was put in charge of Apparels
business and sales increased 5 times. One of the conditions for sales that dealers can return the unsold stocks
within one month of the end of season. Sales return for the year was 25% of sales. Suggest a suitable Revenue
Recognition Policy with references to AS-9. (CA Final Nov 08)
Solution : Revenue from jewellery business:
(a) Revenue from sale of jewellery article is recognized in the book as and when sold to buyers against receipt
of cash. b) sales return is recognized/recorded against cash refund, if any, as and when it arises.
(b) General revenue from sale of apparels is recognized in the books as and when sold, subject to the
following condition - Create a provision of 25% of the value of goods for one month.
Goods supplied on sale or on return basis: revenue is recognized in respect of such supply if and only if-
The buyer of the goods has communicated his acceptance of the goods so supplied, or the buyer of the goods
has not returned the goods within 1 month from the date of supply.
Problem 18: A Ltd prepared an invoice for an export consignment on FOB basis on 30th March 2010.
The goods were dispatched from the factory on 30th March 2010 and bill of lading was made on 3rd April
2010. A Ltd had booked the invoice in the Sales Register for March 2010.
(CA Final Audit May 2010 marks 5)
Solution: As per AS-9 Revenue Recognition revenue on sale of goods should be recognized only when
significant risk and rewards of ownership have been transferred to the buyer. Normally in FOB sales, the risk
passes on from the seller to the buyer at the point when the goods have been loaded and placed on board the
ship at the shipping port. At this point, the bill of lading is prepared.
Since in the given case the bill of lading has been made out only on 3 rd April 2010 the risk has been
passed only on that date. As such, sales should be booked only for 2010 – 11 and not 2009-10.
CHAPTER 10
AS-10: Accounting for
Fixed Assets

Chapter Outline
vv Some Definitions vv Special Discussions on Few
vv Valuation Aspect Important Aspects of AS-10
vv Disclosures in AS-10

Some Definitions
Fixed Asset
Fixed Asset is an asset held with the intention of being used for the purpose of producing or providing good
or services and is not held for sale in the normal course of business.
Example: Land and Building, Plant and Machinery, Furniture and Fitting; etc.
Fair Market Value: It is the price at which knowledgeable and willing parties deals at arm’s length price who
are fully informed and are not under any compulsion to transact.

DO YOU KNOW
The provisions relating to borrowing costs, intangible assets and leases that were originally contained in this
standard were withdrawn once new AS were developed in these areas?

At what Value should Any Asset be Shown?


Valuation of Fixed Asset: Fixed Assets can be shown in the Financial Statements at
i. Historical Cost
ii. Revalued Price

Historical Cost
Cost of an item of fixed asset comprises of its purchase price, including import duties and other non refundable
taxes or levies and any directly attributable cost of bringing the assets to its working condition (even trial run
expenses are to be added).’
80 Accounting Standards

Particulars Amount
Purchase Price XXX
(+) Carriages XXX
(+) Professional fees XXX
(+) Import Duties (always as a % of Purchase price) XXX
(+) Site preparation costs XXX
(+) Professional Fees (fees of architect) XXX
(+) Handling charges XXX
(+) % of Allocated cost XXX
(Administration & general expense) XXX
(+) Designs & know how XXX
(+) Interest costs (if allowed by AS – 16) XXX
GROSS VALUE: XXX
(-) Government grants (AS-12) (XXX)
(-) Trade Discount (%) (XXX)
(-) Modvat credit (refundable taxes) (XXX)
CURRENT COST: XXX
(+)/(-) Price fluctuation adjustment XXX
(+)/(-) Forex Adjustment (AS-11) XXX
HISTORICAL COST XXX
XXX

Notes:
• Cost incurred after the asset is put to use is to be excluded.
• In case of assets manufactured in house all direct costs incurred are to be taken excluding internal
profits included in the cost.
• Administrative and selling expenses are to be excluded.

Costs under Different Circumstances


(i) Self Constructed fixed assets: Materials + Labour + Overheads directly attributable towards
construction. Exclude any internal profits. Under the New Schedule III there is a separate line – item
named “Capital W-I-P”.
(ii) Fixed assets exchanged with another asset: Fair market value of the asset given up if it is more
evident otherwise, FMV of the asset acquired.
(iii) Cost of asset acquired in exchange of shares or other securities: Market value of the shares or
securities issued.
(iv) Cost of asset acquired through Government grants: Concessional price paid or if it is free of cost
then nominal value.

Assets Acquired under Special Case


(i) Cost of jointly held assets: Pro-rata cost of such jointly owned asset is grouped together with similar
fully owned assets. Depreciation should be charged on the proportionate share of asset only.
(ii) Cost of assets acquired under consolidated basis: Cost of each fixed asset should be determined
on a fair basis as per valuation by competent valuation officer.
AS-10: Accounting for Fixed Assets 81

(iii) Asset acquired on hire purchase basis: Such assets are recorded at their cash price. Interest
element is to be apportioned over the period of the life of asset.

Treatment of Improvements and Disposals


Improvements and Repairs
Frequently, it is difficult to determine whether subsequent expenditure related to fixed asset represents
improvements that ought to be added to the gross book value or repairs that ought to be charged to the profit
and loss statement.

Enhancing the Economic Benefits and cannot Exist Independently


Only expenditure that increases the future benefits from the existing asset beyond its previously assessed
standard of performance is included in the gross book value, e.g., increase in capacity. Hence, the cost of
addition or extension to the existing asset which is of a capital nature and which becomes an integral part of
the existing asset is usually added to its gross book value.

Separately Identifiable
Any addition or extension, which has a separate identity and is capable of being used after the existing asset
is disposed of, is accounted for separately e.g: Body of an aircraft and its engine.

Retirements and Disposals


When the fixed asset is disposed off, it will be deleted from the Financial Statement.

If Assets are carried at historical cost:


◘◘ Gains or losses arising on disposal are generally recognized in the statement of profit and loss in the
same year in which asset is disposed off.
◘◘ Items of fixed assets that have been retired from active use and are held for disposal are stated at
the lower of their net book value and net realizable value and are shown separately in the financial
statements.
◘◘ Any expected loss is recognized immediately in the profit and loss statement.

If Assets are carried at revalued price: But, if asset was previously revalued then the profit and loss on
sale should be first adjusted against revaluation reserves and then the balance is to be transferred to profit
and loss account. Balance in revaluation reserve after the sale of asset shall be closed by transferring it to the
general reserve account.

Valuation Aspect
Valuation of Fixed Assets Retired from Active use and held for Disposal
Such asset is stated at the lower of net book value and the net realizable value in the financial statement.

DO YOU KNOW
Assets held for disposal under the new Schedule VI will be disclosed under the head current assets (other
current assets).

Valuation of Fixed Assets in Use (Revaluation)


i. Revaluation is the excess of market value over and above the carrying amount.
82 Accounting Standards

ii. Revaluation profit is not revenue profit hence it cannot be used for declaring dividend or issue of
bonus shares (SEBI Guidelines).
iii. Whenever fixed assets are revalued, such fixed assets are shown at revalued price in the financial
statement:
ww Competent valuers do valuation
ww For entire class of asset/s selected on systematic basis, the basis should be disclosed
ww Revaluation of fixed assets should be restricted to the Net Recoverable Amount.
iv. Accounting treatment on revaluation
First time revaluation: (Upward revaluation)
Asset A/c ...Dr.
To Revaluation Reserve A/c
First time revaluation: (Downward revaluation)
Profit and Loss A/c ... Dr.
To Asset A/c
First time Upward revaluation and then subsequently downward
At the time of Upward revaluation initially pass entry as discussed above:
Then for subsequent downward revaluation pass the entry as:
Revaluation Reserve A/c ... Dr. (Utilise fully)
Profit and loss A/c ... Dr. (balancing figure)
To Asset A/c
First time downward revaluation and then subsequently upward revaluation
At the time of downward revaluation initially pass entry as discussed above
Then for subsequent upward revaluation pass the entry as:
Asset A/c ... Dr.
To Profit and loss A/c Dr. (restricted to the amount debited previously)
To Revaluation Reserve A/c Dr. (balancing figure)

Disclosures in AS-10
The following information should be disclosed in the financial statements:
(i) Gross and net book value of fixed assets at the beginning and end of an accounting period showing
additions, disposals, acquisition and other movements
(ii) Expenditure incurred on account of fixed assets in the course of construction or acquisition; and
(iii) Revalued amounts substituted for historical costs of fixed assets, the method adopted to compute
the revalued amounts, the nature of indices used, the year of any appraisal made, and whether an
external valuer was involved, in case where fixed assets are stated at revalued amounts.

Special Discussions on Few Important Aspects of AS-10


(i) Stand by Equipment
ASI-2 also contains provision relating to stand-by equipment. The Stand-by equipments is a separate fixed
asset in its own right and should be depreciated like any other fixed asset. Stand-by equipment is not of the
nature of a spare but is of the nature of another piece of equipment which is being used in the manufacturing
process, for example, a generator set kept in store as a stand-by to the generator set which is being used
in the manufacturing process. Therefore the stand-by equipment is a separate fixed asset in its own right
and is depreciated like any other fixed asset. Depreciation on the generator should start immediately on its
purchase.
AS-10: Accounting for Fixed Assets 83

(ii) Special Tools, Jigs and Fixtures


Special tools, jigs and fixtures, are assets as these objects carry probable future benefits. The jigs and fixtures,
on the other hand, are not consumed in the production of goods; their life span varies from 18 months to 36
months, and they can be modified for alternative use. Thus, they are meant for use in the production process
over a longer period and are, therefore, of the nature of fixed asset.
Schedule XIV to the companies Act, 1956, classifies moulds, patterns, dies and templates, and wooden
lasts used in the manufacture of shoes as items of plant and machinery. On this basis, jigs and fixtures fall
within the head ‘plant and machinery’ under Schedule VI to Companies Act, 1956.

(iii) Accounting of MODVAT on Capital Goods


AS-10 on ‘Accounting for Fixed Assets’ states, inter alia, in para 9.1, as follows; “The cost of an item of fixed
asset comprises its purchase price, including input duties and other non-refundable taxes or levies and
any directly attributable cost of bringing the asset to its working condition for its intended use; any trade
discounts and rebates are deducted in arriving at the purchase price”. MODVAT credit can be considered to
be of the nature of a refundable tax. Therefore, MODVAT credit should be reduced from the purchase cost
of capital goods concerned. In view of the above, the specified duty on capital goods should be debited to
separate account, e.g., MODVAT Credit Receivable (Capital Goods) Account.
On actual utilization, the account will be adjusted against excise duty on final products. Accordingly, the
purchase cost of the capital goods would be net of the specified duty on capital goods. The unadjusted balance
standing in the MODVAT Credit Receivable (Capital Goods) Account, if any, should be shown on the assets side
under the head ‘advances’. MODVAT credit in respect of capital goods should be recognized in the books of
account when the following conditions are satisfied:
(i) The enterprises is entitled to the MODVAT credit as per the Rules.
(ii) There is a reasonable certainty that the MODVAT credit would be utilized.

(iv) Start-up and Commissioning Cost


The expenditure incurred on start-up and commissioning of the plant, including the expenditure incurred on
test runs and experimental production is usually capitalized as an indirect element of the construction cost.
However, the expenditure incurred after the plant has begun commercial production, is not capitalized and is
treated as revenue expenditure even though the contract may stipulate that the plant will not be finally taken
over until after the satisfactory completion of the guarantee period.
A point to be noted is that there is a difference between start-up and commissioning costs incurred on a
business that is not related to any particular fixed asset. The former is capitalisable and the latter is not. In
fact, in respect of the latter AS-26 contains express prohibition on its deferral.

(v) Cost of Project – delays and Wastages


“If the interval between the date a project is ready to commence commercial production and the date at which
commercial production actually begins is prolonged, all expenses incurred during this period are charged to
the profit and loss statement.

(vi) Cut off date for Capitalization


Costs incurred for making the asset ready for its intended use are capitalized.
What is important in this context is that the stage referred to is the stage of bringing the asset ready for its
intended use and not the stage where an enterprise may be ready to use the asset. For example, a plant may
be ready for its intended use, however, due to lack of demand the plant may not have been started. In these
circumstances, subject to other considerations, all relevant costs incurred up to the stage the plant is ready
for its intended use are capitalized, cost incurred after that stage are not capitalized.
84 Accounting Standards

PROBLEMS AND SOLUTIONS

Problem 1: A Computerised Machinery was purchased by two companies jointly. The price was shared
equally. It was also agreed that they would use the machinery equally. They decided to show in their Balance
Sheets 50% of the value of the machinery and charge 50% of the depreciation in their respective books of
accounts. Comment? (C.A. Final Nov. 1997) (5 Marks)
Solution: As per para 15 where an enterprise owns fixed assets jointly with others the extent of its share in
such assets and the proportion in the original cost, PFD and WDV is to be stated in the balance sheet. At least
details of jointly owned assets are to be indicated separately in the fixed assets register.
Problem 2: During the current year 2006-07, X Limited made the following expenditure relating to its plant
building:

Particular ` in lakhs
Routine Repairs 4
Repairing 1
Partial replacement of roof tiles 0.5
Substantial improvements to the electrical wiring system which will increase efficiency 10

What will be the amount of capitalization?


Solution: As per para 12 of AS-10 expenditure that increases the future benefits from the existing asset
beyond its previously assessed standard of performance is to be capitalized.

Particulars ` Answer / Reason


Routine Repairs 4,00,000 Revenue – Normal repairs
Repairing 1,00,000 Revenue – Normal repairs
Partial replacement of roof tiles 50,000 Revenue – Expenditure just maintain the asset
Substantial improvements ….. 10,00,000 Capitalised – increases efficiency as previously assessed.

Problem 3: Amna Ltd. contracted with a supplier to purchase a specific machinery to be installed
in Department A in two months time. Special foundations were required for the plant, which were to be
prepared within this supply lead time. The cost of site preparation and laying foundations were `47,290.
These activities were supervised by a technician during the entire period, who is employed for this purpose
for `15,000 per month. Technician’s services were given to Department A by Department B, which billed the
services at `16,500 per month after adding 10% profit margin.
The machine was purchased at `52,78,000. Sales tax was charged at 4% on the invoice, `18,590
transportation charges were incurred to bring the machine to the factory. An Architect was engaged at a fee
of `10,000 to supervise machinery installation at the factory premises. Also, payment under the invoice was
due in 3 months. However, the Company made the payment in 2nd month. The company operates on Bank
Overdraft @ 11%.
Ascertain the amount at which the asset should be capitalized under AS 10.
Solution:
Calculation of Cost of Fixed Asset (i.e. Machine) as per AS 10
Particulars `
Purchase price Given 52,78,000
Add: Sales tax at 4% `52,78,000 × 4% 2,11,120
AS-10: Accounting for Fixed Assets 85

Site preparation cost Given 47,290


Technician’s salary Specific/Attributable overheads for 2 months [See Note (ii)] 30,000
Initial delivery cost Transportation 18,590
Professional fee for installation Architect’s fee 10,000
Total cost of asset 55,95,000

Note:
(i) Interest on bank overdraft for earlier payment of invoice is not relevant under AS 10.
(ii) Internally booked profits should be eliminated in arriving the cost of fixed assets as per para 10 of AS
10 ‘Accounting for Fixed Assets’.
(iii) It has been assumed that the purchase price of `52,78,000 excludes amount of sales tax.
Problem 4: The company has written up its fixed asset by ` 50 lacs. The total depreciation of ` 10lacs stands
transferred to P/L accounts. The BOD seeks your advice in the finalisation of accounts for the year ended
June 2005. (C.A. Final Audit Nov. 1999) (3 Marks)
Solution: The companies treatment is not correct for accumulated depn. As it is a case of upward revaluation
of ` 50,00,000. The PFD should be kept in the Fixed asset schedule. Also the depreciation for the current year
should be credited to PFD account. The depn relating to Revaluation figure should be charged to revaluation
reserve account and the balance is to be charged to P/L.
Problem 5: As at the beginning of the year, the company has a capital of ` 2.50 crores, free reserves of
` 0.50 crores and Revaluation Reserve of ` 4.50 crores. In the relevant year under audit the company has
incurred a loss of ` 4 crores. The company proposes to adjust the loss with the Revaluation Reserve.
(C.A. Final Audit May 2001) (6 Marks)
Solution: Guidance note on Revaluation reserve as well as AS-10 states that revaluation reserves should not
be used for any purpose except for the fixed asset for which such revaluation reserves is created. Even SEBI
prohibits revaluation reserves for declaring bonus. Only revenue reserves can be used for written off past
losses and not revaluation reserves. It is an unrealized profit.
Problem 6: J Ltd. purchased machinery from К Ltd. on 30.09.2007. The price was `370.44 lakhs after charging
8% Sales-tax and giving a trade discount of 2% on the quoted price. Transport charges were 0.25% on the quoted
price and installation charges come to 1% on the quoted price.
A loan of ` 300 lakhs was taken from the bank on which interest at 15% per annum was to be paid.
Expenditure incurred on the trial run was Materials ` 35,000, Wages `25,000 and Overheads ` 15,000.
Machinery was ready for use on 1.12.2007. However, it was actually put to use only on 1.5.2008. Find out the
cost of the machine and suggest the accounting treatment for the expenses incurred in the interval between
the dates 1.12.2007 to 1.5.2008. The entire loan amount remained unpaid on 1.5.2008.
(C.A. Final Nov. 2003, Nov 2010 NS + CA Final May 2013) (7/5/5 Marks)
Solution:
(i) Let the quoted price be ` P. Therefore (P – Trade Discount) + S Tax = ` 370.44 lacs
Solving the equation we get P = ` 350.00lacs

Particulars ` In lacs
Quoted Price 350
Trade Discount (7)
343
+ Sales tax 27.44
Net Price 370.44
86 Accounting Standards

Transportation costs 0.88


Installation charges 3.50
Total cost of purchase 374.82
+ Trial run expenses (Material + Labour + Overhead) 0.75
Total Cost 375.57

Borrowing costs on loan outstanding for the period of installation can be assumed to be charged to
Profit and Loss account and not to be capitalized because 2 months cannot be taken as substantial
period as per AS-16. Alternatively it can be taken as substantial period of time which ICAI has taken
a view in its solution.
(ii) The expenses incurred in between 1.12.2007 to 1.5.2008 should be charged to Profit and Loss
account because machine is ready for use as on 1.12.2007.
Problem 7: Yours client is purchasing its fixed assets from indigenous and overseas sources. On these fixed
assets, rebate and discounts are sometimes allowed by the supplier. In other situations, the client also deducts
at the time of payment, liquidated damages because of late delivery of the machines. Advice your client on
accounting for rebates, discount and liquidated damages?
Solution: Cost of assets excludes trade discount and rebates i.e. they are deducted from the cost of fixed
asset……..Para 9 of AS-10. Liquidated damages are in the nature of penalty which is recovered by the purchaser
from the supplier of the asset. LD is not to be deducted from the asset but has to be shown separately in
profit/loss account as an income.
Problem 8: Your client is in the process of setting up a production line for manufacturing a new product.
Based on trial runs conducted by your client, it was noticed that the production lines output was not of the
desired quality. However, your client has taken a decision to manufacture and sell the sub-standard product
over the next one-year due to huge investment involved.
Advise your client on the cut-off date for the purpose of capitalization. (CA Final May 2007) (Marks 4)
Solution: As per provisions of AS 10 ‘Accounting for Fixed Assets’, expenditure incurred on startup and
commissioning of the project, including the expenditure incurred on test runs and experimental production,
is usually capitalized as an indirect element of the construction cost. However, the expenditure incurred after
the plant has begun commercial production i.e., production intended for sale or captive consumption, is not
capitalized and is treated as revenue expenditure even though the contract may stipulate that the plant will
not be finally taken over until after the satisfactory completion of the guarantee period. In the present case,
the company did stop production even if the output was not of the desired quality, and continued the sub-
standard production due to huge investment involved in the project. Capitalization should cease at the end of
the trial run, since the cut-off date would be the date when the trial run was completed.
Problem 9: Welcome Ships Limited incurred dry-docking of its ship (major repairs and overhaul expenses)
of ` 2 crores for the year 2006-07. Advise WS Ltd. on the treatment of the dry – docking expenses?
Solution: As per para 12 of AS-10 expenditure that increases the future benefits from the existing asset beyond
its previously assessed standard of performance is to be capitalized. Dry docking expenses/overhauling
expenses are heavy expenses incurred on periodic basis say 2-3 years for the ships/air craft’s. Under Indian
GAAPs such expenditure is charged to P/L as major repairs. But under IFRS such heavy expenditure is
capitalized and written over the period till the next overhaul expense.
Problem 10: An old car of a company having a nominal book value has found a buyer, who is willing to pay
` 1 lakh for it. The company proposes not to sell the car, but to neglect its valuation in its accounts at ` 1 lakh
should the auditor permit the company to do so? (CA Final Audit Nov. 2003) (Marks 8)
Solution: In the present case car can be either retired from active use or in use. If it is in use then it should be
included in the block of assets. But if it is retired from active use then it should be valued at lower of WDV or
AS-10: Accounting for Fixed Assets 87

net realizable value under the head fixed assets only…..para 14 of AS-10. If the car is in use then revaluation
on such car can be possible. But revaluation should not be done for single car but all the cars. The company
has neglected the valuation of ` 1,00,000 is correct.
Problem 11: Shivaji Ltd. purchased Fixed assets worth `90,00,000 on 1st April 2002. The lifestof the assets
is 10 years and they are to be depreciated on straight line basis. The assets were revalued on 1 April, 2004
when 50% of the assets were assessed at 10% less than the book value, and the remaining assets were
revalued at 15% higher than book value. The assets were ultimately sold on 1.4.2006 for ` 54,80,000. Excess
depreciation on revaluation, if any, should be charged to Revaluation Reserve.
Show Fixed Assets A/c., Depreciation A/c. and Revaluation Reserve A/c. supported by working wherever
necessary. (C.A. Final Nov. 2007) (Marks 8)
Solution:
In the books of Shivaji Ltd.
Fixed Assets Account
Dr Cr
Date Particular ` Date Particular `
1.4.2002 To Bank 90,00,000 31.3.2003 By Depreciation 9,00,000
By Balance c/d 81,00,000
90,00,000 90,00,000
1.4.2003 To Balance b/d 81,00,000 31.3.2004 By Depreciation 9,00,000
By Balance c/d 72,00,000
81,00,000 81,00,000
1.4.2004 To Balance b/d 72,00,000 1.4.2004 By Revaluation Reserve (WN2) 3,60,000
To Revaluation Reserve (WN1) 5,40,000 31.3.2005 By Depreciation (WN3) 9,22,500
77,40,000 77,40,000
1.4.2005 To Balance b/d 64,57,500 31.3.2006 By Depreciation 9,22,500
By Balance c/d 55,35,000
64,57,500 64,57,500
1.4.2006 To Balance b/d 55,35,000 1.4.2006 By Bank 54,80,000
By Revaluation Reserve (loss) 55,000
55,35,000 55,35,000

Depreciation Account
Dr Cr
Date Particular ` Date Particular `
31.3.2003 To Fixed assets 9,00,000 31.3.2003 By Profit and Loss Account 9,00,000
31.3.2004 To Fixed assets 9,00,000 31.3.2004 By Profit and Loss Account 9,00,000
31.3.2005 To Fixed assets 9,22,500 31.3.2005 By Profit and Loss Account 9,00,000
By Revaluation Reserve Account 22,500
(WN4)
9,22,500 9,22,500
88 Accounting Standards

Revaluation Reserve Account


Dr Cr
` `
1.4.2004 To Fixed assets 3,60,000 1.4.2004 By Fixed Assets 5,40,000
(Loss on revaluation) (upward revaluation)
31.3.2005 To Depreciation (WN1) 22,500
To Balance c/d 1,57,500
5,40,000 5,40,000
31.3.2006 To Depreciation 22,500 1.4.2005 By Balance b/d 1,57,500
To Balance c/d 1,35,000
1,57,500 1,57,500
1.4.2006 To Fixed assets 55,000 1.4.2006 By Balance b/d 1,35,000
(Loss on safe of revalued assets)
To General Reserve 80,000
1,35,000 1,35,000

Workings Notes:
1. Calculation of Revaluation amount to be shown in revaluation reserve account:
Downward revision of 50% of fixed assets = (`72,00,000 × ½) × 10% = `3,60,000
Upward revision of remaining 50% of fixed assets = (`72,00,000 × ½) × 15% = `5,40,000
2. Value of Fixed assets after revaluation as on 1.4.04:
[(72,00,000 × ½) – 3,60,000] + {(72,00,000 × ½) + 5,40,000} = `32,40,000 + 41,40,000 = `73,80,000
3. Depreciation per annum for remaining 8 years
Value of Fixed Assets = `73,80,000 (W.N.2)
Remaining life of the asset = 8 years
New Depreciation = `73,80,000/8 = `9,22,500
4. Depreciation to be charged to Revaluation Reserve Account
= `9,22,500 – `9,00,000 = `22,500
Students can alternatively transfer downward revaluation to P/L also instead of revaluation reserve and
accordingly the answer will change.
Problem 12: A conveyor system was capitalized on 01-01-2011 with value of ` 41.37 crores.
The break-up of the capital cost was as follows:
Civil & Mechanical structure 11.72
Driving units and pluming 05.4
Rope 02.83
Belt 11.17
Safety and electrical equipments 06.15
Other accessories 04.10
41.37
During the financial year 2014-2015 due to wear and tear, the rope used in the conveyor system was
replaced by a new one at cost of ` 8 crores. As new rope did not increase the capacity and is a component
of the total assets. The company charged the full cost of the new rope to repairs and maintenance. Old rope
continues to appear in the books of account and is charged with depreciation every year.
AS-10: Accounting for Fixed Assets 89

Whether the above accounting treatment is correct? If not, give the correct accounting treatment with
explanation.
Solution: As per Para 23 of AS-10 - Subsequent expenditure relating to an item of fixed asset should be
added to its book value only if it increases the future benefits from the existing asset beyond its previously
assessed standard of performance. In the instant case, the new replaced rope does not increase the future
benefits from the assets beyond their previously assessed performance, therefore the cost of replacement of
rope should be charged to revenue, however in doing so the estimated scrap value of the old rope should be
deducted from the cost of new rope. The old rope will be continued to be depreciated over the balance life.
P roblem 13: A company has scrapped a semi-automatic part of a machine (not written-off) and replaced
with a more expensive fully automatic part, which has doubled the output of the machine. At the same time
the machine was moved to more suitable place in the factory, which involved the building of new foundation
in addition to the cost of dismantling and re-erection. The company wants to charge the whole expenditure
to revenue. As an auditor, what would you do in this situation? (C.A. Inter, Nov. 2001)
Solution: If the subsequent expenditure increases the expected future benefits from the asset beyond its
pre-assessed standard of performance then as per AS-10 it should be capitalized. Otherwise it should be
expensed. In this case, the replacement of semi-automatic part with a fully automatic part has doubled the output
of the machine thus, it has increased future benefits beyond the machines’ pre-assessed standard performance,
hence this expenditure should be capitalized as part of cost of the machine. However, the expenses for shifting the
machine and building of a new foundation in addition to the cost of dismantling and re-erection do not contribute
to any new future benefits from the existing asset. They only serve to maintain performance of the machine.
Hence, this cost should be charged to revenue.
Problem 14: On 01.04.2010 a machine was acquired at `4,00,000. The machine was expected to have a
useful life of 10 years. The residual value was estimated at 10% of the original cost. At the end of the 3rd year,
an attachment was made to the machine at a cost of `1,80,000 to enhance its capacity. The attachment was
expected to have a useful life of 10 years and zero residual value. At the beginning of the 4th year, the original
machine was revalued upwards by `90,000 and remaining useful life was reassessed at 9 years and residual
value was reassessed at NIL value. Find depreciation for the year 2013-14, if
(i) attachment retains its separate identity.
(ii) attachment becomes integral part of the machine.
Solution:
1. Depreciation of Original Machine

`
Original cost of Machine as on 01.04.2010 4,00,000
Less: Residual Value @ 10% (40,000)
Depreciable Value (a) 3,60,000
Useful life is 10 years
Depreciation per year 36,000
Depreciation for 3 years (36,000 x 3) (b) 1,08,000
Written down value at the end of 3 years (as on 31.03.2013)
[4,00,000 – 1,08,000] 2,92,000
Add: Revaluation 90,000
Total book value after revaluation 3,82,000
Remaining useful life was reassessed as 9 years
Depreciation per year from 2013-14 42,444
90 Accounting Standards

2. Depreciation of Attachment
Original cost of Attachment as on 01.04.2013 1,80,000
Useful life is 10 years
Depreciation per year from 2013-14 18,000

Depreciation for the year 2013-14


(i) When Attachment retains its separate identity:
Depreciation of Original Machine `42,444
Depreciation of Attachment `18,000
Total Depreciation for 2013-14 `60,444
(ii) Attachment becomes integral part of the Machine:
As per para 9 of AS 6 ‘Depreciation Accounting’, any addition or extension to an existing asset which
is of a capital nature and which becomes an integral part of the existing asset is depreciated over the
remaining useful life of that asset. Accordingly,
Total value of Machine as on 01.04.2013
Original Machine at revalued cost (W.N.1) `3,82,000
Cost of attachment `1,80,000
`5,62,000
Remaining useful life of Original Machine 9 years
Depreciation for 2013-14 ` 62,444
CHAPTER 11
AS-11 (Revised):
The Effect of Changes in
Foreign Exchange Rates

Chapter Outline
vv Need and Objective vv Part II: Translation of Financial
vv Part I: Accounting for Foreign Statements of Foreign Operations
Exchange Transactions vv Part III: Accounting for Foreign Exchange Contracts

Need and Objective


We know that financial statements of our companies are recorded in terms of our domestic currencies viz.,
in terms of “Rupees”. But if our company in India is having a Branch or Subsidiary etc. in a foreign country
or our company is exporting or importing goods and services with foreign countries, then how to record
such transactions in terms of domestic currencies in our books of accounts? Hence a need arises to translate
foreign currency transactions or foreign branch financial statements into reporting currency.

Scope
AS – 11 (Revised 2003) issued by ICAI within its ambit covers the following:
PART I → Accounting of foreign exchange transactions and
PART II → Converting the financial statements of the foreign operations.
PART III → Accounting for foreign exchange contracts.

IMPORTANT TERMS
(a) Reporting Currency: It is the currency of the country whose financial statements are reported.
Example: ` is the reporting currency of India.
(b) Foreign Currency: It is the currency other than the reporting currency of an enterprise.
(c) Exchange Rate: It is the ratio for exchange of two currencies. Example: US $ 1 = ` 60.
(d) Average Rate: It is the mean of exchange Rates in force during a period.
(e) Forward Rate: It is the predetermined exchange rate for settlement at a future specified date.
(f) Closing Rate: Rate of exchange that exists on the date of Balance Sheet.
(g) Monetary items: Monetary items are money held, assets to be received and liabilities to be paid which are
fixed and determinable. Example: Cash, bank balance, debtors, creditors, Loan etc.
(h) Non-Monetary items: Non-monetary items are assets and liabilities other than monetary items.
Example: Fixed assets, Inventories, Investments is equity shares, prepaid expenses etc.
92 Accounting Standards

Part I: Accounting for Foreign Exchange Transactions


A foreign currency transaction is a transaction which is denominated in or requires settlement in foreign
currency. A company having some of the transactions with foreign countries is a normal feature of any
business. Foreign exchange transactions include the following:
a) buying and selling of goods and services
b) borrowing or lending of money (Loans transactions)
c) acquiring and selling of assets
d) incurring or settling of liabilities
e) Un-performed foreign exchange contract (forward contracts outstanding on balance sheet date)
In this case of Part I of AS–11 (Revised) hereinafter refer to as AS–11 [R] the accounting treatment is
based on the following 3 issues:
◘◘ Initial recognition of a transaction.
◘◘ Valuation on the balance sheet date.
◘◘ Treatment of exchange difference.

Initial Recognition of a Transaction


Any transaction in a foreign currency should be recorded in the reporting currency (Example: Indian currency)
by applying the exchange rate of foreign currency and reporting currency as at the date of the transaction. It
is also called spot rate. For convenience one can use even weekly average rate.

Valuation as on the Balance Sheet Date


It is very much possible that the foreign exchange transactions after the initial recognition is still outstanding
as on the balance sheet date. AS-11 [R] has suggested the following treatment for such outstanding assets/
liabilities as on the Balance sheet date:
(i) Valuation of Monetary items: Apply closing rate. If closing rate does not reflect the amount to be
realized/disbursed then such other rate which is likely to be realized/disbursed should be considered.
It may be possible that applying closing rate seems to be unrealistic. Example: restrictions imposed
on remittances. Then, as per AS-11 [R] apply most likely realizable rate.
(ii) Valuation of Non - Monetary items: For the purpose of conversion as on the date of balance sheet,
non-monetary has been categorized further into two parts:
Non-Monetary items carried at Historical Cost (Original Cost) e.g. Fixed Asset carried at Cost Less PFD.:
Such Non-Monetary items will be recorded at original exchange rate only i.e. they will have no exchange
difference.
Non-Monetary items carried at Fair Value e.g. Current investments carried at Market value: Exchange rates
applicable when fair valuation was determined. Most probably it may be taken as the closing rate on the
Balance Sheet date.

Treatment of Exchange Difference


Exchange difference arises because of the following:
◘◘ Rate on the date of initial recognition and rate on the date of settlement (receipt/payment).
◘◘ Rate on the date of reporting (Balance Sheet) and rate as on the date of initial recognition
◘◘ Rate on the date of balance sheet and rate on the date of settlement.
ww All the above exchange differences should be transferred to “Profit and Loss statement”.
AS-11 (Revised): The Effect of Changes in Foreign Exchange Rates 93

DO YOU KNOW
Under the New Schedule III forex gain/loss will be disclosed under “Other Income or Other Expense”.

Exception: If the investor invests in a non-integral foreign operation in Monetary item, the exchange
difference is accumulated in Foreign Currency Translation Reserve Account instead of transfer to
income and expense account. When such investments are sold then the exchange difference accu-
mulated in foreign reserve account is to be transferred to income and expense account (refer special
problem number 3)

Part II: Translation of Financial Statements of Foreign Operations


Foreign Operations
A parent body which is situated in domestic country can have many operations in foreign countries. AS – 11[R]
defines Foreign Operations as: (i) a Subsidiary, (ii) an Associate, (iii) a Joint Venture, (iv) a branch/depot of the
reporting (domestic) enterprise, the activities of which are based or conducted in the country other than the
reporting enterprise country.
AS – 11[R] further classifies the foreign operations into two categories: (For translation purpose)
(i) Integral foreign operations (ii) Non-integral foreign operations.

Integral Foreign Operations (Highly Dependent Operations)


In case of Integral foreign operations, the foreign operation activities are the integral activities of the reporting
enterprises. In other words the activities of foreign Subsidiary/Associate/Joint Venture/Branch/Depot are
part and parcel activities of the reporting enterprise. The foreign operations set up in the foreign country are
just an extended arm of the reporting enterprises. Mainly integral foreign operations are its foreign branches.
A few symptoms/tips for better understanding of the meaning of Integral foreign operations are
enumerated below:
◘◘ Foreign operations to a large extent controlled one.
◘◘ Acting as a selling agent i.e. receiving goods on behalf of reporting enterprises and remitting the
proceeds back to the investing company.
◘◘ Foreign operation has been set up for raising financial activities.
◘◘ Procuring or producing raw materials by foreign depots and sending it to head office (reporting
enterprises).
◘◘ Expenses are all paid by the reporting enterprise in its local currency.
◘◘ Exchange rates have a direct impact on the cash flows of the reporting enterprises.

Non-Integral Foreign Operations (Less Dependent Operations)


As such AS-11[R] has not defined non-integral operations but a few hints/symptoms is being provided.
◘◘ Foreign branch/Subsidiary etc can independently generate sales or incur expenses.
◘◘ Expenses of foreign operations are paid in local currency rather than in the reporting currency.
◘◘ A change brought about in the cash flows of foreign operations have a least impact on the cash flows
of the reporting enterprises.
◘◘ Foreign operations even enjoys fixation of selling prices of their product, having active sales market
for their product. Sales are mainly in foreign currencies.
◘◘ Even loan borrowings baskets may be filled up locally by the foreign operation.
◘◘ Only fewer transactions are entered into by the reporting company with the foreign operation.
94 Accounting Standards

One can conclude that Integral operations are in substance reporting enterprises activities only and
Non–Integral operations are set up as a net investments in foreign countries.
All the above indications are only illustrative and not exhaustive. Due weightage is to be given to actual
facts and circumstances and even exercising proper judgment. Hence the issue whether a foreign operation
is integral or non – integral is still a bone of contention.

Translation Aspects of Financial Statement of Integral Operations


Financial Statements of integral operation should be translated as if the reporting enterprises has transacted
the business.
As from our preceding discussions it is clear that the activities of foreign operations are part and parcel of
the activities of the reporting enterprises. It follows that even the principles for conversion factor is as if the
foreign operation is itself a reporting enterprise.
(i) Income and expenses → Date of transaction or average rate
(ii) Cost and Depreciation of tangible fixed assets → Date of acquisition of asset
(iii) Assets carried at fair value → Date on which such fair value is determined
(iv) Assets shown at realizable amount like stock carried at NRV → Date on which realizable value
determined (generally closing rate)
(v) Contingent liability → Closing rate
(vi) Closing stock of inventories → Date on which such costs were actually incurred
(vii) HO Account → Equivalent amount from HO
(viii) Goods sent to Branch → Equivalent amount from HO

Note: Average rate may be used only when there is not significant violation in exchange rates and subject
to materiality.

◘◘ Profit and Loss account gives shelter to all the above exchange differences.

Translation Aspects of Financial Statement of Non - Integral Operations


(i) Balance Sheet items i.e. Assets and Liabilities → Closing rate (irrespective of monetary or non–
monetary items)
(ii) Share Capital → Opening rate (a very special case)
(iii) Income and expenses → Date of transaction or average rate
(iv) Contingent liability → Closing rate
◘◘ Unlike integral operations, the exchange differences are accumulated in a “Foreign Currency
Translation Reserve” (FCTR) until the disposal of “net investment in such non – integral operation”.
◘◘ Under the New Schedule III FCTR is disclosed under the Line item Reserves & Surplus → Other Reserves.

Consolidation Procedures
In case of non – integral foreign operation having a foreign subsidiary or JV then following consolidation
procedure is to be followed:
(i) Goodwill or Capital Reserve arising on acquisition should be translated at closing rate.
(ii) Usual consolidation procedures such as elimination of intra-group transactions of a subsidiary,
associate, JV, etc., should be applied.
(iii) Exchange difference arising out of consolidation will have to be accumulated in foreign translation
reserve account.
(iv) Share of Minority in translation reserves is to be proportionately accounted.
AS-11 (Revised): The Effect of Changes in Foreign Exchange Rates 95

(v) Difference arising out of Mutual indebtness is to be expensed.


(vi) Regarding financial statements having different reporting dates or policies shall be followed same as
per AS-21.
(vii) Principles as laid down in AS 23 and AS 27 is also to be followed.

Note: Exchange difference arising out of consolidation in case of integral foreign operations will have to be
passed through profit and loss account.

Disposal of Non-Integral Operations


Disposal of non – integral foreign operation covers – Sales, liquidation, and repayment of share capital,
abandonment of whole or part of non – integral foreign operation. However disposal does not include writing
down of assets of non – integral foreign operation (AS-28)
On such a disposal, previously accumulated foreign translation reserve account should be released and
thereby recognizing it as income or expense in the same year in which such disposal takes place. In case of
partial disposal proportionate income or expense should be recognized.

Change in Classification
Translation procedure shall be applicable from the date of reclassification.
For integral foreign operation to Exchange difference arising for non–monetary assets at the date of reclassification is
non-integral foreign operation to be accumulated in foreign translation reserve account.
For non-integral foreign The translation amount of non-monetary items at the date of reclassification is treated
operation to integral foreign at historical cost. Exchange difference accumulated in foreign currency translation
operation reserve should not be recognized as profit and loss till the disposal of operation even
if the foreign operations becomes integral.

◘◘ Question of exchange difference arising for monetary asset does not arise because treatment for
monetary asset is same irrespective of integral foreign operation or non-integral foreign operation.

Part III: Accounting for Foreign Exchange Contracts


(i) Forward exchange contract for hedging purposes (risk management),
(ii) Forward exchange contract for trading or speculative purposes: All the contracts in this case are to be
mark to market. (refer special problems on accounting for derivatives in AS-30)

Transactional Provisions
If a foreign branch of the reporting enterprises applies the revised standard i.e. non-integral operation for the
first time, the treatment will be same as if “Change in the classification of a foreign operation”.

Disclosures Requirements
◘◘ Amount of exchange difference included in the profit and loss account.
◘◘ Amount accumulated in the foreign exchange reserve account (in case of non-integral operation)
◘◘ Reconciliation between opening and closing balance the said reserve account.
◘◘ If the reporting currency is different from the currency of the country in which entity is domiciled, the
reason for such difference if any.
◘◘ A change in classification form Integral to Non–Integral and vice–versa following are the disclosures- Nature
of change in classification, Reasons, Impact of such change on shareholders funds, Effect on change in
net profit or loss for each prior period presented, Disclosure is very much appreciated for enterprises
foreign exchange risk management.
96 Accounting Standards

COMPANIES ACCOUNTING STANDARD RULES VS AS -11 (Revised). Exchange difference on long term
foreign currency monetary item:
Note: The notification dated 31.03.2009 and amended from time to time by the Ministry of Corporate
Affairs seeks to insert paragraph 46A after paragraph 45 in the Accounting Standard (AS) 11 relating to the
“The Effects of Changes in Foreign Exchange Rates”. The renewal of notification was inserted by MCA on
29th December 2011.

Features are as follows:


◘◘ It requires that exchange differences arising on reporting of long term foreign currency monetary
items at rates different from those at which they were initially recorded during the period (in respect
of accounting periods commencing on or after 7th December 2006 i.e from 2006-07) financed for
acquisition of a depreciable capital asset can be added to or deducted from the cost of the asset.
◘◘ The notification also requires that depreciation on such capital asset be provided over the balance life
of such an asset.
◘◘ If the foreign currency monetary item is used for other than acquiring depreciable asset than such
exchange difference is transferred to Foreign Currency Monetary item A/C. The balance in the FCMI
A/C is to be amortized over the period of loan. Repayment of loan is considered as a separate/distinct
loan.
◘◘ The interpretation of the maximum period till 31.03.2020: It means such benefit is to be consumed
only from 2006-07 till 2019-2020 and not beyond it.
◘◘ FCMI A/C under the New Schedule VI is disclosed under the line item “Reserves and Surplus”
◘◘ The notification issued by MCA is just an irrevocable option. It means the company can either follow
MCA notification or can continue to follow AS-11 (Rev), in which case the exchange difference will be
transferred to income account.
◘◘ The above notification is not applicable to non – corporate entities.
◘◘ Foreign currency monetary item should be long term asset or liability (more than 12 months).
◘◘ The above notification will have no effect on para 4 (e) of AS-16 i.e. AS-16 will have an over-riding
effect on Exch. diff. para 4(e).

PROBLEMS AND SOLUTIONS

Problem 1: Mr Choksi an agent of Balwas total Limited purchased goods 500 units costing $10 each on
1/3/2013. The exchange rates on 1/3/2013 was 1$ = `55.33 and on 31/3/2013 it was 1$ = `55.24. Pass
journals in the books of Balwas Limited as per AS 11 (R) assuming the net realisable value of the inventories
on 31/3/2013 is (i) $12, (ii) $9.
Solution: Inventories originally costs = 500 × 10 × 55.33 = `2,76,650
On 31/3/2013: $12 or $10 whichever is less. Then inventories will be valued @ $10 (cost). As per AS-11
(R) if non monetary item is carried at cost then exchange rate will be the original rate. Valuation of inventories
= `2,76,650.
On 31/3/2013: $9 or $10 whichever is less. Then inventories will be valued @ $9 (nrv). As per AS-11
( R ) if non monetary item is carried at fair value then exchange rate will be the closing rate. Valuation of
inventories = 500 x 9 x 55.24 = Rs 2,48,580.
Journals:
Loss on inventories Dr 28,070 (trf to p/l)
To Stock 28,070
AS-11 (Revised): The Effect of Changes in Foreign Exchange Rates 97

Problem 2: A company had imported raw materials worth US Dollars 6,00,000 on 5th January, 2005, when
the exchange rate was ` 43 per US Dollar. The company had recorded the transaction in the books at the above
mentioned rate. The payment for the import transaction was made on 5th April, 2005, when the exchange
rate was ` 47 per US Dollar. However on 31st March. 2005, the rate of exchange was ` 48 per US dollar. The
company passed an entry on 31st March, 2005 adjusting the cost of raw materials consumed for the difference
between ` 47 and ` 43 per US Dollar.
In the background of the relevant accounting standard, is the company’s accounting treatment correct?
Discuss. (CA Final Nov. 2006) (4 Marks)
Solution: The A/c treatment given by the company is incorrect. Creditors are Monetary item, hence as per
valuation principle Monetary items should be valued at Closing rate existing on the date of B/S i.e. 48 per $.
Also the exchange loss/gain should be adjusted in income statement and not in the cost of raw materials. The
Auditor should qualify the report.
Problem 3: Derivatives Ltd. has a subsidiary Option Inc. in USA. Derivatives Ltd. has advanced $10,000 on
1.3.2012 as a part of long term investment. The exchange rates prevailing on the respective dates are: 1.3 - 1$
= ` 48, 31.3.- 1$ = ` 48.50.
Pass necessary journals to record the above transaction.
Solution: Normally forex exchange difference is transferred to Profit/Loss A/C. But para 15 of AS-11 (Rev.)
makes an exception to the general rule. As per para 15 exchange difference arising on a monetary item, that in
substance, forms part of an enterprise’s net investment in a non – integral foreign operation, are accumulated
in foreign currency translation reserve account.
1.3.2012 Loan to Option Inc. Dr. 4,80,000
 To Bank 4,80,000
31.3.2012 Loan to Option Inc. Dr. 5,000
 To Exchange gain 5,000
31.3.2012 Exchange gain Dr. 5,000
 To Foreign Exchange Reserve 5,000
Problem 4: ABCD Ltd., Delhi has a branch in New York, USA, which is an integral foreign operation of the
company. At the end of 31st March, 2015, the following ledger balances have been extracted from the books
of the Delhi office and the New York Branch:

Particulars Delhi New York


(` thousands) ($ thousands)
Debit Credit Debit Credit
Share Capital 1,250
Reserves and Surplus 940
Land 475
Building (cost) 1,000
Buildings Depreciation Reserve 200
Plant & Machinery (cost) 2,000 100
Plant & Machinery Depreciation Reserve 500 20
Trade receivables/payables 500 270 60 20
Stock (01-04-2014) 250 25
Branch Stock Reserve 65
Cash & Bank Balances 125 4
98 Accounting Standards

Purchases/Sales 275 600 25 125


Goods sent to Branch 1,500 30
Managing Director’s salary 50
Wages & Salaries 100 18
Rent 6
Office Expenses 25 12
Commission receipts 275 100
Branch/H.O. Current A/c 800 15
Total 5,600 5,600 280 280

The following information is also available:


(1) Stock as at 31-03-2015
Delhi - ` 2,00,000
New York - $ 10 (all stock received from Delhi)
(2) Head Office always sent goods to the Branch at cost plus 25%.
(3) Provision is to be made for doubtful debts at 5%.
(4) Depreciation is to be provided on Buildings at 10% and on Plant and Machinery at 20% on written
down values.
Required:
(a) To convert the branch Trial Balance into rupees, using the following rates of exchange:
Exchange:
Opening rate 1$ = ` 50
Closing rate 1$ = ` 55
Average rate 1$ = ` 52
For fixed assets 1$ = ` 45
(b) To prepare the Trading and Profit & Loss Account for the year ended 31st March, 2015, showing to
the extent possible, Head Office results and Branch results separately.
Solution:
ABCD Ltd.
New York Branch Trial Balance As on 31st March 2015 (Integral Operation)
($ ‘000) (` ‘000)
Dr. Cr. Conversion Dr. Cr.
rate per $
Plant & Machinery (cost) 100 `45 4,500
Plant & Machinery Dep. Reserve 20 `45 900
Trade receivable/payable 60 20 `55 3,300 1,100
Stock (1.4.2012) 25 `50 1,250
Cash & Bank Balances 4 `55 220
Purchase / Sales 25 125 `52 1,300 6,500
Goods received from H.O. 30 Actual 1,500
Wages & Salaries 18 `52 936
Rent 6 `52 312
Office expenses 12 `52 624
AS-11 (Revised): The Effect of Changes in Foreign Exchange Rates 99

Commission Receipts 100 `52 5,200


H.O. Current A/c 15 Actual 800
1394 14,500
Exchange loss (bal. fig.) 558
280 280 14,500 14,500
Closing stock `55 0.55

(b) Trading and Profit & Loss Account for the year ended 31st March, 2015
(`000)
H.O Branch Total H.O Branch Total
To Opening Stock 250 1,250.00 1,500.00 By Sales 600 6,500.00 7,100.00
To Purchases 275 1,300.00 1,575.00 By Goods sent to Branch 1,500 -- 1,500.00
To Goods Received from By Closing Stock 200 0.55 200.55
H.O. -- 1,500.00 1,500.00
To Wages & Salaries 100 936.00 1,036.00
To Gross profit c/d 1,675 1,514.55 3,189.55
2,300 6,500.55 8,800.55 2,300 6,500.55 8,800.55
To Rent -- 312.00 312.00 By Gross profit b/d 1,675 1,514.55 3,189.55
To Office expenses 25 624.00 649.00 By Commission receipts 275 5,200.00 5,475.00
To Provision for Doubtful
Debts @ 5% 25 165.00 190.00
To Depreciation(W.N.1) 380 720.00 1,100.00
To Balance c/d 1,520 4,893.55 6,413.55
1,950 6,714.55 8,664.55 1,950 6,714.55 8,664.55
To Exchange loss 558.00 By Balance b/d 6,413.55
To Managing Director’s 50.00 By Branch Stock Reserve 64.89
Salary (W. N. 2)
To Balance c/d 5,870.44
6,478.44 6,478.44

Working Notes:
(1) Calculation of Depreciation

H.O ` Branch
‘000 `‘000
Building – Cost 1,000
Less : Dep. Reserve (200)
800
Depreciation @ 10% (A) 80
Plant & Machinery Cost 2,000 4,500
Less : Dep. Reserve (500) (900)
1,500 3,600
Depreciation @ 20% (B) 300 720
Total Depreciation (A+B) 380 720
100 Accounting Standards

(2) Calculation of Additional Branch Stock Reserve

(`’‘000)
Closing stock of Branch 0.55
Reserve on closing stock (0.55 × 1/5) 0.11
Less : Branch Stock Reserve (as on 1.4.2014) (65)
Reversal of Stock Reserve (64.89)

Problem 5: Value Ltd has taken a loan of $ 40 lakhs on 1/4/2011. Exchange rates on various dates were :
1/4/2011 : 1$ = ` 60.45, 31.3.2012- 1$ = ` 59.55, 31.3.2013 1$ = ` 61.23. The Loan will be repaid by the end
of 31/3/2019. Pass Journals in the books of the Value Limited for 2011-12; 2012-13 assuming:
(1) Loan was used for the purpose of acquiring Depreciable Fixed Assets (Value Limited follows MCA
circular para 46 (A).
(2) Loan was used for other purpose i.e. other than Depreciable Fixed Assets (Value Limited follows MCA
circular para 46 (A).
(3) If Value Limited goes for the option for AS-11 (Rev) what would be your answer in question (1) and
(2) above.
(4) If Value Limited is a co-operative society what would be your answer in question (1), (2) and (3)
above.
Solution: (` in lakhs)

Companies Act (1) Companies Act (2) AS-11 (Rev) (3) & (4)
1/4/2011 1/4/2011 Same as column 1 or 2
Fixed Assets Dr 2,418 Bank Dr 2,418
To Loan a/c 2,418 To Loan a/c 2,418
31/3/2012 31/3/2012 31/3/2012
Loan Dr 36 Loan Dr 36 Loan Dr 36
To Fixed Asset 36 To FCMR a/c Dr 36 To Forex gain 36
(60.45-59.55)40 (60.45-59.55)40 (60.45-59.55)40
FCMR a/c 4.5 Forex gain a/c 36
To Profit / loss a/c 4.5 To Profit / loss a/c 36
(` 36 lakhs w-off over 8 years)
31/3/2013 31/3/2013 31/3/2013
Fixed Asset Dr 67.2 FCMR a/c Dr 67.2 Forex loss Dr 67.2
To Loan a/c 67.2 To Loan 67.2 To Loan 67.2
(59.55-61.23)40 Profit / loss a/c 5.1 Profit / loss a/c 67.2
To FCMR a/c 5.1 To Forex loss 67.2
(36- 4.5 - 67.2) / 7

Answer for question no. 3 and 4 remain same.


Notes: (i) FCMR is Foreign Currency Monetary Reserve a/c; (ii) Depreciation and Interest on loan is ignored
because rates are not given.
CHAPTER 12
AS-12: Government Grants

Chapter Outline
vv Meaning, Objective and Scope of Government Grants vv Treatment of Grants Received for Carbon Credits
vv Types of Government Grants vv Disclosures Under AS-12
vv Treatment of Refund of Government Grants

Meaning, Objective and Scope of Government Grants


Government Grants are assistance by government in cash or in kind to an enterprise in return for some past or
future compliance with certain conditions to be satisfied by the company receiving grant. It is also called subsidies,
cash incentives etc. Thus to receive Government Grants the enterprise has to fulfill certain conditions. Conditions
may be that some part of the products to be sold to the government undertaking companies or a company may
have to open an industrial unit in backward areas etc. Government grants may be for capital expenditures or
revenue expenditures.

Objective
One should know what should be the proper treatment of Government Grants and how much the company is
benefited from such grants.

Exclusions from Government Grants


Government Grants excludes the following:
(i) If government deals with the company in normal trading activities, the grants received by the
company is not covered under the definition of AS-12.
(ii) Tax holidays and tax exemptions received by a company under section 10, section 80.
(iii) Government participation as equity holders. It means government is also one of the shareholders of
the company.

Is Sales tax a Grant?


As sales tax exemption is neither cash nor a kind grant, hence Sales tax is not a grant as per AS-12.

When to Recognize Government Grants?


A company can recognize receipt of government grants on satisfying the following conditions:
(1) Conditions for receipt of Government Grants should be fulfilled.
(2) Grants are certain to be received.
102 Accounting Standards

Mere receipt of a grant is not the conclusive evidence that grant has to be immediately recorded. Following
points has to be considered:
(i) Where the grant is in the nature of compensation for any past losses (sick companies) or immediate
financial support (flood affected company) then such grant should be treated as per AS – 5 (Extra-
ordinary activity).
(ii) After receipt of grant, if there is any contingency then it is to be governed by AS – 4 (now AS – 29).

Approaches to Government Grants


There are two views for recognizing Government Grants:
(1) Capital Approach: Reduce the Government Grants receipt from Gross Book Value of the Asset. Those who
support Capital approach says that Government Grants received is an incentive and in the nature of capital
receipt.
(2) Income Approach: Defer the Government Grants receipt because cost is incurred for complying the
conditions.

Types of Government Grants


Following are the types of government grants:
(i) Grants related to depreciable assets.
(ii) Grants related to non - depreciable assets.
(iii) Grants related to revenue.
(iv) Grants towards promoter’s contributions.

(i) Grants related to depreciable assets (like Plant, Machinery etc.)

GRANTS RELATED TO DEPRECIABLE ASSETS


MONETARY GRANTS (CASH GRANTS) NON-MONETARY GRANTS (ASSET)
(i) Capital Approach: In this approach cash grants Non-Monetary grants received like receiving Plant and
received towards an asset is to be reduced from the Machine etc. at concessional price. Such asset acquired is
Gross Book Value (GBV) of the asset. Depreciation is to be recorded at cost price. In other words the amount
to be charged on the Net Book Value hence forth. If paid for acquiring the asset is to be recorded. It is mostly a
the NBV is reduced to Zero then in such case the NBV concessional price. If the asset is received free of cost then
should be recorded at Nominal Value i.e. `1/-. in such case the NBV should be recorded at Nominal Value
(ii) Income Approach: In this approach cash grants i.e. ` 1/-.
received towards an asset is to be transferred to
“Deferred Grants” under the head Shareholders
Funds. Such “Deferred Grants” are to be transferred to
Profit and Loss Account every year in the ratio of the
depreciation of the Asset.

(ii) Grants related to non - depreciable assets (like Land):

GRANTS RELATED TO NON - DEPRECIABLE ASSETS


MONETARY GRANTS (CASH GRANTS) NON -MONETARY GRANTS (ASSET)
(i) Capital Approach: Non-Monetary grants received like receiving Land etc. at
Possibility 1: If conditions for grants is already fulfilled: In such concessional price. Such asset acquired is to be recorded
a case cash grants received towards non-depreciable asset at Cost Price. In other words the amount paid for acquiring
is to be transferred to “Capital Reserve A/C”. the asset is to be recorded. It is mostly a concessional price.
If the asset is received free of cost then in such case the
NBV should be recorded at Nominal Value i.e. ` 1/-.
AS-12: Government Grants 103

(ii) Income Approach:


Possibility 2: If the conditions for grants are yet to be fulfilled:
In such a case cash grants received towards an asset is
to be transferred to “Deferred Grants” under the head
Shareholders Funds. Such “Deferred Grants” are to be
transferred to Profit and Loss Account. every year in the
ratio of the costs to be incurred in future for fulfilling such
conditions.

Note: Deferred grants a/c. will be disclosed under the line item “Other long term liabilities” as per the New
Schedule VI. Also the 1 year deferred portion will be disclosed under the “Other current liabilities”
(iii) Grants related to revenue:
Such grants can be for sale of products to government agencies at concessional price, expenses incurred by
the company for quality control, training expenses, high costs incurred for a creation of a product etc. Such
grants has to be recognize on systematic basis to profit and loss account to match revenue against cost. It can
be recorded as “Other income” or reduced from the concerned expenses.
(iv) Grants towards promoter’s contributions:
Promoters contribution means major investments in the project is made by the government. No repayment
of such grants is ordinarily expected. Such receipt is always a huge amount so it is credited to Capital Reserve
Account under the head Shareholders Funds. Such amounts credited to Capital Reserve should not be utilized
for any purpose like dividends/writing off losses.

Treatment of Refund of Government Grants


Government grants may be refunded by the company if it has not complied with the conditions for which
grants are received.

Treatment of Refund
(i) If the grant received was treated as deferred income: If the grant received was treated as deferred
income the refund cost is to be (–) from the unamortised deferred income and the balance is to be
charged to profit and loss account.
(ii) If the grant received was transferred to Capital Reserve: If the grant received was transferred to
Capital Reserve, refund is to be deducted from such Capital Reserve.
(iii) If the grant received was deducted from GBV of the Asset: If the grant received was deducted
from fixed asset then in such case the refund cost is to be added back to the NBV of the asset and
depreciation is to be charged on the revised value henceforth.
(iv) If the grant received was credited to Profit and Loss Account: If the grant received was adjusted
in revenue account, then such refund cost is to be debited to the revenue account (extraordinary
activities).

Treatment of Grants Received for Carbon Credits


◘◘ Carbon credits are received to reduce the emission of greenhouse gases and carbon dioxide.
◘◘ Enterprises receive carbon credits for specified projects, if they fulfill the necessary project parameters.
◘◘ Countries like India, China and Brazil earns substantial amount of Carbon Credit by selling the carbon
credit to other countries.
◘◘ Carbon credits are revenue grants and hence credited to income statement (other income as per New
Schedule III).
104 Accounting Standards

Disclosures Under AS-12


(i) Accounting policy adopted.
(ii) Nature of Government grants received (monetary or non – monetary).
(iii) Amount of government grants.
(iv) Whether the asset acquired at consessional rate or free of cost.

PROBLEMS AND SOLUTIONS


Problem 1: Koit Ltd. receives ` 3,00,000 as share application money from Govt. Comment whether it is a
Government Grants as per AS -12?
Solution: Government participation as equity holders is not covered by AS - 12. Government is also one of
the shareholders of the company hence the receipt is a contribution towards capital.
Problem 2: On 1st April 2010 Sizzler Ltd purchases an asset at ` 10,00,000, Salvage value ` 1,00,000,
life 5 years. Government grant recd in September 2010 ` 3,00,000. Due to non – compliance of some conditions
attached with the asset the whole grant become refundable in the year 2012-13. Compute Depreciation in the
year 2012-13. (Assume SLM depreciation). Sizler Ltd. adopts capital approach.
Solution:
Cost of asset (1.4.2010) 10,00,000
Less: Grant received -3,00,000
Net cost 7,00,000
Depreciation 2010-11 -120000
Depreciation 2011-12 -120000
WDV on 1.4.2012 4,60,000
Add: Refund of grant 3,00,000
Revised WDV 7,60,000
Depreciation 2012-13 2,20,000

*Depreciation 2010-11 = (10-3) – 1 / 5 = 1.2


Depreciation for 2012-13 = 7.6-1 / 3 = 2.2

Problem 3: Fantastic Ltd. has received ` 50,00,000 as grant towards Land purchased on 1.4.2007. Cost of Land is
` 5 crores. The operating cost in the area is likely to be higher due to lack of adequate infrastructural facilities. The
factory commences production on 1.4.2007. The projects for building necessary infrastructure are expected to be
completed by the end of 2009- 2010. The incremental costs incurred due to lack of infrastructural facilities were
` 50 lacs, ` 50 lacs, ` 25 lacs.
Show the accounting treatment as per AS-12 for all the years.
Solution: As the expenses for non – depreciable asset (Land) is going to be incurred in future, but the grant
is already received by the company hence grant has to be deferred:
(CONDITIONAL GRANT)
Bank A/C. Dr. 50,00,000
To Deferred Grant A/C. 50,00,000
Ratio of deferment is calculated below:
AS-12: Government Grants 105

For 2007-2008:
= = 40%

Grant transferred to Profit and Loss from Deferred Grant = 50,00,000 × 40% = ` 20,00,000
For 2008-2009: = = 40%

Grant transferred to Profit and Loss from Deferred Grant = 50,00,000 × 40% = ` 20,00,000
For 2009-2010: = = 20%

Grant transferred to Profit and Loss from Deferred Grant = 50,00,000 x 20% = ` 10,00,000
Problem 4: Top & Top Limited has set up its business in a designated backward area which entitles the
company to receive from the Government of India a subsidy of 20% of the cost of investment. Having fulfilled
all the conditions under the scheme, the company on its investment of ` 50 crore in capital assets, received
` 10 crore from the Government in January, 2005 (accounting period being 2004-2005). The company wants
to treat this receipt as an item of revenue and thereby reduce the losses on profit and loss account for the year
ended 31st March, 2005.
Keeping in view the relevant Accounting Standard, discuss whether this action is justified or not.
(C.A. Final Nov. 1995 & May 2005) (3/4 Marks respectively)
Solution: As per para 10 of AS 12 ‘Accounting for Government Grants’, where the government grants are of the
nature of promoters’ contribution, i.e. they are given with reference to the total investment in an undertaking
or by way of contribution towards its total capital outlay (for example, central investment subsidy scheme)
and no repayment is ordinarily expected in respect thereof, the grants are treated as capital reserve which
can be neither distributed as dividend nor considered as deferred income.
In the given case, the subsidy received is neither in relation to specific fixed asset nor in relation to revenue.
Thus it is inappropriate to recognise government grants in the profit and loss statement, since they are not
earned but represent an incentive provided by government without related costs. The correct treatment is
to credit the subsidy to capital reserve. Therefore, the accounting treatment followed by the company is not
proper.
Problem 5: Dark Ltd. has received grant of ` 20 lacs under the Government Subsidy Scheme for acquiring
imported machinery for setting up an oil exploration plant and the entire grant received is credited to profit
and loss account. (C.A. Final Nov. 2006 Audit)(4 Marks)
Solution: The treatment given is wrong, because when we received grant against machine then it called
capital grant. So it should be reduced from the cost of machinery (capital approach) or transfer to Balance
Sheet on liabilities side as deferred income (income approach). It is incorrect to transfer the grant to profit
and loss a/c.
Problem 6: A Company receives a grant from the State Government as compensation for loss of stocks due
to unseasonal floods. The entire grant received is credited to “Capital Reserve”. Comment on the accounting
treatment. (CA Final Audit)
Solution: Where the grant is in the nature of compensation for any past losses (sick companies) or immediate
financial support (flood affected company) then such grant should be treated as per AS – 5 (Extra-ordinary
activity). In the present case the company has to show such govt receipt as extraordinary receipt.
Problem 7: Primus hospitals Ltd. had acquired 40 units of Doppler scan machines from Holiver USA at a
cost of US$ 1,65,100 per unit in the beginning of Financial Year 2008-09. The prevailing rate of exchange was
` 50 to the US$. The acquisition was partly funded out of a government grant of ` 5 crores. The grant relating
to such machines was given with a rider that in the event of a change in management, the entity is bound
106 Accounting Standards

to return the grant. In April 2011, 51% control in the company was taken over by an overseas investor. The
expected productive period of such an asset is normally reckoned at 5 years. The depreciation rate adopted
was 20% p.a. S.L.M. basis. The company had incurred expenditure of US$ 4,000 towards bank charges and
` 7,500 per unit as sea freight. You are also informed that neither capital reserve nor deferred income account
has been maintained by the company. You are required to suggest the accounting treatment as a result of the
return of the grant, in the light of the relevant AS. (CA Final Nov 2011 Marks 5)
Solution: The grant is received towards fixed asset. Capital approach is adopted: (` in lakhs)
Cost of asset (dopler machines) 165100 × 40 @50 3,302
Add: Freight and other charges 4000 @50 + 7500 × 40 5
Total 3,307
Less: Grants received –500
Cost as per AS-10 2,807
Less: Depreciation for 3 years 2807 × 20% × 3 –1684.2
Carrying amount as on 1/4/2011 1,123
Add: Grant refunded 500
Revised carrying amount 1,623
CHAPTER 13
AS-13: Investment Accounts

Chapter Outline
vv Scope and Objective vv Investments in Variable Income Bearing Securities
vv Meaning of Investments vv Valuation of Investments in Various Cases
vv Types of Investments vv Sale of Investments
vv Cost of Investments vv Reclassification of Investments
vv Investments in Fixed Income Bearing Securities vv Disclosures

Scope and Objective


Non – Applicability:
◘◘ Operating or Finance leases (Operating & Finance lease-covered by AS19)
◘◘ Bases for recognition of Dividend, Interest, Rentals earned on Investments. (AS-9)
◘◘ Investments of retirement benefit plans and life insurance enterprises (covered by AS 15 and IRDA
Act)
◘◘ Mutual Funds (covered by SEBI/Guidance notes)
◘◘ Banks (covered by RBI norms)
◘◘ Public Financial Institutions. (SEBI/RBI)
◘◘ Asset management Companies (covered by SEBI/Guidance notes)
◘◘ Venture capital funds (Limited revision)

Meaning of Investments
It is an asset held for earning income by way of a) Dividend, b) Interest, c) Rentals, d) Capital Appreciation. It
can be Shares, Debentures and Investments in Properties. Assets held in stock in trade are not investments.
Investments are broadly classified into Current investments and Long-term investments. Further
classification can be:
(a) Investments in Government Securities
(b) Investments in Shares, Debentures or Bonds
(c) Investments in Investment properties
(d) Others
108 Accounting Standards

DO YOU KNOW
The New Schedule III has a welcome and broad disclosure of Investments in terms of equity, preference, debt,
investments in trust, investments in govt securities, partnership firms, quoted, unquoted etc.

Does AS 13 applies to dealers in Shares and Debentures?


Shares and Debentures held in stock in trade is not specifically covered by AS-13. But the accounting
treatment seems to be similar to what we do in current investments. So, though not specifically mentioned
but a securities dealer can treat his stock as current investments.

Types of Investments
(i) Current Investments: Such investments are: readily realizable in the market, and held for less than
1 year from the date of investments made. Investments in marketable securities are normally current
investments. Such investments are made out of surplus cash. Purpose of Current Interest is to tide up
with the working capital of the business.
(ii) Long Term Investments: Investments other than current investments are Long-term investments.
Intention of long-term investments is to gain significant influence or control. Investments held
primarily to protect, facilitate or further existing business or trading relations are often called as trade
investments. Trade investments are held for a longer period of time; hence they are in the nature of
Long Term Investments. Example: Investments made by a holding company in its subsidiary.
(iii) Property Investments: Fixed assets like land and building kept as investments not for running
own business or self occupied and it is held for earning Rent or Capital Appreciation are Investment
Properties. They are held for a number of years to generate income and capital gain. As they are
permanently held for earning income, they are also classified as Long Term Investment.

Cost of Investments
Cost of Investments = Purchase Cost + Brokerage + Transfer Fees + Stamp duty + Service taxes – Pre Interest/
Dividend. It even includes fees paid to SEBI and merchant bankers and advertisement costs if incurred for
acquiring shares. Further legal costs incurred to acquire shares and any fees paid to CLB/High Court is also
capitalized.

Cost of Investments in Case of Consideration Other Than Cash


i) Direct issue of Securities: Cost of Investment = Fair value of the securities issued. Fair value may
or may not be equal to Face Value of securities. The statutory authorities determine fair value in
appropriate cases.
ii) Investment is acquired by exchange of another asset: Cost of Investment = Fair value of the asset
given up or = to Fair value of investments acquired if it is more evident and convincing.

Investments in Fixed Income Bearing Securities


Fixed Income bearing securities are Debentures, Bonds etc.
i) Cum Interest: Generally Interest received on account of Debentures/bonds/Loans etc. held as
investments are regarded as Income. But sometimes the Interest Income is regarded as a recovery
against cost of Investment. If a person purchases debentures in the middle of the dates of Interest
accrual and interest payment then in such case unpaid Interest up to date of acquisition of
Debentures is included in the Cost of Investments. Hence such type of Investments is called Cum
Interest Investments. The price paid by the buyer for the investment is inclusive of interest element.
AS-13: Investment Accounts 109

The total Interest received after the acquisition of Debentures (on respective due dates) are to be
allocated between pre acquisition and post acquisition Interest.
Total Interest received

Pre acquisition Income Post acquisition Income

Go to reduce cost treated as revenue income

ii) Ex - Interest: In case of Ex – Interest transaction the price paid by the buyer to the seller is itself
the pure cost of investment. Interest accrued up to the date of purchase is not included in the cost
of Investment. But the buyer of the investment has to pay for the interest accrued up to the date of
purchase separately.

Investments in Variable Income Bearing Securities


Equity Shares: Dividend received on Equity Shares is not fixed or pre – determined. Hence concept of Cum –
Dividend or Ex – Dividend cannot be possible in case of Equity Shares.
If the Dividend is declared from pre-acquisition profits then the dividend received by the purchaser of
the Shares is to be treated as pre-acquisition dividend and hence such a dividend should go to reduce cost of
Investments.
◘◘ Equity Shares issued in different forms and their accounting treatment:
(i) Bonus Shares: Bonus Shares are the extra/gifted shares received by the shareholders from the
company without paying anything.
◘◘ Cost of Bonus Shares: Cost of Bonus Shares for the shareholder will be NIL.
(i) Right Shares: These are the shares issued by the company to its existing shareholders on a
concessional basis. The cost incurred to acquire the right shares will be added to the costs of
investments. The Shareholder can even renounce (sell) partly or fully the shares received on
right basis.

Cost of Right Shares acquired from the point of view of

(i) Original Shareholder (ii) Any person other than the original Shareholder


Simple Purchase Cost Purchase Cost of Acquisition
+
Premium paid to the original Shareholder
for the right shares

Sale of Rights
As a general rule sale proceeds of rights should be transferred to Profit and Loss Account.
◘◘ Exception:
When both the following conditions are satisfied then treatment of sale proceeds of rights differs:
(i) Original shares on which the shareholder gets the right shares was acquired on cum-right basis
(recent purchase after announcement of rights).
(ii) Market Value after the original shares becomes ex-right is lower than the cost of acquisition
110 Accounting Standards

◘◘ Sale proceeds then will be first applied towards reduction of carrying amount of investments to its
market value and balance sale proceeds is to be transferred to Profit and Loss Account..........(Para 13)

Valuation of Investments in Various Cases


i) If Investments are held as Property Investments: [Cost of purchase – Depreciation – Amortization
– Impairment] (in 2009 properties in Dubai were impaired by atleast 50%).
ii) If Investments are held as Current Investments:
1. Current Investments are always valued at Cost or M.V. (market value) whichever is less. Such
amount as valued at Cost or M. V. whichever is less is called Carrying Cost of Investments.
2. If M. V. is less than Cost then (Cost – M. V.) is known as diminution in value of shares, which is
debited to Profit and Loss A/c………………………Prudence!
3. If there are various types/lots of shares or shares of different companies then either Individual
method or Category method is to be applied:
4. Global Method is not appropriate for valuing current investments. Category method may be
applied for same group of investments.
5. Individual method is the more prudent and appropriate to be applied:
Reduction in the value of investments is to be charged to Profit and Loss Account and reversals
credited to Profit and Loss Account.
iii) If Investments are held as Long Term Investments: Since long term Investments are not intended
for disposal in the near future, the market values of such investments are not relevant. For this reason
LTI is to be shown at “Cost” only. The carrying amount of Long Term Interest is therefore determined
on an individual investment basis. However a provision for diminution in value of shares can be
made if the decline in shares is other than temporary. This means if there is any “permanent fall” in
the value of investments then show the Long Term Investments at M.V. or Cost whichever is less. But
it all depends upon the decision of the Board of Directors (Also many other factors contributes for
writing off LTI).
Reduction in the value of investments is to be charged to Profit and Loss and even reversals (recovery)
of investments should be credited to Profit and Loss Account.

Sale of Investments
(I) Computation of Profit and loss on Sale of Investments:
ww Profit = Net Sale proceeds – Carrying Cost (such profit or loss as per AS-13 is transferred to
profit and loss account)
ww Net Sales proceeds = Sale value – Expenses incurred on sale
ww Carrying Cost of Investments = Costs of Investments - Diminution in the value of shares
(II) Computation of Net Cost on Sale of Investments when only a portion of Investments are sold:
ww As per AS 13 when sale of investment takes place Average Cost method is to be followed for
determining Cost of Investments.
ww Other methods for determining Cost of Investments are 1) FIFO method, 2) Specific Identification
method. But such methods are useful for Income Tax purposes and not for accounting purpose.

Reclassification of Investments
Sometimes a Company may classify its Investments from Current into Long term Investments & vice versa.
For such reclassification Special Resolution at the Board Meeting has to be passed.
AS-13: Investment Accounts 111

Following are the provisions made by AS 13 with respect to Reclassification of Investments.


i) Long term Investments are reclassified as Current Investments:
If Long term Investments are reclassified as Current Investments then transfer has to be made at
Cost or Carrying Amount whichever is less on the date of transfer. Difference is to be transferred to
Profit and Loss Account in the name of Diminution in the value of Shares.
ii) Current Investments are reclassified as Long Term Investments:
If Current Investments are reclassified as Long Term Investments then transfer has to be made at
Lesser of Cost or Fair Value on the date of transfer. Difference is to be transferred to Profit and Loss
Account in the name of Diminution in the value of Shares.

Disclosures
In addition to providing guidance for accounting treatment, AS 13 also gives a list of disclosure to “Notes to
Accounts” which are attached to and form part of Financial Statements. The disclosures as per AS 13 are:
a) The accounting policies for determination of carrying amount of investments.
b) Classification of investments (whether Current/Long Term or Shares or Governments securities or
Inv. Properties etc.)
c) The amount included in the Profit and Loss Statement for:
ww interest, dividends(showing separately dividend received from subsidiary companies), and
rentals from long term as well as current investments separately.
ww profit and loss on sale of current investments and changing in carrying cost on such investments.
ww profit and loss on sale of long term investments and changing in carrying cost on such investments.
ww gross Income received, T.D.S. and Net Income thereof.
d) Tax deducted at source to be shown under Advance Taxes Paid.
e) Significant restrictions on the right of ownership, reliability of investments or remittance of income
and proceeds of disposal.
f) The aggregate amount of quoted or unquoted investments, giving the aggregate market value of
quoted or unquoted investments.

DO YOU KNOW
Till MCA will not mandate AS-30/31/32, companies are required to follow AS-13 and not AS-30/31/32.

DO YOU KNOW
In IFRS, AS-13 type of standards are abolished. They adopt standards similar to AS-30/31/32. For Property
Investments IFRS have a seperate Standard IAS-40.
112 Accounting Standards

PROBLEMS AND SOLUTIONS

Problem 1: Shri Gajab acquired 2,000 Shares of Acme Products Ltd. from a broker @229/-. The Broker
charged him 1%. Service tax 5% on brokerage and stamp duty ` 500. Compute Cost of Investments as per
AS-13.
Solution:
Basic Cost (2,000 × 229) 4,58,000
+ Brokerage 4,580
+ Service Tax (5% on 4,580) 229
+ Stamp Duty 500
4,63,309
Problem 2: COSA Ltd. gives an offer of 2,000 Equity Shares on Right basis @160/- to Shree. Shree renounces
30% of the right shares to Hamd by charging a premium of ` 12 per share. Compute Cost of Right Shares
acquired by Shree &Hamd.
Solution:
Cost of Right Shares
(i) For Shree = 2,24,000 (2,000 Shares × 70% × 160/-)
(ii) For Hamd = 1,03,200 (2,000 Shares × 30% × {160 + 12})
Problem 3: Natrajan is expected to get 5,000 shares on right basis at ` 90. He sells all the rights @ ` 55. How
to account for the sale proceeds. Please advice.
Solution: In the absence of any information full sale proceeds of rights sold is to be credited to profit and loss
account. Hence ` 2,75,000 is to be credited to Profit and Loss Account.
Problem 4: Maazaa Limited’s Share Capital of 40,000 ` 100 equity shares. Maazaa Limited offered 1 right
share for every 4 held at 135/-.Immediately after the offer, an investor Mr. Mehta purchased 1,000 such shares
cum – right @150/-. Mr. Mehta renounced all the rights offered to him. The market value of MaazaaLimited
shares came down to ` 148/- immediately after the offer. The sale proceeds of rights were `3,000.
Show the entries in the books of Mr. Mehta.
Solution: As per para 13 of AS-13 where the investments are acquired cum right basis and the Market Value
of investments after right is lower than the cost of acquisition, then sale proceeds of rights should be applied
towards the Cost of investments to bring down it to the market value.
Ex – right (after right) actual market value of investments = 1,000 x 148 = ` 1,48,000
Cost of acquisition, = 1,000 x 150 = ` 1,50,000
Difference = ` 2,000 i.e. sale proceeds applied towards cost of investments.

Books of Mr. Mehta


Investments in Maazaa Ltd Dr. 1,50,000
  To Bank 1,50,000
Bank Dr. 3,000
  To Renouncement Fees 3,000
Renouncement Fees Dr. 3,000
  To Investments in Maazaa Ltd 2,000
  To Profit and Loss Account 1,000
AS-13: Investment Accounts 113

Problem 5: Gajini Ltd has 25% shares in Rajini Ltd acquired in the year 2006-07 @24/-. On 31/3/2009
shares of Rajini Ltd valued at stock exchange @20. Gajini Ltd wants to value investments in Rajini at ` 20/-.
Please advice.
Solution:
Valuation of Investments : AS 13 on “Accounting for Investments” requires investments to be classified
as long term and current investments distinctly in its financial statements. The investments in shares of
associate companies can very well be considered as trade investments since they would not be intended to
be liquidated within a period of one year from its acquisition. Hence they would be classified as long term
investments.
Investments in associates 25% is enough. They are LTI. Temporary fluctuations should not affect
investments made by G Ltd in R Ltd. Therefore LTI held by G Ltd in R Ltd is c/f @COST.
Problem 6: On 1.07.2007, A Ltd. acquired 2,000 shares in B Ltd. at cum-dividend price of ` 50,000. B Ltd.
declared and distributed 20% dividend on 30/09/07 in respect of year 2006-07 on its Share Capital, which
consisted of 20,000 shares of ` 10 each. On the same day, B Ltd. declared and distributed 10% interim dividend
for half year ended 30/09/07. Pass journal entries.
Solution:
1.7.2007 Investments in B Ltd Dr. 50,000
  To Bank 50,000
30.9.2007 Bank Dr. 6,000 (20000 × 20% + 20000 × 10%)
  To Dividend 6,000
31.3.2008 Dividend Dr. 6,000
  To Investments in B Ltd 5,000
  To P/L a/c. 1,000
(final dividend is full pre and interim dividend is ½ pre and ½ post)
Problem 7: Following is the Investment made by Ranka Ltd. as on 31.03.2005.
Type of Share Quantity Cost Market Value
(i) Equity Shares:
X Ltd. 250 124 175
Y Ltd. 400 106 156
Z Ltd. 200 97 95
(ii) Preference Shares:
F Ltd. 400 102 88
M Ltd. 10 115 149
R Ltd. 55 116 121
Value the investments as per Global/Category/Individual method.
Solution:
(I) Global Method: In case of global method, cost column & market value column are totaled up and
then Cost & M. V. are compared, whichever is less is shown in Balance Sheet. In the given problem
global method cannot be applied because there are various types of Investments. Thus, Category or
Individual method may be applied.
As per Para 31 of AS- 13 Global method should not be followed. Companies can use either Category
or Individual investment method.
114 Accounting Standards

(II) Category Method:

Type of Share Quantity Cost M. V. Amount taken for Valuation =


C.A./Cost
(i) Equity Shares:
X Ltd. 250 31,000 43,750
Y Ltd. 400 42,400 62,400
Z Ltd. 200 19,400 19,000
Sub – total: (A) 92,800 1,25,150 92,800 (Cost or M.V. whichever is less)
(ii) Pref. Shares:
F Ltd. 400 40,800 35,200
M Ltd. 10 1,150 1,490
R Ltd. 55 6,380 6,655
Sub – total: (B) 48,330 43,345 43,345 (Cost or M.V. whichever is less)
Grand Total: (A + B) 1,36,145

(III) Individual Method:

Type of Share Quantity Cost M. V. Amt. taken for Valn. = C.A./Cost


(i) Equity Shares:
X Ltd. 250 31,000 43,750 31,000
Y Ltd. 400 42,400 62,400 42,400
Z Ltd. 200 19,400 19,000 19,000
Sub – total: (A) 92,800 1,25,150 92,400
(ii) Preference Shares:
F Ltd. 400 40,800 35,200 35,200
M Ltd. 10 1,150 1,490 1,150
R Ltd. 55 6,380 6,655 6,380
Sub – total: (B) 48,330 43,345 42,730
Grand Total: (A + B) 1,35,130

Problem 8: As a Statutory auditor of a Public, how would you deal with the following:
The company had subscribed to shares of associate company amounting to ` 5 crores. These associate
companies have incurred substantial losses and have been referred to BIFR for being declared as sick
companies. The company does not want to provide for the loss.
(C.A. Final –May 1998, May 2001) (Marks 5)
Solution:
Valuation of Investments : AS 13 on “Accounting for Investments” requires investments to be classified
as long term and current investments distinctly in its financial statements. The investments in shares of
associate companies can very well be considered as trade investments since they would not be intended to
be liquidated within a period of one year from its acquisition. Hence they would be classified as long term
investments.
AS 13 states, “long-term investments should be carried in the financial statements at cost. However,
provision for diminution shall be made to recognise a decline, other than temporary, in the value of the
investments, such reduction being determined and made for each investment individually”. In the instant
case, these associate companies have incurred substantial losses and have been referred to BIFR for being
AS-13: Investment Accounts 115

declared as sick companies. The net worth of these companies would have been wiped out resulting in a fall
in the value of the investments. Therefore, such fall cannot be merely temporary as the companies could take
a long time to turn around (if at all) and again have a positive net worth. The auditor would therefore have to
qualify his report by saying that no provision for diminution for fall in the value of investments as required
by AS 13 has been made and to that extent the profits and reserves have been overstated.
Problem 9: During the year 31.03.2002, Natal Ltd. have acquired following shares of Beetel Ltd.
Date of Acquisition No. of Shares Cost per Share (`)
10.05.2001 200 50.00
16.06.2001 500 70.00
22.08.2001 6,000 75.00
11.01.2002 100 Bonus
03.03.2002 500 75.00

On 21st March 2002, the Company sold 2,000 shares @ ` 94/-.


Compute: 1) Profit & Loss of Shares sold. 2) Cost of unsold Shares.
Solution:
As per AS 13, Profit on Sale of Investment is to be determined with reference to Average Cost of Investments.

Date of Acquisition No. of Shares Cost per Share Total Cost


(`) (`)
10.05.2001 200 50.00 10,000
16.06.2001 500 70.00 35,000
22.08.2001 6,000 75.00 4,50,000
11.01.2002 100 Bonus 0
03.03.2002 500 75.00 37,500
Total 7,300 5,32,500

As per AS 13:

Average Cost = = 72.95/-

1) Profit & Loss of Shares sold:


Sale Value of 2,000 Shares (Sales) ` 1,88,000 (2,000 × 94/-)
Less: Cost of Shares sold ` 1,45,900 (2,000 × 72.95/-)
Profit on Shares sold ` 42,100
2) Cost of unsold Shares:
Total Cost of all Shares ` 5,32,500
Less: Cost of Shares sold ` 1,45,900
Cost of unsold Shares ` 3,86,600
Problem 10: Lacto Ltd. held following investments of Zed Ltd. as Long term Investments.
Equity Shares: Cost ` 52,000, Carrying Cost ` 51,000, Market Value ` 56,600.
As on 30.09.2005 the Directors want to reclassify the Long term Investments as Current Investments.
(CA Final May 2010 OS)
116 Accounting Standards

Solution: If Long term Investments are reclassified as Current Investments then transfer has to be made at
Cost or Carrying Amount whichever is less on the date of transfer. Difference is to be transferred to Profit and
Loss Account in the name of Diminution in the value of Shares.
In the present problem Carrying Amount = ` 51,000 & even lesser of Cost or Carrying Amount is also
` 51,000, Hence no adjustment is to be done.
P roblem 11: Lacto Ltd. held following investments of Zed Ltd. as Current Investments.
Equity Shares: Cost ` 52,000, Carrying Cost ` 51,000, Market Value ` 56,600.
As on 30.09.2005 the Directors want to reclassify the Current Investments as Long Term Investments.
(CA Final May 2010)
Solution: If Current Investments are reclassified as Long Term Investments then transfer has to be made at
Lesser of Cost or Fair Value on the date of transfer. Difference is to be transferred to Profit and Loss Account
in the name of Diminution in the value of Shares.
Carrying amount = ` 51,000
Valuation = Lesser of FV or Cost = Lesser of 56,600 or 52,000 = ` 52,000
Value of investments is to be increased by ` 1,000
P roblem 12: On 01.01.2007 Bevan Limited purchase a flat in Ratnajar Co-op. Soc. Limited for ` 30 lakhs.
However for purchasing the flat, it was necessary to obtain membership of the society priced at ` 1 lakh. The
company acquired shares ` 50,000 as per the bye laws of co-operative societies act. The company charged off
the membership price of ` 1 lakh to P/L account and disclosed the cost of shares as separate investments?
Comment?
Solution: The treatment of charging membership ` 1,00,000 is incorrect. Also the cost of shares should be
capitalized to the cost of property. Any cost incurred to acquire the investment property is to be added to the
Cost of Property. The total cost of investment property = ` 31,50,000.
P roblem 13: Happy Limited acquired 20,000 equity shares of Pretty Limited of FV ` 10 by issuing 25,000
equity shares of its own company. FV and MV of Happy Limited is` 10 and ` 40 respectively. Compute Cost of
Investments as per AS-13.
Solution:
Investments in Pretty Limited Dr. 10,00,000
To Equity Share Capital 2,50,000 (25,000 × 10)
To Securities Premium 7,50,000 (25,000 × 30)
(Costs of investments acquired for non – cash consideration should be taken as the value of shares issued
Para 10 of AS-13)
P roblem 14: X Limited on 1.1.2012 had made an investment of ` 600 lakhs in the equity shares of Y limited
of which 50% is made in the long term category and the rest as temporary investment. The realisable value
of all such investment on 31.3.2012 become ` 200 lakhs as X Limited lost a case of copy right. How will you
recognize the reduction in financial statements for the year ended on 31.3.2012?
(CA Final May 2012 Marks 4)
Solution: Current Investments are always valued at Cost or M.V. (market value) whichever less is. Since long
term Investments are not intended for disposal in the near future, the market values of such investments are
not relevant. For this reason LTI is to be shown at “Cost” only. However a provision for diminution in value of
shares can be made if the decline in shares is other than temporary.
Accordingly loss of ` 100 relating to Current investment should be debited to Profit and Loss. The
loss amounting to Long Term Interest ` 100 should not be written-off. Hence the carrying amount of the
investments as on 31-3-2012 is: [`600 lakhs – `200 lakhs] = `400 lakhs.
CHAPTER 14
AS-14: Amalgamation
and Absorption

Chapter Outline
vv Meaning of Amalgamation vv Purchase Consideration
vv Types of Amalgamation vv Treatment of Reserves Specified in
vv Accounting treatment of Amalgamation a Scheme of Amalgamation
in the Nature of Merger vv Disclosures
vv Accounting Treatment of Amalgamation
in the Nature of Purchase

Meaning of Amalgamation
The ‘term amalgamation is defined as “an amalgamation pursuant to the provisions of the Companies Act,
1956, or any other statute which may be applicable to companies”
An amalgamation is an arrangement whereby a company (called ‘transferor Company) is amalgamated
into another company (called ‘Transferee Company’) and loses its existence. In some amalgamation, there
may be more than one Transferor Company.

Types of Amalgamation
The practice of amalgamation of business enterprises has been in existence for a very long time. Earlier, most
amalgamation used to be principally a means by which one company acquired the net assets of another’
company. The company whose assets were acquired lost its existence, its shareholders were paid off out of
the consideration received from the acquiring company, and they ceased to have any further stake in the risks
and rewards of the business of the acquired company. However, in recent times, amalgamation has often been
used principally as a means by which the resources of two (or more) companies are pooled together. In such-
cases, even though one of the companies loses its corporate existence, the business of the acquired company is
continued by the acquiring company. Moreover, the shareholders of the acquired company are allotted shares
in the acquiring company. As such, they have a continuing interest in the risks and benefits of the acquiring
company Thus, the substance of such amalgamation is that the businesses of the acquired and the acquiring
companies continue to be run as before, but under a single corporate name. AS 14 recognizes the fact that
amalgamation may be of different nature - some of them may be really in the nature of purchase while the
others represent a genuine pooling of shareholders. From this angle, AS 14 classifies amalgamation into two
categories: (a) amalgamation in the nature of merger, and (b) amalgamation in the nature of purchase.
118 Accounting Standards

An amalgamation in the nature of merger is one, which satisfies all the following conditions.
(a) All the assets and liabilities of the transferor company (i.e., the company which is amalgamated into
another company, the latter being called the transferee company) become, after amalgamation, the
assets and liabilities of the transferee company.
(b) Shareholders holding not less than 90% of the face value of the equity shares of the transferor
company (other than, the equity shares already held therein, immediately before the amalgamation,
by the. transferee company or its subsidiaries or their nominees) become equity shareholders of the
transferee company by virtue of the amalgamation.
(c) The consideration for the amalgamation receivable by those equity share holders of the transferor
company who agree to become equity shareholders of the transferee company is discharged by the
transferee company wholly by the issue of equity shares in the transferee company except that cash
may be paid in respect of any fractional shares.
(d) The business of the transferor company is intended to be carried on, after the amalgamation, by the
transferee company.
(e) No adjustment is intended to be the made to the book values of the assets and liabilities of the
transferor company when they are incorporated in the financial statements of the transferee
company except to ensure uniformity of accounting policies.
From the above, it can be seen that in an amalgamation in the nature of merger, there is a genuine
pooling not merely of the assets and liabilities of the amalgamation companies but also of the
shareholders of both the transferee and the transferor companies.

Accounting treatment of Amalgamation in the Nature of Merger


The standard requires that amalgamation in the nature of merger should be accounted for under the pooling
of interests method.
Under the Pooling of Interests Method, an amalgamation is accounted for as if the separate businesses
of the amalgamating companies were intended to be continued by the transferee company. Accordingly, only
minimal changes are made in aggregating the individual financial statements of the amalgamation companies.
The application of this method involves the following.
(1) In preparing the .transferee company’s financial statements, the assets, liabilities and reserves
(whether capital or revenue or arising on revaluation) of the transferor company should ‘be recorded
at the existing carrying amount and in the same form as at the date of the amalgamation. For example,
land of the transferor company should be clubbed with the land of the transferee company and
shown at combined figure. Similarly, general reserve of the transferor company should be clubbed
with the general reserve of the transferee company. The balance of the profit and loss account of the
transferor compare should be aggregated with corresponding balance of the transferee company or
transferred to General Reserve, if any. Thus, the financial statements of the transferee company after
the amalgamation would represent, more or less, a line-by-line addition of the financial statements
of the amalgamating companies.
(2) If, at the time of the amalgamation, the transferor and the transferee companies have conflicting
accounting policies, a uniform set of accounting policies should be adopted following the
amalgamation the, effects on the financial statements of any changes in accounting policies should
be appropriate! disclosed.
(3) The difference between the amounts recorded as share capital issued (plus any additional
consideration in the form of cash or other assets) and the amount of share capital of the transferor
company should be
PC x
(–) Net-assets (x)
Adj. in Reserves x
AS-14: Amalgamation and Absorption 119

DO YOU KNOW
Merger method is now no more acceptable/adopted under IFRS – 3.

Accounting Treatment of Amalgamation in the Nature of Purchase


The standard requires amalgamations in the nature of purchase should be accounted for under the purchase
method. Under this method, the amalgamation is accounted for more or less in the same manner as normal
acquisition of assets.
1. In preparing the transferee company’s financial statement, the assets and liabilities of the transferor
company should be incorporated either at their fair values at the date of amalgamation or at their
existing carrying amounts.
2. Any excess of the amount of the consideration over the value of the net assets of the transferor
company acquired by the transferee company should be recognized in the transferee company’s
financial statements as goodwill arising on amalgamation. On the other hand, if the amount of the
consideration is lower than the value of the net assets acquired, the difference should be treated as
capital reserve.
3. The goodwill arising on amalgamation should be written off on a systematic basis. The period of
write-should not exceed five years unless a somewhat long period can be justified.
4. As a general rule, the reserves (whether capital or revenue or arising on revaluation) of the transferor
company should not be included in the financial statements of the transferee company. An exception
is, however, made in the case of statutory reserves. Statutory reserves are those reserves, which
have been created by the transferor company pursuant to the requirements of, or to avail of the
benefits under the Income-tax Act, 1961. Statutory reserves also include other reserves created in
the financial statements of the transferor company in terms of the requirements of other statutes.
The statute under which a statutory reserve is created may require the identity of such reserve to
be maintained for a specified period. The standard provides that where the requirements of the
relevant statute for recording the statutory reserves of the transferor company in the books of the
transferee company are complied with, such statutory reserves should be recorded in the financial
statements of the transferee company. The corresponding debit should be given to a suitable account
head (e.g., “Amalgamation Adjustment Account’), which should be disclosed as a part of “Non-current
Asset” in the balance sheet. When the identity of the statutory reserves is no longer required to be
maintained, both the reserves and the aforesaid account should be reversed.

Purchase Consideration
Purchase consideration for an amalgamation (whether in the nature of merger or purchase) means the
aggregate of the shares and other securities issued and the payment made in the form of cash or other assets
by the transferee Company to the shareholders of the transferor Company. It should include any non-cash
element at fair value. In case of issue of securities, the value fixed by the statutory authorities may be taken
to be the fair value. In the case of other assets, the fair value may be determined by reference to the market
value of the assets given-up. Where the market value of the assets given up cannot be reliably assessed, such
assets may be valued at their respective net book values.

Treatment of Reserves Specified in a Scheme of Amalgamation


Where the scheme of amalgamation sanctioned under a statute prescribes the treatment to be given to the
reserve of the transferor company after amalgamation, the same should be followed.
120 Accounting Standards

Disclosures
The standard requires the following disclosures to be made in the financial statements.
1. For all amalgamations, the following disclosures should be made in the first financial statements
mowing the amalgamation.
(a) Names and general nature of business of the amalgamating companies;
(b) Effective date of amalgamation for accounting purpose;
(c) The method of accounting used to reflect the amalgamation; and
(d) Particulars of the scheme sanctioned under a statute.
2. Amalgamations accounted for under the pooling of interests method, the following additional
disclosures should be made in the first financial statements following the amalgamation.
(a) Description and number of shares issued, together with the percentage of each company’s
equity shares exchanged to effect the amalgamation; and
(b) The amount of any difference between the consideration and the value of net assets acquired,
and the treatment thereof.
3. For amalgamations accounted for under the purchase method, the following additional disclosures
should be made in the first financial statements following amalgamation.
(a) Consideration for the amalgamation and a description of the consideration;
(b) The amount of any difference between the consideration and the value of net assets acquired,
and the treatment there of including the period of amortization of any goodwill arising on
amalgamation.
CHAPTER 15
AS-15 (Revised):
Employees Benefits

Chapter Outline
vv Basic objective of AS-15 (Revised) vv Accounting for Retirement Benefits
vv Accounting for Short Term Benefits vv Termination Benefits
vv Accounting for Long Term Benefits

Basic objective of AS-15 (Revised)


Accounting of wages and salaries is a straight forward exercise. But the retirement benefits requires too
much of complex calculations, discounting, actuarial valuation which is further open for debate. The objective
of AS-15 (Rev) is to provide the accounting and disclosure principle for payments made by employer to
employees (including absence leave, retirement benefits, bonus plan etc.).

Scope of AS-15 (Revised)


◘◘ short-term employee benefits, such as wages, salaries and social security contributions (e.g., contri-
bution to an insurance company by an employer to pay for medical care of its employees), paid annual
leave, profit-sharing and bonuses (if payable within twelve months of the end of the period) and non-
monetary benefits (such as medical care, housing, cars and free or subsidised goods or services) for
current employees;
◘◘ post-employment benefits such as gratuity, pension, other retirement benefits, post-employment life
insurance and post-employment medical care;
◘◘ other long-term employee benefits, including long-service leave or sabbatical leave, jubilee or other
long-service benefits, long-term disability benefits and, if they are not payable wholly within twelve
months after the end of the period, profit-sharing, bonuses and deferred compensation; and
◘◘ termination benefits.
Exclusion: AS does not cover:
a) Accounting for employee share-based payments’ (to be accounted as per Guidance Note Issued by
ICAI)
b) Accounting and reporting by employee benefit plans.

What are the Types of Employees Covered Under AS-15 (Revised)?


As per para 5…………….
◘◘ Benefits provided to employees or their family (spouse, parents, dependent children) or to some trust
on his behalf……….
122 Accounting Standards

◘◘ Employee may provide services to an enterprise on a full-time, part time, permanent, casual or tempo-
rary basis.
◘◘ Employees include whole-time directors and other management personnel.

Which Discounting Rate has to be Used for Discounting Long Term Payments?
As per AS-15 discounting rate to be used for payments made after 12 months period will be as per the yield
on Government bonds existing on the balance sheet date.
But as per IAS it can be even on high level quality corporate bonds.

Accounting for Short Term Benefits


Short-term employee benefits includes:
Wages, salaries and social security contributions (e.g., contribution to an insurance company by an
employer to pay for medical care of its employees), paid annual leave, profit-sharing, bonuses and Non-
monetary benefits (such as medical care, housing, cars and free or subsidised goods or services) for current
employees.
(i) Accounting for wages/salaries etc:
ww Straight forward accounting – No discounting or Actuarial Valuation
ww As a liability if unpaid
ww As an expense unless capitalized by any other standard (AS-10)
ww Excess paid to be shown as prepaid expenditure
(ii) Accounting for leave benefits/paid leaves:
It includes vacation for sickness, maternity, paternity.
(a) in case of non-accumulating compensated absences, no provision is required to be made at the
end of the year if leaves are not utilized. Example: paternity leaves cannot be carried forward.
(b) in the case of accumulating compensated absences, when the employees render service that
increases their entitlement to future compensated absences:
(i) If Vesting = In such case full provision is required to be made because such leaves unuti-
lized entitles an employee to cash payments at the year end.
(ii) If it is Non - Vesting = No cash entitlements here. In case of accumulating leaves but
non-vesting, provision is required to be made only for the leaves that are likely to be utilized
in future.
(iii) Accounting for bonus/profit sharing plans:
recognize when and only when
• the enterprise has a present obligation to make such payments as a result of past
events; ……(AS-29) and
• a reliable estimate of the obligation can be made…..(AS-29)
• A present obligation exists when, and only when, the enterprise has no realistic alter-
native but to make the payments…….(AS-29)
Obligation can arise because of constructive obligation (legal bonus) or out of any tradition
followed by the employer to pay the bonus.
• Disclosure:
– No specific disclosure.AS-18 Related Party disclosure –Payment to KMP.

Post Employment Benefits (PEB)


◘◘ PEB does not include Termination benefits (VRS).
◘◘ PEB is payable on or after the completion of employment – Retirement benefits.
AS-15 (Revised): Employees Benefits 123

◘◘ PEB Plans can be:


ww Defined Contribution Plan (DCP) or
ww Defined Benefits Plan (DBP)
ww DCP & DBP can be administered in the four ways:
A) State plans:
Established by Legislation to cover all enterprises and are operated by a National or local Government
or autonomous body. The obligation of the employer is restricted to the stipulated periodical
contribution. Nevertheless, State plan can also bear the characteristics of a ‘benefit’ plan
B) Multi-employer plans are operated and managed by third party entities: e.g. Group gratuity
fund may be managed by an Asset Management Co. Film\TV industries- these industries receive
contribution from business enterprises, invest these funds, and disburse the funds to employees,
without regard to the identity of the enterprise.
C) Group Administration Plan:
Enterprise may be inter-related to each other like parent\ subsidiary etc. Group Administration Plan
are aggregation of all plans of participating employers where by plans are operated & managed at
single point to save the cost of administration.
D) Insured Benefits Plans:
• The employer arranges with an insurer to manage the post employment benefits for employees.
• It is not possible to determine the characteristic of the plan, then, it may be considered as
“defined contribution’ plan”.
(i) DEFINED CONTRIBUTION PLAN
◘◘ Fixed contribution into a separate entity (fund) and will have no obligation to pay further contribution
even if fund does not hold sufficient assets to pay all employees benefits e.g. Provident Fund.
◘◘ The employer does not bear ‘Actuarial risk’- the benefit may accrue to an employee may be less than
expected. It also does not bear the ‘Investment risk’ – the value of assets may be inadequate to meet the
payments when they fall due.
(ii) DEFINED BENEFIT PLAN
◘◘ Under this plan (e.g. Gratuity),employee benefits are determined by two or more variable factors:
like length of service, last drawn salary, grade etc. The amount of benefits payable to an employee
at the time of retirement may not be easily determinable. Thus, the liability of the employer is not
constant but variable. So “actuarial” & “investment” risks are borne by the employer.
(iii) Accounting for Defined Contribution Plans
◘◘ Within 12 months:
ww Recognize the contribution payable in DCP in exchange for services as a liability or accrued
expense after deducting any contribution paid.
ww In the books of account DCP is charged as an expense .
ww If the accounting standard requires or permit the inclusion of the contribution in the cost of an
asset then AS10 or AS26 would be applicable.
◘◘ After 12 month of contributions: (Not applicable to level II and level III enterprises)
ww The amount of contribution payable should be discounted. The rate to be used to discount the
post employment benefit obligation (funded & un- funded) should be determined by reference
to market yields at the B/S date on Govt.Bonds.
◘◘ Disclosure:
ww In Profit and loss A/C, the amount of DCP should be recognized as an expense .
ww In notes to accounts an enterprise should disclose DCP for related parties e.g. KMP (AS-18).
124 Accounting Standards

(iv) Accounting for ‘Defined Benefits Plans’


Under defined benefit plans, an enterprise undertakes to pay specific amount of post-employment benefits
to its employees. If the contribution made to the fund during service period of an employee do not grow
into a sum sufficient for meeting the post-employment benefit committed, the employer is required to make
further contributions to make good for the short fall. In other words, actuarial risk and investment risk fall,
in substance, on the enterprise. The employers therefore ensure that:
a) Sufficient funds are available for meeting the post-employment benefit obligations on their falling due
and
b) Appropriate amount is charged against revenue in each of the accounting periods during which the
employees are entitled to the benefits during the period when the services are rendered.
To attain the twin objectives stated above, the employer usually takes the following steps.
a) The employer makes an assessment of post-employment benefit payable in respect of service
rendered by an employee during accounting period. Present value of this liability is the value of
service rendered by the employee. This is called the Current Service Cost. The employer charges
Current Service Cost to its income statement and recognizes the corresponding liability.
b) The sum retained by charge against revenue, are paid to a body (fund) to make investments. The
investments are called plan assets.
c) The liability for post-employment benefit payable grow in each accounting period due to interest and
fresh obligations arising from services rendered by employees. Accordingly, the employer estimates
interest costs and current service costs in each accounting period, adds them with the liability and
recognizes them by charge against income statements.
d) The employer makes fresh assessment of liability at the end of each accounting period. The difference
between revised value of liability and that as per step above is the actuarial gain/actuarial loss. The
employer writes down the liability and recognizes the actuarial gain/actuarial loss.
e) The plan assets grow in each accounting period due to interest earned and fresh investments. The
employer recognizes interest earned on plan assets and adds them with the previous value of plan
assets.
f) The employer makes fresh assessment of value of liability and plan assets at the end of each year. The
difference between fair values of plan assets estimated at the end of an accounting period and that
as per step above is the actuarial gain/actuarial loss. The employer writes up/writes down the plan
assets and recognizes the actuarial gain/actuarial loss.
g) On post-employment benefit falling due, the plan assets are sold to meet the obligation.
(v) Some important definitions:
Present value of a defined benefit obligation. The present value before deducting any plan assets or any
expected payments required to settle the obligation that has occurred as a result of the service of employees
in the current and previous periods.
Current service cost. The increase in the present value of the defined benefit obligation that occurs as a
result of employee service in the current period.
Interest cost. The increase in the period in the present value of the defined benefit obligation that arises
because the benefits payable are one year closer to the settlement of the scheme.
Plan assets. Those assets held by the employee benefit fund, including any qualifying insurance policies.
Return on plan assets. The interest, dividends, and any other income that is derived from the plan assets
together with any realized or unrealized gains or losses on those assets less the cost of administering the plan
and any tax payable by the plan.
Actuarial gains and losses. Experience adjustments and the effects of any changes in actuarial assumptions.
Experience adjustments are differences between the previous actuarial assumptions and what has actually
happened.
AS-15 (Revised): Employees Benefits 125

Past service cost. The increased present value of a defined benefit obligation for employee service in
previous periods that has arisen because of the introduction of changes to the benefits payable to employees.
Past service costs may be positive or negative depending on whether the benefits are improved or reduced.

DO YOU KNOW
Most of the investments made in plan assets are in fixed income securities like corporate bonds, govt securi-
ties, LIC policies.

(vi) Defined Benefit Plan: Recognition and Measurement


This requires complex calculation taking actuarial assumption and discounting such cost to present value.
The method of calculations involved in DBP is known as “projected credit unit method”
In Balance Sheet: Net defined benefit liability shall be presented as under:
Value of defined benefit liability for current and prior year services —
Less : Unrecognised past service cost —
Less : Fair value of the plan asset —
Net defined benefit liability —

Note: Negative result will mean asset and will be so recognized but this value not to exceed the present
value of economic benefit available in the form of refund from plan or reduction in future contribution.

In Profit and Loss Account: Net total of following should be recognized as an expense, unless some other
accounting std requires its inclusion as cost of some asset.
Current service cost —
+ Interest cost —
- Expected return on plan asset and on any reimbursement right —
+ Actuarial gain and losses —
- Past service cost to the extent required to be recognized —
+ Effect of curtailment/settlement —
+ Effect of amount not recognized as asset in case of negative result referred above —
Amount to be recognized as expense —
Above is as per Para 61 of AS 15 (Rev) which states that net total of the following be recognised as an
expense whereas para 118 states that AS doesn’t specify whether various components be shown as a
single item of income or expense on the face of Profit and Loss Account. It is desirable that ICAI clarifies
this.

(vii) Interval for Measurement: Detailed actuarial calculation for ascertaining present value of Defined
Benefits Obligation may be made at intervals not exceeding three years.
Fair value of plan assets should be ascertained at each Balance sheet date.
Present Value of Defined Benefit Obligation and Current Service Cost:
As these benefits are payable after a very long period, determination of its cost and its allocation over service
periods requires application of actuarial method and making actuarial assumption like, Year of service, rate
of increments, final salary, expected premature termination, Death rate, cost of Medical benefits, return
on investment etc. This is the responsibility of the management of the enterprise, who can take help of
professional actuaries.
126 Accounting Standards

(viii) Actuarial Valuation Method:


The projected unit credited Method (also known as accrued benefit prorated on service or Benefit divided by
number of years of services). Such benefits of respective years are then discounted to get the present value
obligation for current and prior years of services because such costs are payable in later years.
(ix) Actuarial Assumption:
Actuarial Assumptions are Classified into

Demographic Assumptions Financial Assumptions


(regarding current and former employees and
their dependents)
Example: Example:
1. Mortality (death rate) 1. Discount rate
2. Rate of employee turnover, disability and early retirement. 2. Future salary & benefit level.
3. Proportion of those entitled to benefit 3. Future medical Cost
4. Claim rate under medical plan 4. Expected rate of return on plan asset
Actuarial Assumption should be unbiased (i.e. neither imprudent nor excessively conservative) and
mutually compatible (i.e. they reflect economic relationship between factors such as inflation, rate of salary
increase, return on plan asset, discount rate etc.)
(x) Salaries, Benefits and Medical Cost :
Factors:
(a) Future salary increase in view of inflation, seniority, promotion, demand and supply of such
employees etc.
(b) Medical cost measurement should consider estimated future changes in the cost of medical service,
technological changes, health care utilization etc.
(xi) Actuarial Gains and Losses:
It is the result of difference between pervious actuarial assumption and what has actually occurred as also
changes in actuarial assumption.
Actuarial Gain or Losses is net of actuarial gain or loss on obligation and on plan asset and is to be
immediately taken to P and L a/c. ................... (para 92)
It arises because of changes in discount rate, expected return on plan asset, mortality rate, employee
turnover, salary, benefits, medical cost etc.
(xii) Past Service Cost:
It arises due to introduction of a defined benefit plan or changes in the benefit payable under an existing plan.
Past service cost which is already vested is written off immediately. Unvested past service cost is written
off over the period until vesting on straight line basis. Such unrecognized past service cost is deducted
while recognizing net employee benefit obligation in the balance sheet. Thus it is not reflected like deferred
expenditure.
Negative result due to reduction in benefit will also be given same treatment.
(xiii) Fair Value of Plan Asset:
Fair value of the plan asset is
i) Market Value
ii) If market is not available, then a valuation like present value of cash flow until disposal Discount rate
should be considering risk associated with plan asset and period up to disposal/maturity.
AS-15 (Revised): Employees Benefits 127

Plan Assets Exclude:


a) Unpaid contribution due by reporting enterprise.
b) Any non-transferable financial instrument issued by the enterprise and held by the fund.
Plan assets shall be net of its liabilities excluding liability on account of employee benefits.
Qualifying Insurance Policies are included in plan Asset. Others will be considered in Reimbursement
asset. Qualifying insurance policies are those (a) which are not issued by a related party insurer and (b) its
proceeds are available only for paying or funding employee benefits under a defined benefit plan, and are not
available for other creditors of enterprise even is bankruptcy.
(xiv) Reimbursement:
Any right for reimbursement for defined benefit obligation from another party including under insurance
policies which are not qualifying insurance policies is recognized only when it is virtually certain that the
other party will reimburse.
It is measured at fair value and is recognized as a separate asset in Balance sheet (i.e. not deducted from
present value of employee benefit obligation) In Profit and Loss account it can be reduced from the related
employee cost.
(xv) Return on plan Asset:
Expected return on plan asset is the return at the rate expected at the beginning of the year over the whole
life of the obligation. It is calculated making due consideration of contribution received and benefits paid.
Actual Return on Plan Asset is the difference between what the fair value was expected at the end of year
and what is the actual fair value.
Difference between expected return and actual return is actuarial gain/loss and is recognized in P and L.
account immediately.
(xvi) Curtailment and Settlements:
Curtailment Occurs:
(a) When there is present obligation to make material reduction in number of employee covered by plan
either statutorily or otherwise
Or
(b) When scheme is amended such that future services of employees will not be entitled for benefits or
for reduced benefits. Like: Due to closing of a plant, Discontinuation of an operation.
Settlement Occurs: When obligation for all or part of benefit under a plan are fully eliminated say by lump sum
cash payment etc.
In case of curtailments and settlements, re-estimate present value of obligation and fair value of assets.
Change in such valuations and the unrecognized past service cost will be the amount of gain or loss to be
recognized into P and L account.
It curtailment/settlement relates to only part of employees then only proportionate part of unrecognized
past service cost shall be eliminated.

Termination Benefits
◘◘ Termination benefits are payable as a result of :
ww Termination before the normal retirement date; or
ww Employees decision to accept VRS.
◘◘ Recognition, Measurement and Disclosure:
The benefit payable due to premature termination of employment either on the insistence of employer
or voluntarily by employee are known as termination benefit. Termination Benefits are usually
lumpsum payments payable under scheme, statutory regulations, under contractual requirement or
otherwise. It is immediately recognized as an expense. When this falls due more than 12 months after
the balance sheet date then the same are discounted.
128 Accounting Standards

Accounting Entries for Retirement Benefits


Current Service Cost:
Current Service cost xx → Charged to P/L
To PV of obligation xx
Interest on obligation:
Interest cost xx → Charged to P/L
To PV of obligation xx
Past service cost:
PSC (vested) xx → Charged to P/L
PSC (unvested but amortized portion) xx → Charged to P/L
PSC (unvested but unamortized portion) xx → (-) from PV of obligation
To PV of obligation xx
Benefits paid on retirement:
PV of obligation xx
To FV of plan assets xx
Contributions towards plan assets:
FV of plan assets xx
To Cash / Bank xx
Income accrued on plan assets:
FV of plan assets xx
To Interest income xx → (-) from P/L i.e. employee cost
Curtailment with Settlement:
PV of obligation xx
To Past Service Cost xx → Unamortized portion
To Cash / Bank xx
(the difference will be gain / loss goes to p/l)
Actuarial gain or loss:
Actuarial loss xx → Charged to P/L
To PV of obligation xx
(For actuarial gain the entry will be reversed)
FV of plan assets xx → Charged to P/L
To Actuarial gain xx
(For actuarial loss the entry will be reversed)
AS-15 (Revised): Employees Benefits 129

PROBLEMS AND SOLUTIONS

Problem 1: Tata Cosmetics Limited has 300 employees as on 31/3/2013. 5 days leaves are allowed in a
year which can be carried forward till next year only. The company adopts LIFO method of utilizing leaves
(first current year leave will be used and then last year). As on 31/3/2013, the average unutilized leaves per
employee are 2 days. Based on the past experience it is expected that out of 300 employees, 210 will take a
leave of 5 days in next year and remaining 90 employees will take 6 ½ days leave in the next year. You are
required to calculate the amount of liability to be provided as on 31/3/2013 based on number of days?
Solution: For 210 employees 5 days leave as per LIFO will be consumed for March 2014. Leave of 31/3/2013
will lapse.
For 90 employees 5 days leave as per LIFO will be consumed for March 2014. But the actual leave estimated
to be 6 ½ which means for 31/3/2013 the company has to provide 1 ½ x 90 employees = 135 days salary.
Problem 2: Kumar Ltd is an engineering industry. The company received an actuarial valuation for the first
time for its pension scheme which revealed as surplus of `6 lakhs. It wants to spread the same over the next
2 years by reducing the annual contribution to `2 lakhs instead of `5 lakhs. The average remaining life of the
employee is estimated to be 6 years.
You are required to advise the company. (C.A. Final May 2011) (Marks 4)
Solution: According to Para 92 of AS – 15, any actuarial gains and losses should be recognized immediately
in the Statement of Profit & Loss Account as income or expenses.
In the given case, the amount of surplus from pension scheme of ` 6 Lakhs is an actuarial gain, which
should be recognized as income in the Profit & Loss Account of the current year and not to be adjusted from
the amount of annual contribution.
Disclosure: The change relating to actuarial valuation for its pension scheme should be treated as a change
in an Accounting policy and disclosed in accordance with AS – 5. Disclosure required by AS – 15 should also
be made in the Financial Statements.
Problem 3: An actuary has made some changes in his assumptions for retirement benefits e.g. changes in
medical costs to be paid, changes in discounting rate etc. Are these changes in the assumptions changes in
accounting policy or changes in accounting assumption?
Solution: A number of actuarial valuation methods have been developed by the actuarial profession to
estimate employer’s obligations under defined benefit schemes. While these methods are primarily designed
to calculate funding requirements, they are also frequently used to determine retirement benefit costs for
accounting purposes. The actuarial method selected for determining accrual of liability and the assumptions
made can have a significant effect on the expense to be recorded in each accounting period. Therefore, in
carrying out a periodical valuation, an actuary chooses a suitable valuation method and, in consultation
with the employer, makes appropriate assumptions about the elements affecting the computations. The
assumptions relate to the expected inflow from future contributions and from investments as well as to the
expected outgo for benefits. In making an actuarial valuation, the actuary may sometimes effect a change in
the actuarial method used or in the assumptions adopted for determining the retirement benefit costs. Any
alterations in the retirement benefit costs so arising are charged or credited to the statement of profit and
loss for the year. Accordingly, a change in the actuarial method used for determining the retirement benefit
costs constitutes a change in an accounting policy and thus, the auditor would have to ensure that the same
is disclosed and impact on accounts is expressed in quantitative terms.
130 Accounting Standards

Problem 4: Zodiac Enterprsies pays salary/wages per day = ` 200. Leaves utilized during the year say 300
days. Leaves unutilized at Balance sheet date are non-accumulating 200 days and accumulating are 500 days
out of which 40% are vesting. 60% of the non – vesting leaves are expected to be utilized in future years.
Calculate relevant figures.
Solution:
1) 300 days are paid leave hence it will be paid through normal salary/wage paymeny =
300 × @200 = ` 60,000.
2) Non – accumulating leaves means that leaves which is not utilized and will not be c/f. Therefore
these leaves will lapse. Hence no provision is required.
3) Accumulating leaves = 500 days = 40% vesting (cash entitlement) + 60% (c/f with adjustment in
future years) = 200 + 300.
Accumulating leaves = Vesting (cash entitlement) = 200days @200 = ` 40,000.
Non - Accumulating leaves = 300 days x 60% utilization factor @200 = ` 36,000.
Entry for payment and provision for paid leaves c/f:
PAID LEAVES UTILISED:
Employees cost A/C Dr. 60,000 (transfer to Profit/Loss A/C )
To Cash/Bank A/C 60,000
PAID LEAVES UNUTILISED:
Employees cost A/C Dr. 76,000 (transfer to Profit/Loss A/C )
To Provision for Employees Benefits A/C 76,000
Problem 5: Wage of Mr A, an employee of Grasim Ltd is ` 200 per day. He was on leave from 26/3/2009 to
31/3/2009. Show accounting entries in the books of Grasim Ltd to record the compensated absence in each
of the following cases:
(i) Mr. A is entitled to 8 days of compensated absence, which is non – accumulating.
(ii) Mr. A is entitled to 8 days of compensated absence, which is accumulating.
Solution: Mr A has taken 6 days actual leave during the year (leave utilized). But he is entitled to 8 days leave
during the year in both the cases given in Qt 7 (i) and (ii).
i) Non – accumulating leaves means that leaves which is not utilized and will not be c/f. Therefore
these leaves will lapse. Hence no provision is required.
PAID LEAVES UTILISED:
Employees Benefits A/C Dr. 1,200 (transfer to Profit/Loss A/C )
To Cash/Bank A/C 1,200 (6days @200).
ii) PAID LEAVES UTILISED:
Employees Benefits A/C Dr. 1,200 (transfer to Profit/Loss A/C )
To Cash/Bank A/C 1,200
PAID LEAVES UNUTILISED:
Employees Benefits A/C Dr. 400 (transfer to Profit/Loss A/C )
To Provision for Employees Benefits A/C 400 (2days @200).
Problem 6: Gross wage bill of Ferrous Limited for the month of April 2009 is ` 30,00,000. PF contribution
(employers share) is 8% of gross wage. The employees make a matching contribution to PF. Ferrous Ltd.
deposited a total sum of ` 4,50,000 on 30/4/2009 towards PF obligations for April 2009. Show accounting
entries to record the above transactions.
AS-15 (Revised): Employees Benefits 131

Solution:
Employees Benefits A/C Dr. 32,40,000 (30lakh + 8% of 30lakh) (transfer to Profit/Loss A/C )
To PF Contribution A/C 4,80,000 (16% x 30lakh)
To Cash/Bank A/C 27,60,,000 (16,20,000 – 2,40,000)
PF Contribution A/C Dr. 4,80,000
To Bank A/C 4,50,000
To Liability for PF 30,000
Problem 7: Calculate FV of Plan Assets at the end:
FMV (beginning) = ` 7,00,000, Employers Contribution = ` 1,00,000, Benefits paid = ` 40,000, Actual
return = ` 50,000.
Solution: FV of plan assets = 7,00,000 + 1,00,000 – 40,000 + 50,000 = ` 8,10,000 (For explanation refer
further problems).
Problem 8: Nature of benefit payable is Lumpsum - gratuity
Basis of payment 1 % of final salary for each year of service
Salary for current year ` 10,000
Increase each year 7% compounding
Number of years of service = 5
Discount rate = 10% (Quality Bonds YTM)
Expected return on plan assets = 11%.
Calculate: Also draw ledgers for all the years for Gratuity a/c and plan assets a/c
Solution:
Step 1) Calculation of Final Salary at the 5th year = 10,000 x (1.07)4 ,= ` 13100 (app).
Step2) Gratuity = 13,100 x 1% = ` 131 every year.
Step3) Gratuity for 5 years = 131 x 5 = ` 655.
To pay 131 at the end 5th year we have to provide ` 131 × 1 for 5th year = ` 131
To pay 131 at the end 5th year we have to provide ` 131 × 0.909 for 4th year (@10%) = ` 119
To pay 131 at the end 5th year we have to provide ` 131 × 0.826 for 3rd year (@10%) = ` 108
To pay 131 at the end 5th year we have to provide ` 131 × 0.751 for 2nd year (@10%) = ` 98
To pay 131 at the end 5th year we have to provide ` 131 × 0.683 for 1st year (@10%) = ` 89
` 89, ` 98, ` 108, ` 119 and ` 131 are known as Current Service Cost charged to Profit and Loss.
Lets now build up GRATUITY of obligation as per Projected Unit Credit Method (PUCM):

Year 1 Year 2 Year 3 Year 4 Year 5


Opening obligation b/d — 89 196 324 476
+ Interest Cost 10% — 9 20 22 48
+ Current Service Cost 89 98 108 119 131
Closing obligation balance c/f 89 196 324 476 655

DO YOU KNOW
Closing obligation for any year = n × Current Service Cost

Example: Closing obligation for year 1 = 1 × 89 = 89, year 2 = 2 × 98 = 196 and so on.
132 Accounting Standards

Lets us now build up growth in investments for paying obligations at the end
To receive 131 at the end 5th year we have to invest ` 131 × 1 for 5th year = ` 131
To receive 131 at the end 5th year we have to invest ` 131 × 0.901 for 4th year = ` 118
To receive 131 at the end 5th year we have to invest ` 131 × 0.811 for 3rd year = ` 106
Similarly for 2nd and 1st year also.
` 86, ` 96, ` 106, ` 118 and ` 131 are known as Contribution towards Plan investments.

Year 1 Year 2 Year 3 Year 4 Year 5


Opening b/d — 86 192 319 472
+ Interest earned 11% — 10 21 35 52
+ Cash investments 86 96 106 118 131
Closing bal c/f 86 192 319 472 655

Gratuity Payable A/C


YEAR 1
To balance c/f 89 By Current Service cost 89
YEAR 2
To balance c/f 196 By balance b/d 89
By Interest cost 9
By Current Service cost 98
YEAR 3
To balance c/f 324 By balance b/d 196
By Interest cost 20
By Current Service cost 108

Similarly we can go for the fourth and the fifth year.

Plan Assets A/C


YEAR 1
To bank a/c 86 By balance c/f 86
YEAR 2
To balance b/d 86
To Expected return on plan assets 10
To bank a/c 96 By balance c/f 192
YEAR 3
To balance b/d 192
To Expected return on plan assets 21
To bank a/c 106 By balance c/f 319

Similarly we can go for the fourth and the fifth year. At the end of fifth year Gratuity payable a/c and Plan
assets a/c get canceled as it is a payment towards retirement benefits.
AS-15 (Revised): Employees Benefits 133

Problem 9: Following information is given about a funded benefit plan. The PV of obligation and the FV of
assets planned were both ` 1,000 as on April 2009.

2009-10 2010-2011 2011-12


Discount rate 10% 9% 8%
Expected return on plan assets at the 12% 11.10% 10.30%
start of the year
Current service cost 130 140 150
Benefits paid 150 180 190
Contributions paid 90 100 110
PV of obligations on 31/3 1,141 1,197 1,295
FV of plan assets on 31/3 1,092 1,109 1,093
A vg remaining life of employees 10 10 10

In year 2, the plan was amended to provide additional benefits effective from year 1. The PV at the end of
year 1 for additional benefits is ` 50 vested and ` 30 non - vested benefits. As on 1/4/2010 the enterprise
estimated that the avg period until non -vested benefits would become vested was 3 years; the past service
cost arising from non vesting benefit should be w-off on SLM basis over 3 years. The past service cost arising
from vesting benefit should be immediately w-off.
You are required to:
Complete the following table, Also calculate the amount charged to P/L Account as per AS-15 and draw
Balance Sheet disclosure.

2009-10 2010-11 2011-12


Opening PV of obligation
Interest cost
Current service cost
Past service cost (non vested benefits)
Past service cost (vested benefits)
Benefits paid
SUBTOTAL
Closing PV of obligation (actuarial valuation)
Actuarial gain/loss ……(a)
FV of plan assets as on 1/4
Expected return on plan assets
Contributions paid
Benefit paid
SUB TOTAL
FMV of plan assets
Actuarial gain/(loss)….. (b)
Net Actuarial gain/(loss) a + b
134 Accounting Standards

Solution:
BUILDING UP PV OF OBLIGATION 2009-10 2010-11 2011-12
Opening PV of obligation 1000 1141 1197
Interest cost 100 103 96
Current service cost 130 140 150
Past service cost (non vested benefits) — 30 —
Past service cost (vested benefits) — 50 —
Benefits paid (150) (180) (190)
SUBTOTAL 1080 1264 1253
Actuarial gain/(loss) à (a) (61) +87 (42)
Closing PV of obligation (actuarial valuation) 1141 1197 1295
BUILDING UP FV OF PLAN ASSETS 2009-10 2010-11 2011-12
FV of plan assets as on ¼ 1000 1092 1109
Expected return on plan assets 120 121 114
Contributions paid 90 100 110
Benefit paid (150) (180) (190)
SUB TOTAL 1060 1133 1143
Actuarial gain/(loss)à (b) 32 (24) (50)
FV of plan assets as on 31/3 (market valuation) 1092 1109 1093
Net Actuarial gain/(loss) a+b (29) 63 (92)

DO YOU KNOW
Closing PV of obligation and FV of plan assets is to be calculated by Actuary and hence students are not al-
lowed to analyze the valuation work on 31/3. It is even applicable in practice.

Statement of Profit/Loss (Extract) Para 55


2009-10 2010-11 2011-12
Current service cost 130 140 150
Interest cost 100 103 96
(–) Expected returns (120) (121) (114)
Net Actuarial gain/(loss) 29 (63) 92
Past service cost (vested benefits) — 50 —
Past service cost (non vested benefits) — 10 10
TOTAL CHARGED TO Profit and Loss 139 119 234

Balance Sheet Disclosures (Extract) Para 55


2009-10 2010-11 2011-12
PV of Obligation 1141 1197 1295
(–) FV of plan assets (1092) (1109) (1093)
49 88 202
Unrecognised past service cost — (20) (10)
NET SHOWN IN THE B/S 49 68 192
AS-15 (Revised): Employees Benefits 135

Problem 10: At 01.04.2009 the FV of plan assets was ` 10,000. On 30.09.2009, the plan paid benefits of
` 1,900 and received contributions of ` 4,900. At 31.03.2010, the FV of the plan assets was ` 15,000 and the
PV of the defined benefit obligation was ` 14,792. Actuarial losses on the obligation for 2009-2010 were ` 60.
At 01.04.2009, the reporting enterprise made the following estimates, based on the market prices at that
date:
Interest and dividends (after tax by the fund) 9.25%
Realized and unrealized gains on plan assets (after tax) 2.00%
Administration costs (1.00)%
Expected rate of return 10.25%

For 2009-10, Find out:


(i) Expected and actual return on plan assets
(ii) Actuarial gain/(loss) to be recognized in 2009-10.
(CA Final Nov 2009/May 2011/May 2013 / Nov 2015) (Marks 4)
Solution: Actuarial gain/loss can be calculated by comparing FV of plan asset with Estimated value of asset
OR by comparing Expected return with Actual return from the plan assets.
Lets compute expected return on plan assets:
Return on opening balance = 10,000 × 10.25% = 1,025
+ Return on the net cash contributions = (4,900 – 1,900) = 3,000 × 10.25 % × ½ = ` 154
Total expected return = 1,025 + 154 = ` 1,179
Lets compute actual return on plan assets: `
Opening FV of plan assets = 10,000
+ Net cash contributions = 3,000
Expected FV 13,000
Less: Actual FV of plan assets 15,000
Actual return earned by plan assets 2,000
Comparing with expected return 1,179
Actuarial gain = 821
Actuarial loss due to obligation difference = (60)
Net actuarial gain = 761
Problem 11: A Company- has a scheme for payment of settlement allowance to retiring employees. Under
the scheme, retiring employees are entitled to reimbursement of certain travel expenses for class they are
entitled to as per company rule and to a lump-sum payment to cover expenses on food and stay during the
travel. Alternatively employees can claim a lump-sum amount equal to one month pay last drawn.
The Company’s contentions in this matter are-
(a) Settlement allowance does not depend upon the length of service of employees. It is restricted to
employees’ eligibility under the Travel rule of the Company or where option for lump-sum payment
is exercised, equal to the last pay drawn.
(b) Since it is not related to the length of service of the employees, it is accounted for on claim basis.
State whether the contentions of the Company are correct as per relevant Accounting Standard. Give
reasons in support of your answer. (CA Final June 2009 OS, CA Final Nov. 13 Audit)
Solution: Refer Audit & Prof. Ethics November 2013 Paper Question 2.
Problem 12: A pension plan of a company provides a pension of 1.5% of final salary for each year of service.
The benefits are vested after 4 years of service. However on 1/1/2010 the company changes the pension to
136 Accounting Standards

1.25% of final salary for each year of service starting from 1/1/2003. The present value of the differential
benefits for service from 1 January 2003 to 1 January 2010 at the date of changes as follows:
Employees with more than 4 years of service on 1/1/2010 ` 1,25,000
Employees with less than 4 years of service on 1/1/2010 ` 1,21,000
(Average period until vesting – four years) Determine the amounts recognized in respect of past service
costs.
Solution: Past service cost which is already vested is written off immediately. Unvested past service cost is
written off over the period until vesting on straight line basis. Such unrecognized past service cost is deducted
while recognizing net employee benefit obligation in the balance sheet.
Employees with more than 4 years of service on 1/1/2010 ` 1,25,000 should be immediately w-off
Employees with less than 4 years of service on 1/1/2010 ` 1,21,000 should be charged to Profit and Loss
Account on SLM basis till balance period on vesting.
CHAPTER 16
AS-16: Borrowing Costs

Chapter Outline
vv Meaning of Borrowing Costs (BC) vv Treatment of Borrowing Costs Incurred
vv Elements of Borrowing Costs or vv Meaning of Expenditure Incurred
Scope of AS-16: Para 4 on Qualifying Asset: Para 15
vv Conditions for Borrowing Costs vv Types of Borrowings/BC Eligible for Capitalization
vv What are Qualifying Assets (Q. A.) vv Disclosures

Meaning of Borrowing Costs (BC)


Whether the company is a small company or having huge turnover it requires borrowing to fulfill its capital
expenditure needs or working capital needs. Such borrowings always costs to the company. Previously such
borrowing costs (interest etc.) were being charged to revenue accounts. We know that any costs incurred to
bring fixed asset to its present location are to be capitalized. Purchase price + Labours Charges + Installation
expenses have to be capitalised. If loan is taken to acquire a fixed asset, then why not interest cost to be
added to the cost of asset? There is, therefore, an imperative need to accord proper accounting treatment for
recognising and recording borrowing costs incurred during the period of construction/improvement in the
asset. Once more accounting standard adhered to accrual and matching concept principle.

Elements of Borrowing Costs or Scope of AS-16: Para 4


Inclusions in Borrowing Costs:
BC is not restricted to interest costs only but it includes:
(a) Interest expenses on borrowings & Commitment charges on borrowings (charges in lieu of interest
between the period of loan sanctioned and loan disbursement)
Interest includes interest on short term or long term borrowings.
(b) Annual amortisation of premium paid or discount expense on debentures/bonds.
(c) Annual amortisation of costs incurred for arranging borrowings (Brokerage)
(d) Finance charges under finance lease (Interest component of MLP AS-19).
(e) Exchange difference on foreign currency loan to the extent they are regarded as an adjustment
towards interest costs. (para 4 ( e )
Explanation of para 4 (e): This phrase refers to exchange difference on the amount of principal of the
foreign currency borrowings to the extent the difference between interest on local currency borrowings and
the interest in foreign currency borrowings. In other words AS-16 deals with loss of exchange differences
138 Accounting Standards

compensated by gains due to reduced amount of interest. It does not deal with gains in exchange differences.
If the exchange difference, on principal amount is not borrowing cost as per AS-16, then is to be taken care
off by AS – 11/Companies Act.
Exclusions in Borrowing Costs: (i) Cost of Equity (Ke), (ii) Cost of Preference Share Capital (Kp), (iii)
Imputed cost of equity (Kr).
Interest u/s 208: As per Companies Act a company can pay interest on Share Capital for projects under
progress. Even though such interest (imputed cost) is excluded from AS-16, it is still to be added to Cost of
project because provisions of companies act overrides accounting standards.

DO YOU KNOW
Borrowing Costs includes not only interest on long term loans but also on working capital loans. Had the cash
generated from operations would have been used for repaying working capital loans, the borrowing costs
would have been lower. Hence Interest on working capital is also to be capitalised.

Conditions for Borrowing Costs


(1) Borrowing costs should have been incurred (see elements of BC).
(2) The asset should be a qualifying asset.

What are Qualifying Assets (Q. A.)


BC can be capitalised only if the asset is a Q. A.
(i) Q.A. is an asset which takes substantial period of time to get itself ready for its intended use (Fixed
asset) or sale (Inventories).
(ii) Substantial period of time → It’s a period greater than 1 year (any longer or shorter period can be
justified).
(iii) The substantial time should be due to natural conditions. Ex.: Wine takes substantial time to mature.
There is a clear change in the condition of the inventory through natural processes during the period
it is lying in store. Hence wine is a Q.A. But stock kept due to loss of demand is not a Q.A.
(iv) Thus Q.A. is an asset which takes a very long time to construct/manufacture.
(v) An asset after being ready for use, no BC is to be added henceforth.

DO YOU KNOW
Qualifying assets covers fixed assets (lathe machine under construction) as well as stock in trade (wine, genetic
seeds).

Other Examples
◘◘ A toll bridge that takes a couple of years to construct before it is ready for use and is opened to the
public.
◘◘ A power plant that takes a substantial period of time to get ready for its intended use.
◘◘ A hydroelectric dam that services the needs of a village and takes a considerable period of time to
construct
The following are not Qualifying assets:
(i) Inventories routinely manufactured.
AS-16: Borrowing Costs 139

(ii) Inventories produced in large quantities and for short term.


(iii) Assets are ready for use/sale.
Substantial period of time: (ASI- 1). The issue as to what constitutes a substantial period of time primarily
depends on the facts and circumstances of each case. However, ordinarily, a period of twelve months is
considered as substantial period of time unless a shorter or longer period can be justified on the basis of
facts and circumstances of the case. In estimating the period, time which an asset takes, technologically and
commercially, to get it ready for its intended use or sale should be considered.
The following assets ordinarily take twelve months or more to get ready for intended use or sale unless
the contrary can be proved by the enterprise:
(i) assets that are constructed or otherwise produced for an enterprise’s own use, e.g., assets constructed
under major capital expansions.
(ii) assets intended for sale or lease that are constructed or otherwise produced as discrete projects (for
example, ships or real estate developments).

Treatment of Borrowing Costs Incurred


(i) BC is to be added to the cost of asset during its manufacturing/constructing stage.
(ii) An asset after ready for use, no BC is to be added henceforth. Such BC is to be transferred to profit
and loss account.
(iii) Deferment (c/f.) of BC is not allowed by the standard in any case (supported by para 56 of AS-26).

Meaning of Expenditure Incurred on Qualifying Asset: Para 15


It means:
(i) Cash payments made during the year xxx
(ii) Transfer of Assets during the year xxx
(iii) Assumption of interest bearing liabilities xxx xxx
Less: Progress payments received (xxx)
Government Grants received (xxx)
Expenditure on Q. A. xxx
Expenditure on Q. A. may be incurred during the whole year; hence average carrying amount of Q. A. can
be the most suitable basis of ascertaining the amount of expenditure.
Para 15 provides that the average carrying amount of the asset during a period including borrowing costs
previously capitalized is normally a reasonable approximation of the expenditure to which the capitalization
rate is applied.

Types of Borrowings/BC Eligible for Capitalization


As far as the rate of BC is concerned following are the two possibilities of borrowings and treatment of rate
will depend on such type of borrowing:
i) Special Borrowings: Funds borrowed specifically for purchasing/constructing/manufacturing
a Qualifying Asset is Special Borrowing. For example : A Ltd. finances 100% cost of Machinery by
taking loan from SBI Bank of ` 10,00,000. Rate of Interest (%) for BC = Actual rate of Interest (%)
paid for the concerned loan.
Also Borrowings Costs = Interest as per actual rate of interest xxx
Less: Income from temporary investments - xxx
xxx
140 Accounting Standards

ii) General Borrowings: Here funds are borrowed generally for many assets or may be some part of the
borrowings may be used for working capital requirements. Examples: Issue of Debentures, Working
Capital Loans etc. used for general purpose.

Conditions for General Borrowings


i) Amount of BC to be capitalized should be determined by applying a capitalization rate to the
expenditure on that asset.
ii) Amount of BC capitalized should not exceed the amount of BC incurred during that period.
The above conditions are interpreted through the following steps:
Step 1 Compute total borrowing costs (including interest etc.) incurred.

Step 2 Use the formula: Weighted Average @ (%) =

Step 3 Find total Avg. Capital Expenditure on General Borrowings


Step 4 Borrowing Costs to be capitalized = Weighted Average @ (%) × Avg. Capital Expenditure which should not
exceed borrowing costs incurred.

Borrowing specifically used for a Used in general for different Not used for any qualifying asset say
qualifying asset qualifying asset working capital loans

Borrowing cost should be fully Weighted average rate of borrowing Borrowing cost should be fully
debited to qualifying asset cost should be charged on average charged to profit & loss account
expenditure on qualifying asset. The
total amount charged should not
Borrowing cost for the period after exceed the actual borrowing cost
the completion all qualifying asset
will be charged to P & L A/c.

Commencement of Capitalization of Borrowing Costs: Para 14


Following three conditions must be fulfilled before the commencement of capitalization of borrowing cost.
◘◘ Activities, which are essential to prepare the asset for its intended use, should be in progress.
◘◘ Borrowing cost is incurred.
◘◘ Expenditure for acquisition, construction or production of a qualifying asset is being incurred.

Suspension of Capitalization of Borrowing Costs: Para 17/18


When borrowing costs should be suspended:
i) When active development is interrupted due to avoidable reasons and no work is carried out.
ii) When active development is interrupted but some minor work like security, maintenance etc. may be
carried out.
When borrowing costs should not be suspended:
i) When active development is going on without any interruption.
ii) When technical work is carried out but active development may be interrupted.
iii) When temporary delay is necessary for the maturity of the product for its intended use and sale.
iv) When suspension is a natural circumstance like for example Heavy rains during construction of
Bridge.
AS-16: Borrowing Costs 141

Cessation of Capitalization of Borrowing Costs: Para 19-22


Cessation is different from Suspension. Capitalization of borrowing cost should cease when substantially all
the activities necessary to prepare the qualifying assets for its intended use or sale are completed.
It means all relevant activities, which are essential for intended use or sale of qualifying assets, should be
completed. BC should be totally stopped when physical construction or production is substantially completed
though some routine administrative work may be still carried out.
If the construction of the qualifying asset is carried in parts/phase and each part/phase can be used
independently, then capitalization of borrowing cost for such phase/part will cease, but can continue for
other parts as well.
Example: A business park comprising several buildings, each of which can be used individually, is an example
of qualifying assets for which each part is capable of being used as intended while construction continues for
the other parts.

Period of Time during which Borrowing Costs incurred should be Capitalized


The time period in respect of which borrowing costs incurred to be capitalize has three aspects:
1. When should capitalization start (commencement of capitalization)
2. For what period of time should capitalization be temporarily stopped (suspension of capitalization)
3. When should capitalization stop (cessation of capitalization)

Disclosures
The financial statement should disclose:
1) The accounting policy adopted for borrowing cost.
2) The amount of borrowing cost capitalized during the period.

PROBLEMS AND SOLUTIONS

Problem 1: R. Ltd. has borrowed ` 25 crores from financial institution during the financial year 2004-05.
These borrowings are used to invest in shares of A Ltd., a subsidiary company, which is implementing a new
project estimated to cost 50 crores. As on 31st March, 2005 since the said project was not yet complete, the
Directors of R Ltd., resolved to capitalize the interest on the borrowings amounting to ` 3 crores and add it to
the cost of investments. As a statutory auditor, please comment.
(Final C.A. Audit, Final C.A. Nov. 2004) (4 Marks)
Solution:
Capitalisation of Interest on Borrowings in respect of investments: As per Accounting Standard 13 on
“Accounting for Investments,” the cost of investment includes acquisition charges such as brokerage, fees and
duties. In the instant case, R Ltd has used borrowed funds for making investment in shares of a subsidiary
company. For acquiring shares of a subsidiary company, apart from any fees, duties etc there are no cost
incurred for investing in shares. Hence, any borrowing costs incurred cannot be treated as part of cost of
investments and cannot be added to the cost of investments. The Accounting Standard 16 on “Borrowing
Costs” also does not consider investment in shares as qualifying assets that can enable a company to add the
borrowing costs to investments. In the instant case therefore, the statutory auditor would qualify his report
by stating that the borrowing costs have wrongly added to the cost of investments rather than charging
them to the profit and loss account. The effect of the same on the profit for the year would also have to be
mentioned in the audit report.
142 Accounting Standards

Problem 2: How would you, as a statutory auditor, deal with the following situation? The company deals
in purchase and sale of timber and has included notional interest charges (calculated on the paid up share
capital and free reserves) in the value of stock of timber as at the Balance Sheet date as part of cost of holding
the timber. (Final C.A. Nov. 1995) (4 Marks)
Solution: The companies accounting policy of capitalising the cost of interest to the stock is incorrect. The
cost of reserves and capital cannot be taken as BC. Even as per AS-2 the valuation of stock is incorrect, which
excludes interest as cost of inventory. Also the BC is to be capitalised only for manufacture or acquiring
Qualifying asset. BC cannot be added during the holding period of the asset. The company should exclude such
imputed cost of capital from timber stock otherwise the auditor will have to qualify the report. Practically
speaking company purchasing wet timber is kept for substantial period of time to make itself ready (dry
timber) for intended sale. In such BC would have been capitalized.
Problem 3: Palms Trees Ltd. has taken a loan of $ 10,000 on April 1, 2005, for a specific project at an interest
rate of 6% p.a.. Installment is payableannually into 5 equal installments. On April 1, 2005, the exchange
rate between the currencies was ` 45 per $. The exchange rate, as at March 31, 2006, is ` 48 per $. The
corresponding amount could have been borrowed by Palms Trees Ltd. in local currency at an interest rate of
11 % p.a. as on April 1, 2005.
1. You are required to ascertain the amount of borrowing cost incurred and to be included under:
a. Para 4(a) of AS 16
b. Para 4(e) of AS 16
2. Is AS 11 applicable in the above case?
3. What would be your answer if interest rate prevailing locally is 15% p. a.? (CA Final A/Cs May
2012)
Solution:
Step 1) Interest as per Foreign Loan = 10000x 6% × @48= ` 28,800 AS-16 para 4(a)
Step 2) Exchange Difference = ($10000 × 48-45) = ` 30,000
Step 3) Interest as per Indian Loan = $10000 × 45 × 11%= ` 49500 (Maximum Notional)
Step 4) Exchange Difference covered by AS – 16 para 4 (e) = 49500 – 28800 = ` 20,700 out of ` 30,000
Total BC to be capitalised = Interest on Foreign Loan para 4(a) + Exchange difference para 4(e) = 28,800 +
20,700 = ` 49,500
Exchange Difference of ` 9,300 is covered by AS-11.
If interest rate prevailing in India is 15% p.a.:
Step 1) Interest as per Foreign Loan = 10000 × 6% x @48 = ` 28,800 AS-16 para 4(a)
Step 2) Exchange Difference = ($10000 × 48-45) = ` 30,000
Step 3) Interest as per Indian Loan = $10000 × 45 × 15%= ` 67,500 (Maximum Notional)
Step 4) Exchange Difference covered by AS – 16 para 4 (e) = ` 30,000 (Full)
Total BC to be capitalised = Interest on Foreign Loan para 4(a) + Exchange difference para 4(e) = 28,800 +
30,000 = ` 58,800
Exchange Difference covered by AS-11 is Nil.
Solution: Refer para 15…..
Borrowing Cost incurred = 13% [2,00,000 + 2,00,000] = ` 52,000
Expenditure on Q. A. may be incurred during the year, hence average carrying amount of Q. A. can be the most
suitable basis of ascertaining the amount of expenditure.
Opening WIP of Q. A.= (500000+26000) = 5,26,000
i) Cash payments made during the year 75,000
ii) Transfer of Assets during the year 1,00,000
AS-16: Borrowing Costs 143

iii) Assumption of interest bearing liabilities —


Less: Progress payments received (3,50,000)
Expenditure on Q. A. 3,51,000

Captalisation @ = . . × 100 = 13%

Notional Interest = 13% × 351000 = ` 45,630 : Borrowing Cost incurred = ` 52,000


Hence Borrowing Cost to be capitalised = ` 45,630 : Borrowing Cost transferred to Profit and Loss = 6,370
Problem 4: XYZ Ltd., has undertaken a project for expansion of capacity as per the following details:
  Plan ` Actual  `
April, 2015 2,00,000 2,00,000
May, 2015 2,00,000 3,00,000
June, 2015 10,00,000 —
July, 2015 1,00,000 —
August, 2015 2,00,000 1,00,000
September, 2015 5,00,000 7,00,000
The company pays to its branches at the rate of 12% p.a., interest being debited on a monthly basis. During
the half year company had ` 10 lakhs overdraft upto 31st July, surplus cash in August and again overdraft of
over `10 lakhs from 1.9.2015. The company had a strike during June and hence could not continue the work
during June. Work was again commenced on 1st July and all the works were completed on 30th September.
Assume that expenditure were incurred on 1st day of each month.
Calculate: (i) Interest to be capitalized. (ii) Give reasons wherever necessary.
Assume:
(a) Overdraft will be less, if there is no capital expenditure.
(b) The Board of Directors based on facts and circumstances of the case has decided that any capital
expenditure taking more than 3 months as substantial period of time.
(Final C.A. May 2003) (8 Marks)
Solution:
Qualifying Asset A/C
2002 Particulars ` 2002 Particulars `
1 April To Bank O/D 2,00,000 1 April By Balance c/d 2,02,000
30 April To Interest 2,000
(1% × 200000)
1 May To Balance b/d 2,02,000 1 April By Balance c/d 5,07,020
1 May To Bank O/D 3,00,000
31 May To Interest 5,020
(1% × 502000)
1 June To Balance b/d 5,07,020 1 April By Balance c/d 5,07,020
1 June To Bank O/D —
30 June To Interest —
(Note 2)
1 July To Balance b/d 5,07,020 1 April By Balance c/d 5,12,090
1 July To Bank O/D —
31 July To Interest 5,070
(1% × 507020)
144 Accounting Standards

1 Aug To Balance b/d 5,12,090 1 April By Balance c/d 6,12,090


1 Aug To Bank O/D 1,00,000
31 Aug To Interest —
(Note 3)
1 Sep To Balance b/d 6,12,090 1 April By Balance c/d 13,22,090
1 Sep To Bank O/D 7,00,000
30 Sep To Interest 10,000
(Note 4)

Important Notes:
1) O/D is above 10lacs but expn on QA is less than 10 lacs, therefore 1% interest rate is applied on
expenditure of Q.A. As per para 15, the average carrying amount of the asset during a period, including
borrowing cost previously capitalized, is normally a reasonable approximation of the expenditure to
which the capitalization rate is applied in that period.
2) In the month of June the activities were stopped due to strike. Hence BC was suspended…….para 17
of AS-16.
3) In the month of August there is no O/D or positive cash balance. No interest cost is incurred and
hence the same is not capitalized.
4) During September, actual overdraft is ` 10 lakhs. Hence, only ` 10,000 interest has been capitalised
even though actual expenditure exceeds ` 10 lakhs. Maximum BC to be capitalized will be ` 10,000
in Sep.
Problem 5: A company obtained term loan during the year ended 31st March, 2002 to an extent of
` 650 lakhs for modernization and development of its factory. Buildings worth ` 120 lakhs were completed
and Plant and Machinery worth ` 350 lakhs were installed by 31st March, 2002. A sum of ` 70 lakhs has
been advanced for assets, the installation of which is expected in the following year. ` 110 lakhs has been
utilized for Working Capital requirements. Interest paid on the loan of ` 650 lakhs during the year 2001-2002
amounted to ` 58.50 lakhs. How should the interest amount be treated in the Accounts of the Company?
(C.A. Final Nov. 2002, Nov 2005) (6 Marks)
Solution: Building & Plant are qualifying assets because Building is being constrcuted, P & M is installed for
modernization of factory. Payments for assets is a capital advance i.e. we are contractee hence we have to
make fixed payments. Money spent on the working capital is also not for the purpose of Q.A. Hence interest
pertaining to construction of building & installation of plant is required to be capitalised and all the other
activities should be charged off to the Profit and Loss Account. In the present case, the interest amount of
` 58.50 lakhs shall be treated as follows: Capitalisation of interest ` 42.30 lakhs and interest charged to Profit
and Loss ` 16.20 lakhs.
Problem 6: On 01.04.2006 Nidhi Ltd. borrowed ` 2,00,000 @12% p. a. for construction of qualifying asset.
The construction commences on 01.06.2006 and completes on 28.02.2007. Income earned on investing the
same borrowings temporarily was ` 5,000 of which` 2,000 represents, income earned after 01.06.2006.
Ascertain the amount of borrowing cost eligible for capitalisation. (CA Final May 2010 NS similar qt)
Solution:
Gross Interest paid = 200000 × 12% = 24,000
Interest eligible for BC as per AS-16 = 24000 × 9/12 = 18,000
Net Interest transferred to Profit and Loss = 24000 - 18000 = 6,000
Specific borrowing case:
Interest eligible for BC as per AS-16 = 18,000
Less: Interest on temporary investments = –2,000
Net Interest capitalised for BC as per AS-16 = 16,000
AS-16: Borrowing Costs 145

Problem 7 Kirloskar Ltd. had following general borrowings and investments in qualifying assets.
Source Date of raising Amount (`)
12% Debentures 01.04.2005 1,500,000
15% Term Loan 01.04.2005 600.000
18% Term Loan 01.04.2005 400,000

Qualifying Assets Amount (`) Date of Commencement Date of Completion


A 600,000 01.04.2005 31.01.2006
В 500,000 01.05.2005 31.03.2006
С 400,000 01.07.2005 —

With the help of these details determine for the year ended 31.03.2006
1. the amount of borrowing cost incurred
2. the amount of borrowing cost to be capitalised for qualifying assets
3. the amount of borrowing cost to be charged to revenue
Solution:
Step 1) Calculation of BC incurred:
SOURCE BORROWINGS AVG O/S INTEREST
12% Debn. 15,00,000 15,00,000 1,80,000
15% Loans 6,00,000 6,00,000 90,000
18% Loans 4,00,000 4,00,000 72,000
Total: 25,00,000 25,00,000 3,42,000
Step 2) Calculation of Capitalisation rate for general borrowings: (Effective Interest @)
Capitalisation % = Interest/Avg O/S  13.68 % p.a.
Step 3) Calculation of Capitalisation of borrowings costs:
Q. A. EXPENDITURE INCURRED ON QA % PERIOD PRODUCT
A 6,00,000 13.68% 10/12 68,400
B 5,00,000 13.68% 11/12 62,700
C 4,00,000 13.68% 9/12 41,040
Total : 15,00,000 1,72,140
Step 4) Capitalisation of borrowing cost = Notional BC 1,72,140
Step 5) Borrowing cost charged to Profit and Loss = 3,42,000 – 1,72,140 = 1,69,860
Problem 8: X Ltd began construction of a new building on 1st January. It obtained ` 1 Lakh special loan to
finance the construction of the building on 1st January at an interest rate of 10%. The Company’s other two
outstanding non-specific loans were-
(a) ` 5,00,000 Loan at 11 % Interest, and (b) ` 9 ,00,000 Loan at 13% Interest.
The expenditure that were made on the building project were as follows -
(b) January ` 2,00,000, (b) April ` 2,50,000, (c) July ` 4,50,000, (d) Dec. `1,20,000.
Building was completed by 31st Dec 2007. Following the principles prescribed in AS-16 “Borrowing Cost,”
calculate the amount of interest to be capitalized and pass one Journal Entry for capitalizing the cost and
borrowing cost in respect of the building. (CA Final Nov 2008 Marks 10)
146 Accounting Standards

Solution:
Calculation of Capitalisation rate: (5,00,000 x 11% + 9,00,000 x 13%)/14,00,000 = 12.29%
Calculation of BC:
` 1,00,000 × 12/12 = ` 1,00,000
` 2,50,000 × 9/12 = ` 1,87,500
` 4,50,000 × 6/12 = ` 2,25,000
` 1,20,000 × 1/12 = ` 10,000
` 5,22,500

Cost of building `(2,00,000 + 2,50,000 + 4,50.000 + 1,20,000) 10,20,000


Add: interest cost to be capitalised: specific borrowings (` 1,00,000 × 10%) 10,000
General borrowing (` 5,22,500 × 12.285%) 64,189
Total amount to be capitalized for the building 10,94,189

Journal entry
Date Particulars Dr.(`) Cr.(`)
31st Dec. Building account Dr. 10,94,189
  To bank account 10,94,189
(being amount of cost of building and borrowing cost thereon capitalized)

Problem 9: On April 1, 2014, MGH constructions undertook construction of a factory building for expansion
purpose. Total cost of project was ` 3,00,00,000. The building was completed by end of March 2015 and
during the period following payments were made :
Payment made Amount
st
1 April 2014 20,00,000
30th June 2014 60,00,000
st
31 December 2014 1,80,00,000
st
31 March 2015 40,00,000
Total 3,00,00,000
MGH constructions borrowings as at March 31, 2015 were as follows :
a) 9% term loan amounting to ` 80,00,000 taken on December 31, 2013. Simple interest is payable
annually. Amount outstanding as at March 31, 2014 and during 14-15 is ` 80,00,000. The loan was
taken specifically for the project.
b) 11% debentures issued on March 31, 2013 with simple interest payable annually. Amount outstanding
for the year 14-15 is ` 1,50,00,000.
c) 10% bonds issued on December 31, 2012 amounting to ` 1,70,00,000. Simple interest payable at
annual rest. Amount outstanding for the year 14-15 is `1,60,00,000.
How much borrowing cost should be capitalized for construction of the building as per AS-16 “Borrowing
Cost”?
Solution:
Step 1) Calculation of BC incurred: (Only for GB) (` in lakhs)
SOURCE BORROWINGS AVG. O/S INTEREST
11% Debn. 150.000 150.000 16.500
10% Bonds 160.000 160.000 16.000
Total 310.000 310.000 32.500
AS-16: Borrowing Costs 147

Step 2) Calculation of Capitalisation rate for general borrowings:


Capitalisation % = Interest/Avg. O/S 10.484 % p.a.

Step 3) Calculation of Average Investments in QA:


Q. A. EXPENDITURE INCURRED ON QA PERIOD PRODUCT
1st April 2005 — Specific B —
30th June 2005 — Specific B —
31st Dec.2005 180 3/12 45
31st March 2006 40 0/12 0
Total: 220 45
Net Investments in General Borrowings 45
Capitalisation rate 10.484%

Step 4) Capitalisation of borrowings cost = Notional BC on GB 4.716


+ Interest on Specific Borrowings * +5.85
Total BC capitalized = 10.566
* [20 lacs + 60 lacs x 9/12] x 9%

Step 5) Borrowings cost charged to Profit and Loss 29.134


[32.5 + 80 × 9%] – [10.566 capitalized]
Problem 10: Venus Ltd. made the following borrowings:
Borrowing Date of Amount (`) Purpose Related
Borrowing Expenses
15% Term Loan 01-01-2008 6,00,000 General 50,000
14.5% Term Loan 01-07-2008 4,00,000 Specific to acquisition of plant & machinery 20,000
14% Term Loan 01-10-2008 5,00,000 General 40,000

Qualifying assets for borrowing are:


Factory Shed 1,00,000
Plant & Machinery 9,00,000
Other Fixed Assets 1,00,000
11,00,000
Assume that the project is ready for commercial production as on 01-01-2009. Calculate the borrowing
cost to be capitalized? Year ends on 31/12.
Solution:
Step 1) Calculation of BC incurred:
SOURCE BORROWINGS AVG O/S INT+OTHERS
15% Term Loans 6,00,000 6.00.000 1,40,000 (90 + 50)
14% Term Loans 5,00,000 1.25.000 57,500 (17.5 + 40)
Total: 11,00,000 7,25.000 1,97,500

Step 2) Calculation of Capitalisation rate for general borrowings:


Capitalisation % = Borrowing Costs/Avg O/S 27.24 % p.a.
Note: Out of total expenditure of ` 9,00,000 on Plant & Machinery ` 4,00,000 is allocated to Specific
Borrowings. This means ` 5,00,000 is towards General Borrowings.
148 Accounting Standards

Step 3) Calculation of Average Investments in QA:


Q. A. EXPENDITURE INCURRED ON QA PERIOD PRODUCT
Factory Shed 1,00,000 12/12
P & M 5,00,000 12/12
Other F. A. 1,00,000 12/12
Total: 7,00,000 7,00,000
Capitalisation rate = 27.24% = 700000 × 27.24% = 190680

Step 4) Capitalisation of borrowing cost =Notional BC on GB 190680


BC on SB (400000 × 14.5% × ½ + 20000) 49,000
Total BC Capitalised 239680
Problem 11: Given below are expenses incurred in three phases of a project relating to construction of a
captive power plant: (`)

Phase 1 Phase 2 Phase 3 Total


Cash payments 5,00,000 7,00,000 5,00,000
Transfer of asset 5,00,000 2,00,000 3,00,000
Total 10,00,000 9,00,000 8,00,000 27,00,000

Money borrowed @ 12% p.a. ` 22,00,000


Phase 1 is completed as on the beginning of the year. Calculate the capitalization of borrowing costs?
Solution: As Phase 1 is completed hence capitalisation will be ceased. But capitalisation will be applicable
on other Phases.
Capital expenditure = 27,00,000 and borrowings = 22,00,000. This means the capital expn is incurred
from internal resources.
Capital Expenditure = Phase 2 + Phase 3 = 9,00,000+8,00,000 = 17,00,000
Capitalisation rate = 12% – Borrowing costs incurred = 12% × 22,00,000 = `2,64,000.
Bifurgation of borrowings into proportion of Phase I and (II + III) = 10 : 17.
B.C. (to be capitalised) = 2,64,000 × 17/27 = ` 1,66,222; B. C. transferred to Profit and Loss Account =
2,64,000 × 17/27 = ` 97,778
Problem 12: P Ltd. has undertaken a significant expansion programme for its manufacturing facilities. The
details are as follows: (RTP)
Details of Capital expenditure: (` in lakhs)

Facility Amount Remarks


Plant A 3400 Specific borrowing to the extent of ` 2400 lakhs
Plant B 2000 Specific borrowing to the extent of ` 600 lakhs
Power generation facility at Plant C 400 No specific borrowings
Plant D 400 No specific borrowings
Corporate offices 800 No specific borrowings

Facility expansion commenced on 01-06-2008 on all facilities. As of 31-03-2009, the status of completion
of these facilities was as follows:
◘◘ Plant A was completed as of 31-12-2008.
◘◘ Plant B was completed as of 30-11-2008.
◘◘ Power generation facility at Plant C is still under construction as of 31-03-2009.
◘◘ Plant D was completed as of 28-02-2009.
◘◘ Corporate offices were still under construction as of 31-03-2009.
AS-16: Borrowing Costs 149

Details of debt-equity profile of JP Ltd.


Equity: 1000 lakhs equity shares of ` 5 each fully paid.
500 lakhs 6% redeemable cumulative preference shares.
Debt:
◘◘ ` 2000 lakhs 11% p.a. secured debentures raised on 01-07-2006 redeemable in four equal install-
ments commencing 01-07-2008.
◘◘ ` 1500 lakhs secured working capital loan obtained on 01-04-2008. The loan carries an interest of
14% p.a. This loan was partially repaid on 31-12-2008 - amount repaid ` 500 lakhs.
◘◘ Loan from Financial Institutions amounting to ` 3000 lakhs bearing interest at 14% p.a. obtained for
construction of Plant A& B on 01-05-2008.
◘◘ Foreign currency loans from financial institutions amounting to $6 million bearing LIBOR plus 2%
obtained on 01-06-2008. The contracted LIBOR rate was 6% p.a. The exchange rate as of that date
was US$ 1 = ` 43. The exchange rate as of 31-03-2009 was US$ 1 = ` 46.50. The average exchange rate
during 2008-2009 was ` 45. None of the loans were repaid as of 31-03-2009. Debt is secured by a
paripassu first charge on all fixed assets of the company, both present and future. The working capital
loan is also secured against inventories and book debts of the company.
Calculate only the interest capitalized as borrowing costs.
Solution: This problem is of General Borrowing as well as Specific Borrowing both.
Step 1) Calculation of BC incurred: (Only GB)
SOURCE BORROWINGS AVG O/S INTEREST
11% Debt 2000 1,625 178.75
14% WC loan 1500 1,375 192.50
8% Foreign Loan $ 6million 2,250 180.00
5,250 551.25
Calculation of Avg Borrowings:
1) 11% Debt = 2000 × 3/12 + 1500 × 9/12 = 1625, 2) 14% WC Loan = 1500 × 9/12 + 1000 × 3/12 = 13753)
Foreign loan = $6m = $60lakhs → $60 × 10/12 = $50 lakhs (effective), Further conversion rate will be avg rate
= 50 $ × 45 = `2250 lakhs. (Interest can alternatively calculated on closing rate)
Capitalisation rate = 551.25/5250 = 10.50% p.a.

Statement of Capitalisation of BC
(1) PLANT A: Expn = 3400 lakh
General = 1000 lakhs Specific = 2400 lakhs Total
1000 × 10.50% × 7/12 2400 × 14% × 7/12 = 257.25
(2) PLANT B: Expn = 2000 lakh
General = 1400 lakhs Specific = 600 lakhs Total
1400 × 10.50% × 6/12 600 × 14% × 6/12 = 115.50
(3) PLANT C: Expn = 400 lakh
General = 1000 lakhs Specific = 0 Total
400 × 10.50% × 10/12 35.00
(4) PLANT D: Expn = 400 lakh
General = 1000 lakhs Specific = 0 Total
400 × 10.50% × 9/12 31.50
(5) CORPORATE OFFICE = 800 lakh
General = 1000 lakhs Specific = 0 Total
800 × 10.50% × 10/12 70.00
Total BC capitalized = 257.25 + 115.50 + 35 + 31.50 + 70 = ` 509.25 lakhs
CHAPTER 17
AS-17: Segmental Reporting

Chapter Outline
vv Scope and Objective vv Reportable Segments
vv Types of Segments vv Disclosure Requirements

Scope and Objective


Nowadays a company cannot survive running with single product. Diversification is a necessity in today’s
market, to secure itself from the attacks and counter attacks of the competitors on any product. Specially in a
multi product company or a company operating in different geographical areas traditional method of financial
disclosures to the shareholders could not depict the complete picture regarding the risk and returns. Hence,
there arises a need for segmental reporting.
The significance of Segment Reporting is as under:
◘◘ Evaluation and understanding of the overall performance of the company.
◘◘ Identification of loss making and profit making units.
◘◘ How various products/various areas are exposed to various risk and returns. 
◘◘ To effectively assess the underlying risks and returns of an enterprises and make more informed deci-
sions relating to the enterprise.
◘◘ To know the return on Capital employed for a segment.
◘◘ The shareholders can make correct decisions based on the segmental performance reported to them.

Types of Segments
i) Business Segment:
If a company is dealing in various products/services having different risk - return profile then we say
that the enterprise is having Business segments. A BS is a distinguishable component of an enterprise
that is engaged in providing an individual product or service or a group of related products or services
and that is subjected to risks and returns that are different from those of other business segments. A
coaching classes is running 3 divisions Commerce, Science and School. Risk and returns of individual
division might be different. This is an example of Business Segment. 
Factors to be considered for BS:
i) Nature of product, ii) Different process involved, iii) Type or class of customers, iv) Methods of
distribution, v) Nature of regularity environment (laws, regulations).
AS-17: Segmental Reporting 151

Meaning of Risks and rewards:


If a segment includes two products having different risks and rewards factors then they are two
different segments. For Example: Risk and rewards profile of doing Broking business is different
compared to trading on one’s own account; therefore they fall under two different segments.
The two factors are very important in the context of interpreting ‘risks and rewards’ under AS-17 for
defining BS. The word significant which is added before the term risks and rewards is very important.
The risk and reward of doing business in jeans would not be significantly different compared to
doing business in shirts. Also manufacturing chairs and tables carry similar risks and returns. If we
take the example of hair oil and cooking oil, they would fall under two different segments, because
their nature is different, the production process is different, the customers are different, channels
are different.
ii) Geographical Segment:
If a company is providing products/services catering geographically exposed to different risk and
returns then it is a Geographical segment. Again Geographical Segment is divided as per Location of
Assets and Location of Customers.
Geographical Segment can be within a region or within a city, within a country or scattered in
different countries.
Factors to be considered for GS:
(i) Economic and political conditions, (ii) Proximity between operations, (iii) Special risks involved,
(v) Currency risks, (iv) Exchange control risks.
Advantage of having geographical segment:
Shehnaaz Global products has been in the business of beauty products. The products were sold in
Asia, London and France. Previously the business was doing well. However in the last three years
there has been a decline in the profitabilty of the company. The company is not able to know the exact
reason for the decline in the profitabilty because the company use to prepare a consolidate income
statement (combining all the locations). After preparing segment wise profit and loss statement, the
company came to know that the sales in France was declining in the last few years. Now the company
can take corrective actions based on this information.

SOME DEFINITIONS
i) Reportable Segment: An enterprise may be engaged in ‘n’ number of products/areas. But only some of
the segments need to be compulsorily disclosed in the financial statements. Such mandatory disclosed
segments are known as Reportable Segment.
ii) Segment Revenues: It represents revenue of a particular segment.
Segment Revenue = (Direct revenues from outside customers + Inter segment sales + Allocated income). As
segment revenue includes allocable income the basis for sharing common income between the segments
can be agreed between the segments, internal MIS, transfer-pricing policy.
What about other income: Other income should be included as a part of segment revenue if they do not
fall under the exclusion definition (discussed below) and if it is essentially operating in nature. For Example:
Export incentives are price subsidies for achieving exports which is indirectly a component of export
turnover and should be included in segment revenue. Some times segment assets are idle, and these may
be used to earn rentals. Such income would be operating income and consequently form part of segment
revenue.
Exclusions: (i) Income like Interest, dividend earned unless the operations of the segment are primarily of
a financial nature; (ii) Gains on Sale of investments; (iii) Extraordinary items.
152 Accounting Standards

iii) Enterprise Revenues:


Enterprise Revenue = Revenues from external customers only as reported in profit and loss account.
iv) Segment Expenses: It represents expenses of a particular segment.
Segment Expenditure= (Direct expenses related to the unit + Expenses incurred on dealing with other
department + Allocated expenses). It excludes (i) Interest expense, (ii) Capital loss on Sale of investments,
(iii) Corporate expenses like selling and distribution expenses, office expenses, (iv) Tax expenses, (v)
Extraordinary items.
Why interest in not included in segment expenses: As per ASI-22, interest expense relating to overdrafts/
other borrowings identified to a particular segment should not be included as a part of segment expense
unless the operations of the segment is of financial nature or unless interest is included as a part of cost of
inventories as per AS –16. Segment results are net of operating results rather than net of financing.
v) Segment Result: (Segment Revenue – Segment Expenses)
vi) Segment Assets: Segment’s own assets used for its operating activities + Allocable assets including allocable
Goodwill. For Example: Milk powder purchased to make choclates is a segment asset if not consumed
before the reporting period for the Dairy product segment. Segment revenue generally includes debtors,
inventories, advances, fixed assets.
It excludes the following:
(i) Advance Taxes, T.D.S.(Taxes), (ii) Assets used by the Head-Office for general purpose.

DO YOU KNOW
If item of expense (say depreciation) is included in P & L A/C. for a particular segment, then the corresponding
asset will also go to that segment assets head.

vii) Segment Liabilities: Segment’s own operating liabilities + Share of Common liabilities. It excludes; (i)
Provision for Taxes (ii) Liabilities incurred at Head – Office level for general purpose.
viii) Segment Accounting Policies: AS-1 prescribes accounting policies at the total company level as a whole.
But segment accounting policies are specific accounting policies for the concerned segment to be reported.
Even accounting policies at the enterprise level may be passed on (allocated to) the segment. For ex.: If all
the segments use one particular Asset commonly then, depreciation charged on the enterprise level will
be allocated to different segments.
Examples of segment policies:
Policies relating to, (i) Basis of allocation of revenues/expenses between segments, (ii) Transfer pricing
policies.

Identification of Segments
Identification of Segments mostly depends upon the Organizational and Management structure. The internal
financial reporting to the BOD and CEO provides the best evidence for the basis of identification of segments.

Deciding between Primary or Secondary Segments


If an enterprise has both business as well as geographical segment then the question arises as to which is
primary and which is secondary segment. This is because the intensity of disclosures being low for secondary
segments and high in the case of primary segments. Following points are worth noting for such differentiation:
1) Largely it depends upon the information provided by the internal financial reporting to the BOD and
CEO.
AS-17: Segmental Reporting 153

2) The important factor for differentiation is “dominant source and nature of risks and return”.
3) If risks and returns of enterprises are largely affected by the products and services then business
segment will be the primary reporting segment. Here business segment is the dominant factor.
4) If risks and returns of an enterprise are largely affected by the operations in different areas then
geographical segment will be the primary reporting segment. Here geographical segment is the
dominant factor.
5) If suppose the risks and returns of both the business as well as geographical segment is equally
dominant then business segment will be primary segment.
However, if confusion arises as to differentiation between primary and secondary segment, one has to
understand the facts and circumstances of the case. Ultimately the financial disclosure should depict true and
fair view.

Segment in CFS
There is no such rule that a subsidiary company would constitute a reportable segment. Segments reportable
in the stand-alone financial statements may not be reported at the CFS level. If there are similar segments in
parent and subsidiary then they are treated as one segment for CFS purpose. Example: If HO and Subsidiary
have a common segment say FMCG, then they are merged at the CFS level.

Reportable Segments
An enterprise may be engaged in ‘n’ number of products/areas. But only some of the segments need to
be compulsorily disclosed in the financial statements. Such mandatory disclosed segments are known as
Reportable Segment.
However following are the tests applied to decide the Reportable Segment:

Test 1: REVENUE TEST


As per Revenue test Reportable Segment is:

Segment Revenue ≥ 10%(Total Revenue of all segments)

DO YOU KNOW
Total Revenue of all Segments ≠ Enterprise Revenue
Enterprise Revenue = Only external revenues of all segments (excluding inter-segment revenues).

Test 2: RESULT TEST


As per Result test Reportable Segment is:

Segment Result ≥ 10% of Higher of (Total Results of profitable segments OR Total Results of loss making
segments). Loss making segments will be considered in absolute terms.

Test 3: ASSETS TEST


As per Assets test Reportable Segment is:

Segment Assets ≥ 10% (Total Segment Assets)


154 Accounting Standards

Test 4: 75% TEST


As per 75% test Reportable Segment is:

If External Revenue of Reportable Segments < 75 % (Enterprise Revenue), then


Add more and more segments such that;
External Revenue of Reportable Segments ≥ 75 % (Enterprise Revenue)

Test 5: MANAGEMENTS CHOICE:


As per Management Test Reportable Segment is:

If any of the above tests are not satisfied then management at its discretion choose Reportable Segment.

DO YOU KNOW
If ‘X’ is a Reportable Segment last year, then ‘X’ will continue to be a Reportable Segment every year irrespective
of the above tests gets satisfied or not.

What about Non-Reportable Segments ?


Non-Reportable Segments are not reported as such but they are shown as reconciling segments.

Disclosure Requirements
Two styles of formats are required for reporting purposes:

Primary Reporting Format


Every enterprises has to report the following as Primary Reporting:
1) Revenue items:
a) Revenues from external customers. b) Inter-segment Revenues. c) Depreciation/impairments/
amortizations. d) Non-cash expenses other than ‘c’ above. e) Segment Results.
2) Balance Sheet items:
a) Total Carrying amounts of Segment Assets. b) Total amounts of Segment Liabilities. c) Tangible
and Intangible assets acquired.
3) Other items:
a) Reconciliation between (Different Segments Vs. Whole Enterprise’s) of Segment Revenues/
Results/Assets/Liabilities.

Secondary Reporting Format


(i) If Business Segment is Primary Segment: Then Geographical segment will be Secondary.
Following is to be reported by the Secondary Segments:
ww Segment Revenues from external customers of geographical segment of only those segments
who’s External Revenues is 10 % ≥ Total Enterprises Revenues.
ww Carrying amount of segment assets, of those segment assets whose assets ≥ total assets of
geographical segments.
ww Tangible and Intangible assets acquired.
(ii) If Geographical Segment is Primary Segment: Then Business will be Secondary.
Following is to be reported by the Secondary Segments:
AS-17: Segmental Reporting 155

ww Segment Revenues from external customers of business segment of only those segments whose
External Revenues is 10% ≥ Total Enterprises Revenues.
ww Carrying amount of segment assets, of those segment assets whose assets is 10% ≥ total assets
of business segments.
ww Tangible and Intangible assets acquired.
(iii) If Geographical Segment based on location of assets is Primary Segment: Then Geographical
Segment based on location of customers will be Secondary.
Following is to be reported by the Secondary Segments:
ww Segment Revenues from external customers of business segment of only those segments who’s
External Revenues is 10%≥ Total Enterprises Revenues.
(iv) If Geographical Segment based on location of customers is Primary Segment: Then Geographical
Segment based on location of assets will be Secondary.
Following is to be reported by the Secondary Segments:
ww Carrying amount of segment assets, of those segment assets whose assets is 10% ≥ total assets
of business segments.
ww Tangible and Intangible assets acquired.

Other Optional Disclosures


1. Transfer pricing basis.
2. Changes in segment accounting policies.
3. For Business Segments types of products and services.
4. Composition of each Geographical Segment.

Additional disclosures required in respect of secondary segments


In addition to reporting the aforementioned segment information in respect of primary segments, enterprises
are required to make additional disclosures in respect of secondary segments. The disclosure required
depends on type of primary segment. The principal disclosure requirements are as below:

Where primary segments are business segments (Para 48)


a) segments revenue from external customers by geographical area based on the geographical location
of its customers, for each geographical segment whose revenue from sales to external customers is
10% or more of enterprise revenue;
b) the total carrying amount of segment assets by geographical location of assets, for each geographical
segment whose segment assets are 10% or more of the total assets of all geographical segment; and
c) the total cost incurred during the period to acquire segment assets that are expected to be used
during more than one period (tangible and intangible fixed assets) by geographic allocation of
assets, for each geographical segment whose segment assets are 10% or more of the total assets of
all geographical segments.

Where Primary segments are geographical segments (Para 49)


Where primary segment is geographical segment (whether based on location of assets or location of
customers), the following segment information should be disclosed for each business segment whose revenue
from sales to external customers is 10 % or more of the total assets of all business segments:
a) segment revenue from external customers:
b) the total carrying amount of segment assets; and
the total cost incurred during the period to acquire segment assets that are expected to be used
during more than one period (tangible and intangible fixed assets)
156 Accounting Standards

Where primary segments are geographical segments based on location of assets (Para 50)
If locations of customers are different from location of assets, in addition to disclosures required pursuant
to paragraph 49 above, an enterprise is required to report revenue from sales to external customers for
each customer-based geographical segment whose revenue from sales to external customers is 10% more of
enterprise revenue.

Where primary segments are geographical segments based on location of customers (Para 51)
If locations of assets are different from location of customers, in addition to disclosures required pursuant
to paragraph 49 above, an enterprise is required to report the following segment information for each asset-
based geographical segment whose revenue from sales to external customers is 10% or more of enterprise
revenue or whose segment assets is 10 % or more of total enterprise assets.
a) the total carrying amount of segment assets by geographical location of the assets; and
b) The total cost incurred during the period to acquire segment assets that are expected to be used
during more than one period (tangible and intangible fixed assets) by location of the assets.

Some Examples of Segmental Disclosure (For Your Knowledge)

Name of the Co. Industry Segments


Dabur India Ltd. Pharma FMCG
Ayurvedic products
Others
SAIL Steel Geographical segment
Bhilai Steel Plant
Durgapur Steel Plant
Rourkela Steel Plant
Bokaro Steel Plant
IISCO Steel Plant
Alloy Steel Plant
Salem Steel Plant
Visvesaraya Steel Plant
Others
Indian Hotels Ltd. Hotel Hotel services
Air Catering
Lifetree IT Primary reporting
Convergence Ltd. Business segment: Software products, Software development products,
Software maintenance.
Secondary reporting
Geographical segment:
India
Europe
South Asia
Other part of the world
Tata Chemical Ltd. Fertilizers and Inorganic chemical
Chemical Fertilizers
AS-17: Segmental Reporting 157

Barclays PLC Banking UK Banking


Barklay Card
Barklays Capital
Barclays Wealth
Retail and Commercial Banking
Insurance activities
HO and other operations
Axis Bank Banking Treasury
Corporate Banking
Retail Banking
Others
Tech Mahindra IT and technical Telecom Service provider
services Telecom equipment manufacturer
BPO
Others
Sonata Software IT Geographical Segment:
Domestic Services (Sales and Services)
International Services (Sales and Services)
Religare Ltd Financial Services Investments and Finance
Financial Advisory Services
Broking
Support Services Fees
Custodian Fees
Life Insurance
ITC Conglomerate Tobacco
Hotels
Agri products
Toilet products

PROBLEMS AND SOLUTIONS

Problem 1: Mictrotech Ltd. produces batteries for scooters, cars, trucks and specialised batterries for
invertors and UPS. Are these products different business segments or a part of the same business segment.
(RTP)
Solution: As per AS-17 segments are identified based on different risk/rewards factors. The company is
basically producing batteries. But the batteries are further meant for (i) auto/vehicles and (ii) invertors/
UPS mostly useful for household purposes i.e. indoors. The risk and rewards in auto and invertors are
significantly different. Auto batteries are affected by governent policy, road conditions, number of accidents
etc and batteries for invertors/UPS depends upon number of power supppliers, standard of living, corporate
use or household use etc. Hence there are two business segments for Microtech Ltd. ‘Auto batteries’ and
‘batteries for invertors/UPS’.
Problem 2: If by applying the 10% thresholds, one reportable segment is identified but there are 5 other
business/geographical segments which do not meet individually any of the 10% thresholds, what should the
enterprise do in this case.
158 Accounting Standards

Solution: In such case the decision regarding reportable segment lies with the management. As per TEST 5
(Management Choice) if any of the previous tests are not satisfied then management at its discretion choose
reportable segment. Such segments are disclosed under unallocated column.
Problem 3: M Ltd. Group has three divisions A, B and C. Details of their turnover, results and net assets are
given below :
` (’000)
Division A
Sales to B 3050
Other Sales (Home) 60
Export Sales 4090
7200
Division B
Sales to C 30
Export Sales to Europe 200
230
Division C
Export Sales to America 180

Head Office Division


A B C
`(’000) `(’000) `(’000) `(’000)
Operating Profit or Loss before tax 160 20 (8)
Re-allocated cost from Head Office 48 24 24
Interest costs 4 5 1
Fixed assets 50 200 40 120
Net current assets 48 120 40 90
Long-term liabilities 38 20 10 120
*Long term liabilities does not include interest bearing liabilities.
Prepare a Segmental Report for publication in M Ltd. Group.
(Final C.A. Nov. 2000 & May 2005) (8 Marks)
Solution:
M Ltd.
Segmental Report
` (‘000)
Divisions Inter-segment Consolidated
A B C Eliminations Total
Segment Revenue          
Sales :          
Domestic 60 - - - 60
Export 4,090 200 180 - 4,470
External sales 4,150 200 180 - 4,530
Inter Segment sales 3,050 30 - (3,080) -
Total revenue 7,200 230 180 (3,080) 4,530
AS-17: Segmental Reporting 159

Segment result (given)  160 20 (8)   172


 Head office expenses         (96)
Operating profit         76
Interest expense         (10)
Net Profit 66
Other Information          
Fixed assets 200 40 120   360
Net current assets 120 40 90   250
Segment assets 320 80 210   610
Unallocated corporate          
Assets         98
Segment liabilities 20 10 120   150

Sales Revenue by Geographical Market


Home Sales Export Sales Export to Europe Export to America Consolidation Total
(by division A)
60 4,090 200 180 4,530

Problem 4: Segment revenue does not include the following: (i) Indirect income like Interest, dividend,
Rent etc. (ii) Capital Gains on Sale of Assets. (iii) Extraordinary items.
Determine whether other items, such as export incentives, lease rent, interest from customers, can form
part of segment revenue as per AS – 17.
Solution:
Export incentives & Interest from customers: Both are part of operating income. Hence such operating receipts
has to be a part of segment revenue.
Lease Rent: A company may engage in leasing when it is profitable, in addition to the of sale of goods. For
example: An automobile company may be engaged both in selling the cars as well as leasing the car. In that
case leasing may be treated as a separate segment.
Problem 5: Superb Ltd. is a multinational company having registered office in Mumbai. The following details
are available from the books and other records of the company for the year ended 31st March, 2014:
` (’000) ` (’000)
Sales:
Domestic 7,625
Europe 1,676
America 2,325
Australia 766 12,392
Inter-unit sales between geographic areas (not included above)
Domestic 523
Europe 760 1,283
Operating profit:
Domestic 3,575
Europe 762
160 Accounting Standards

America 1,262
Australia 344 5,943
Other items:
General corporate expenses 362
Interest expenses 274
Income from investment 165
Identifiable assets:
Domestic 10,620
Europe 5,635
America 3,205
Australia 1,560 21,020
General corporate assets 750
Investments 675
Operating profit includes ` (’000) 62 on inter-geographical segment sales.
Prepare a statement showing financial information about the operations of Superb Ltd. in different
geographical segments.
Solution: Information about Superb Ltd.’s Operations in Different Geographical Segments

Geographical Segments
(Fig. in ` ’000)

Item Domestic Europe America Australia Inter-Area Consolidated


Eliminations figs.
Revenues:
Sales to unaffiliated
Customers 7,625 1,676 2,325 766 12,392
Inter unit sales
between geographic areas 523 760 (1,283)
Total revenue 8,148 2,436 2,325 766 (1,283) 12,392
Operating profit 3,575 762 1,262 344 5,943
Other items:
General expenses (362)
Interest expenses (274)
Income from investments 165
Net Profit 5,472
Assets:
Identifiable to
geographical area 10,620 5,635 3,205 1,560 21,020
General corporate assets 750
Investments 675
Total assets 22,445
AS-17: Segmental Reporting 161

Problem 6: The Chief Accountant of Sports Ltd. gives the following Data regarding its six segments:
` in Lakhs

Particulars M N O P Q R Total
Segment Assets 40 80 30 20 20 10 200
Segment results 50 –190 10 10 –10 30 –100
Segment Revenue 300 620 80 60 80 60 1,200

The Chief accountant is of the opinion that segments “M” and “N” alone should be reported. Is he justified
in his view? Discuss. (Final C.A. A/cs Nov 2010 OS, Audit May 2002)(Final C.A. May 2006)(5/4 marks)
Solution: As per para 27 of AS 17 Segment Reporting , a business segment or geographical segment should
be identified as a reportable segment if:s
(i) Its revenue from sales to external customers and from other transactions with other segments is
10% or more of the total revenue- external and internal of all segments; or
(ii) Its segment result whether profit or loss is 10% or more of:
(1) The combined result of all segments in profit; or
(2) The combined result of all segments in loss,
whichever is greater in absolute amount; or
(iii) Its segment assets are 10% or more of the total assets of all segments.
If the total external revenue attributable to reportable segments constitutes less than 75% of total
enterprise revenue, additional segments should be identified as reportable segments even if they do not meet
the 10% thresholds until atleast 75% of total enterprise revenue is included in reportable segments.
(a) On the basis of turnover criteria segments M and N are reportable segments.
(b) On the basis of the result criteria, segments M, N and R are reportable segments (since their results
in absolute amount is 10% or more of ` 200 lakhs).
(c) On the basis of asset criteria, all segments except R are reportable segments.
Since all the segments are covered in atleast one of the above criteria all segments have to be reported
upon in accordance with Accounting Standard (AS) 17. Hence, the opinion of chief accountant is wrong.
Problem 7: Fellicy Limited has investments (equity and debt) in the books of HO. The investments constitute
10% of total assets. There is some income of interest and dividend. Is it necessary to show this activity of the
HO as a separate segment or can it be shown as unallocated corporate assets?
Solution: As far as 10% limit is concerned it is material. If HO is actively engaged in trading in investments
and earning income out of it then it can be treated as a separate segment. But if only surplus cash in invested
and incidental income is earned then it can be taken to unallocated column.
Problem 8: A Company has an inter-segment transfer pricing policy of charging of at cost less 10%. The
market prices are generally 25% above cost. Is the policy adopted by the company correct?
(C.A. Final May 2007) (4 marks)
Solution: AS 17 ‘Segment Reporting’ requires that inter-segment transfers should be measured on the basis
that the enterprise actually used to price these transfers. The basis of pricing inter-segment transfers and any
change therein should be disclosed in the financial statements. Hence, the enterprise can have its own policy
for pricing inter-segment transfers and hence, inter-segment transfers may be based on cost, below cost or
market price. However, whichever policy is followed, the same should be disclosed and applied consistently.
Therefore, in the given case inter-segment transfer pricing policy adopted by the company is correct if,
followed consistently.
162 Accounting Standards

Problem 9: The management of Airways Ltd provides you the information related to one of its segment.
You are required to calculate Segment assets from the given information Plant, property, equipments =
` 24,00,000, Inventories = ` 7,00,000, Loans to employees = ` 4,00,000, Accounts receivable = ` 5,00,000,
DTA = ` 45,000 (C.A. Final NS) (5 marks)
Solution: Segment assets = 24,00,000 + 7,00,000 + 5,00,000 = ` 36,00,000.
Segment assets includes the operating assets employed for the operations of the segments. Any loan even
to employees also should not be considered segment assets.
Problem 10: A multinational enterprise by the name of Torrential International has business activities
located in three segments. The relevant details are as follows:

Allocation of net income and net assets


Location Relevant Percentages for Allocation of:
Revenue & Costs Assets & Liabilities (See note 2)
% %
Europe 60 40
North America 20 40
Asia 20 20
* The allocation percentage to be applied to revenue and cost for net of inter-group revenue (see note 3)
1. Details relating to head office
The head office procures all necessary finance for the enterprise’s activities and allocates this finance
to operating units through current accounts. Some costs, assets and liabilities relate solely to head
office and cannot be allocated to segments on a rational basis. These amounts are as follows:
ww Operating costs of ` 80 lakhs at 31st March 2015
ww Non-current financial assets
ww Bank balance of Head office is ` 140 lakhs at 31st March 2015
ww All liabilities except trade payables
2. Inter group revenues – year to 31st March 2015

Selling Segment Inter-Group Sales Inter-Group Sales made to


Europe North America Asia
` ’000 ` ’000 ` ’000 ` ’000
Europe 16,000 11,200 4,800
North America 12,800 8,800 4,000
Asia 10,400 5,600 4,800
Total 14,400 16,000 8,800

Extracts from the consolidated financial statements of Torrential International for the year ended
31st March 2015:

Statement of comprehensive income – year ended 31st March 2015


` ‘000
Revenue 532,000
Cost of sales (249,600)
Gross Profit 282,400
AS-17: Segmental Reporting 163

Distribution Costs (79,200)


Administrative expenses (94,400)
Profit from operations 108,800
Income from investments 4,800
Finance costs (20,000)
Profit before tax 93,600
Income tax expenses (22,400)
Profit after tax 71,200
Non controlling interest (6,400)
Net profit for the period 64,800

Statement of financial position as at 31st March 2015


` ’000 ` ’000
Assets
Non-current Assets
Property, Plant & Equipment 272,000
Financial assets 40,000 312,000
Current Assets
Inventories 60,000
Trade receivables 83,200
Bank balances 19,200 162,400
474,400
Equity and Liabilities
Capital and Reserves
Issued Capital 120,000
Accumulated profits 144,000 264,000
Non-current liabilities
Interest bearing borrowings 112,000
Deferred tax 28,800 140,800
Current Liabilities
Trade and other payables 56,000
Short-term borrowings 13,600 69,600
474,400

Required: Prepare a segment report for Torrential International for the year ended 31st March, 2015 that
complies with AS-17.
Solution:
Segment report for Torrential International
Amount in ` ‘000
Europe North America Asia Total
REVENUE
External Sales (60 : 20 : 20) 319,200 106,400 106,400 532,000
Inter-Segment sales 16,000 12,800 10,400 39,200
Total Revenue 335,200 119,200 116,800 571,200
164 Accounting Standards

RESULT
Segment Result (W1) 71,680 20,160 24,960 116,800
Unallocated corporate expenses (H O Operating cost) (8,000)
Profit from operations 108,800
Investment Income 4,800
Finance cost (20,000)
Income taxes (22,400)
Non controlling interest (6,400)
Net Profit 64,800
OTHER INFORMATION
Segment assets (W2) 168,160 168,160 84,080 420,400
Unallocated corporate assets (40,000 + 14,000*) 54,000
Consolidated assets 474,400
Segment liabilities (W3) 22,400 22,400 11,200 56,000
Unallocated corporate liabilities (140,800 + 13,600) 154,400
Consolidated liabilities 210,400
* Total bank balance – amount allocated to segments = ` 19,200 – ` 5,200 = ` 14,000 (000)

Workings (all figures in ` ’000)

W1 Segment result
Europe North America Asia Total
Segment Revenue 335,200 119,200 116,800 571,200
Segment costs (249,120) (83,040) (83,040) (415,200)
External*
Inter-Group (Note 2 to question) (14,400) (16,000) (8,800) (39,200)
Segment Result 71,680 20,160 24,960 116,800
* Total operating costs (excluding inter-group items) are 423,200 (249,600 + 79,200 + 94,400)
Head Office costs are 8,000
So costs to be allocated are 415,200 in the ratio (60 : 20 : 20)

W2 : Segment Assets all allocated in the ratio 40 : 40 : 20


Europe North America Asia
Property, plant & equipment (272,000) 108,800 108,800 54,400
Inventories (60,000) 24,000 24,000 12,000
Trade receivables (83,200) 33,280 33,280 16,640
Bank Balance (5,200) 2,080 2,080 1,040
168,160 168,160 84,080

W3 : Segment liabilities all allocated in the ratio 40 : 40 : 20


Europe North America Asia
Trade Payables (56,000) 22,400 22,400 11,200
AS-17: Segmental Reporting 165

Problem 11: M/s XYZ Ltd. has three segments namely X, Y, Z. The total assets of the Company are ` 10.00
crs. Segment X has ` 2.00 crs., segment Y has ` 3.00 crs. and segment Z has ` 5.00 crs. Deferred tax assets
included in the assets of each segments are X- ` 0.50 crs., Y—` 0.40 crs. and Z—` 0.30 crs. The accountant
contends that all the three segments are reportable segments. Comment. (CA Final Nov 2008 Marks 5)
Solution: According to AS 17 “Segment Reporting”, segment assets do not include income tax assets.
Therefore, the revised total assets are 8.8 crores [10 crores – (0.5+0.4+0.3)]. Segment X holds total assets of
1.5 crores (2 crores – 0.5 crores); Segment Y holds 2.6 crores (3 crores – 0.4 crores); and Segment Z holds
4.7 crores (5 crores – 0.3 crores). Thus all the three segments hold more than 10% of the total assets, all
segments are reportable segments.
CHAPTER 18
AS-18: Related Party
Disclosures

Chapter Outline
vv Introduction vv Exemptions
vv Definition of Related Party, Control, vv Exceptions
Significant Influence vv Disclosure Requirements of AS-18
vv Related Party Relationship to Which
AS-18 Applies [Para 3]

Introduction
AS-18 is more of a Disclosure Standard rather than a Standard on accounting treatment. It prescribes no
accounting requirements. It only prescribes disclosure requirements. AS-18 applies to the financial statements
of each reporting enterprise as also to consolidated financial statements presented by a holding company.

Objective of AS-18
It is a normal feature in a business to have related party (RP) in any business transactions. This is because RP
are very well known and the company can place faith on such RP. But RP’s can even siphon funds and affects
the volume and decision of business of one enterprise for the benefit of the other enterprise or for own self
interest.
Following are the consequences of RP transactions:
(i) Transactions may not be at arms length price (sometimes no price is charged)
ww providing free management services.
ww providing interest free loan.
ww sales at a discounted price.
ww payment at inflated price.
ww pledging personal assets/giving guarantees.
(ii) Existence of RP affects financial and operating decisions.
(iii) Transactions would not have been taken place if there were no related party relationship.
(iv) RP may be formed to fulfill the bad desires of Management.
(v) Many frauds previously took place because of RP relationship. The parties behind the fraud were
supposed to be RP.
Thus a related party relationship could have an effect on the financial position and operating results of
the reporting enterprise. The purpose of the Standard is to make the reader of financial statements aware of
the existence of related party relationships and the extent to which an entity’s financial position, profitability,
AS-18: Related Party Disclosures 167

or cash flows may have been affected by transactions with such parties. It should be noted that this is a
disclosure Standard and does not deal with recognition or measurement issues, all of which are dealt with by
other relevant Standards. Disclosure of RP transaction is a good practice of Corporate governance reporting.

Scope of AS-18
AS-18 prescribes disclosures of:
(i) Related party relationships; and
(ii) Transactions between a reporting enterprise and its related parties.
Such disclosures convey to the users that certain RP relationship exists and RP transactions took place
during the year. Company should be fair enough to disclose the names and transaction of RP.

Definition of Related Party, Control, Significant Influence


(i) Definition of Related Party: Parties are RP if at any time during the reporting period one party has the
ability to control or exercise significant influence on another.
(ii) Definition of Control: Control can be exercised in the following ways:
(1) Control by ownership by purchasing directly or indirectly more than 50% of shareholding.
(2) Control by controlling the composition of BOD (appointment/removal of directors).
(3) Control by acquiring the substantial interest in voting power and the power to direct the financial
and operating policies of the companies.
Control is even defined in AS-21, but it is not as stringent as defined in AS-18. In case of AS-21 the first two
points on control is being covered.
(iii) Definition of Significant Influence: It is a position whereby no control but has significant influence
over decisions making directly or indirectly. If 20% interest is acquired than it is enough to show that
there is significant influence, unless such influence can
otherwise to be demonstrated. It means that acquiring List of relatives
a substantial interested in other company means that Section 2 (77) defines relatives as: (Under the
there is a significant influence, unless it can be clearly New Companies Act)
demonstrated that this is not the case.   Members of HUF, Husband and Wife, Father
(including step father), Mother (including step
SI can be exercised in the following ways: mother), Son (including step son), Son’s wife,
ww by representation on the BOD. Daughter, Daughter’s husband, Brother and
ww participation on policy making decisions. Sister including the step brother and step sister.
  As compared to the old companies act now
ww inter – change of managerial personnel.
the list is quite short.
ww material inter – company transactions. KMP:
ww dependence on technical information. Section 2 (51) defines KMP as: (Under the New
Significant influence can be acquired by voting power, Companies Act)
statute or agreement. MD, CEO, Manager, WTD, CS.

Related Party Relationship to Which AS-18 Applies. [Para 3]


or
Analysis of Related party relationships [Para 3] (C. A. Final Nov. 2001)
AS-18 applies only to the following related party relationships described below:
Clause a: Enterprises that directly, or indirectly through one or more intermediaries, can control or are
controlled by or are under a common control with the reporting enterprise.
168 Accounting Standards

Enterprises that directly, or indirectly through one or more intermediaries


(i) Controls the reporting enterprise (e.g., holding company of the reporting enterprise)
(ii) Are controlled by the reporting enterprise (e.g., subsidiaries of the reporting enterprise)
(iii) Are under the common control of the reporting enterprise (e.g. fellow subsidiaries/co-subsidiaries).
ww Meaning of directly or indirectly means sub – subsidiaries.
ww According to ASI-19- ‘Interpretation of the term Intermediaries’, covers only ‘subsidiaries’ of the
reporting enterprise as defined in AS-21. The term should not be extended to cover associates
etc.
Clause b: Associates and Joint Ventures of the reporting enterprise and the investing party or venture in
respect of which the reporting enterprise is an associate or a joint venture.
Example: C Ltd. is a joint venture operated by co venturer A Ltd. and B Ltd. For both A Ltd. and B Ltd., C Ltd.
(joint venture) is a related party. For C Ltd. both A Ltd. and B Ltd. Co ventures are related parties. But A Ltd.
and B Ltd. are not related parties only by virtue of they being co - venturers.
Also Associates and the investing parties (reverse) are covered, but co – associates are not RPs. For ex.: A
Ltd. has 21% shares in B Ltd. and further B Ltd. has 21% shares in C Ltd. than A Ltd. and C Ltd. are not said to
be RP’s.
Clause c: Individuals owning, directly or indirectly, an interest in the voting power of the reporting enterprise
that gives them control or significant influence over the enterprise, and their relatives.
Clause d: Key Management Personnel (KMP) and their relatives.
Clause e: Enterprises over which any person described in clause 3 [c] or 3 [d] have control or are able to
exercise significant influence.

Special Discussion on (Key Management Personnel) KMP : ASI -21


KMP are the personnel of the company who has authority and responsibility to plan, control and direct the
operating and financial policies of a company. They participate at all the levels i.e. planning, controlling and
directing activities of the company. M.D., WTD and Managers are the examples of KMP. KMP are related parties
as per para 3(d) of AS-18. If a person acting in the capacity of KMP but not termed as MD/KMP is KMP in
substance if he is involved in planning, controlling and directing. ASI – 21 states that – Merely being a director
of the company is not sufficient unless that director has the authority and responsibility to plan, control and
direct the activities of the company. Remuneration paid to KMP’s is a RP transaction.
Who is KMP Who is not a KMP
Whole Time Directors Simplicitor Director
Managing Directors CFO
Managers Functional Head
Chief Executive Officer Departmental Head

Exceptions to Related Party Relationships or Who are not Deemed to be RPs [Para 4]
Following are not deemed to be RP’s:
(i) Two Companies simply because they have director in common (unless the director is able to affect
the policies of both the companies in their mutual owings).
(ii) Single customer, supplier, distributor, franchise, agent etc. with whom significant volume of business is
transacted. These are those parties on which the company has economic dependency.
(iii) The parties listed below, in the course of their normal dealings with an enterprise by virtue only of
those dealings (Although they may circumscribe the freedom of action or may even participate in the
decision making process).
Providers of financers, trade unions, public utilities, government departments & government agencies.
AS-18: Related Party Disclosures 169

Exemptions in Disclosure Requirements of AS-18 [Para 5]


There may be a RP relationship but as per para 5 following are the exemptions granted for disclosures:
(i) If confidentiality is required by statute or an agreement then the reporting party may not disclose
such transactions. For example Banks are not required to disclose the banking transactions which
conflict with their duties. This exemption is not available in respect of confidentiality in a commercial
contract between two enterprises where confidentiality is not required as per any statute or any law.
(ii) Intra – group transactions in Consolidated Financial Statements. It means transactions between
Holding and its subsidiary should be eliminated while presenting group consolidated accounts. This
is not applicable to an Associate, because associate is not consolidated but accounted as per equity
method.
(iii) Relationship and transactions between one state controlled enterprise with the other state controlled
enterprise. State controlled enterprise is that company which is controlled by State Government.

Details of disclosure if transactions take place during the existence of related party relationship.
1. Name of the related party. 2. Description of relationship. 3. Description of nature of transaction. 4. Volume of transaction
either as amount or as proportion. 5. Any other important element of the transaction. 6. Outstanding at balance sheet
date related to related parties and provisions for bad debts. 7. Amount written off or written back in the period in
respect of debt due from or due to related party.
Examples of related party transactions which will be covered:
a. Purchase or sale of goods, service and fixed assets
b. Agency arrangements, leasing and hire purchase arrangement
c. License agreements
d. Transfer of research and development
e. Finance including loans and equity contribution
f. Guarantees and collaterals
g. Management contracts including for deputation of employees.
h. Providing consultation services.

IMPORTANT POINTS
(1) Where purchase is made in one year and payments are made in the subsequent year then reporting for RP
is done in the first year. This is because transfer of resources takes place in previous year and in the next
year it is the consideration paid towards resources already transferred.
(2) A member of Local Advisory Board of a Bank is not a KMP.
(3) A shareholder holding less than 20% shares and is the only shareholder holding maximum shares is not an
RP.
(4) A Non – Executive director is not a KMP because he only participates in the financial and operating decisions
and does not control.
(5) AMC is the RP in case of Mutual Fund.
(6) Reimbursement of expenses incurred by KMP on behalf of the company is not a related party transaction.
How related Party is covered by other sections/act
(1) Companies Act - Section 297
(2) Companies Act - Section 299
(3) Companies Act - Section 301
(4) Income Tax Act – Section 40 A (2) (b)
(5) CARO – Clause 4 (iii)
(6) CARO – Clause 4 (v)
(7) Auditing Practices – AAS 23 (now SA 550)
(8) SEBI Act – Yearly disclosures are made mandatory.
170 Accounting Standards

PROBLEMS AND SOLUTIONS

P roblem 1: NANO Limited has set up a trust for gratuity and pension fund. It transfers Funds of ` 20,00,000 in
the financial year 2009 -2010. Will the transfer of funds to the trust a Related Party transaction?
Solution: Every company who forms trusts/Gratuity Funds have full control over the trusts. Therefore such
trusts/Gratuity Funds are RP. But the trusts/Gratuity Funds is formed for the benefits of the employees,
hence such transactions are not treated as RP transactions.
Problem 2: Apple Limited has two Associates, Mango Limited and Orange Limited, owning 25 per cent of
the voting power of Mango Limited and 30% of the voting power of Orange Limited. Would Mango Limited be
considered a related party in the financial statements of Orange Limited?
Solution: As per para 3 (b) of AS-18 Associates and Joint Ventures of the reporting enterprise and the
investing party or venturers in respect of which the reporting enterprise is an associate or a joint venture;
are RP. But sub associates (associates of associates) or co – associates (fellow associates) are not covered. In
the present problem Mango is fellow associate of Orange Ltd. hence they are not RP.
Problem 3: P Ltd. has 60% voting right in Q Ltd. Q Ltd. has 20% voting right in R Ltd. Also, P Ltd. directly
enjoys voting right of 14% in R Ltd. R Ltd. is a listed company and regularly supplies goods to P Ltd. The
management of R Ltd. has not disclosed its relationship with P Ltd.
How would you assess the situation from the viewpoint of AS-18 on Related Party Disclosures?
(CA Final Nov 2007, Nov 2012)
Solution:
Investing Investee % of RP
Company Company holding
P Ltd Q Ltd 60% Yes – para 3 (a) Control
Q Ltd R Ltd 20% Yes – para 3 (b) Sig influence
P Ltd R Ltd * 34% Yes – para 3 (b) indirect Sig influence

* P is holding directly 14% and through its subsidiary Q 20% = 14+20 = 34%.
Problem 4:
60%
A B
15% 40%
C

Solution:
Investing Investee % of RP
Company Company holding
A Ltd B Ltd 60% Yes – para 3 (a) Control
B Ltd C Ltd 40% Yes – para 3 (b) Sig influence
A Ltd C Ltd * 55% Yes – para 3 (a) indirect Control
* A is holding directly 15% and through its subsidiary B 40% = 15 + 40 = 55%.
AS-18: Related Party Disclosures 171

Problem 5:
60%
A B
60%

X C
40%

Solution:
Investing Investee % of RP
Company Company holding
A Ltd B Ltd 60% Yes – para 3 (a) Control
B Ltd C Ltd 60% Yes – para 3 (a) Control
X Ltd C Ltd 40% Yes – para 3 (b) Sig influence
Co. X and A are
not RP.

Problem 6:
30%
A B
25%

Solution:
Investing Investee % of RP
Company Company holding
A Ltd B Ltd 30% Yes – para 3 (b) Sig influence
B Ltd C Ltd 25% Yes – para 3 (b) Sig influence
A Ltd C Ltd - Sub associates are not covered
Co. A and C are not RP.

Problem 7:
60%
A B
70%

Solution:
Investing Investee % of RP
Company Company holding
A Ltd B Ltd 60% Yes – para 3 (a) Control
A Ltd C Ltd 70% Yes – para 3 (a) Control
Co. B Ltd and C ltd is RP because they are under a common control with Company A Ltd. Refer para 3 (a).
172 Accounting Standards

Problem 8: Narmada Limited sold goods for ` 90 lakhs to Ganga Limited during 31.3.2006. The Managing
Director of Narmada Limited owns 100% of Ganga Limited.
Sales are made to Ganga Limited at Normal Selling Price. The Chief Accountant of Narmada Limited contends
that no disclosure is required transactions are at arm length price.
(C A Final Nov 2001/May 2006)
Solution:
M. D. of Narmada Ltd is KMP para 3(d) → RP
M. D. of Narmada Ltd is also holding 100% of Ganga Ltd 3(c) → RP
Therefore; Narmada Ltd and Ganga Ltd are RP of each other para 3(e) → RP
Therefore the contention of the chief accountant is wrong.
Problem 9: A husband and wife are controlling 34% of voting power in XY Limited. They are having a
separate partnership firm, which supplies mainly the raw-material to the company. The Management says
that the above transaction need not be disclosed.
(C.A. Final Nov. 2003)
Solution:
(Husband+Wife) holding 34% shares of XY Ltd is para 3(c) → RP (Sig influence)
Also. (Husband+Wife) have a separate partnership (PF) para 3(c) → RP (Control)
Therefore;
PF and XY Ltd both are RP of each other para 3(e) → RP
Therefore the contention of the management is wrong. Raw materials supplied by partnership firm to the
company should be covered as a RP transaction.
Problem 10: Heavy Limited divested 65% shares in its subsidiary Light Limited on 7/1/2009 and as such
no relationship as per AS-18 exists on the year end. Should the H Co. disclose the relationship and the related
transaction on 31/3/2009?
Solution: As per AS-18 para 23 provides that if there happens to be a RP relationship during the year
then reporting is mandatory. Holding Co is required to report it in its separate financial statement. It is not
necessary that the relationship should exist on the year end. Even if the relationship existed at any time in
the year then AS-18 is attracted. In the present case even if there would not be any transaction then also
reporting would be attracted because relationship is that of Control.
Problem 11: Mr. Rajjab KMP of Shehzadaa Limited received a cheque of ` 1,40,000 towards reimbursement
of expenses incurred on stay in hotel, conference expenses etc. visiting some meeting on behalf of the
company. You are required to guide whether the above transaction is covered under AS-18?
Solution: KMP is of course a RP to the company, but the reimbursement of expenses are not in the nature
of remuneration. Reimbursement of expenses is for business purpose. Hence such transaction will not be
disclosed in the financial reporting.
CHAPTER 19
AS-19: Leases

Chapter Outline
vv Objective and Importance of AS-19 vv Leasing Activities Undertaken
vv Types of Lease by Manufacturer/Dealer
vv Change in Classification vv Sale and Lease Back

vv Recognition Principles or Accounting vv Initial Direct Costs


Treatment for Leases vv Disclosure Requirements

Objective and Importance of AS-19


Lease financing by companies has acquired a great importance during the last few years. Previously the
lessor use to claim depreciation on the leased assets. But now the dispute has been settled down with the
introduction of AS-19. Lease is nothing different what we know regarding Lease i.e. one party to the lease
is the owner of the asset and further he lets out the asset for use to another party in consideration of lease
rent. What the Standard wants to resurface is the real substance attached with some of the Lease agreements
which happens to be a Finance Lease. Curtains have to be raised in case of Finance Leases. What appears
as legal form is that a Lessor is the owner of the asset hence he should record the asset. But in case of the
Finance Leases the substantial risks and rewards in the assets are transferred by the lessor to the user of
the asset for the maximum life of the asset. The lease arrangements are effectively sale transactions whose
consideration is received in installments. Hence as per the standard lessee should be the owner because the
lessee enjoy all the rights as a owner. Installments not only cover principals but also compensate the lessor
for the finance income.
Leasing is one of the major important points to be considered for the application of the accounting policies,
viz “substance over form”. The relevance of this principle is that, in legal form the lessor is the owner of the
asset, but in actual economic reality (substance) lessee is the owner who uses the asset.

Scope of AS-19
AS-19 applies to all types of leases except the following:
◘◘ Lease agreements entered into for the exploration or use of resources such as oil, gas timber etc.
◘◘ Motion picture, manuscripts, plays, video recordings, copyrights, patents etc. given on license.
◘◘ Lease agreements for the use of vacant lands.
◘◘ Service contracts – The reason being, one contracting party does not transfer right to use the asset to
another party.
174 Accounting Standards
174

IMPORTANT TERMS & THEIR DEFINITIONS


(a) Lease: Simply speaking lease is an arrangement whereby the lessor lets out its asset to the lessee for the
receipt of periodical lease rentals. Lease also includes Hire purchase agreements also.
(b) Finance lease: Finance lease is an agreement whereby the lessor transfers to the lessee substantially
whole of the risks and rewards of the asset incidental to the ownership.
(c) Operating Lease: Operating lease is an agreement whereby the lessor does not transfers to the lessee
substantially whole of the risks and rewards of the asset incidental to the ownership. It is the lease other
than the finance lease. In this none of the above conditions are met.
(d) Contingent Rent: It is the lease rent received not on the basis of time factor but based on sales made, price
indices, units produced etc. In other words the rent is not fixed but fluctuating.
(e) Minimum Lease Payments [MLP]:
ww MLP for lessee = (Lease Rentals + GRV as per lessee). MLPs excludes contingent rent, cost for services,
maintenance and taxes to be paid by and reimbursed to the lessor by the lessee.
ww MLP for lessor = (Lease Rentals + GRV as per lessee or any other third party capable to pay such
residual value). This means GRV for lessee and lessor may be different.
(f) Guaranteed Residual Value [GRV]:
ww For lessee: Any residual value guaranteed by or on behalf of the lessee.
ww For lessor:
(i) Any residual value guaranteed by or on behalf of the lessee or
(ii) by any independent third party.
(g) Unguaranteed Residual Value [UGRV]: It’s always from the point of view of the lessor.
UGRV = [Estimated Residual Fair Value of the asset - GRV]
(h) Gross Investments in lease: It’s always from the point of view of the lessor.
Gross Investments = (MLP + UGRV) = (Lease Rentals + GRV as per lessor +UGRV)
(i) Unearned Finance income: It’s always from the point of view of the lessor.
It is the amount of Finance income at the inception of the lease which is yet not earned by the lessor.
Unearned Finance Income = [Gross Investment – PV of (MLP + UGRV)]
(j) Net Investments in lease: It’s always from the point of view of the lessor.
Net Investments represents the Fair Value on the date of inception.
Net Investment (which is nothing but present value of Gross Investments)
= Gross Investment – Unearned finance income
(k) Interest rate: Interest rate implicit in the lease: It is the rate at which interest is included in the lease
installments. More accurately it can be defined (at the inception of the lease) as, that discount rate which
makes, [PV of MLP (from lessors point of view) + UGRV accruing to lessor] = Fair Value of the asset (at the
beginning of the lease).
Which interest factor should be considered: Lessor should consider its own IRR as discussed above. For lessee if
lessors IRR is not appropriate then lessee’s incremental borrowing rate is to be taken or even weighted avg cost of
capital.

DO YOU KNOW
Whether the lease is a finance lease or operating lease is always decided as on the Zero date.
AS-19: Leases 175

Types of Lease
Lease

Finance lease Operating lease

What is transfer of substantially the whole of risks and rewards?


Transfer of risks means risks due to the loss of demand, technology obsolescence, and losses from idle
capacity.
Transfer of rewards means heavy demand in the products, residual value of the asset.
One should note that the transfer is of risk and reward only, and not of the legal ownership. The legal
ownership lies with the lessor only. But what AS-19 observes is the substance and not the legal form.
In many companies like car leasing companies (lessor) borne the repairs and maintenance or insurance
so that they can attract customers. In such a case one cannot say that it is not a finance lease. The reason
being, the lessor companies do not want to trouble the lessees in which case the lessors are expert in taking
out insurance themselves, having Repairs workshop etc. The second reason is that the cost of repairs etc is
inbuilt in the lease rent.
Under the terms of a lease the gas cylinders were delivered by manufacturer/supplier at the location of the lessee. Lessor
was not responsible for any damage before or after the delivery of the cylinders. The Lessee was responsible to pay all
the related taxes, charges, license fees, rates, surcharges, registration in respect of the cylinders. The lessor was required
to only purchase the asset. The equipment was also to be selected by the lessee. The Lessor party has to recover an
amount higher than the cost of cylinders over the period of three years. After that the asset will be transferred to the
lessee.
Yes; it is a case of Finance Lease.
Proof: Ultimately the asset is transferred to the lesse after the expiry of the lease period. Right from the beginning the
lessor doses not have any control over the equipments. The Lessor has just finance the asset. It is a finance arrangement.

Conditions for Finance lease: para 9 (Any one condition should be satisfied)
(i) Transfer of ownership of the asset to the lessee at the end of the lease term.
(ii) Lessee is given an option to purchase the asset at the end of the lease term, at a price that is sufficiently
lower than the fair value on the date of exercise of the option.
(iii) Lease term is for the major part of the economic life of the asset even if the title is not transferred.
(iv) At the inception (beginning) of the lease the PV(MLP) recovers substantially the portion of the Fair
Value of asset on the date of inception of lease. This is known as Fair value recovery test.
(v) The lease is a non-cancellable lease agreement. A non-cancellable lease agreement is that which is
cancelled only due to the following circumstances:
ww Occurrence of some remote contingency.
ww With the permission of the lessor.
ww The lessee enters into a fresh agreement for same or similar asset with the same lessor.
ww Payment made by the lessee of such an additional amount (at the inception), which shows the
continuation of lease is certain.
(vi) The Leased asset is of a specialized nature such that only the lessee can use it, without much
modifications being made.
The above factors are very important one for lease classification. Satisfaction of any one of the above
factors is enough to entitle the lease agreement to make it a finance lease. For ex. : A Co. lets its
176 Accounting Standards

Machine for 9 years. The useful life of the machine is 9yrs. Here in this case, condition number 3 gets
satisfied, other conditions remains silent or might not have been satisfied. Thus, it is a finance lease.
It is also been observed that when one condition is satisfied, then other conditions do get attracted
to classify to make the agreement a lease agreement.
However some additional conditions are also given with either individually or in combination with
other conditions may be considered as finance lease. (These are suggestive conditions only)
(vii) Lessee cancels the lease agreement, and the losses associated with the cancellation of lease are borne
by the lessee.
(viii) Gains or losses fluctuation in the residual value of the asset falls on the lessee.
(ix) Lease rentals for the secondary period of the lease are substantially low than the market rent.

DO YOU KNOW
In case of operating lease there can be multiple lessees. Even the lessor uses the asset for some time.

Change in Classification
Whether a lease is a finance lease or operating lease is decided at the very inception of the lease. If one come
across a change in the classification of the lease in future then following treatment should be given:
(a) If the change results into a different category of lease, then the lease agreement thereafter to be
treated as a totally separate agreement for the balance period.
(b) But say if some estimates go wrong like for ex. wide change in useful life of asset or default made by
the lessee etc. then the classification remains same i.e. no change in classification.

Recognition Principles or Accounting Treatment for Leases


If the Lease is Classified as a Finance Lease
Accounting in the books of Lessee Accounting in the books of Lessor
As already discussed in preceding paras that though the Initial Recognition: Under the finance lease lessor should
lessor is the owner of the leased asset in legal form but by record the lease rentals receivables as follows:
substance lessee should be treated as the owner as per the
standard. So the leased asset for the lessee is as good as
asset purchased.
Initial Recognition: At the inception of the lease the lessee Journal Entry:
should record the asset purchased on the asset side of the Lease rentals receivable A/C ...Dr.
balance sheet and payment to the lessor as a liability. Value (Equal to net investments in lease)
of the asset to be recorded should be lesser of [Fair Value   To Asset A/c
on inception or PV of MLP from lessees point of view].
Journal Entry: Lease Receipts: Here again the lease installments are to
Asset A/c ...Dr. be segregated into Principal + Finance income. Finance
To Lessor A/c income should be based on the pattern reflecting a
constant periodic rate of return on the net investment of
the lessor outstanding.
Interest Rate: It is the rate at which interest is included Interest rate: Interest rate implicit in the lease: It is the
in the lease installments. For lessee the interest rates can rate at which interest is included in the lease installments.
be incremental borrowing rate/weighted average cost of More accurately it can be defined (at the inception of the
capital/lessors IRR. lease) as, that discount rate which makes, [PV of MLP(from
lessors point of view) + UGRV accruing to lessor] = Fair
Value of the asset (at the beginning of the lease).
AS-19: Leases 177

Computation of Depreciation: As of now it is well settled


that the lessee will be the owner of the asset, thus he should FMV =
enjoy the depreciation claim also. The basis of charging
depreciation will be normal as applicable to other assets.
Depreciation policy will be as per AS- 6.
Lease Payments: The lease installments paid by the lessee Review of UGRV: A Lessor is required to reassess and
to the lessor comprised of principal and interest element. review at every Balance Sheet the value of UGRV. Any
Thus the bifurcation is necessary for proper accounting. change in UGRV will change the Income allocation and
Principal amount element is to be reduced from the even the interest rate implicit in the lease rent.
outstanding balance year by year. Finance charge (Interest)
is to be allocated over the lease term in such a manner that
the rate of interest remains constant throughout the lease
term.

DO YOU KNOW
Many a times the interest and principal component is different for lessor and lessee. Following are the reasons
for such difference:
yy Difference due to GRV guaranteed by third party
yy Difference due to different discounting factor used by the lessor and lessee.
yy Difference due to UGRV.

If the Lease is Classified as a Operating Lease


Accounting in the books of Lessee Accounting in the books of Lessor
1. Unlike Finance lease, lease payments should be 1. In case of operating lease, the lessor is the owner of the
recognized in profit and loss account on straight-line asset. The Lessor should present the leased asset under
basis over the lease period. Even if the rentals are spread the head fixed asset.
uneven during the lease period then also recognition
of lease should be on SLM basis. If any other method
is more representative of the time pattern of the user’s
such method can be used for example Lease rent
allocated on the basis of number of units produced.
2. Costs of repairs, insurance, maintenance etc. paid by 2. Depreciation will be charged as per AS-6 by the lessor as
lessee should not be spread equally over the lease term. he is the owner of the asset.
It should be expensed in the year it is incurred.
3. Lease income should be recognized in profit and loss
account on straight-line basis over the lease period. Even
if the rentals are spread uneven during the lease period
then also recognition of lease should be on SLM basis.
If any other method reflects more systematic allocation
of earning derived from the diminishing value of leased
asset, that approach can be adopted.
4. Receipts for services provided for repairs, maintenance
etc. is to be recognized in the year it is earned.
5. Costs like depreciation, maintenance etc., incurred for
operating the lease is to be expensed in the year it is
incurred.
178 Accounting Standards

Leasing Activities Undertaken by Manufacturer/Dealer


Whatever we have discussed till now is the case of direct financing lease where the lessor (bank or any other
financing institution) purchases the asset and let out the asset for lease rentals. But in case of manufacturer/
dealer leases, the manufacturer in addition to direct sales of goods/assets he may even go for leasing the
manufactured asset. This means partly the assets are kept for sale and partly kept for leasing. It is also known
as sale - type – lease. Again one has to classify the sale-type-lease as finance lease or operating lease.
If it is a Finance lease: If the manufacturer/dealer go for finance lease, it will give rise to two types of
income accruing to him:
(i) profit or loss resulting from outright sale of the asset being leased, at normal selling prices.
(ii) finance income spread over the lease period based on IRR basis.

Measurement of income: In case of manufacturer/dealer lease Profit on sale = FMV – Cost of goods
Finance Income = (Gross Investments in lease – FMV).
The Finance income is to be allocated as per IRR over the lease period.
Low interest rate case: Sometimes the manufacturer/dealer keeps interest rate artificially low as compared
to commercial rates in order to attract customers. Even some finance leases are sold at ‘zero interest rate’.
If this is done, then much of the portion of income will be absorbed by selling profit (at the inception). So, to
curb this, AS-19 suggests: Compute the selling profit as per the normal prevailing market interest rates.
If Operating lease: In case of operating lease there is nothing as such sale. No selling profit is to be recognized
at the inception of lease. The Lease payments received will be the sole income and no selling profit is to be
accounted.
Distinction between Direct Finance Lease and Manufacturer Lease
Direct Finance Lease Manufacturer Lease
(i) Here one party manufactures the asset and sell it to (i) Here manufacturer of the lease himself leases the asset.
a finance company. The finance company in further
leases it to the third party.
(ii) Only Finance Income is recorded as the income for the (ii) Total income of the Lessor consists of selling profit +
lessor. Finance income.
(iii) Here Fair Value at the inception of the leasing becomes (iii) Here Sale Value at the inception of the leasing becomes
the Net Investments. the Net Investments.

Sale and Lease Back


Previously many companies use to enter into Sale & lease back transactions and record huge notional profits
in their books. But after the introduction of AS-19 such unrealized profits is not accounted for.
Sale & lease back transaction involves sale of asset by the vendor and leasing back the same asset to the
same vendor. In other words the original vendor will now becomes the lessee. Both the arrangements viz;
Sale of Asset and Leasing are in fact assigned as a single package. The sale price and the lease rent may not
represent the fair value. Once again in case of Sale & Lease back accounting treatment depends upon whether
the lease is finance or operating lease. As far as accounting of Lease Rent is concerned it is same as explained
earlier. But accounting treatment for the profit or loss arising on sale of asset is explained as follows:
◘◘ If Sale and lease back is a Finance lease:
The difference between (Sale proceeds - Carrying amount of asset) will be either profit or loss. Such
profit or loss is to be spread over the lease term in proportion to the depreciation of asset. One should
not recognize full profit/loss in the same year. The reason being the same asset is received back and
capitalized by the lessee with different amount, which is nothing but revaluation of asset. Therefore
the profit/loss on sale of asset should be amortized over the useful life of the asset.
AS-19: Leases 179

◘◘If Sale and lease back is a Operating lease:


For theoretical discussion let’s say Sale Price = SP, Carrying Amount = CA and Fair Value = FV.
Now consider 3 cases as follows:
Case I:If Sale price of asset = Fair Value (means price is not manipulated)
Profit or loss on sale of asset (SP – CA) is to be recognized immediately.
Case II: If Sale price of asset < Fair Value (means price is not manipulated)
Profit or loss on sale of asset (SP – CA) is to be recognized immediately.
Exception: If loss is compensated by charging future lease rentals lower than the market price, then
defer and amortize such loss in proportion to lease payments over the lease term.
Case III: If Sale price of asset > Fair Value
ww The excess of SP over FV (SP – FV) → amortised over the useful life of the asset
ww Profit or loss is due to (FA – CA) → can be immediately recognised. Alternatively profit or loss
can be balancing figure.
Word of Caution
Remember if in any of the above case I, II or III CA exceeds FV then provide for the loss of asset first and
then apply the above principles. (Conservatism).

Initial Direct Costs


Initial direct costs are expenses incurred initially in arranging a lease. Example are legal charges, commissions,
stamp duty, brokerage, consultancy fees etc. Such costs may be incurred by the lesee or lessor. But mostly it
is incurred by the lessor to attract the lessee customers.
Accounting treatment
Type of lease In the books of lessee In the books of lessor
Finance Lease Capitalized as a cost of asset Expensed immediately or apportion in the ratio of finance income
Operating Lease AS-19 is silent but it may be Expensed immediately or apportion in the ratio of finance income
expensed immediately
Manufacturer or Capitalized as a cost of asset Expensed immediately because it forms part of Profit and Loss
dealer lease account as normal selling expense

10. Disclosure Requirements


For Finance Lease
In the books of lessee:
◘◘ Segregation of lease assets and own assets and the carrying amount of the assets.
◘◘ Total MLP and P.V. of MLP as follows:

Period (Months) Absolute P.V.


0 – 12 xxx xxx
12 – 60 xxx xxx
60 or more xxx xxx

◘◘ Contingent rent (expense) recognized in P & L A/c


◘◘ Total sub-lease payment receivable on the date of balance sheet.
◘◘ Signification terms of lease agreement like further lease, contingent rent etc.
180 Accounting Standards

In the books of lessor:


◘◘ Gross investment and PV (MLP) receivable as follows:

Period (Months) Absolute P.V.


0 – 12 xxx xxx
12 – 60 xxx xxx
60 or more xxx xxx

◘◘ Unearned finance income.


◘◘ GRV
◘◘ Contingent rent recognized in P & L A/c.
◘◘ Significant terms of lease agreement like further lease, contingent rent etc.
◘◘ Accounting for initial direct cost.

For Operating Lease


In the books of lessee:
◘◘ Total of further MLP as follows:

Period (Months) Absolute P.V.


0 – 12 xxx xxx
12 – 60 xxx xxx
60 or more xxx xxx

◘◘ Lease payment debited to P & L A/c


◘◘ Contingent rent recognized in P & L A/c.
◘◘ Total sub lease payments receivable on the date of balance sheet date.
◘◘ Significant terms of lease agreement like further lease, contingent rent etc.
In the books of lessor:
◘◘ For each class of asset:
(a) Gross carrying amount
(b) Accumulated depreciation
(c) Impairment of asset if any
(d) Net carrying amount
◘◘ Finance income recognized during the year in P & L A/c
◘◘ Future lease payments as follows:

Period (Months) Absolute


0 – 12 xxx
12 – 60 xxx
60 or more xxx

◘◘ Contingent rent recognize in P & L A/c


◘◘ Significant terms of lease agreement like further lease, contingent rent etc.
◘◘ Accounting policy for initial direct cost.
AS-19: Leases 181

PROBLEMS AND SOLUTIONS

Problem 1: State whether the following comes within the scope of AS-19.
1. A Ltd. gives its well for extraction of oil to B Ltd. for 7 years at lease. .
2. Cola Ltd. entered into copyright agreement with Coke Ltd.
3. M Ltd. lets out its vacant land to P Ltd. at a rent of ` 50,000 p.m.
Solution:
Scope of AS – 19:
1) Not covered by AS – 19 (refer para 2)
2) Not covered by AS – 19 (refer para 2)
3) Not covered by AS – 19 (refer para 2)
Problem 2: Following are some of the cases given. Classify with reasons whether the following is Finance
or Operating lease –
(a) Y Ltd. wishes to obtain a machine tool costing ` 20 lakhs by way of lease. The effective life of the
machine tool is 12 years but the company requires it only for the first five years. It enters into an
agreement with R Ltd. for a lease rental of ` 2 lakhs p.a. The Finance Director of Y Ltd. is not sure
about the treatment of these lease rentals and hence requests your assistance in proper disclosure
of the same. For calculation purposes, the implicit rate of interest may be taken at 15%. Discount
factors : 0.87, 0,76, 0.66, 0.57 and 0.50. (C.A. Final May 2002/New syllabus June 09) (5 Marks)
(b) F Ltd. has a newly set up its pharmaceuticals business. It has installed a huge plant, which will be
used only by F Ltd. without any modifications at a monthly rent of ` 10,00,000.
(c) K Ltd. leased out its assets to R Ltd. at ` 5,000 p.m. for 6 years after which the asset will be retained
by the lessee by paying ` 30,000 at the end of 5th year. Residual value estimated at the end of 5th year
is ` 40,000.
Solution:
(a) Lets calculate the PV of MLP of the leased asset.
Cost of the asset = ` 20,00,000, Agreement period = 5 years, Rent = ` 2,00,000.
PV of MLP’s @15% = 2,00,000 x Cum. PVF@15% = 2,00,000 x 3.36 = ` 6,72,000.
PV of MLP’s is less than the FMV on the date of lease agreement. Also none of the other conditions of
FL are being satisfied.
Hence it is a Operating Lease.
(b) The Leased asset is of a specialized nature such that only the lessee can use it, without much
modifications being made. Hence it is a FL.
(c) Lessee is given an option to purchase the asset at the end of the lease term, at a price that is sufficiently
lower than the fair value on the date of exercise of the option. The FV at the end of the 5th year is `
40,000 and the lessee is given the option to purchase the asset at a price lower than the FV.
Problem 3: Lessee Ltd. acquired a machinery from Lessor Ltd. The following information is available to you:
Lease term 5 years
Fair value at the inception of lease `45,00,000
Fixed Lease Rent paid at the end of every year : Year 1 ` 11,00,000
Year 2 ` 13,00,000
Year 3 ` 16,00,000
Year 4 ` 13,00,000
Year 5 ` 11,00,000
182 Accounting Standards

The above lease payments includes ` 1,00,000 for maintenance and services to be reimbursed by lessee.
GRV (by lessee) ` 1,75,000
Estimated residual value ` 2,75,000
Lessor IRR @11.16% p.a.
Depreciation charged on similar assets (SLM) 19% p.a.

At the end of year 5 Lessee Ltd. purchased machinery @Market Value of ` 2,60,000 from the Lessor. Lessee
Ltd. immediately sold the asset at the same value.
Show the journal in the books of Lessee Ltd. and Lessor Ltd. for all the years.
Solution:
Books of Lessee (Lessee Ltd.)
(` In lakhs)
STEP I) Calculation of PV of MLP: STEP II) Calculation of Finance Charges:
Year MLP PVF @ PV(MLP) Installment = (Principal + interest) O/S.
11.16% Principal
0 — — — — — — 44.41
1 10 0.900 9.00 10 5.04 4.96 39.37
2 12 0.809 9.71 12 7.61 4.39 31.76
3 15 0.728 10.92 15 11.46 3.54 20.30
4 12 0.655 7.86 12 9.73 2.27 10.57
5 11.75 0.589 6.92 11.75 10.57 1.18 nil
(10 + 1.75)
Total 60.75 44.41 60.75 44.41 16.34

◘◘ Last year’s MLP includes GRV also.


◘◘ Yearly interest is calculated as a % on O/S. Principal.
◘◘ Principal means Cash Price excluding finance charges.
Accounting treatment in the books of Lessee Ltd. (lessee):
Cost of the asset = Lesser of (FMV or PV of MLP) = ` 45 or 44.41=` 44.41 lacs
Even Principal O/S. opening balance is to be taken as 44.41
Depreciation = 44.41 × 19% = ` 8.44.
Journal Entries for the first year: (` In lakhs)
YEAR 1:
Initial recognition:
Machinery Dr. 44.41 (Full Cash Price without finance costs)
 To Lessor A/c 44.41
At the end of the year:
Maintenance Charges Dr. 1 (maitn. Exp due)
Finance Expense Dr. 4.96 (fin. Cost due)
  To Lessor A/c 5.96
Lessor A/c Dr. 11 (installment paid) = Maintenance + Finance chs + Principal
  To Bank 11
Depreciation Dr. 8.44 (Depn. for the yr.)
  To Machinery A/c 8.44
*(Finance expense and Depreciation will be then transferred to Profit and Loss).
AS-19: Leases 183

LEDGERS:
Lessor A/c
Year Particulars ` (lakh) Year Particulars ` (lakh)
1 To Bank 11.00 1 By Machine 44.41
To Balance 39.37 By Maintenance Chs 1.00
By Finance Chs 4.96
2 To Bank 13.00 2 By balance b/d 39.37
To Balance 31.76 By Maintenance Chs 1.00
By Finance Chs 4.39
3 To Bank 16.00 3 By balance b/d 31.76
To Balance 20.30 By Maintenance Chs 1.00
By Finance Chs 3.54
4 To Bank 13.00 4 By balance b/d 20.30
To Balance 10.57 By Maintenance Chs 1.00
By Finance Chs 2.27
5 To Bank 13.60 5 By balance b/d 10.57
By Maintenance Chs 1.00
By Finance Chs 1.18
By Machine 0.85

Note:
Lease payments 11,00,000
towards GRV 1,75,000
towards UGRV (2,60,000 – 1,75,000) 85,000
Payment in 5th year 13,60,000

Machinery A/c
Year Particulars ` (lakh) Year Particulars ` (lakh)
1 To Lessor 44.41 1 By Depreciation 8.44
By balance c/d 35.97
2 To Balance 35.97 2 By Depreciation 8.44
By balance c/d 27.53
3 To Balance 27.53 3 By Depreciation 8.44
By balance c/d 19.09
4 To Balance 19.09 4 By Depreciation 8.44
By balance c/d 10.65
5 To Balance 10.65 5 By Depreciation 8.44
To Lessor 0.85 By Bank 2.60
By Profit and Loss A/c 0.46
184 Accounting Standards

Books of Lessor (Lessor Ltd.)


(` in lakhs)
STEP I) Calculation of PV of MLP: STEP II) Calculation of Finance Income:
Year MLP PVF PV(MLP) Installment = (Principal + interest) O/S.
@11.16% Principal
0 — — — — — — 45.00
1 10 0.900 9.00 10 4.98 5.02 40.02
2 12 0.809 9.71 12 7.53 4.47 32.49
3 15 0.728 10.92 15 11.38 3.62 21.11
4 12 0.655 7.86 12 9.64 2.36 11.47
5 11.75 6.92 12.75 11.47 1.28 Nil
(10 + 1.75) 0.589
Total 60.75 44.41 61.75 45.00 16.75
ugrv 1.00 0.589 0.59
45.00

◘◘ MLP = ` 60.75 lakh ◘◘ PV of UGRV = ` 0.59 lakh


◘◘ UGRV = ` 1 lakh ◘◘ Unearned Finance income = ` 61.75 lakh –
◘◘ Gross Investments = ` 61.75 lakh (44.41 + 0.59) = ` 16.75 lakh
◘◘ PV of MLP = ` 44.41 lakh ◘◘ Net investment in lease = ` 45 lakh
Accounting treatment in the books of Lessor:
Journal Entries for the first year:
YEAR 1:
Initial recognition:
Lessee Ltd Dr. 45.00
  To Machine 45.00 (Net Investments)
At the end of the year:
Lessee Ltd Dr. 6.02 (Income due)
  To Finance income 5.02
  To Maintenance Chs 1.00
Bank Dr. 11.00 (installment received)
  To Lessee Ltd 11.00
(Finance income and maintenance collected will be transferred to Profit and Loss)

Lessee A/c
Year Particulars ` (lakh) Year Particulars ` (lakh)
1 To Machine 45.00 1 By Bank 11.00
To Maintenance Fees 1.00 By Balance c/d 40.02
To Finance income 5.02
2 To balance c/d 40.02 2 By Bank 13.00
To Maintenance Fees 1.00 By Balance c/d 32.49
To Finance income 4.47
3 To balance c/d 32.49 3 By Bank 16.00
To Maintenance Fees 1.00 By Balance c/d 21.11
AS-19: Leases 185

To Finance income 3.62


4 To balance c/d 21.11 4 By Bank 13.00
To Maintenance Fees 1.00 By Balance c/d 11.47
To Finance income 2.36
5 To balance c/d 11.47 5 By Bank 13.60
To Maintenance Fees 1.00 By Profit and Loss a/c (Bal. Fig.) 0.15
To Finance income 1.28

Problem 5: A. Ltd. leased a machinery to B Ltd. The following information is available to you:
Lease term 5 years
Fair value at the inception of lease (01.01.2005) ` 20,00,000
Fixed Lease Rent paid at the end of every year ` 5,00,000
GRV (by lessee) ` 1,00,000
Estimated residual value ` 2,00,000
Lessors IRR 15%
Depreciation 10% p.a.

You are required to work out the following:


Unearned Finance Income. Entries in the books of Lessee for the first two years.
(C.A. Final Nov. 2004 – Accounts) (8 Marks)
Similar Qt asked in Nov 2010 NS as well as CA Final Nov 2012.
Solution: Note: In the given problem Depn is 10% and lease period is 5 years. Depn. should have been 20%.
Computation of Unearned Finance Income
As per AS 19 on Leases, unearned finance income is the difference between (a) the gross investment in the
lease and (b) the present value of minimum lease payments under a finance lease from the standpoint of the
lessor; and any unguaranteed residual value accruing to the lessor, at the interest rate implicit in the lease.
where:
(a) Gross investment in the lease is the aggregate of (i) minimum lease payments from the stand point
of the lessor and (ii) any unguaranteed residual value accruing to the lessor.
Gross investment = Minimum lease payments + Unguaranteed residual value
= (Total lease rent + Guaranteed residual value) + Unguaranteed residual value
= [(` 5,00,000 × 5 years) + ` 1,00,000] + ` 1,00,000
= ` 27,00,000
(b) Table showing present value of (i) Minimum lease payments (MLP) and (ii) Unguaranteed residual
value (UGRV).

Year MLP inclusive of UGRV Internal rate of return Present Value


(`) (Discount factor 15%) (`)
1 5,00,000 .8696 4,34,800
2 5,00,000 .7561 3,78,050
3 5,00,000 .6575 3,28,750
4 5,00,000 .5718 2,85,900
5 5,00,000 .4972 2,48,600
1,00,000 .4972 49,720
186 Accounting Standards

(guaranteed residual value) 17,25,820 (i)


1,00,000 .4972 49,720 (ii)
(unguaranteed residual value)
(i) + (ii) 17,75,540 (b)

Unearned Finance Income = (a) – (b) = ` 27,00,000 – ` 17,75,540 = ` 9,24,460


Journal Entries in the books of B Ltd. `
At the inception of lease
1) Machinery account Dr. 17,25,820
 To A Ltd.’s account 17,25,820
(Being lease of machinery recorded at present value of MLP)
At the end of the first year of lease
2) Finance charges account Dr. 2,58,873
 To A Ltd.’s account (1725820 × 15%) 2,58,873
(Being the finance charges for first year due)
3) Depreciation Dr. 1,72,582
 To Machinery account (10% × 17,25,820) 1,72,582
(Being the finance charges for first year due)
*As per para 11 of AS 19, the lessee should recognise the lease as an asset and a liability at an amount equal
to the fair value of the leased asset at the inception of lease.
Problem 6: Global Ltd. leased an equipment costing ` 1,50,000. The asset would revert back to Global Ltd.
The other information is as follows:
Lease term 3 years
Useful life 4 years
Unguaranteed Residual Value ` 20,000
IRR @ 10%

The annual payments are determined in such a way that the present value of lease payments plus the
residual value is equal to the Cost of Asset.
You are required to work out the following for Global Limited:
i) Unearned Finance Income.
ii) Annual Lease payments.
iii) Yearly Finance income
iv) Show how the items will appear in the Profit and Loss Account and Balance Sheet for the various
years.
(Similar Qt. asked in Nov. 2013 (4 marks) + (Final C.A. May 2006 – Accounts) (8 Marks)
Solution:
Calculation of PV of MLP: (From Global Limited i.e. Lessors point of view)
Let MLP every year be ` X.

Year MLP PVF @ 10% PV(MLP)


0 — — —
1 X 0.9090 0.9090X
2 X 0.8264 0.8264X
3 X + 20000 (Rent + UGRV) 0.7513 0.7513X + 15,026
Total 3X + 20000 2.4867X + 15026
AS-19: Leases 187

As per the data given in the problem PV of MLP + RV = Cost of Asset


Therefore, 150000 = 2.4867X + 15026
X = ` 54,300.
(i) Annual Lease payment = ` 54,300 (rounded off)
(ii) UFI = Gross Investments – Net Investments = (3X + 20000) – 150000 = ` 32,900
Here Net Investments = Lesser of PV of MLP or FMV & both are same i.e. `150000.
(iii) Segregation of Finance Income

Year Lease Rentals Finance Charges @ 10% on Repayment Outstanding


outstanding amount of the year Amount
0 — — — 1,50,000
I 54,300 15,000 39,300 1,10,700
II 54,300 11,070 43,230 67,470
III 74,300** 6,805 67,470 —
1,82,900 32,875 1,50,000
* Annual lease payments are considered to be made at the end of each accounting year.
** ` 74,300 includes unguaranteed residual value of equipment amounting ` 20,000.
(v) Profit and Loss Account ( Relevant Extracts)
Credit side `
I Year By Finance Income 15,000
II year By Finance Income 11,073
III year By Finance Income 6,752
Balance Sheet (Relevant Extracts)
Assets side ` `
I year Lease Receivable 1,50,000
Less: Amount Received (39,275) 1,10,725
II year Lease Receivable 1,10,725
Less: Received (43,202) 67,523
III year: Lease Amount Receivable 67,523
Less: Amount received (47,523)
Residual value (20,000) NIL
Notes to Balance Sheet
Year I: `
Minimum Lease Payments (54,275 + 54,275) 1,08,550
Residual Value 20,000
1,28,550
Less: Unearned Finance Income(11,073+ 6,752) (17,825)
Lease Receivables 1,10,725
Classification:
Not later than 1 year 43,202
Later than 1 year but not more than 5 years 67,523
Total 1,10,725
Year II:
188 Accounting Standards

Minimum Lease Payments 54,275


Residual Value (Estimated) 20,000
74,275
Less: Unearned Finance Income (6,752)
Lease Receivables (not later than 1year) 67,523
Year III:
Lease Receivables (including residual value) 67,523
Less: Amount Received (67,523)
NIL
Problem 7: Madhav Transport Ltd. has given 4 newly purchased fleets of trucks to Anna Services Ltd. on
lease. The lease is for 3 years only. Rentals agreed in the contract are as follows:
Year 1 = ` 2,70,000, Year 2 = ` 2,40,000, Year 3 = ` 2,10,000. The Cost of the total fleet of vehicles =
` 9,00,000. The life of the trucks may be taken as 9 years. Costs of yearly insurance borne by the lessee is
` 500 per truck for first year & thereafter 600 per truck.
How will you treat the above in the books of both lessor and lessee.
Solution: As no information regarding apportionment of Lease Rent is given, hence SLM method is to be
adopted.
Total Rent = 270000 + 240000 + 210000 = 720000
Rent p.a. = 720000/3 = 240000.

Journal Entries in the books of Madhav Ltd. ( Lessor)


Year 1 Year 2 Year 3
Bank Dr. 270000 Dr. 240000 Dr. 210000
Lease Suspense Cr. 30000 — Dr. 30000
Lease Rent Cr. 240000 Cr. 240000 Cr. 240000
Cash Dr. Dr. 2000 Dr. 2400 Dr. 2400
Insurance Cr. 2000 Cr. 2400 Cr. 2400
Insurance Dr. 2000 Dr. 2400 Dr. 2400
Lease Rent Dr. 240000 Dr. 240000 Dr. 240000
Profit/Loss Cr. 242000 Cr. 242400 Cr. 242400

In the books of Lessee the above entries will be reversed.


Problem 8: Hamu Texturising Limited has manufactured a texturising machine at a cost of ` 16,00,000.
Its major revenue is from sale of the machines. Now it has also decided to lease out the machine.
A prospective lessee M/S. Dijirix Ltd. comes forward and following terms are agreed upon:
Fair value (direct sale value if purchased now from the market) = ` 20,00,000, Lease term = 5 years, Lease
Rent = ` 6,00,000 p.a. , Guaranteed RV = Nil, IRR = 15.24%.
You are required to calculate the following:
i) Selling Profit, ii) Finance income.
Solution: This is the case of Manufacturer dealer (Manufacturer Cum Lessor). Thus, Manufacturer dealer
earns selling profit + finance income.
◘◘ Normal SP is to be considered for profit calculation.
◘◘ Normal discounts and rebate is allowed from the SP.
i) Selling Profit = FMV – Cost = 20,00,000 – 16,00,000 = ` 4,00,000
Profit of ` 4,00,000 is to be recognised immediately.
AS-19: Leases 189

ii) Finance Income:


Lease Rent = ` 6,00,000
IRR = 15.24%
GRV = `Nil.
Lease term = 5 years.
Gross Investments = 6,00,000 × 5 = ` 30,00,000
Net Investments = Fair Value = PV (MLP’s+UGRV) = 6,00,000 x Cum.PVF@15.24% =
= 6,00,000 × 3.33333 = 20,00,000
As PV (MLP’s) = Fair Value = ` 20,00,000, it is a Finance Lease.
Calculation of Finance income:

Installment (Principal + Interest) O/S. Principal


— — — 20,00,000
6,00,000 2,95,200 3,04,800 17,04,800
6,00,000 3,40,189 2,59,811 13,64,611
6,00,000 3,92,033 2,07,967 9,72,578
6,00,000 4,51,780 1,48,220 5,20,798
6,00,000 5,20,798 79,202 Nil

Initial Recognition:
Dr.
Lease Rent receivable 20,00,000 (Cash Price)
 To Revenue A/c 20,00,000
Problem 9: Factory Works Limited is a manufacturing – cum – leasing company presents the following
information regarding one of the asset to be leased for the financial year 2005 – 2006.
Cost of the asset = ` 5,00,000, Total MLP = ` 7,00,000, UGRV = ` 50,000, Unearned Finance Income =
` 1,50,000. Compute profit to be recognised by the dealer as per AS-19.
Solution:
Lets calculate the Fair Value (FV) of the asset:
FV of the asset at the inception of the lease = Net Investments
But Net Investments = Gross Investments – UFI.
But GI = Total MLP’s + UGRV = 7,00,000 + 50,000 = ` 7,50,000
Net Investments = Gross Investments – UFI. = 7,50,000 – 1,50,000 = ` 6,00,000
Selling Profit = FV – Cost = 6,00,000 – 5,00,000 = ` 1,00,000
Problem 10: Bandivli Works Ltd. own an asset of which is depreciated @10% on SLM basis. The asset was
purchased 5 years back for ` 1,00,000. The company has decided to sell the machine by the end of 5th year at
` 60,000 (being Fair Value) to Mayur Enterprises and leases it back for the remaining 5 years. Bandivli Works
Ltd. agrees to pay a lease rent of ` 15,000 p.a. for the remaining 5 years. Lessee’s incremental borrowing cost
7.93% p.a. (CA Final May 2013)
Advice on accounting in the books of Lessor and Lessee.
Solution: If Sale and lease back is a Finance lease: The difference between (Sale proceeds - Carrying amount
of asset) will be either profit or loss. Such profit or loss is not recognized immediately bu is spread over the
lease term in proportion to the depreciation of the asset. As the asset is sold at the end of 5th year, carrying
amount has to be calculated:
190 Accounting Standards

Step 1 → Carrying Amount = 100000 – 5 × 10000 = 50,000


Selling Price = ` 60,000 (assume to be fair value)
Step 2 → Determination of Finance Lease or Operating Lease:
Standing as on the sixth year opening day on which asset is sold out on Sale – lease back basis lets
determine whether the lease is FL or OL.
PV (MLP) @7.93% = 15,000 (Rent) × Cum. PVF@7.93% = 15,000 × 4 = 60,000
PV of Total MLP’s = 60,000
PV of Total MLP’s is equal to the selling price and hence it is a Finance Lease
Lessee will record the asset at ` 60,000.
Step 3 → Calculation of profit and treatment:
Profit on sale = Sale proceeds – CA = 60,000 - 50,000 = 10,000 (deferred)
Journal Entry:
Cash/Bank A/C. Dr. 60,000
 To Asset A/C. 50,000
 To Deferred Income A/C. 10,000
But as per the standard such profit should not be immediately recorded but should be deferred in the ratio
of future depreciation.
Therefore Deferment = ` 2,000 (10,000/5)
Calculation of Finance Charges:
Installment = (Principal + interest) O/S. Principal
— — — 60,000
15,000 10242 4758 49758
15,000 11054 3946 38703
15,000 11931 3069 26772
15,000 12877 2123 13895
15,000 13895 1105 nil
75,000 60,000 15,000
Further entries in the books of Lessee (Bandivli Ltd) can be passed easily, just by referring question no:
3,5……
As there is no Unguaranteed RV the finance income of Lessor will be same as Finance charges for Lessee
(refer table above).
Problem 11: If Mamta Ltd. sold an asset to Samta and lease back the same asset from Samta Ltd. Assuming
it is an Operating Lease give the treatment in the following cases: (All figures are in `)
(I) Problem on Case I type i.e if SP = FV:
1) SP = 15,000, CA = 12,000, FV = 15,000
(II) Problem on Case II type i.e if SP < FV:
1) SP = 15,000, CA = 13,000, FV = 19,000. 2) SP = 15,000, CA = 17,000, FV = 16,000 assuming loss is
compensated by future lease rental equally over 2 years
(III) Problems on Case III type i.e if SP > FV:
1) SP = 15,000, CA = 12,000, FV = 13,000.
Solution:
If Sale and lease back is a Operating lease:
AS-19: Leases 191

Case I
If Sale price of asset = Fair Value
Profit or loss on sale of asset (SP – CA) is to be recognized immediately.
(1) Cash/Bank Dr. 15,000
To Asset 12,000
To Profit and Loss 3,000
Case II
If Sale price of asset < Fair Value
Profit or loss on sale of asset (SP – CA) is to be recognized immediately.
Exception: If loss is compensated by charging future lease rentals lower than the market price, then defer and
amortize such loss in proportion to lease payments over the lease term.
(1) Cash/Bank Dr. 15,000
To Asset 13,000
To Profit and Loss 2,000
(2) First of all write off part of asset (because CA > FV)
P/L a/c Dr. 1,000
To Asset a/c 1,000
Then,
Cash/Bank Dr. 15,000
Def. Loss Dr. 1,000 (deferred over 2 years)
To Asset 16,000
Case III
If Sale price of asset > Fair Value then excess of sp – fv is to be deferred
(1) Cash/Bank Dr. 15,000
To Asset 12,000
To Deferred Income (deferred over lease paid) 2,000 (15-13)
To Profit and Loss A/c 1,000
Problem 12: Core Ltd. transfers both Land and Building on lease basis for the major useful life of the
building. Advise on the accounting of lease rentals paid by the lessee to Core Ltd.?
Solution: The total lease rentals should be bifurcated into Land and Building on fair value basis. On Land
operating lease will be applicable, unless ownership transfers to the lessee after the lease term. AS-19 is not
applicable on the use of Land (having unlimited life). AS- 19 is applicable to Building component. Under IAS
-17 on Leasing, even IASC considers Land and Buildings as two separate elements.
Problem 13: Magnum Division Ltd. availed a finance lease from Sterling Ltd. on the following terms:
Lease term 3 years
Useful life 5 years
Unguaranteed Residual Value (estimated at the end of third year) `75,000
IRR @ 10%
Fair Value of the asset as on today ` 7,00,000
PV of ` 1 at the end of third year 0.7513
PV of annuity of ` 1 for three years = 2.4868

Prove that the lease is a Finance Lease.


192 Accounting Standards

Solution:
Computation of annual rent =
PV of Residual value = 75,000 × 0.7513 = 56,348
PV of lease payments = 7,00,000 – 56,348 = 6,43,652
Annual payments = 6,43,652/2.4869 = 2,58,817
PV of lease payments constitutes 92% of the FMV hence it is a finance lease.

REMEMBER:
If in the exam, ICAI asks you problems in lease on:
(i) Immediate lease payments
(i) Semi - annual lease
You can refer chapter on Leasing in SFM module.
CHAPTER 20
AS-20: Earning per Share

Chapter Outline
vv Scope and Objective vv Potential Equity Shares
vv Basic EPS vv Dilutive and Non – Dilutive EPS
vv Restatement vv Disclosures
vv Timing for Inclusion in WANES

Scope and Objective


For the purpose of performance evaluation of two companies and same company for two different years EPS
is one of the very important indicators. So, for correct derivation of the EPS AS-20 has been introduced. The
main focus is on the denominator. ICAI has clarified in a circular that every company which is required to
give information under Schedule VI to the Companies Act, should compute and disclose EPS in accordance
with AS-20, whether or not the shares of that company is listed in a stock exchange. In consolidated financial
statement the holding company is required to present EPS on the basis of consolidated information.

Basic EPS
The standard requires an enterprise to present the Basic EPS on the face of financial statements. Basic EPS
is simply the Net Profit available to Equity Shareholders divided by the Weighted Average Number of Equity
Shares outstanding during the year (WANES).

Basic EPS =

(i) Net Profit available to Equity Shareholders:


ww Concept of Net Profit available to Equity Shareholders:
Net Profit available to equity shareholders (Numerator) is the profits after all expenses and
payments to preference shareholders. It is the profits attributable to the ESH who has provided
funds for earning that profits.
ww Calculation of Net Profit available to Equity Shareholders:
Net profit/loss during the year XXX
  – Tax expenses XXX
  – Preference Dividend XXX
  – Preference Dividend Tax XXX
Net Profit available to Equity Shareholders(Numerator) XXX
194 Accounting Standards

ww Other points on Net Profit available to Equity Shareholders:


yy Dividend on non-cumulative preference shares is to be deducted only if provided in the
accounts. But Dividend on cumulative preference shares for current year is to be deducted
even if not provided in the accounts because dividend on cumulative preference shares
accrues every year just like interest expense.
yy Even dividends to participating preference shares have to be deducted from NP.
yy Statutory Reserves is not required to be deducted from NP like transfer of Reserves to
Sinking Fund.
(ii) Weighted average number of equity shares outstanding during the year (Denominator):
ww Concept of WANES:
Traditionally for calculating EPS denominator (No. of Shares) closing shares as on the last day
of the year was taken. This means that earnings per share are due to the shares outstanding at
the end of the year. Is it justified? No. During the year there may be various issues and buybacks
who proportionately have contributed towards the EPS. It is not necessary that Opening number
of shares = Closing number of shares. There might be several issues (additions) or buy-backs. So
such adjustments of proportionate issues and proportionately buy-backs are very much neces-
sary to the Opening number of shares. These issues and buybacks contribute proportionately
towards the net profits. For this proportionate time - weights has to be assigned to the fresh
capital and buy-backs.
ww Calculation of WANES
WANES outstanding during the year
Opening Shares b/d xxx
(+) Fresh Capital (Number of Shares × Time weight) xxx
(–) Capital Buy-backed (Number of Shares x Time weight) xxx
WANES xxx

• Time weight for Inflow of Capital =

• Time weight for Outflow of Capital =

ww Calculation of WANES in different cases:


Case 1 → Bonus Shares:
In case of Bonus issue equity holders are issued shares for “no additional consideration”. So
the number of shares is increased without any increase in the inflow of capital. As per AS-20
whenever Bonus, Share are issued, they are included in the number of shares as from the begin-
ning. The concept which works behind this logic is that the bonus shares are nothing but capi-
talisation of reserves. And capitalising reserves does not change the equity of a company. The
reserves from which the bonus shares are issued are very well available at the beginning of the
year hence number of shares for bonus issue is taken from the beginning of the year irrespective
of the issue of the bonus.
Case 2 → Share split (Subdivision) or Reverse split (Consolidation) :
Again in case of Share split or Consolidation, equity holders are issued shares for “no additional
consideration”. So the change in number of shares is not compensated by any increase in inflow
of capital resources. Thus even in case of Share split or Consolidation number of shares is to be
considered as from the beginning. Date of Share split or Consolidation is immaterial.
AS-20: Earning per Share 195

DO YOU KNOW
In case of Bonus/Share split/Consolidation the ownership percentage is not affected.

Case 3 → Partly paid- up Shares:


In case of partly paid up shares the concept of equivalent units as seen in costing is applied. The
partly paid up shares will be treated as an equivalent unit as a fraction of fully paid up shares.
It is assumed that the partly paid up shares are entitled to proportionate dividends. If cash or
any consideration is received towards the partly paid up shares then such inflow of resources
is to be treated as a fresh issue of shares. But the fresh issue is to be converted into equivalent
shares. Further the equivalent shares are to be given time weights. If partly paid-up shares are
not entitled to dividends then they are treated same as options.
Case 4 → Different Nominal values of shares:
If a share is having different nominal values then the same treatment as given in case of partly
paid up shares viz., concept of equivalent units is to be applied here also.
If different dividend rights are attached to different nominal shares then equivalent number of
shares is to be further adjusted by the proportion of dividend.
Case 5 → Right Shares:
Right shares are those shares, which are issued to the existing shareholders at a price which is
less than the market price. If it is issued at market price then it is a normal issue of shares. If it is
issued at less than the fair value then there is an inclusion of a disguised bonus element. (refer
special problem number 6)

DO YOU KNOW

Shares Normal Shares Right Shares Bonus Shares


Exercise Price (EP) EP = Current MP EP < Current MP EP = 0

Case 6 → Amalgamation effect on EPS:


If it is Amalgamation in the nature of purchase From the date of purchase
If it is Amalgamation in the nature of merger From the beginning of the year

In case of amalgamation in the nature of purchase the amalgamated company records only the
assets and liabilities therefore the date of acquisition is the date relevant for WANES. But in
case of amalgamation in the nature of merger the amalgamated company even incorporates the
Reserves and Profit & Loss of the amalgamating company as if the amalgamated company exists
from the beginning.
• If it is Amalgamation in the nature of purchase WANES = WANES of New Co. + Shares issued to Old Co. × Time weight
• If it is Amalgamation in the nature of merger WANES = WANES of New Co. + Shares issued to Old Co. × 12/12.....para 18

Case 7 → Buy-Back:
If Buy - back takes place at the Fair Value then we have to just simply deduct the Shares buy-
backed (after time-factor adjustment) from Opening number of Shares for WANES. But Buy –
back is effected at a premium which happen in most of the cases then this will be a reverse case
to the treatment of right issue of shares i.e. inclusion of bonus element, but in a reverse direc-
tion. (refer special problem no. 13)
196 Accounting Standards

Restatement
From our above discussions it is clear that Bonus, Share – split, Consolidation are issued free of cost to
shareholders i.e. there is a change in the denominator, numerator remaining same. Every company is required
to report current years EPS with the last years report. Thus, last year’s accounts are also to be given effect,
otherwise the financial statements will disclose misleading information. So adjusting previous years EPS is
known as Restatement. Hence to harmonise the EPS, restatement of EPS is required by the Standard. AS-20
further carries the logic when bonus/share split etc., occurs after the year closes but before signing of the
accounts the EPS is to shown only after adjusting for the new number of shares.

Timing for Inclusion in WANES


Shares issued Weight to be considered from
• For cash Cash is receivable
• Conversion of debt Date of Conversion
• In lieu of interest or principal Date on which interest ceases to accrue
• Settlement of liability Date on which settlement becomes effective
• Acquisition other than cash Acquisition is recognized
• Services rendered When the service is rendered
• Contingently issuable shares Date on which conditions are met
• Amalgamation in the nature of purchase On the date of purchase
• Amalgamation in the nature of merger From the beginning of the year

Potential Equity Shares


Potential Equity Shares (PES) are those instruments or contracts that entitles the holder to receive equity
shares in future. They have been so named because they have got the potential to claim EPS in future. Potential
Equity Shares have a tendency to increase future EPS or dilute future EPS. Potential Equity Shares play a very
important role in calculating Diluted EPS. PES not only affects the numerator (NP) of the Basic EPS but also
the denominator (WANES). If PES is converted into actual equity shares during a year, then such PES are no
more PES. Such PES then enters into calculation of Basic EPS.
Examples of Potential Equity Shares are: Convertible Debentures, Convertible Preference shares, Share
warrants, Employee Stock option, Contingently issuable shares, Advance money received as share application,
pending allotment which is used in the business.

Dilutive and Non – Dilutive EPS


The main objective of AS – 20 is to inform the prospective shareholders regarding the dilution in EPS. If
inclusion of Potential Equity Shares dilutes (reduces) the Basic EPS then such Potential Equity Shares are
called Dilutive securities and such reduced EPS is called as Dilutive EPS. Dilutive EPS is a predictor which
signals dilution in future EPS, which is to be disclosed for transparency of financial accounts, presented to
the shareholders. Dilutive EPS indicates the risk attached to the basic EPS. Dilutive EPS definitely affects the
portfolios of shareholders hence reporting requirements are mandatory. Potential Dilutive securities have a
tendency of lowering EPS. Potential Equity Shares are considered dilutive only when their conversion would
lead to decrease in net profit per share from continuing operations. Potential Equity Shares are considered to
be anti - dilutive only when their conversion would lead to increase in net profit per share or decrease in loss
per share from continuing operations.
AS-20: Earning per Share 197

Reporting requirements: Dilutive EPS is to be reported (mandatory as per AS-20 as well as New Schedule
VI). Anti – dilutive is never to be reported in the financial statements.
PES effect on Basic EPS at a glance:

Basic EPS = Effect on Basic EPS Future EPS =

Computation of Diluted EPS


1) Computation of EPS denominator for Diluted EPS:
WANES outstanding as on opening date XXX
+ Normal issue if any XXX
+ Potential Equity Shares which are not converted during the reporting period and still outstanding XXX
Total : XXX

2) Computation of EPS numerator for Diluted EPS:


PES not only affects the denominator but it also affects the numerator as well. In case of Dilutive Potential
Equity Shares earnings available to equity share holders will be affected in future for ex.: In case of Convertible
Debentures in future earnings will be increased by way of savings in Interest cost, of course net – off tax.
Net profit available for equity shares in case of Diluted equity shares is computed as follows:
Net profit for the year including prior period adjustment etc. XXX
+ Preference Dividend (savings) XXX
+ Preference Dividend Tax (savings) XXX
+ Interest on Debentures net of tax (savings) XXX
Total : XXX

3) Sequence to be considered for Potential Diluted equity shares :


As per our preceding discussion it is well clear that Dilutive EPS is to be reported and Anti- Dilutive EPS is
to be ignored. In considering whether potential equity shares are dilutive or anti-dilutive each series is to be
considered separately rather than in aggregate. But how the sequence is to be considered?
Step 1: Calculate incremental earnings for each series separately.
Step 2: Calculate incremental equity shares for each series separately.
Step 3: Divide Step (1) by Step (2) which is nothing but Incremental EPS.
Step 4: The series having least Incremental EPS as derived in Step (3) is the most dilutive.
Step 5: Consider the most dilutive first then the lesser one and so on.
Note: The above procedure is to be followed only when there are more than one PES in the series. But if
there is only a single PES then;

Dilutive EPS =

Note: Dilutive is to be calculated till the PES were outstanding. Once it is converted then from that date such
converted PES enters into Basic EPS calculations.
198 Accounting Standards

Diluted EPS and Other Related Issues


Options and Diluted EPS
1) Employees stock options are contracts by an employee with its employee for allotting shares to its
employees in future for past services rendered by its employee. Such stock options are given to some
senior most employees.
2) Options are considered for dilutive equity shares only if the fair value > exercise price of the shares.
If Options are exercised at par with the fair value then they are normal shares. Fair value means
average price of the equity shares during the year.
3) The difference between the number of shares that would have been issued at fair value and the
actual shares issuable at exercise price is equity shares for no consideration.
4) Options are considered for Basic EPS only after the exercise of the options.
5) Options are most dilutive among all PES.
6) The computation assumes the most advantageous conversion rate or exercise price from the point of
view of the holder of the Potential Equity Shares. For example : In a situation where an exercise price
is between a range of ` 60 to ` 90, for computing Dilutive EPS the least price i.e. ` 60 is to be taken.
The reason being computation assumes the most advantageous conversion rate or exercise price
from the point of view of the holder of the Potential Equity Shares Conservatism!
7) Costs of Unamortised ESOP is to be written – off throughout the life of options on SLM basis
How to Compute Equity Shares for no Consideration in case of Options:
Shares under Option ……….(i) → Will be given in the problem XXX
Less: Equity Shares for which proceeds receivable = ( i × EP ÷ MP) (XXX)
Less: Unamortised employee costs = ( Unamortised cost ÷ MP) (XXX)
Equity shares for no consideration. → XXX

Note: (a) EP: Exercise price, (b) MP: Fair Value or Market Value per share during that year.

DO YOU KNOW
Options are always dilutive: The above method of calculating Equity shares for no consideration for options
is given by IAS 33 also known as Treasury Stock method.

Loss Making Companies and Diluted EPS


An enterprise which is showing loss per share is also required to disclose Basic as well as Dilutive EPS.
Potential Equity Shares are considered dilutive only when their conversion would lead to increase in net loss
per share from continuing operations. Potential Equity Shares are considered to be anti - dilutive only when
their conversion would lead to decrease in net loss per share from continuing operations.

Continuing Vs Discontinuing Operations and Diluted EPS


1) For checking purpose whether the PES are dilutive/anti-dilutive consider results of continuing
operations only. In other words results from discontinuing operations are to be ignored.
2) But for the final reporting purpose consider Continuing as well as Discontinuing operations for
computing Basic as well as Dilutive EPS.

Contingently Equity Shares


1) Contingently equity shares are equity shares isssuable upon satisfaction of certain conditions. When
the conditions are met immediately Equity shares are issued.
AS-20: Earning per Share 199

2) If conditions are met then equity shares will be issued and for computing Basic EPS the date will be
reckoned from the date when conditions are met upto the reporting date.
3) If conditions for Contingently equity shares are not met throughout the complete financial year then
it remains potential equity shares at the end of reporting date.

Buy Back of Shares & Diluted EPS


1) It may be possible that at the year end Buyback shares are still outstanding. In such a case Buy back
is considered for dilutive equity shares only if the fair value < exercise price of the shares. Fair value
means average price of the equity shares during the year.
2) The difference between the number of shares that would have been bought back if issued at fair
value and the actual shares bought back at exercise price is equity shares for no consideration.

Disclosures
(a) Mandatory Disclosures:
ww Positive as well as Negative EPS.
ww Basic & Diluted EPS on the face of Profit & Loss Accounts.
ww Restatement of EPS – for post balance sheet bonus issue.
ww Numerator used for Basic & Diluted EPS – with reconciliation.
ww Denominator used for Basic & Diluted EPS – with reconciliation.
ww Nominal value of shares along with EPS figure.
(b) Additional Disclosures:
ww Terms and conditions of contracts eligible for Potential Equity Shares.
ww Composition of Net profit viz., Net Profit from ordinary activities.

List of Restatement of EPS in which circumstance it is applicable:


Situation Why
Bonus No Consideration received
Right Shares Less Consideration received
Buy Back More Consideration is paid
Shares Consolidation No change in proceeds but number of shares changes
Shares Subdivision --- Same as above ---

PROBLEMS AND SOLUTIONS

Problem 1: For the year 2013-2014 Tuts Limited earns ` 3 lakhs before tax. Other details are as follows:
Opening Equity Share Capital ` 500,000
5,000 Shares issued on 01.11.2013 (`10/-).
On 1.3.2014 1,000 Shares were buy backed.
Corporate Tax 33.33%. Compute Basic EPS.
Solution:
Basic EPS = Net Profit/WANES = 2,00,000/52,000 = ` 3.85
Net Profit = 3,00,000 – 1,00,000 = ` 2,00,000
200 Accounting Standards

WANES = 50000+5000x5/12-1000x1/12 = 50000+2083.33-83.33 = 52,000.


Problem 2: Financial information of X Limited is extracted as follows:
Opening Equity Shares on 01.01.2006 1,00,000
Shares issued as Bonus as on 01.05.2006 20,000
Profits for the year ended 31.12.2006 (`) 40,00,000
Profits for the year ended 31.12.2005 (`) 55,00,000
Calculate Basic EPS for 2006 and adjusted EPS for 2005.
Solution:
Basic EPS 2006 (Reported) = Net Profit/WANES = 40,00,000/1,20,000 = `33.33
Basic EPS 2005 (Reported) = 55,00,000/1,00,000 = ` 55
Basic EPS 2005 (Restated) = 55,00,000/1,20,000 = ` 45.83
Assume Opening shares are numbers.

Note: One should compare Restated EPs with Reported EPS i.e. ` 44.83 with ` 33.33.

Problem 3: Tedy Limited earns a Net Profit for the year 2005 – 2006 ` 5,00,000 before Tax. Tax @ 40%.
Transfer to Sinking Fund is ` 10,000. Transfer to General Reserves ` 2,000. Preference Dividend ` 10,000.
Equity Dividend = ` 16,000. Dividend Tax 10%. Compute Net Profit for Basic EPS.
Solution:
Net Profit for Basic EPS = 5,00,000 – 40% tax – 10,000 Pref. Div. – 1,000 CDT on Pref. Dividend = ` 2,89,000.
Trf to general reserves, sinking fund, equity dividend should be ignored.
Problem 4: COSY COMFORTS Limited earns a Net Profit for the year 2006 – 2007 ` 20,00,000 after tax. The
corresponding figure for the last year was ` 16,00,000. Its Capital Structure contains 80,000 Shares of ` 10
each. On 07.11.2006 it has decided to consolidate the shares from ` 10 each to ` 100 each. Calculate Basic
EPS and Restated EPS.
Solution:
Basic EPS 2006 – 07 (Reported) = Net Profit/WANES = 20,00,000/8,000 = `250
Basic EPS 2005 – 06 (Reported) = 1600000/80000 = ` 20
Basic EPS 2005 – 06 (Restated) = 1600000/8000 = `200
Problem 5: STAR LIMITED has three classes of shareholders in its capital structure viz ; Class A : 5,000
Shares, Class B : 1,000 Shares Class C : 3,000 Shares. Dividends rights attached to this class is 90%, 110% and
100%respectively. Net Profit for the year 2006 is ` 1,80,000.
Compute EPS for each class of shareholders for the year 2006.
Solution:
EPS per full fledge share. 180000/(5000 × 0. 9 + 1000 × 1.1 + 3000 x 1) = 180000/8600 = `20.93
Proportionate Dividend for CLASS A = `18.837 (90% X 20.93)
Proportionate Dividend for CLASS B = `23.023 (1.1 X 20.93)
Proportionate Dividend for CLASS C = `20.93.
Problem 6: As on 01.01.2006 Pistons Limited had 5,00,000 shares. It issued 1 right share for every 5 shares
held at ` 15/- on 01.03.2006. Market price per share just before the right issue is ` 21. Net profit for 2005 was
`11,00,000 and for 2006 `15,00,000.
Calculate EPS for 2006 and restated EPS for 2005.
(Similar to CA Final Nov 04 8 Marks; May 2014 5 Marks)
AS-20: Earning per Share 201

Solution:
OS × MP + RI × Subscription Price 5,00,000 × 21 + 1,00,000 × 15
Theoretical Ex-right MP = = = ` 20
Total Shares 6,00,000
Irrespective of any method Theoretical Ex-right MP is required to be calculated as a first step.
Formula Method:
WANES = 5,00,000 + Right shares × 9/12 × Adjusting Factor (A.F.)
= 5,00,000 + 1,00,000 × 10/12 × 1.05 = 500000+87500
Calculation of A.F.:
Adjusting Factor = MP before rights = 21/20 = 1.05
MP ex- right
Basic EPS 2006 (Reported) = Net Profit/WANES = 15,00,000/(5,00,000+87500) = ` 2.55
Basic EPS 2005 (Reported) = 1100 000/5,00,000 = ` 2.20
Basic EPS 2005 (Restated) = 1100000/500000 × 1.05 = ` 2.10
Logical Method:
Total Right shares received = 1,00,000
Less : Shares for which proceeds are received - 75,000 (1,00,000 × 15/20)
Net RI for which proceeds not recd. OR Bonus element 25,000
Eligible RI = 75000 × 10/12 + 25000 × 1/1 = 87500
Basic EPS 2006 (Reported) = Net Profit/WANES = 15,00,000/(5,00,000+87500) = ` 2.55
Basic EPS 2005 (Reported) = 1100 000/5,00,000 = ` 2.20
Basic EPS 2005 (Restated) = 1100000/(500000 + 25000 Bonus) = ` 2.10
Problem 7: Max Ltd. takes over Zee Ltd. as on 01.04.2005.
Max Ltd. Zee Ltd.
Original Shares 200,000 1,00,000
Face Value 100 100
Share Value for P.C. 210 105
Profits (01.01.2005 –31.03.2005) 1,00,000 70,000
Profits (01.04.2005 – 31.12.2005) 4,30,000 —
Compute EPS assuming:
(a) Amalgamation in the nature of Purchase
(b) Amalgamation in the nature of Merger
Solution:
Calculation of PC = 100000 × 105 = 1,05,00,000
Number of shares issued = 10500000/210 = 50,000

Amalgamation in the nature of Purchase Amalgamation in the nature of Merger


Net Profit = 530000 Net Profit = 600000
WANES = 237500 (200000+50000X9/12) WANES = 250000 (200000+50000)
Basic EPS = ` 2.23 Basic EPS = ` 2.40
202 Accounting Standards

Problem 8: Mohur Limited has equity capital of ` 40,00,000 consisting of fully paid up shares of ` 10 each.
Net Profit after tax for 2004 – 2005 was ` 60,00,000. It has also issued 36,000, 10% Convertible Debentures
of ` 50 each. Each debenture is convertible into 5 equity shares. Tax rate 30%. Compute diluted EPS.
(C.A. Final Nov. 2006)(Similar Problem Nov. 2009)(NS)
Solution:
(1) Basic EPS = = = 15

(2) Diluted EPS = = = 10.56

Note – Adjust PAT


Net profit before conv. debenture in to share = 60,00,000
Interest saving on debenture
{36000*50*10%*70%(100-30)} = 1,26,000
Adj. PAT 61,26,000
New no share for dilution:
Old shares = 4,00,000
Convertible debenture (36000*5) = 1,80,000
5,80,000
Problem 9: Esteem Ltd. has formulated a employees stock option plan for its employees. Options were
granted on 01.04.2007 at a price of ` 30 per share. Fair Value per share on 01.04.2007 was ` 50/-. The
options would vest in 3 years. The company amortises the difference between the FV and exercise price over
3 years equally. Esteem Ltd. has 6,00,000 shares as on 31.03.2008 as well as on 31.03.2009 but on 31.03.2010
it has 6,30,000 shares. Options were of 1,50,000 shares. Fair value of equity shares on average basis as on
31.03.2008 ` 45, as on 31.03.2009 ` 47, as on 31.03.2010 ` 40. Net profits before ESOP costs is ` 55,00,000,
70,00,000 and ` 64,00,000 respectively.
Please calculate Dilutive as well as Basic EPS for all 3 years after considering the options.
Solution:
Particulars 31.03.2008 31.03.2009 31.03.2010
Profits before amortization of ESOP cost 55,00,000 70,00,000 64,00,000
Less: ESOP written off – amortised
(1/3rd of [50-30] 150000) (10,00,000) (10,00,000) (10,00,000)
Net profits available to equity share holders: 4,500,000 6,000,000 5,400,000
WANES ......... (a) 600,000 600,000 630,000
Basic EPS 7.50 10.00 8.57
Shares under options 1,50,000 1,50,000 1,50,000
Less: Equity shares for which proceeds received (100,000) (95,745) (112,500)
(1,50,000 х 30/45, 1,50,000 х 30/47, 1,50,000 х 30/40)
Less: Unamortised ESOP cost treated as proceeds received
(20.00.000/45, 10,00,000/47, 0) (44.444) (21,277) —
Shares for which no consideration recd ...... (b) 5,556 32,979 37,500
Shares eligible/or Dilutive EPS .....(a+b) 605,556 632,979 667,500
Dilutive EPS = [Net Profits/(a+b)] = 7.43 9.48 8.09
AS-20: Earning per Share 203

Note: There is only Incremental shares and no Incremental earnings in options.


The calculations of ESOP are quite difficult. Therefore students are required to acquaint themselves
with the topic of Share based payments from Financial Reporting Module.

Problem 10: From the information furnished you are required to compute the Basic and Diluted EPS
(earnings per share) for accounting year 0.04.2011 to 31.3.2012 and adjusted EPS for the year 01.04.2010 to
31.03.2011.
Net profit for year ended 31.03.2011 ` 75,50,000
Net profit for year ended 31.03.2012 ` 1,00,25,000
No. of Equity shares as on 01.04.2011 50,00,250
Bonus issue on 01.01.2012 1 share for every 2 held
No. of 12% Convertible Debentures of ` 100 1,00,000
each issued on 01.01.2012
Conversion Ratio of Debentures 10 shares per debenture
Tax Rate 30 percent
(CA Final May 2012) (Marks 4)
Solution:
a) Basic EPS for the year ended 31-3-2012:-

= = = ` 1.33 per share

b) Diluted EPS:-

= = ` 1.32 per share

*Convertible debentures were issued on 1-1-2012 hence weight of ¼ is applied.


c) Adjusted EPS:-
Net Profit for 31-3-2011 ` 75,50,000
No. of equity shares including Bonus 75,00,375

Adjusted EPS = = ` 1.00 per share

Problem 11: Scrap Ltd. has 27,00,000 Equity shares outstanding. In addition to this, it has also 9% 50,000
Convertible Preference shares ` 10 each which will be converted into 1,50,000 Equity shares in future. Now
for the financial year 2013-2014 Scrap Ltd. had made a net loss of ` 9,60,000 available to equity shareholder.
Prove that Basic EPS = Dilutive EPS. Assume Dividend Tax @ 11%.
Solution:
Basic EPS (Loss per share) = (–9,60,000/27,00,000] = (0.36)
For Diliutive EPS (Loss per share) = (–9,60,000 + 49,950/27,00,000 + 1,50,000) (anti-dilutive) (0.32)
As loss per share is reduced therefore 9% Conv. Pref. Shares are anti-dilutive and ignored. Hence it is
proved that Basic EPS = Dilutive EPS = (0.36) in case of loss making enterprises.
204 Accounting Standards

Problem 12: Undivided Ltd. having a net profit of ` 40,000 from continuing operations and net loss of
` 30,000 from discontinuing operations. The company has 20,000 Equity shares and 500 Potential equity
shares.
Compute Basic EPS & Dilutive EPS of Undivided Ltd.
Solution:
Profit/Loss position is as follows :
Net profit from continuing operations 40,000
Net loss from discontinuing operations (30,000)
Net profit to the company as a whole - 10,000
Recall Potential Equity Shares are considered dilutive only when their conversion would lead to decrease
in net profit per share from “continuing operations”.
For checking purpose whether the PES are dilutive/anti-dilutive consider results of continuing operations
only. In other words results from dis-continuing operations is to be ignored.
But for the final reporting purpose consider both Continuing as well as Discontinuing operations for
computing Basic as well as Dilutive EPS.
Therefore for dilutive EPS profits of `40,000 is the control figure which is from continuing operations.
Accordingly 500 Potential equity shares are dilutive as it decreases the net profit per share from “continuing
ordinary operations” from ` 2.00 to ` 1.95.
Reporting and Disclosures :
Basic EPS = (10,000/20,000) = 0.50
Dilutive EPS = (10,000/20,500) = 0.49
Problem 13: Find out the WANES:
Date Particulars Shares
01.04.2009 Opening shares (Face value `10) 14,400
01.06.2009 Merger with A Ltd (` 6 paid up) 9,000
01.10.2009 Issued shares – assets purchased 1/7/2009 7,200
01.11.2009 Amalgamation – purchase with X Ltd
Shares issued @ 6/- 9,600
01.02.2010 25% Shares buy back from opening shares
Shares issued to X Ltd were not eligible for dividend as decided by both the companies.

Solution:
Date Particulars Shares
01.042009 Opening shares 14,400
01.06.2009 Merger with A Ltd (` 6 paid up) [9,000 × 60%] 5,400
01.10.2009 Issued shares – assets purchased 1/7/2009 [7,200 × 9/12] 5,400
01.11.2009 Amalgamation – purchase with X Ltd —
Shares issued @6/- (it will be considered in the Diluted EPS)
01.02.2010 25% Shares buy back from opening shares (600)
WANES 24,600

Problem 14: As on 31/3/2001, the equity share capital of Q Ltd. is 10 crores divided into shares of `10 each.
During the financial year 2001/02, it has issued bonus shares in the ratio of 1:1. The net profit after tax for
the years 31/3/2001 and 31/3/2002 is ` 8.50 crores and ` 11.50 crores respectively. The Earnings per share
AS-20: Earning per Share 205

(EPS) disclosed in the accounts for two years is ` 8.50 and ` 5.75 respectively. Comment on the Discloser as
an Auditor. (C.A. Final Nov. 2002 Audit)(5 Marks)
Solution: The earnings per share for both the years will have to be calculated taking the equity share capital
after the bonus issue as the denominator. If the same is done the earnings per share for 31/3/2001 will be
` 4.25 and that for 31/3/2002 will be ` 5.75. Since the above figures of earnings per share have not been
disclosed, the company has not complied with the provisions of the standard. If the same is not followed, he
would then have to qualify his report in terms of section 143 of the Companies Act, 2013.
Problem 15: From the Books of Bharati Ltd., following information is available as on 31.3.2011 and
31.3.2012.
(1) Equity shares of `10 each 1,00,000
(2) Partly paid Equity Shares of ` 10 each ` 5 paid 1,00,000
(3) Options outstanding at an exercise price of ` 60 for one equity share of `10 each. Average Fair Value 10,000
of equity share during both years ` 75
(4) 10% convertible preference shares of `100 each. Conversion ratio 2 equity shares for each preference 80,000
  share
(5) 12% convertible debentures of ` 100. Conversion ratio 4 equity shares for each debenture 10,000
 
(6) 10% dividend tax is payable for the years ending 31.3.2013 & 31.3.2012.
(7) On 1.10.2012 the partly paid shares were fully paid up
(8)  On 1.1.2013 the company issued 1 bonus share for 8 shares held on that date.

Net profit attributable to the equity shareholders for the years ending 31.3.2013 and 31.3.2012 were
`10,00,000. Tax = 30%
Calculate: EPS in accordance with AS-20. (C.A. Final May 2003) (14 Marks)
Solution:
Calculation of Basic EPS:
31/3/2013 31/3/2012
(Restated)

Basic EPS = NPAT – PD 10,00,000 10,00,000


WANES 2,00,000 1,75,000
`5 `5.71
Calculation of WANES:
Opening Shares
Fully paid up (100000 × 1) 100000 100000
Partly paid up (100000 × 50%) 50000 50000
Cash Calls made (100000 × 50% × 6/12) 25000 -
Bonus (200000 × 1/8) × 12/12 25000 25000
WANES = 200000 175000

Ranking for Dilution:


Particulars 10% Conv. Preference 12% Conv. Debentures Options
Shares
Savings 880000 = 5.5 84000 = 2.1 0 =0
No of shares 160000 40000 2000
Ranking III II I
206 Accounting Standards

No of shares for no consideration for options = 10000 × (75 – 60)/75 = 2000 tax rate 30%.
(ii) Diluted earnings per share: Options are most dilutive as their earnings per incremental share is
nil. Hence, for the purpose of computation of diluted earnings per share, options will be considered
first. 12% convertible debentures being second most dilutive will be considered next and thereafter
convertible preference shares will be considered

For 31/3/2013 For 31/3/2012 (Adj)


NP for ESH No of shares EPS No of shares Adj EPS
Basic 1000000 200000 5 175000 5.71
+ Options 0 2000 2000
1000000 202000 4.95 177000 5.65
Dilutive Dilutive
Conver. Debentures 84000 40000 40000
1084000 242000 4.48 217000 5.00
Dilutive Dilutive
Conver. Preference 880000 160000 160000
1964000 402000 4.89 377000 5.21
Anti Dilutive Anti Dilutive

Problem 16: From the following information of Beta Ltd., Calculate EPS in accordance with AS - 20 -
Particulars 31.3.2008 31.3.2007
Net Profit before Tax ` 3,00,000 ` 1,00,000
Current Tax ` 40,000 ` 30,000
Tax relating to earlier years ` 24,000 ` (13,000)
Deferred Tax ` 30,000 ` 10,000
Profit After Tax ` 2,06,000 ` 73,000
Other information:
(a) Profit of 31.03.2008 includes ` 1,00,000 compensation from Central Government towards loss on
account of earthquake in 2005 (non-taxable)
(b) Outstanding convertible 6 % Preference Shares 1,000 issued and paid on 30.9.2006. Face value `100,
Conversion ratio 15 equity shares for every preference share.
(c) 15% Convertible Debentures of `1,000 each Total Face Value `1,00,000 to be converted into 10
Equity Shares per Debenture issued and paid on 30.6.2006.
(d) Total Number of Equity Shares outstanding as on 31.3.2008, 20,000 including bonus shares issued
on 1.1.08 of 10,000 shares, face value `100. (CA Final Nov 2008) (Marks 8)
Solution:
(a) Calculation of Earnings Per Share (EPS) of Beta Ltd.
` `
Year ended Year ended
31.3.08 31.3.07
1. A. Earning before extra ordinary items (after Pref. Div) 1,00,000 70,000
B. No. of Equity Shares 20,000 20,000*
C. Basic Earnings Per Share [A/B] (EPS excluding extra-ordinary) 5.00 3.50
2. Tax Rate applicable
40,000 + 30,000/2,00,000 35%
30,000 + 10,000/1,00,000 40%
AS-20: Earning per Share 207

Ranking System
3. A. Dividend on Weighted Average Preference Shares 6,000 3,000
B. Incremental Shares 15,000 7,500
C. EPS on Incremental Shares [A/B]---------Rank I 0.40 0.40
4. Convertible Debentures
A. Increase in earnings 9,750
` (1,00,000 × 15/100 × .65)
` (1,00,000 ` 15/100 × .60 × 9/12) 6,750
B. Increase in shares 1,000 750
C. Increase in EPS [A/B]---------Rank II 9.75 9.00
It is anti-dilutive as it increases the EPS from continuing ordinary operations (Para 39, AS 20)
Calculation of Diluted EPS
Year ended Year ended
31.3.08 31.3.07
` `
A. Profit from continuing ordinary activities before Preference Dividend 1,06,000 73,000
No. of ordinary equity shares 20,000 20,000
Adjustment for dilutive potential of 6% convertible pref. shares 15,000 7,500
B. Total no. of shares 35,000 27,500
C. Diluted EPS from continuing ordinary operations [A/B] 3.02 2.65

Disclosure of EPS in accordance with AS 20 in the Profit and Loss Account


Earning per share (Face value ` 100) 31.3.08 (`) 31.3.07 (`)
Basic EPS from continuing ordinary operations 5.00 3.50
Diluted EPS from continuing ordinary operations 3.02 2.65
*Remember EPS whether basic or diluted has been calculated by excluding extra-ordinary activities.
**Debentures has not been considered in diluted EPS, as it increases EPS (anti-dilution).
CHAPTER 21
AS-22:
Taxes on Income

Chapter Outline
vv Objective, Meaning and Scope vv Setting off Principles of Deferred Taxes
vv Recognition Principles for Deferred Taxes vv Transitional Provisions
vv Review of Deferred Taxes vv Some Issues Relating to Deferred Taxes
vv Measurement of Current and Deferred Taxes vv Disclosures

Objective, Meaning and Scope


Traditionally taxes on income were provided as a percentage based on the profits from the revenue account.
But now with the introduction of AS-22 the taxes on income scenario is totally changed. Now taxes are
provided based on the yearly charge principle. Just as Salary or any other expenditure of P/L is debited by
the amount of expense accrued during the year, in the same way taxes on income will be expensed based on
the (Taxes paid + outstanding or Taxes paid - Prepaid). In other words now even taxes on income will become
one of the expenditure in P/L Account. Taxes will be worked out on the basis of accrual principle. Thus, AS-22
adhered to the fundamental principle of Matching Concept.
The concept of Taxes on income as per AS - 22 can be well explained as follows: For example lets say
the company has provided for the doubtful debts in year 1. This means income as per P/L will be after this
provision of doubtful debts. But for tax purpose it is disallowed. Hence in the year 1 the company is paying
more taxes as compared to the taxes computed on accounting income. This excess tax is a type of prepaid
taxes. When in the next year the company actually suffers bad-debt loss then a tax deduction can be claimed.
But in year 2 income as per taxable income will be less than the accounting income hence the prepaid taxes
of last year can be recouped.

Scope
The Standard deals with the taxes on income (domestic as well as foreign taxes). It excludes CDT, FBT and
Wealth Tax. But FBT has been otherwise abolished.
AS-22: Taxes on Income 209

SOME TERMS AND THEIR DEFINITIONS


(i) Taxable Income: Income computed as per Income Tax Laws is Taxable Income. Of course the Net profit
which is derived as per the Profit and Loss account cannot be said to be Taxable Income. By taking Net
Profit as the base a separate computation as per the provisions of Income Tax Act is done for computing
Taxable Income.
(ii) Current Taxes: Current Taxes are taxes payable on the Taxable Income as discussed above. It is the actual
tax liability, which has to be paid either today or in a subsequent year.
Current Taxes = Tax rate% × Taxable Income
Accounting Income: Accounting income is the income as per P/L account. It is the profit, which is disclosed
(iii)
to the shareholders in the Financial Statements. Such profit is calculated by complying with the provisions
and laws of Companies Act.
(iv) Tax expense: It is the amount of tax which is debited to the P/L Account. It is the charge against revenue.
Irrespective of Current tax liability, Tax expense is the tax accrued during the year just like any other
expense. Hence tax expense is a yearly charge against profit.
(v) Differences in Taxes: As we have seen in the definitions given above that income as per P/L account
(Accounting income) and income as per Income Tax Laws (Taxable Income) is not the same. Such differences
give rise to tax to be paid and tax provision. Further such difference in both the incomes can arise out of
two differences namely (i) Timing differences and (ii) Permanent Differences.
ww Timing Differences (T.D.) → Differences in accounting income and Taxable income which arises in one
year and is capable of reversal in the subsequent years is known as timing Differences. If the differ-
ence which arises do not reverse in the subsequent years then it is not a timing Difference. Timing
Difference also known as Temporary Differences.
Examples of Timing Differences: Depreciation, Payments as per section 43B, Impairment losses,
Preliminary Expenses, provisions for warranties, provision for excise duties, lease income.
ww Measurement of Timing Differences: For the measurement of T.D. lets say Depreciation as per I. Tax
is `1,00,000 and Depreciation as per Companies Act is ` 70,000, then T. D. = ` 30,000.Further such
difference should reverse in the subsequent years then only it can be TD.
ww Permanent Differences (P.D.) → These are the differences in the Accounting income and Taxable
income which arises in one year and is not capable of reversal in the subsequent years. In other words
these differences are of permanent nature.
Examples of Permanent Differences: Agricultural Income, Personal Expenses, Donations not eligible
under 80G. Lets say Agricultural income reduces taxable income (by claiming exemption) and this
creates a difference between accounting income and taxable income. In subsequent years accounting
income will never get an exemption, in which case the difference will be a permanent difference.

Why We are Learning TD and PD?


We are learning TD and PD to provide for deferred taxes. This is because Deferred taxes are provided only on
TD and not on PD.
(vi) Deferred Taxes: Deferred taxes are taxes on timing differences. Deferred taxes indicate either taxes
payable in future or tax savings in future. If today more taxes have been paid against tax expense,
this mean for future we have created a Deferred tax Asset (DTA). If today less taxes have been paid
against tax expense, then a liability is accrued which is Deferred tax Liability (DTL).
Deferred Taxes = Tax rate% × Timing Differences
210 Accounting Standards

Measurement of Deferred Taxes: For the measurement of D.T. let’s say Depreciation as per I. Tax is
` 1,00,000 and Depreciation as per Companies Act is ` 70,000. Tax rate is 30%.
Then T.D = ` 30,000 and D. T. = 30% × 30,000 = ` 9,000 (DTL) → pay less today and tax authority will
recover it in subsequent year.
Other important points:
◘◘ DTA indicates assets created for the company.
◘◘ DTL indicates liability created for the company.
◘◘ Separate DTA’s/DTL’s are calculated for each timing difference.
◘◘ Once a DTA/DTL is created it should be reversed in the subsequent in years.
◘◘ Deferred taxes are created for all timing differences. Deferred taxes are not created for permanent
differences.
◘◘ Deferred taxes also arise in case of companies paying Nil taxes.
◘◘ Deferred taxes should not be discounted.
◘◘ Deferred taxes should be created for all timing differences. But for DTA the principle of prudence
should also be taken care of.
Tax Expenses = Current Tax + / - Deferred Taxes
◘◘ Journal Entry:
(1) Profit and loss A/c (tax expense) Dr. xxx
 To Current Tax A/c xxx
Profit and loss A/c (tax expense) Dr. xxx
 To Deferred Tax A/c (DTL) xxx

DO YOU KNOW
Current tax and deferred tax is required to be calculated, but P/L Dr (tax charge) is always the balancing figure.

Recognition Principles for Deferred Taxes


As already discussed above that deferred taxes are to be recognised for all the timing differences.
Recognition for DTL: When a company has DTL then it need not worry. It should be immediately recorded and
c/f. DTL should be created even under the circumstances of losses. Tax liability has to be paid in future hence
DTL has to be recorded by adhering to the principle of Prudence.
Recognition for DTA: When a company recognises DTA then it has to be very sure that such DTA will be
reversed (realised) in future. Recording of DTA should be always be supported by enough proof, that in future
sufficient income is available to recover DTA. DTA should be recognised and carry forward only to the extent
that in future sufficient taxable income is available. A company incurring yearly losses may record DTA but
it should produce enough evidence to demonstrate that in future DTA can be well recovered. Special care
is to be taken while creating a DTA. Standard is too strict in case of recovery of DTA’s. Standard has made it
mandatory for the companies creating DTA to produce evidences and proof for the recovery of DTA’s. Binding
Sales Contracts, Lucrative Sales orders on hand, Better turnaround can be some of the possible evidences for
realising DTA’s. If such loss making companies fail to present evidences for the realisation of DTA’s then DTA’s
should not be recognised.
AS-22: Taxes on Income 211

Following is the table showing nature of evidence to be produced:

Incurrence of Profit or Losses Nature of proof / evidence to be produced


1) Profit making Companies DTA can be recorded.
2) Loss making companies DTA can be recognised only if the company is Reasonable
Certain for the future profits.
3) Mounting losses + Unabsorbed Depreciation DTA can be recognised only if the company is Virtual
Certain for the future profits.

Meaning of Virtual Certainty: ASI-9


Standard has not defined as such Virtual Certainty. Virtual Certainty is a matter of judgement. It differs from
case to case. Virtual Certainty should be supported by enough evidences on the reporting date. It means that
it is certain for practical purposes. Mere business plans and forecasts do not fulfill the conditions for Virtual
Certainty. Robust and strong proof should be there to prove Virtual Certainty. For example In case of Fashion
Industries a sudden invention of new and innovative product which has high degree of potential to capture
market can be said to be an example of Virtual Certainty.
If a company suffers from some casual and exception losses then the company will have to prove here that the
losses were an exceptional loss which is not possible in future. For example: In case of oil manufacturing company
sudden discovery of oil fields can lead to virtual certainty.

Review of Deferred Taxes


For this purpose one should see whether already DTA is recognised or not:
(i) If DTA is recognised: If DTA is recognised then see whether conditions relating to reasonable / virtual
certainty are still valid and continuing. If such conditions are still valid or have become more strong
then carry forward the DTA and if such conditions cease to exists then write – off the DTA.
(ii) DTA is not already recognised: If DTA is not recognised then see whether conditions relating to
reasonable/virtual certainty arises and accordingly recognise DTA. If no future visibility of Virtual
Certainty exists then don’t record DTA.
Hence a proper review and reassessment of DTA is very much required by every Company at the end
of Balance Sheet date.

Measurement of Current and Deferred Taxes: (para 20 and 21)


Deferred taxes should be measured using tax rates and tax laws that have been enacted or substantively
enacted by the balance sheet date. Enactment is when a bill is passed by both the houses of parliament and it
receives the consent of president.
◘◘ Current taxes are paid on current tax rates.
◘◘ Deferred taxes are calculated by using tax rates and tax laws that have been enacted or substantively
enacted by the balance sheet date.
For ex: If for the financial year 2008-09 under audit budget is passed in Feb. 2009.The current tax rate is
40% but the Feb. budget declares next year’s tax rate as 35%. Then for the financial year 2008-09 under audit
current tax is calculated on 40% but deferred tax is calculated on 35% (next year’s tax rate).

Setting off Principles of Deferred Taxes


DTA and DTL relating to the same provisions and laws can be set off. For example: DTA relating to Expenditure
u/s. 43B can be set off against DTL relating to Depreciation. This is because both the DT’s come under same
212 Accounting Standards

head of tax laws i.e: Income from Business. DTA relating to Capital Gains and DTL relating to Depreciation
cannot be set off.

Transitional Provisions
If a company for the first item accounts for DT’s then such effect has to be given either by debiting to Opening
Reserves. Again see to it that DT’s are created only for T.D’s and P. D’s. Future in case of DTA’s Prudence and
Virtual Certainty is to be considered.

Some Issues Relating to Deferred Taxes


(1) Effect on Section 115JB (Minimum Alternate Tax):
◘◘ Section 115JB is payment of tax as per Book Profit.
◘◘ As per ASI 6 Current Tax is to be calculated by taking higher of Tax under normal tax rates or Tax
under section 115JB.
Current Tax under section 115JB = ↑ (Tax at normal tax rates or Tax as per book profit)
◘◘ But DTA / DTL is calculated by taking normal tax rates only.
Deferred Taxes = Timing Differences × Normal tax rates
(2) Effect on Tax holidays companies:
ww Calculate Timing differences arising within the tax holiday period.
ww Calculate Timing differences reversing within the tax holiday period.
ww Deduct Timing differences reversing within the tax holiday period from timing differences
arising within the tax holiday period on FIFO basis.
ww Create DTA / DTL on the net timing difference.
ww Current tax = 0 for full tax holiday period.
(3) CFS: ASI-26
ww In case of consolidated financial statements Tax expense of parent and subsidiary is to be just
merged.
(4) MAT Credit: ASI-6
ww The excess of tax paid as per 115JB over current tax rates can be carry forward for the next
7 years.
ww MAT credit should be accounted for as an asset by passing the following entry.
 MAT credit entitlement Dr. xxxx
To P/L account xxxx
ww MAT credit entitlement should be disclosed under the head Loans and advances (New Schedule
VI)
ww MAT credit should be separately shown in the profit and loss statement.

(5) Deferred tax in case of amalgamation: ASI-11


The incorporation of Deferred taxes is not discussed in AS-14 (Amalgamation) but is discussed in AS-22
(ASI-11).The summary of ASI-11 is as follows:
One has to understand that DTA is not recorded by the vendor company on consideration of prudence
(virtual certainty is not met), then the following provisions will be applicable:
AS-22: Taxes on Income 213

Purchase Merger
The criteria of prudence are met on the date of amalgamation. Journal:
If assets/liabilities are recorded at fair values DTA Dr.
  To G/W or C Reserve Not applicable

If assets/liabilities are recorded at carrying values No entry No entry


Subsequently the criteria of prudence are met before the Journal:
first balance sheet date, but after the amalgamation.   DTA Dr. DTA Dr.
  To G/W or C Reserve To P/L A/c
Subsequently the criteria of prudence is met after the first DTA Dr. DTA Dr.
balance sheet after amalgamation   To Profit / loss A/c   To P/L A/c

DO YOU KNOW
AS-22 has maximum number of ASI’s.

Disclosures
◘◘ DTA/ DTL should be shown net under the New Schedule III i.e. DTA (net) will be disclosed under the
head Non Current Assets and DTL (net) under Non Current Liabilities. Net means either DTA or DTL
one will prevail i.e. either DTA – DTL or DTL – DTA.
◘◘ Nature of evidences regarding Virtual Certainty for DTA’s recognition (in notes to accounts).

DO YOU KNOW
With the disclosure requirements of New Schedule III, ASI-7 on Discolusre of DTA/DTL will be scrapped.

PROBLEMS AND SOLUTIONS

Problem 1: Trimurthy Pan Masala (P) Ltd. was incurring heavy losses in the last several years since it could
not withstand the competition in the market. The State in which the company had its registered office and
also its major sales had moved a bill in the State Assembly to ban manufacture and sale of all kinds of Pan
Masalas in the State. While finalizing the accounts for the year ended 31-03-2004, the CFO of the company
created a Deferred Tax Asset for the tax benefits that would arise in future years from the earlier years losses
that had remained unabsorbed in Income Tax. (C. A. Final Audit Nov. 2002 and Nov 2004) (Marks 5/4)
Solution: Creation of Deferred Tax Asset: Accounting Standard 22 on “Accounting for Taxes” requires that
Deferred Tax Asset (DTA) should be recognised for all timing differences, subject to the considerations of
prudence. The Standard further states that unabsorbed losses of the business can be considered for creation
of DTA provided there is virtual certainty supported by convincing evidence that sufficient future taxable
income will be available against which such DTA can be realised. In view of this, the DTA created by the CFO
of Trimurthy Pan Masala (P) Ltd. is not in order. Since the company which is in the business of manufacturing
of panmasala has been incurring heavy losses and in addition to this the State in which the company is having
its major sales is proposing the ban on sale and manufacture of pan masala, the statutory auditor would,
therefore, have to qualify his report and mention the extent of amounts of loss and reserves which are under
and overstated respectively.
214 Accounting Standards

Problem 2: Following is the P/L Statement for various years of KOMAL Limited.
2004 – 2005 2005 – 2006 2006 – 2007
Net Profit before Taxes 10,00,000 12,00,000 15,00,000
(1) Company purchased Fixed Assets at the beginning of 2004-2005 for ` 20,00,000. Depreciation as per I. Tax is 30% WDV
and Depreciation as per Co. Act is 40% SLM basis.
(2) S ales Tax provided in 2005-2006 but deduction received in 2006-07 ` 25,000.
(3) Tax rate 35%.
You are required to: (1) Compute Timing Differences, (2) Deferred Taxes, (3) Tax expenses, (4) Pass necessary journals:
Solution:
INCOME AS PER CO. ACT &I. TAX. + CURRENT TAX ON TAXABLE INCOME
04-05 05-06 06-07
Net Profit before taxes 10,00,000 12,00,000 15,00,000
+ Depn. As per Co. Act 8,00,000 8,00,000 4,00,000
– Dep. As per I. tax –6,00,000 –4,20,000 –2,94,000
Sales Tax 25,000 –25,000
+ Advertisement as per Co.
Advertisement allowed
Taxable Income 12,00,000 16,05,000 15,81,000
Current Tax @35% 4,20,000 5,61,750 5,53,350
04-05 05-06 06-07
Depreciation 2,00,000 3,80,000 1,06,000
Sales Tax 25,000 –25,000
Advertisement
Total : 2,00,000 4,05,000 81,000
+ Opening balance — 2,00,000 6,05,000
Closing bal 2,00,000 6,05,000 6,86,000
LEDGERS OF DTA & DTL:

D T A A/C. 35% OF T. D.

2004-05 2004-05
To P/L 70000 By bal c/f 70000
2005-06 2005-06
To bal c/d 70000 By bal c/f 211750
To P/L 141750
2006-07 2006-07
To bal c/d 211750 By P/L 8750
To P/L 37100 (reverse)
By bal c/f 240100
AS-22: Taxes on Income 215

Journal Entries for the Year

2004-05 2005-06 2006-07


P/L A/c Dr. 420000 P/L A/c Dr. 561750 P/L A/c Dr. 553350
To Current tax/Cash 420000 To Current tax/Cash 561750 To Current tax / Cash 553350
(Being tax liability payable) (Being tax liability payable) (Being tax liability payable)
DTA A/c Dr. 70000 DTA A/c Dr. 141750 DTA A/c Dr. 37100
To P/L A/c 70000 To P/L A/c 141750 To P/L A/c 37100
(Being DTA created for the TD) (Being DTA created for the TD) (Being DTA created for the TD)
P/L A/c Dr. 8750
To DTA A/c 8750
(Being DTA reversed)

Disclosures As Per P/L A/C.


2004-05 2005-06 2006-07
NPBT = 10,00,000 NPBT = 12,00,000 NPBT 15,00,000
Less: Less: Less:
Current Tax 4,20,000 Current Tax 5,61,750 Current Tax 5,53,350
+/– D. Tax – 70,000 +/– D. Tax – 141750 +/– D. Tax – 28,350
(3,50,000) (4,20,000) (5,25,000)
NPAT = 6,50,000 NPAT = 7,80,000 NPAT = 5,25,000
In the Balance sheet DTA (net) is shown as a separate line item. Current tax if not paid then shown as other
current liabilities.
Problem 3: Rama Ltd has provided the following information:
(a) Depreciation as per Accounting Records `2,00,000
(b) Depreciation as per Income Tax Records `5,00,000
(c) Unamortized Preliminary Expenses as per Tax Record `30,000
There is adequate evidence of future profit sufficiency. How much deferred tax asset/liability should be
recognized as transition adjustment? Tax rate 50% (CA Final May 2011)(Marks 5)
Solution: Depreciation as per Accounting Records 2,00,000 - Depreciation as per Income Tax Records
(5,00,000) = (3,00,000) Timing Difference
Create DTL 3,00,000 × 50% = `1,50,000
Unamortized Preliminary Expenses as per Tax Record 30,000 Timing Difference
Create DTA 30,000 × 50% = `15,000
Deferred Tax Liability towards Timing Difference on Depreciation = `1,50,000
Deferred Tax Asset towards Timing Difference on Preliminary Expenses = `15,000
Problem 4: Following is the P/L Statement for two years of BAPTISTA Limited.
2004 – 2005 2005 – 2006
Net Profit before Taxes
3,35,000 3,55,000
(1) D epreciation as per I. Tax is 75000 & 25000 for two years. Depreciation as per Co. Act is 50,000 each year.
(2) S ales Tax provided in 2004-2005 but deduction received in 2005-06 ` 35,000.
(3) P enalties not allowed under I. Tax ` 10,000 – For the year 2005-2006
(4) C ompany received Income from Bonds ` 10,000 in the year 2004-2005 (exempt).
(5) Tax rate 40%.
216 Accounting Standards

You are required to: (1) Compute Timing Differences, (2) Deferred Taxes, (3) Tax expenses, (4) Pass necessary journals.
Solution: Income As Per Co. Act & I. Tax. + Current Tax On Taxable Income
04-05 05-06
Net Profit before taxes 3,35,000 3,55,000
+ Depn.as per Co. Act 50,000 50,000
– Dep. As per I. tax –75,000 –25,000
Sales Tax 35,000 –35,000
+ Penalties disallowed +10,000
Exempt Income u/s. 10(15) –10,000
Taxable Income 3,35,000 3,55,000
Current Tax @40% 1,34,000 1,42,000
Statement of T.D. 2004-05 2005-06
Depreciation –25,000 25,000
Sales Tax 35000 –35000
Total : 10,000 –10,000
+ Opening balance — 10,000
Closing bal 10,000 —
Ledgers of DTA & DTL:
D T A A/C. 40% OF T. D. D T L A/C. 40% OF T. D.
2004-05 2004-05 2005-06 2005-06
To P/L 14000 By bal c/f 14000 To bal c/f 10000 By P/L 10000
2005-06 2005-06 2005-06 2005-06
To bal c/d 14000 By P/L 14000 To P/L 10000 By bal c/d 10000
(reverse) (reverse)

Problem 5: Rane Firms Ltd. a listed company started business on 1.4.2004. The following are the details
from the books of account :
Profit before depreciation and taxes:
2004 – 2005 ` 14,00,000
2005 – 2006 ` 18,00,000
2006 – 2007 ` 24,00,000
2007 – 2008 ` 30,00,000
The company purchased the following machine:
1.4.2004 ` 10,00,000
The company charges depreciation on machines @ 25% on SLM. wherealse the rate of depreciation for tax
purpose is 30% p.a. SLM. Tax rates for the relevant years were 45%, 40% 35%, 32% and 30% respectively.
You are required to: (1) Compute Timing Differences, (2) Deferred Taxes, (3) Tax expenses, (4) Pass
necessary journals.
Solution: Before Referring The Problem On Changes On Tax Rates For Various Years, Refer The Following
Paras Of As-22 :
1. Current taxes are always computed on the basis of tax rates and tax laws applicable for the year…
para 20.
2. Deferred tax assets and liabilities should be measured using the tax rates and tax laws that have been
enacted or substantively enacted by the balance sheet date. …para 21.
AS-22: Taxes on Income 217

But in the problem substantive rates are not provided

Income As Per Co. Act & I. Tax. + Current Tax on Taxable Income

2004-05 2005-06 2006-07 2007-08


Net Profit before dep& taxes 14,00,000 18,00,000 24,00,000 30,00,000
- Dep. As per I. tax -3,00,000 -3,00,000 -3,00,000 - 1,00,000
Taxable Income 11,00,000 15,00,000 21,00,000 29,00,000
Tax % 45% 40% 35% 32%
Current Tax 4,95,000 6,00,000 7,35,000 9,28,000
Depn. as per Co. Act 2,50,000 2,50,000 2,50,000 2,50,000
STATEMENT OF TD 2004-05 2005-06 2006-07 2007-08
Depreciation 50,000 50,000 50,000 -150,000
Total: 50,000 50,000 50,000 -150000
+ Opening balance — 50,000 1,00,000 150000
Closing bal 50,000 1,00,000 1,50,000 -

LEDGER OF DTL:
D T L A/C
2004-05 2004-05
To bal c/f 20,000 By P/L (40% x 50000) 20,000
2005-06 2005-06
To bal c/f 35,000 By bal c/d 20,000
(35% x 100000) By P/L (b.al fig) 15,000
2006-07 2006-07
To bal c/f 48,000 By bal c/d 35,000
(32% x 150000) By P/L (b.al fig) 13,000
2007-08 2007-08
To P / L (reverse) 48,000 By bal c/d 48,000
To bal c/f (30% x 0) 0

Problem 6: Following is the P/L Statement for 3 years of PQR Limited.


31.03.2001 31.03.2002 31.03.2003
Net Profit before Taxes (200000) 100000 120000
(1) Losses can be c/f. in the next 8 years as per I. Tax.
(2) B
 y the end of 31.03.2001 company feels that in future sufficient taxable income is available.
(3) Tax rate 40%.
You are required to: (1) Compute Timing Differences, (2) Deferred Taxes, (3) Tax expenses, (4) Pass necessary journals.
(C. A. Final Nov. 2003) (Marks 4)
Solution:
31.03.2001 31.03.2002 31.03.2003
Net Profit before taxes –2,00,000 1,00,000 1,20,000
Taxable Income 0 0 20,000
Current Tax @40% 0 0 8,000
218 Accounting Standards

DTA/DTL for 31.03.2001 T.D. DTA 40%


Losses gives rise to DTA only 2,00,000 80,000 0
Total: 2,00,000 80,000 0
Journal:
DTA Dr. 80,000
To P/L 80,000
(Current tax = 0)
DTA/DTL for 31.03.2002 T.D. DTA 40%
Losses recovered –1,00,000 –40,000 0
Total: –1,00,000 –40,000 0
Add: Opening 2,00,000 80,000 0
Closing Balance: 1,00,000 40,000 0
Converting losses to profits is recovering DTA (Reversal of TD;s)
Journal:
P/L Dr. (b.f.) 40,000
To Current Tax 0
To DTA 40,000
DTA/DTL For 31.03.2003 T.D. DTA 40%
Balance loss recovered –1,00,000 –40,000 0
Total: –1,00,000 –40,000 0
Add: Opening 1,00,000 40,000 0
Closing Balance: 0 0 0
Journal:
P/L Dr. (b.f.) 48,000
To Current Tax 8,000
To DTA 40,000

Problem 7: The following particulars are submitted by the accountant of Power point Ltd. for the year
31.03.2005:
(1) Profit before taxes ` 16,00,000
(2) Details of Land sold during the year:
ww Sale of Land on : 12.01.2005
ww Sale proceeds of Land :` 14,00,000
ww Date of purchase of Land : 14.11.1979
ww Cost of acquisition of Land :` 1,00,000
ww Market Value of Land on 01.04.1981: ` 3,00,000
ww Indexation for the year 2004-2005 : 480
(3) Capital gains tax 20%.
(4) Business income taxes 40%.
Compute DTA / DTL:
Solution: Lets separate two different types of Incomes heads:
Business Incomes:
Profits before taxes 16,00,000
(-) Profit on Sale of Land
AS-22: Taxes on Income 219

considered separately
(14,00,000 - 1,00,000) –13,00,000
Business Profits as per Income Tax = 3,00,000
Current Taxes @40% 1,20,000
Assuming there is no T.D. in Business Income, Entry will be:
P/L Dr. 1,20,000
  To Current Tax 1,20,000

Capital Gains:
Sale proceeds of Land 14,00,000
(-) Cost of Acquisition -14,40,000
(3,00,000 x 480/100)
Capital Loss: 40,000
Current Taxes @20% 0
DTA on Losses @20% 8,000
DTA  Dr. 8,000
  To P / L 8,000
It is assumed that the company has got enough future capital gains to recover DTA.

Problem 8: Show P/L Account for various years of Reliance Limited from the following particulars:
2001 – 2002 2002 – 2003 2003 – 2004 2004 - 2005
Net Profit before taxes 400,000 450,000 475,000 500,000
Other particulars:
(1) Company has only one Asset.
(2) Date of purchase of Asset 01.04.2001 for ` 4,00,000.
(3) Life 4 years.
(4) Depreciation as per Books SLM with residual Value Nil.
(5) Depreciation as per I. Tax 50% WDV Method.
(6) Recognition of impairment loss on 31.03.2003 ` 60,000.
(7) Sale of asset at the end of 2004 – 2005 for ` 52,000.
(8) Tax – rate @40%.
Solution:
01-02 02-03 03-04 04-05
Net Profit before taxes 4,00,000 4,50,000 4,75,000 5,00,000
+ Depreciation as per Co. Act 1,00,000 1,00,000 70,000 70,000
+ Impairment Loss 0 60,000 0 0
– Dep. As per I. tax –2,00,000 –1,00,000 –50,000 0
– Profit on Sale of Asset 0 0 0 –52,000
(WDV – SP = 0 - 52,000)
Short term capital gains 0 0 0 2,000
(4,00,000 – 2,00,000 –
1,00,000 -50,000) – 52,000
Taxable Income 3,00,000 5,10,000 4,95,000 5,20,000
Current Tax @40% 1,20,000 2,04,000 1,98,000 2,08,000
220 Accounting Standards

DTA/DTL For 2001-2002 T.D. DTA 40% DTL 40%


Excess Depreciation in I Tax -100000 0 40,000
Total: -100000 0 40,000
Journal:
P / L Dr. 1,60,000
To Current Tax 1,20,000
To DTL 40,000

DTA/DTL For 2002 – 2003 T.D. DTA 40% DTL 40%


Excess Depreciation in Co. 60,000 0 -24,000
Total: 60,000 0 -24,000
Add: Opening -1,00,000 0 40,000
Closing Balance: -40,000 0 16,000

Journal:
P / L Dr. 1,80,000
DTL Dr. 24,000
To Current Tax 2,04,000

DTA/DTL For 2003 – 2004 T.D. DTA 40% DTL 40%


Excess Depreciation in Co. 20,000 0 -8,000
Total: 20,000 0 -8,000
Add: Opening -40,000 0 16,000
Closing Balance: -20,000 0 8,000
Journal
P / L Dr. 1,90,000
DTL Dr. 8,000
  To Current Tax 1,98,000

DTA/DTL for 2004 –05 T.D. DTA 40% DTL 40%


Depreciation +/- Gains 20,000 0 –8,000
Total: 20,000 0 –8,000
Add: Opening –20,000 0 8,000
Closing Balance: 0 0 0

Journal
P / L Dr. 2,00,000
DTL Dr. 8,000
  To Current Tax   2,08,000

Problem 9: Compute Deferred Taxes for years given below based on the information provided to you and
also show the effect of MAT Credit:
2006 – 2007 2007 – 2008 2008 – 2009 2009 - 2010
Net Profit before taxes 200,000 110,000 10,000 90,000
Depreciation as per Books 50,000 50,000 1,00,000 1,00,000
Depreciation as per I. Tax 210,000 40,000 40,000 10,000
Tax – rate @35%. MAT @20%. Company is charged under section 115JB.
AS-22: Taxes on Income 221

Solution:
06-07 07-08 08-09 09-10
Net Profit before taxes 2,00,000 1,10,000 10,000 90,000
+ Depreciation as per Co. Act 50,000 50,000 1,00,000 1,00,000
– Dep. As per I. tax –2,10,000 –40,000 –40,000 –10,000
Taxable Income 40,000 1,20,000 70,000 1,80,000
Current Tax as per section 115JB 40,000 22,000 2,000 18,000
(Book Profit x 20%)
Current Tax as per normal tax rates 14,000 42,000 24,500 63,000
(Taxable Income x 35%)
Current tax as per Higher of 40,000 42,000 24,500 63,000
(115JB or Normal Taxes)

Statement of TD 06-07 07-08 08-09 09-10


Depreciation 1,60,000 –10,000 –60,000 –90,000
+ Opening balance - 1,60,000 1,50,000 90,000
Closing bal 1,60,000 1,50,000 90,000 -
DTL (56,000) 3,500 21,000 31,500
IMP: MAT provisions will be applicable.
Journal entry for the year 2006-07
P/L Dr 40000
To Current tax 40000
P/L Dr 56000
To DTL 56000
MAT Cr entitlement Dr 26000
(40000-14000)
To P/L 26000
Current tax Dr 40000
To Advance tax/TDS/Cash 40000

Journal entry for the year 2007-08


P/L Dr 42000
To Current tax 42000
DTL Dr 3500
To P/L 3500
Current tax Dr 42000
To MAT Cr entitlement 20,000
To Advance tax/TDS/Cash 22,000

Problem 10: A company has paid tax under 115JB for the year ended 31/3/2007. The company wants to
claim Mat credit in the next year. (C.A. Final Audit Nov. 2007) (Marks 5) Audit
Solution: The concerned issue is discussed in the Guidance Note issued by ICAI on “Accounting for Credit
available in respect of MAT under the IT Act 1961”. As per para 6 of the said Guidance Note, Although MAT
credit is not a deferred tax asset under AS 22 , yet it give rise to expected future economic benefit in the form
of adjustment of future income tax liability arising within the specified period.
MAT paid in a year in respect of which the credit is allowed during the specified period under the Income
Tax Act is a resource controlled by the company as a result of past event, namely the payment of MAT. MAT
credit has expected future economic benefits in the form of its adjustment against the discharge of the normal
tax liability if the same arises during the specified period. Accordingly, MAT credit is an asset.
According to the framework, once an item meets the definition of the term ‘asset’, it has to meet the criteria
for recognition of an asset so that it may be recognized as such in the financial statements. Para 88 of the
222 Accounting Standards

Framework provides the following criteria for recognition of an asset: an asset is recognized in the balance
sheet when it is probable that the future economic benefits associated with it will flow to the enterprise and
the asset has a cost or value that can be measured reliably.
Thus, if the auditor is satisfied that the probability of the company to claim the said credit is high, it could
recognize the same as an asset. In Balance sheet it should be shown under the head “Loans & Advances” as
“MAT credit entitlement”.
Problem 11: Compute Deferred Taxes and show Profit & Loss for year 2003 – 2004 and 2004 – 2005:
Exempt Limited started its operations on 01.04.2003 and has purchases one machine for ` 600,000 on that
day. For accounting purposes, the company charges SLM method of depreciation over 15 years assuming Nil
scrap value. Depreciation rate as per I. Tax is 25% WDV method. Profits before depreciation and tax for 2003
– 2004 and 2004 – 2005 is ` 400,000 and 425,000 respectively. The company enjoys 100% tax exemption for
10 years starting from 2003 – 2004 under section 80IA. Tax – rate @35%.
Solution:
As for 10 year income will be exempt u/s. 80IA therefore Current Tax = Nil
Only DT’s will be calculated for difference of Depreciation.

Originating Timing Differences


Year DEPRECIATION AS PER TIMING DIFFERENCES
Books I. Tax Arises Reverses Balance
( + ) ( - )
1 40,000 1,50,000 1,10,000 -
2 40,000 1,12,000 72,000 -
3 40,000 84,000 44,000 -
4 40,000 63,000 23,000 -
5 40,000 48,000 8,000 -
Total: 2,57,000
6 40,000 36,000 - 4,000
7 40,000 27,000 - 13,000
8 40,000 20,000 - 20,000
9 40,000 15,000 - 25,000
10 40,000 11,000 - 29,000
Tax holiday period ends 91,000
T.D. arises in year 1 = 1,10,000
Less: Reversed during tax holiday = –91,000
Net: 19,000
Tax 30% 5,700
DTL 5,700
Entry for yr. 1 is: P/L A/c. Dr. 5,700
To DTL 5,700
T.D. arises in year 2 = 72,000
Less: Reversed during tax holiday = 0
Net: 72,000
Tax 30% 21,600
DTL 21,600
Entry for yr. 2 is: P/L A/c. Dr. 21,600
To DTL 21,600
T.D. arises in year 3 = 44,000
Less: Reversed during tax holiday = 0
Net : 44,000
Tax 30% 13,200
DTL 13,200
Entry for yr. 3 is: P/L A/c. Dr. 13,200
To DTL 13,200
AS-22: Taxes on Income 223

Entry for year 6-10 tax holiday period C. tax = 0, Tax expenses = 0, even Full DTLs are recouped in the first year itself.
Hence no entry from 6-10 for taxes.
Net DTL standing as at the end of year 10th = 5,700(Yr.1)+(72,000+44,000+23,000+8,000) x 35% = 57,150
From year 11 Company will start paying taxes and accumulated DTL from year 2-5 (i.e. ` 57,150) will now
reverse from year 11.

Problem 12: Milton Limited is full tax free enterprise for 10 years of its existence and is in second year of
its operations. Depreciation TD resulting in DTL in years 1 and 2 is ` 200lakhs and ` 400 lakhs respectively.
From the 3rd year onwards it is expected that the TD would reverse each year by ` 10 lakhs. Tax rate 35%.
Compute DTL in the second year and any charge to P/L. (C.A. Final May 2006) (4 Marks)
Solution:
T.D. arises in year 1 = 200 lakhs
Less: Reversed during tax holiday = (10lakhs × 8) -80 lakhs
Net : 120 lakhs
Tax 35%
DTL 42 lakhs
Entry for yr. 1 is: P/L A/c. Dr. 42 lakhs
To DTL 42 lakhs
T.D. arises in year 2 = 400 lakhs
Less: Reversed during tax holiday = 0
Net : 400 lakhs
Tax 35%
DTL 140 lakhs
Entry for yr. 2 is: P/L A/c. Dr. 140 lakhs
(Trf. To P/L A/C.)
To DTL 140 lakhs
Total DTL at the end of year 2 = (42+140) = ` 182 lakhs.

Problem 13: Omega Ltd is working on different projects those are likely to be completed within 3 Years
period. It recognizes revenue from these contracts on percentage of completion method for the financial
year 2006, 2007 and 2008 for ` 11 Lakhs, `16 Lakhs and ` 21 Lakhs respectively. However, for Income Tax
purpose, it has adopted the completed contract method under which it has recognized revenue of ` 7 Lakhs,
` 18 Lakhs and ` 23 Lakhs for the years 2006, 2007 and 2008 respectively. Income Tax rate is 35%. Compute
the amount of deferred tax asset/ liability for the years 2006, 2007 and 2008. (C.A. Final New Syllabus)
Solution:
2006 2007 2008
Revenue as per accounts (percentage of completion method) 11,00,000 16,00,000 21,00,000
Revenue as per tax (completion contract method) 7,00,000 18,00,000 23,00,000
Timing Difference 4,00,000 (2,00,000) (2,00,000)
DTL (Timing diff x 35%) 1,40,000 (70,000) (70,000)
DTL arises DTL reverse DTL reverse

Problem 14: Wonder Compliments Ltd has incurred a loss of ` (1,00,000) in the year 2012-13. The loss is an
exceptional loss and will be reversed in the next year. Items disallowed during the year amounts to ` 25,000.
The profit earned during the year 2013-14 is ` 3,40,000. Compute the amount of deferred tax asset/ liability
for the years 2012-13 and 2013-14. Tax rate is 40%.
Solution:
2012-13
Timing difference = -1,00,000 + 25000 = (75,000). Taxable income is Nil, hence Current tax = 0.
DTA = (75,000) x 40%= ` 30,000 + items disallowed 25,000 × 40% = 10,000. Total DTA = ` 40,000 2013-14
224 Accounting Standards

Timing difference reversal = 75,000 + 25000 = 1,00,000. Hence DTA reversed is full ` 40,000. Current tax
= 3,40,000 -75,000 – 25,000 = 2,40,000 × 40% = ` 96,000.
Problem 15: Cool mail distributors had paid the first MLP of a finance lease ` 3,90,000. The Cost of asset is
recorded at ` 15,50,000 on the signing of the lease contract. Depreciation rate is 20%. Interest rate for lessee
is 12%. Compute the net DTA / DTL during the year arising due to AS-19? Tax rate 35%.
Solution:
As per AS-19 following are the items charge to P/L by the Lessee company:
Depreciation = 15,50,000 × 20% = 3,10,000
Finance Charges = 15,50,000 × 12% = 1,86,000
Total items disallowed by I.T. = 4,96,000
Less: MLP allowed as rent (3,90,000)
Net timing difference = 1,06,000
DTA 35% 37,100
Problem 16: The following details are available in the books of RTP Ltd.
Particulars ` in lakhs
Provision for tax:
For 2009-10 400
For 2010-11 600
For 2011-12 500
Advance tax paid:
For 2009-10 350
For 2010-11 700
For 2011-12 540

RTP Ltd. estimates its Deferred Tax Liabilities to be ` 200 lakhs and its Deferred Tax Assets to be ` 40
lakhs. How will the above be disclosed?
Solution: Disclosure of Current and Deferred Tax balances will be on the basis of principles laid down in
AS 22. These are:
Current tax assets and liabilities can be set off, if the enterprise has a legally enforceable right to set off the
recognized amounts and intends to do settle them on a net basis.
Deferred tax assets and liabilities can be set off, it the items relates to taxes on income levied by the same
governing taxation laws.

Disclosure in Balance Sheet


Particulars ` in lacs
Non-Current Liabilities
Deferred Tax Liabilities (Net) [200 – 40 (DTA)] 160
Current Assets:
Advance tax paid 1,590
Less: Provision 1,500 90
AS-22: Taxes on Income 225

Problem 17: What is the tax effect of sale of fixed assets, considering the block of assets approach followed
in the Income-tax Act, 1961?
A company has a block of assets with a written down value of `1,00,000 on April 1, 2012 for tax purposes.
The book value of the assets for accounting purposes is also `1,00,000. The assets are depreciated on written
down value basis at 25 percent per annum for both accounting and tax purposes. Of the entire block, assets
costing `5,000 on April 1, 2012, were sold for `10,000 on March 31, 2014. Compute the deferred tax asset/
liability assuming tax rate of 40 per cent.
Solution: In the case of a company, the following computations will be made:
2012-13
In this year, depreciation for both accounting and taxation purposes would be `25,000 (25 per cent of
`1,00,000). Accordingly, no timing difference arises on this account.
2013-14
Depreciation for the year would be `18,750 (25 percent of `75,000) as per the books of account, while for
tax purposes it would be `16,250 as sale proceeds of `10,000 would be reduced from the block of assets prior
to the computation of depreciation. Accordingly, the following timing differences arise:
• Depreciation for tax purposes is `16,250 and for accounting purposes `18,750 giving rise to a timing
difference of `2,500.
• Profit on sale of fixed asset amounting to `7,188 (`10,000 - `2,812 being the WDV of the asset as on
31st March, 2014) is recognized for accounting purposes. However, for tax purposes this income is
not considered. This will result in a timing difference of `7,188.
The net timing difference would be `4,688 (`7,188 – `2,500) by which the accounting income would
exceed the taxable income, thus requiring creation of a deferred tax liability of `1,875 (4,688 X 0.4).
The difference of `4,688 would reverse in future years when depreciation for accounting purposes would
be higher as compared to depreciation for tax purposes because depreciation for accounting purposes would
be computed on higher carrying amount of fixed assets as compared to carrying amount of those assets for
tax purposes.
CHAPTER 22
AS-24:
Discontinuing Operations

Chapter Outline
vv Objective & Scope vv Initial Disclosure Event
vv Discontinuing Operation (DO): Defined vv Initial Disclosure Description

Objective & Scope


An enterprise may invest its resources on a number of operations and segments. Each such segment contributes
to the revenues and cash flow of the enterprise. Thus AS-24 requires separate disclosure of assets, revenue,
expenses and cash flows generated by each segment in the process of discontinuation. Since discontinuation
affects future performance of an enterprise, such disclosures permit a user of financial statements to make
a more informed assessment of the cash earnings of the enterprise. The objective of AS -24 is to establish
principles for reporting information about discontinuing operations. This will enhance the ability of users
of financial statements to make projections of an enterprise’s cash flows, earnings generating capacity and
financial position by segregating information about discontinuing operation from continuing operation.
The disclosure required includes disclosure of date and nature of initial disclosure event as also the date
by which, the transactions relating to discontinuance is expected to be completed.

DO YOU KNOW
Throughout New Schedule III Profit/Loss Statement information related to the continuing operation is consid-
ered. For discontinuing operations of an enterprise Part II of Schedule III requires single amount of presenta-
tion of profit before tax.

Discontinuing Operation (DO): Defined


A discontinuing operation of an enterprise has the following features:
i. Disposing of or terminating through a single plan: That the enterprise pursuant to a single (co-
ordinate) plan is: (also refer the explanation below#)
a) Disposing of substantially in its entirety, such as by selling the component by a single transaction
or by demerger or by spin-off of ownership of the component to the enterprise’s shareholders:
- selling them; or
b) Disposing of piecemeal, such as by selling off the component’s assets and settling its liabilities
individually: or
c) Terminating through abandonment permanently.
AS-24: Discontinuing Operations 227

ii. Separate major line of business or a geographical area of operation: Segmental Reporting as
defined in AS-17 normally represents or satisfies separate major line of business or a geographical
area of operation. Even if an enterprise operates in a single business or geographical segment, a
major product or service line may also satisfy this criterion.
ii. Distinguishable component - financially and operationally distinguishable: D.O. should
be financially and operationally distinguishable for reporting purposes. This means the assets,
liabilities, and major expenses should be attributable towards such discontinuing operation. If a debt
is attributed to a component, the related interest and other financing costs are similarly attributed to
it. It means had the operation discontinued it would eliminate the related income/expenses/assets/
liabilities.
#EXPLANATION: Manner of Discontinuing:
a1) Disposing of substantially in its entirety: It means a binding agreement of sale is entered with the
buyer. Actual transfer may take place at a later date.
b1) Piecemeal, selling off the component’s assets and liabilities: It means selling Assets and Settling
liabilities individually settlement of assets/lies may even take months perhaps years. There may not
be a binding sale agreement to sell the asset.
c1) Abandonment: An enterprise may terminate an operation by abandonment without substantial sale
of assets. However, changing the scope of an operation or the manner in which it is conducted is
not abandonment because that operation, although changed, is continuing. Business enterprises
frequently close facilities, abandon products, change the size of work force in response to market
forces. While those kinds of terminations generally are not, in themselves, discontinuing operations,
but they may be present in connection with a discontinuing operation.

Example of Abandonment: An event management company managing events for various nature stops
managing birthday parties and focuses on large scale events. If the birthday parties events was contributing
significant portion of the event management business and the plan of abandonment was based on a detailed
laid down pan then such a situation is covered by AS-24 as termination by Abandonment.
I llustration — Disposal of a major line of business
A company manufactures and sells 10 brands of consumer products that are grouped into five major product
lines — beauty care, health care, food, pet care and baby care. Each brand comprises operations and cash
flows that can be clearly distinguished, operationally and for financial reporting purposes, from the rest of
the entity. Therefore, each brand is a component of the entity. Each product line (i.e., a group of components)
represents a major line of business that comprises a major part of the entity’s operations and financial results.
The company decides to exit the beauty care market and disposes of its related product line.
Analysis
The disposal qualifies as a discontinued operation because the beauty care product line is a group of
components of the entity that represents a major line of business that comprises a major part of an entity’s
operations and financial results (i.e., a strategic shift).
Illustration — Disposal that does not qualify for presentation as a discontinued operation:
Assume the same facts as in the above Illustration, except the company decides to dispose of one of its brands
within the beauty care product line. Following the disposal, the company will not have significant continuing
involvement or continuing cash flows with the disposed component.
Analysis
The disposal does not meet the definition of a discontinued operation because the disposed brand does not
228 Accounting Standards

What may not be a discontinuing operation? A restructuring transaction or event that does not

meet the definition of discontinuing operations as per AS-24 should not be called a discontinuing
operation. Examples of activities that do not necessarily on their own satisfy the discontinuing
operation definition, but that might do so in combination with other circumstances, include:……..
para 9
(a) gradual or evolutionary phasing out of a product line or class of service;
(b) discontinuing, even if relatively abruptly, several products within an ongoing line of business;
(c) shifting of some production or marketing activities for a particular line of business from one location to another;
and
(d) closing of a facility to achieve productivity improvements or other cost savings.
(e) change in the scope of operation
(f ) selling the subsidiary whose activities were similar to its holding company under CFS.

Initial Disclosure Event


An enterprise should commence making disclosures required by the standard from the accounting period
in which the initial disclosure event as described below occurs. The initial disclosure event is one of the
following, whichever occurs earlier:
a) The enterprise has entered into a binding sale agreement for substantially all of the assets attributable
to the discontinuing operation: or
b) The enterprise’s board of directors or similar governing body has both (i) approved a detailed, formal
plan for the discontinuance and (ii) made an announcement of the plan.
Announcement for the above purpose means announcement of the main features of the plan to the affected
parties, e.g. trade unions, stock exchanges, creditors, lenders etc. A detailed formal plan normally includes:
a) Identification of the major assets to be disposed of;
b) The expected method of disposal;
c) The expected period required to complete the disposal;
d) The principal locations affected;
e) The location, function and expected number of employees who will be compensated for termination
of their services;
f) The estimated proceeds or salvage value to be realized by disposal.

Note: If initial disclosure event occurs after balance sheet date but before approval of the financial
statements by the board of directors in case of companies, or by the corresponding governing body in case
of other enterprises, the event is treated as event occurring after the balance sheet date, and disclosures
as per AS 4 are required. It will be treated as non – adjusting event and only disclosure in the BOD report
is required.

Initial Disclosure Description


An enterprise should include the following information relating to a discontinuing operation in its financial
statements beginning with the financial statements for the period in which the initial disclosure event occurs:
a) A description of the discontinuing operations;
b) The business or geographical segments in which it is reported as per AS-17, Segment Reporting;
c) The date and nature of the initial disclosure event;
d) The date or period in which the discontinuance is expected to be completed if known or determinable;
AS-24: Discontinuing Operations 229

e) Carrying amounts, as of the balance sheet date, of the total assets to be disposed of and the total
liabilities to be settled;
f) Amounts of revenue and expenses attributable to the operation.
g) Tax expense.
h) Cash flows attributable to DO and continuing operation.

What are the Assets/Liabilities/Income/Expenses Attributable to D.O


The items of assets, liabilities, income, expenses, gains, losses and cash flows can be attributed to a discontinuing
operation only if they will be disposed of, settled, reduced, or eliminated when the discontinuance is
completed. To the extent such items continue after completion of the discontinuance, they are not allocated
to the discontinuing operation. For ex: Salary of the continuing staff of a discontinuing operation.

Updating the Disclosures


In addition to the disclosures required above, an enterprise should include, in its financial statements, for
periods subsequent to the one in which the initial disclosure event occurs, a description of any significant
changes in the amount or timing of cash flows relating to the assets to be disposed or liabilities to be settled
and the events causing those changes.

Continuation of Disclosure
All disclosures required above, should continue in financial statements for periods up to and including the
period in which the discontinuance is completed. Discontinuance is completed when the plan is substantially
completed or abandoned, though full payments from the buyer(s) may not yet have been received.

Withdrawal from Discontinuation Plan


If an enterprise abandons or withdraws from a plan that was previously reported as a discontinuing operation,
that fact, reasons therefore and its effect should be disclosed.

Separate Disclosure for Each Discontinuing Operation


Any disclosures required by AS 24 should be presented separately for each discontinuing operation.

Is Loss Recognized in Respect of Discontinuing Operation an Extraordinary Item


Loss in respect of discontinuing operation is not an extra-ordinary item. This is because AS-5 cites two
examples of extra-ordinary item attachment of property and loss due to earthquake. Discontinuing operation
is an exceptional item.

PROBLEMS AND SOLUTIONS

Problem 1: Compel Ltd. is discontinuing a product segment which is not a reportable segment within the
meaning of AS-17 and reported in segment information as an unallocated reconciling item. Management
intends to sell-off the assets and settle the liabilities by piecemeal. Is it a discontinuing operation within the
meaning of AS-24?
Solution: Normally segments reportable under AS-17 are purely discontinuing operation for AS-24 also.
But what about those segments which act as unallocated reconciling item ? Reportable segment or part of
reportable segment can also be discontinuing operation. Condition for discontinuing operation is that, the
discontinuing operation should be major business line or area of operations.
230 Accounting Standards

In the present case C Ltd. is disposing one of the product segment through piecemeal distribution. Selling
off the attributable assets and liabilities itself indicates that some of the assets and liabilities of the business
are attributable to the segment. Hence it is a discontinuing operation.
Problem 2: Mix Ltd. is having cosmetics division, garment division and beverages division. The beverages
division accounts for 7% of total assets, 10% of enterprise revenue and 10% of combined profits of all divisions.
It is proposed to be demerged by the Board of Directors pursuant to a single plan. It is a distinguishable
component operationally and financially. Is it a discontinuing operation as per AS-24?
Solution: Normally segments reportable under AS-17 are purely discontinuing operation for AS-24. Mix
Ltd. wants to dispose beverages division, which is a discontinuing operation as per AS-24. Following are the
points entitling beverages division as discontinuing operation as per AS-24:
◘◘ 7% of total assets, 10% of enterprises revenue and 10% of combined profits etc. is a clear indication
of the fact that some of the assets and liabilities of the business are attributable to the beverages
segment.
◘◘ It is a distinguishable component operationally and financially.
Problem 3: Silk and Cotton Ltd. has a large network of retail shops which sell various types of clothes. It is
trading in clothes and carries on activities through retail outlets all over the country. It wishes to discontinue
a particular brand of silk sarees. The outlet will be operational even after the product is discontinued. Does it
have to report silk sarees as discontinuing operations?
Solution: The operating assets of the company is the retail outlets and not any type of cloth sold through the
outlet. The company is discontinuing the particular brand of silk sarees. The company can still continue to
operate the outlet by selling some other products through the outlet. Hence, discontinuing the silk sarees is
not discontinuing operation.
Problem 4: X Limited is presenting consolidated financial statements of X Limited and its subsidiaries. It
has sold a subsidiary XYZ Limited whose activities are similar to that of X Limited. Can the sale of XYZ Limited
be treated as discontinuing operation with reference to consolidated financial statements?
Solution: According to para 9 of AS-24 in relation to CFS is selling a subsidiary whose activities are similar to
those of the parent or other subsidiaries does not necessarily satisfy the definition of discontinuing operations.
But these might be discontinuing operations as per AS-24 in combination with other circumstances as per
the particular facts and circumstances of the case.
Problem 5: Does the fact that disposal of component is classified as discontinuing operations call into
question the enterprise’s ability to continue as a going concern?
Solution: According to para 14 of AS-24 the fact that a disposal of a component of an enterprise is classified
as a discontinuing operations does not, in itself, bring into question the enterprise’s ability to continue as a
going concern.
Circumstances under which the going concern ability gets invalid:
1) If on account of discontinuing operation the company discontinue the core operations retaining the
non - core operations then the going concern ability may be under doubt.
2) A company may have only one operation and that operation itself gets discontinued.
Problem 6: What if the component in P.5 above constitutes 70% of the net assets of the enterprise and
80% of its operating revenues and it is expected that net assets are going to realize 50% below the carrying
amount?
Solution: According to para 14 of AS-24 the fact that a disposal of a component of an enterprise is classified as
a discontinuing operations does not, in itself, bring into question the enterprise’s ability to continue as a going
concern. As stated in the problem that the component constitutes 70% of the net assets of the enterprise and
AS-24: Discontinuing Operations 231

80% of its operating revenues and the expectancy that net assets are going to realize 50% below the carrying
amount may cast a doubt on the enterprise’s ability to continue as a going concern.
P roblem 7: Resource Enterprises produces a standard type of tools and equipments for industiral use.
The company carry out its production into 3 processes.. It has been found that over a period of a time cost
of production of the first process is 15% higher than the market price of the intermediate product available
freely in the market. The company has decided to close down the first process as a measure of cost saving.
Should this event be treated as a discontinuing operations?
Solution: The company has made a good decision of outsourcing the service, in respect of a single process.
This has been done by the company with a view to achieve cost reduction. The change is merely a cost –
saving endeavour. Such a change does not meet definition criteria in para 3 (a) of AS-24. Hence, this change
is not discontinuing operation.
P roblem 8: A cosmetic producing Company provides the following information -
Particulars Cold Vanishing
Cream Cream
January 2013 –September 2013 p.m. 2,00,000 2,00,000
October 2013 –December 2013 p.m. 1,00,000 3,00,000
January 2013 – March 2014 p.m. 0 4,00,000

The Company has enforced a gradual change in product-line on the basis of an overall plan. The Board of
Directors of the Company has passed a resolution in March 2013 to this effect. The Company follows calendar
year as its accounting year. Should this be treated as a discontinuing operation? Give reasons in support of your
answer. (C.A. Final May 2008)
Solution: By gradually reducing the size of operations in the product line of Cold Cream, the company has
increased its scale of operation in Vanishing Cream. Such a change is a gradual or evolutionary phasing out of
a product line or class of services and does not meet the definition criteria namely, disposing of substantially
in its entirety, a component of the enterprise. Hence, this changeover is not a discontinuing operation.
P roblem 9: HL Ltd has 3 divisions auto products, auto lubricants and printing divisions. This is classified as
a reportable segment as per AS-17. It has decided to abandon 3 out of 5 assets of printing division and replace
it with new automated assets. Comment whether the sale of assets of printing division is a discontinuing
operation by abondonment?
Solution: It is not a termination by abandonment because only few assets have been abandoned. Changing
the scope of an operation or the manner in which it is conducted is not abandonment because that operation,
although changed, is continuing. Business enterprises frequently close facilities, abandon products, change
the size of work force in response to market forces. While those kinds of terminations generally are not, in
themselves, discontinuing operations, they can in connection with a discontinuing operation.
CHAPTER 23
AS-25:
Interim Financial Reporting

Chapter Outline
vv Meaning of Interim Financial vv Problems Encountered while following AS – 25
Reporting and Its Applicability vv Integral and Discrete View
vv Minimum Financial Statements vv Recognition and Measurement
vv Merits of Interim Financial Reporting Principles as PER AS – 25
vv Meaning of Year to Date Basis

Meaning of Interim Financial Reporting and Its Applicability


Meaning
An ‘interim period’ is a financial reporting period shorter than a full financial year. As per SEBI listed
companies has to present quarterly reports. Interim financial report means a financial report containing
either a complete set of financial statements or a set of condensed (minimum) financial statements for an
interim period.
Exception: The first accounting period of an enterprise which may be shorter than a full financial year is not
an interim period. Similarly any shorter period consequent to a change in the accounting period is not an
interim period.
Previously, SEBI is to take care of IFR. But now after AS-25- even unlisted companies are brought under
IFR. In short, SEBI & AS-25 both have comprehensively covered all the enterprises within the cage of IFR.
The reporting can be on quarterly basis or half yearly basis. But in case of listed companies (SEBI’s listing
agreement clause 41) reporting is to be done quarterly. For a listed company such reporting should be done
for the sake of good governance and even for stock market listing requirements.

Objective
AS 25, deals with the form, contents of an interim financial report, minimum components of an interim
financial report, minimum explanatory notes to the interim financial report, periods for which interim
financial reports should be prepared, recognition and measurement and restatement of previously reported
interim periods. Timely and reliable interim financial reporting improves the ability of investors, creditors,
and others to understand an enterprise’s capacity to generate earnings and cash flows.
The recognition and measurement principles laid down in this Accounting Standard apply also to complete
financial statements for an interim period, and such statements would include all of the disclosures required
under this Accounting Standard as well as those required by other Accounting Standards.
AS-25: Interim Financial Reporting 233

Applicability
As of now, neither the standard nor any regulation in India requires any enterprise to prepare interim financial
reports. This means an enterprise can voluntarily elects to prepare interim financial report. Then what may
be the various situations where AS-25 is applicable (but not mandatory) : (i) IFR for mergers, (ii) IFR for
declaring interim dividends, (iii) IFR for IPOs, (iv) IFR for submitting to the Banks.

SEBI Quarterly Reports


The disclosure and presentation as per AS-25 is not applicable to listed companies who are required to
report quarterly. SEBI has given its own format for the quarterly reporting. However, the recognition and
measurement principles laid down in AS-25 should be applied for recognition and measurement of items
contained in such interim financial results.

DO YOU KNOW
There are more estimates used in IFR as compared to annual accounts.

Minimum Financial Statements


AS 25/IAS 34
AS 25/IAS 34 defines the minimum content of an interim financial report as including condensed financial
statements and selected explanatory notes. It does not detail the information that should be included in these
condensed financial statements. An entity should determine the level of detail and ensure that the condensed
financial statements can be compared with the previous annual financial statements. The interim financial
report should provide an update on the latest financial statements.
The minimum elements specified for an interim financial report are:
◘◘ Condensed balance sheet
◘◘ Condensed income statement
◘◘ Condensed statement of changes in equity
◘◘ Condensed cash flow statement
◘◘ Selected explanatory notes
If the entity publishes interim financial statements that are condensed, then they should include, as
a minimum, the headings and subtotals included in the most recent annual financial statements and the
explanatory notes as required by IAS 34. Additional line items or notes should be included if omitting them
would make the interim financial statements misleading. Basic and diluted earnings per share should be
presented on the face of the income statement. If the entity’s most recent annual financial statements are
prepared on a consolidated basis, the interim financial report should be prepared on the same basis.

Periods for which Interim Financial Report are required to be presented


Interim financial statements (condensed or complete) should include periods as follows:
◘◘ Balance sheet as of the end of the current interim period and a comparative balance sheet as of the end
of the immediate preceding financial year;
◘◘ Statement of profit and loss for the current interim period and cumulatively for the current finan-
cial year to date, with comparative statements of profit and loss for the comparable interim periods
(current and year-to-date) of the immediately preceding financial year;
◘◘ Cash flow statement cumulatively for the current financial year to date, with a comparative statement
for the comparable year-to-date period of the immediately preceding financial year.
234 Accounting Standards

Example 1: Financial year of Bapu Ltd. is April 1 to 31 March. The company presents quarterly interim
reports. The interim report for second quarter of 2009-10 (01/07/09 to 30/09/09) should consist of the
following:
Balance Sheet as at 30 September 2009 31 March 2009
Profit & Loss A/c for:
3 months ending 30 September 2009 30 September 2008
6 months ending 30 September 2009 30 September 2008
Cash flow statement for
6 months ending 30 September 2009 30 September 2008
For an enterprise whose business is highly seasonal may provide financial information for twelve months
ending on interim reporting date as also comparative information for prior corresponding twelve months
period.
Example 2: Financial year of A Ltd. is April 1 to 31 March. Its business is highly seasonal. The company
presents quarterly interim reports. The interim report for second quarter of 2009-10 (01/07/09 to 30/09/09)
may consist of the following:
Balance Sheet as at 30 September 2009 31 March 2009
Profit & Loss A/c. for:
3 months ending 30 September 2009 30 September 2008
6 months ending 30 September 2009 30 September 2008
12 months ending 30 September 2009 30 September 2008
Cash flow statement for
6 months ending 30 September 2009 30 September 2008
12 months ending 30 September 2009 30 September 2008

Selected Explanatory Notes


An enterprise should include the following information, as a minimum, in the notes to its interim financial
statements:
◘◘ A statement that the same accounting policies are followed in the interim financial statement as
those followed in the most recent annual financial statement or, if those policies have been changed, a
description of the nature and effect of the change;
◘◘ Explanatory comments about the seasonality of interim operation;
◘◘ The nature and amount of items affecting assets, liabilities equity, net income, or cash flow that is
unusual because of their nature, size, or incidence;
◘◘ Issuances, buy-backs, repayments and restructuring of debt, equity and potential equity shares;
◘◘ Dividends, aggregate or per share (in absolute or percentage terms) separately for equity shares and
other shares:
◘◘ Segment revenue, segment capital employed (segment assets minus segment liabilities) and segment
results for business segments or geographical segments, whichever is the enterprise's primary basis
of segment reporting (disclosure of segment information is required in an enterprise's interim finan-
cial report only if the enterprise is required in terms of AS-17, segment reporting, to disclose segment
information in its annual financial statement);
◘◘ The effect of changes in the composition of the enterprise during the interim period, such as amalga-
mations, acquisition or disposal of subsidiaries and long-term investments, restructuring and discon-
tinuing operations; and
◘◘ Material changes in contingent liabilities since the last annual balance sheet date.
AS-25: Interim Financial Reporting 235

Merits of Interim Financial Reporting


◘◘ Any time, amalgamation & merger can be possible.
◘◘ Interim dividends can be declared.
◘◘ Loan obtaining procedures can be reduced.
◘◘ Work load for the final audit gets reduced to a certain extent.

Meaning of Year to Date Basis


IST IFR 1/4/2007- 30/6/2007 3 MONTHS
IIND IFR 1/4/2007- 30/9/2007 6 MONTHS
IIIRD IFR 1/4/2007- 31/12/2007 9 MONTHS

Problems Encountered while following AS – 25


There is certain problems accountants face while following AS-25 which is not the case for annual accounts.
The effect of errors in estimation and allocation are magnified.
◘◘ Too much work load on the employees throughout the year which they ignore.
◘◘ Some interim periods shows profit and some interim period’s shows losses. Hence calculation of tax
is often a difficult in interim periods.
◘◘ Expenses incurred in one particular interim period may benefit other interim periods, hence alloca-
tion of expenses becomes difficult.
◘◘ Estimation of provisions like gratuity, pension, and warranties can be very complex and time
consuming and costly.
◘◘ Expenses which are based on annual targets are difficult to ascertain.
◘◘ Stock taking in every interim period may be very time consuming and cumbersome job.
◘◘ Some expenses like advertisement incurred in interim period but benefit is spread during the whole
year.

Integral and Discrete View


Recognition (recording of) income, expenses, assets, liabilities etc. in IFR depends upon following suggested
views.

Integral view Discrete view


• It is a part & parcel of the total financial year. • It is itself an equivalent financial year.
• I t is not favored by ICAI. But some items like tax expenses • It is much favored by ICAI, because it is based on accrual
are to be worked out by integral view. system of accounting.
• Incomes & expenses are first estimated & then allocated. • I ncomes & expenses incurred during the year are
considered (accrual basis)
• P
 reviously recognized incomes & expenses are revised • Not many revisions are done here.
in the succeeding IFR.
• T he integral approach would require more use of • There are no estimations or allocations different from
estimation. those used for annual reporting. Annual operating
expenses are recognized in the interim period in which
they are incurred, irrespective of the number of interim
periods benefited.
236 Accounting Standards

Recognition and Measurement Principles as per AS – 25


Recognition of an Asset and Liability
For assets, the same tests of future economic benefits apply at interim dates and at the end of an enterprise’s
financial year. Cost that, by their nature, would not qualify as assets at financial year-end would not qualify
at interim dates either.
Similarly, a liability at an interim reporting date must represent an existing obligation. A cost that does not
meet the definition of an asset at the end of an interim period is not deferred on the balance sheet either to
await future information as to whether it has met the definition of assets or to smooth earnings over interim
periods within a financial year.

Provisions
The principles for recognizing and measuring losses from inventory write-downs, restructurings, or
impairments in an interim period are the same as those that an enterprise would follow if it prepares only
annual financial statements. However, if such items are recognized and measured in one interim period and
the estimate changes in a subsequent interim period of that financial year, the original estimate is changed
in the subsequent interim period either by accrual of an additional amount of loss or by reversal of the
previously recognized amount. Determination of the appropriate amount of a provision (e.g. warranties
liquidated damages, etc.) may be complex and often costly and time-consuming. Enterprises sometimes
engage outside experts to assist in the annual calculations.

Pension, Gratuity & Leave


AS-25 suggests that provision in respect of defined benefit schemes for an interim period should be calculated
based on the year to date basis, using actuarially determined rates. While finalising the annual accounts the
actuary should be requested to provide the interim period figures on retirement benefits like for curtailments,
current service costs, past service costs etc.

Contingencies
The measurement of contingencies may involve the opinions of legal experts or other advisors. Formal
reports from independent experts are sometimes obtained with respect to contingencies. Such opinions
about litigations, claims assessments, and other contingencies and uncertainties may or may not also be
needed at interim dates depending on the situation. For example, if there are significant developments in the
litigation at interim dates, one should revisit the need for making or reversing a provision for which a legal
opinion may be useful.

Contingent Lease Rentals


Sometimes lease arrangements may have contingent rentals built into it, which take effect on the happening
of an event or fulfillment of a condition. For example, a lease of premises may require additional rentals
to be paid if a particular turnover is achieved by the tenant. AS-25 does not contain any guidance on how
contingent lease rentals are to be dealt with in the interim periods. IAS-34 requires contingent lease payment
to be recognized as a liability if legal or constructive obligation exists. If the contingent lease payment is based
on sales target then a fair estimation of annual sales has to be made and contingent rentals per interim period
has to be appropriated (this is an example of integral view followed by IAS – 34 similar to volume rebates).

Inventories
Inventories are measured for interim period on the same basis as at year-end. To save time and costs,
enterprises often use estimates to measure inventories at interim period than at year-end. Full stocktaking
and valuation procedures may not be required at interim dates, hence estimations are inevitable. Inventory
AS-25: Interim Financial Reporting 237

losses from market declines should not be deferred beyond the interim period in which the decline occurs.
The standard requires the determination of net realizable value by reference to selling prices and related
costs to complete at the end of the interim period. An enterprise will reverse a write-down in a subsequent
interim period when the recoveries take place, i.e., subsequent recoveries against the losses recognized
should be treated as gains in the interim periods when the recoveries happen.

Intangible Assets
The deferment of costs incurred in developing intangible assets in an interim period in the hope that the
recognition (capitalization) criteria will be met later in the financial year is not justified. Reinstatement of
expenditure recognized as an expense in previous annual financial statements or interim financial reports
are prohibited. This means once expensed these costs cannot be later on resurrected and capitalized even if
conditions are met.

Revenues Received Seasonally, Cyclically, or Occasionally


(e.g. dividend, interest or royalty income)
These should not be anticipated or deferred as of an interim period if anticipation or deferral would not be
appropriate at the end of the financial year. Some enterprises consistently earn more revenues in certain
interim periods; such revenues are recognized when they occur. For ex: Sugar industry earns more revenue
in a particular interim period hence revenue should not be deferred but recorded in the period when
such revenues arises. Similarly Dividend received is to be recorded only when right to receive dividend is
established in an interim period.

Costs Incurred Unevenly or Irregularly during the Financial Year


Costs that are incurred unevenly during the financial year should be anticipated or deferred for interim
reporting purposes if, and only if, it is also appropriate to anticipate or defer that type of cost at the end of the
financial year. For example, the cost of a planned major periodic maintenance or overhaul or other seasonal
expenditure that is expected to occur late in the year is not anticipated for interim reporting purpose unless
as event has caused the enterprise to have a legal or constructive obligation. Accounting for future expenses is
prohibited. But if a legal or constructive obligation arises then such expenses can be anticipate and deferred
over the interim periods. For ex: VRS payments may be made in the future interim periods, they would be
accrued in the interim period when the liability occurred, i.e., when the employees signed up for the VRS
scheme.

Bonus / ex - gratia
Conditions for recording bonus in interim periods:
i) the bonus is a legal obligation or past practice (no realistic alternative but to pay)
ii) a reliable estimate of the amount can be made.
Bonus based on yearly results (discretionary) = Do not recognize in interim periods.
Bonus based on yearly results (past practice to pay) = Recognize in interim periods.
There should be either legal or statutory liability for provision of bonus. In other words the bonus should
be constructive on the interim period. Bonus can also arise due to past practice, to maintain good business
relations or act in an equitable manner.

Volume Rebates
Volume rebates or discounts are anticipated in interim periods, by both the payer and the recipient, if it is
probable that they have been earned or will take effect. An intergral view has been followed. But discretionary
rebates and discounts are not anticipated / deferred even if typically experienced in earlier periods.
238 Accounting Standards

Depreciation
Depreciation for the interim period is based only on assets owned during that interim period.

Foreign Currency Translation Gain and Losses


These are measured for interim results by the same principles as at the end of the financial year-end, which
is essentially the discrete approach ex: Monetary items are valued at closing rate and Non – Monetary items
at acquisition rate if carried at historical cost or at closing rate if carried at fair value.

Bad - debts
Provision/actual bad debts is to be recorded in the interim period when it arises. Don’t spread over the loss
over the entire year even if it is uncurred in a quarter.

Income Tax Expenses


Income tax expense for interim period income tax expense is estimated by applying average annual income
tax rate of the interim period. The estimated average annual income tax rates are re-estimated on year to date
basis. With the limited revision in AS-25, ICAI has deleted the word effective from average effective annual
income tax rate. The effective tax rate concept applies in cases where a country has a system of progressive
taxation (slab wise). In India there is only single corporate tax rate, hence the term effective was actually
unnecessary in the Indian context.

Changes in Accounting Policies and Estimates


An enterprise is required to apply the same accounting policy as applied in the most recent financial
statements. If accounting policy of interim accounts are required to be changed then annual and interim
accounts may not be comparable. But if the forthcoming year is expected to undergo a change in AP then
the AP in the interim period is also required to be changed. Also in case of quarterly reporting as per SEBI
guidelines the previous period IFR is required to be restated, where in case of AS-25 (adopting principle of
AS-5) only disclosure of change in AP is required and not the restatement of IFR.

DO YOU KNOW
The recognition and measurement principles laid down in AS-25 are applicable to SEBI quarterly reports by
the listed companies. But as far as the disclosure and presentation is concerned format prescribed by SEBI is
to be followed.

PROBLEMS AND SOLUTIONS

Problem 1: An enterprise reports quarterly. At the end of Q1 estimate of pre-tax annual profit was ` 6 lacs
and the aggregate of deductions from the GTI under I. Tax Act was estimated at ` 1 lac.
At the end of Q2 estimate of pre-tax annual profit was ` 6.30lacs and the aggregate of deductions from the
GTI under I. Tax Act was estimated at ` 84,000.
The pre-tax annual profits for Q1 & Q2 were ` 1.2 lacs and ` 1.3 lacs respectively. The tax rate is 30%.
Compute profit after tax for the quarters.
Solution: Estimated income pre-tax for Q1 is ` 5,00,000 and c/f. loss is ` 1,00,000, tax@40%, then Avg.
annual tax @ =
AS-25: Interim Financial Reporting 239

For Q1 : Annual tax liability = 30% (6-1) = ` 150000


AAET@ = 150000 / 600000 = 25%
Tax expense for Q1 = Actual earnings for Q1 x 25% = 120000 x 25% = 30000
For Q2 : Annual tax liability (reassessed) = 30% (6.3-0.84) = ` 1.638
AAET@ = 1.638 / 6.3 = 26%
Tax expense for Q2 = (Total actual earnings for Q1+Q2) x 26% - Tax expense already recognised = (120000
+ 130000) x 26% - 30000 = ` 65000 – ` 30,000 = ` 35,000.
Disclosures of P/L : Q1 Q2
PBT 120000 130000
Less: Tax (30000) (35000)
PAT 90000 95000
Problem 2: Estimated annual income ` 1,00,000 (inclusive of estimated capital gains of ` 20,000 earned
in second quarter). Assumed tax rates: Capital Gains: 10%, Other income: first 40,000 = 30% and balance
income = 40%. Assuming there is no difference between taxable income and accounting income. Income
earned every quarter is estimated to be ` 25,000 (including ` 20,000 capital gains earned in second quarter).
Solution: Total estimated income = 25,000 × 4 = ` 1,00,000 out of which ` 20,000 is capital gains which
means other income is ` 80,000
Avg. Annual tax % under capital gain = 10%
Tax on other income based on slab = 30% on first 40,000 and 40% on balance 40,000 = ` 28,000
Avg. tax rate = 28,000/80,000 = 35%
Tax expense for Q1, Q3 and Q4 = 25,000 × 35% = ` 8,750.
Tax expense for Q2 = 20,000 × 10% + 5,000 × 35% = ` 3,750.
Problem 3: Batuk Pvt. Ltd. receives dividend income of ` 4,00,000 on its investments in the third quarter.
The management wants to recognise dividend of `1,00,000 per quarter. Comment.
Solution: Dividend income is always recorded when the right to claim the dividend is established. In case
of Dividend income Discrete view has to be adopted, which means income will be recorded when accrued.
Hence full dividend of ` 4,00,000 is to be recorded in the third quarter……para 36 of AS-25
Problem 4: For the first quarter ending 31st March, 2013, Majhe Co. gives you the following information:
` crores
Sales 50
Salary and other expenses 30
Advertisement expenses (routine) 02
Administrative and selling expenses 08
While preparing interim financial report for the first quarter ‘Majhe Co.’ wants to defer ` 21 crores
expenditure to third quarter on the argument that third quarter is having more sales, therefore third quarter
should be debited by higher expenditure, considering the seasonal nature of business. The expenditures are
uniform throughout all quarters.
Calculate the result of first quarter as per AS 25 and comment on the company’s view.
(C.A. Final May 2005) (4 Marks)
240 Accounting Standards

Solution: Result of the first quarter ended 31st March, 2013


(` in crores)
Turnover 50
Add: Other Income Nil
Total 50
Less: Change in inventories Nil
Salaries and other cost 30
Administrative and selling expenses (8 + 2) 10 40
Profit 10

As per AS 25 on Interim Financial Reporting, the income and expense should be recognised when they are
earned and incurred respectively. As per para 38 of AS 25, the costs should be anticipated or deferred only
when
(i) it is appropriate to anticipate that type of cost at the end of the financial year, and
(ii) costs are incurred unevenly during the financial year of an enterprise.
Therefore, the argument given by Majhe Corp. relating to deferment of ` 21 crores is not tenable as
expenditures are uniform throughout all quarters.
Problem 5: Accounts of Poornima Ltd. show a net profit of ` 7,20,000 for the third quarter of 2005 after
incorporating the following:
(i) Bad debts of ` 40,000 incurred during the quarter. 50% of the bad debts have been deferred to the
next quarter.
(ii) Extra ordinary loss of ` 35,000 incurred during the quarter has been fully recognised in this quarter.
(iii) Additional depreciation of ` 45,000 resulting from the change in the method of charge of depreciation
to be applied from next financial year.
Ascertain the correct quarterly income. (C.A. Final May 2006) (4 Marks)
Solution: In the above case, the quarterly income has not been correctly stated. As per AS 25 Interim Financial
Reporting, the quarterly income should be adjusted and restated as follows:
(1) Bad debts of ` 40,000 have been incurred during current quarter. Out of this, the company has
deferred 50% (i.e.) ` 20,000 to the next quarter. Therefore, ` 20,000 wrongly deferred should be
deducted from ` 7,20,000.
(2) The treatment of extra-ordinary loss of ` 35,000 being recognized in the same quarter is correct.
(3) Recognising additional depreciation of ` 45,000 in the same quarter is in tune with AS 25.
Poornima Ltd should report quarterly income as `7,00,000 (` 7,20,000 – ` 20,000).
Problem 6: Practice Limited provided for a loss of ` 2,00,000 being lower of (NRV or Cost) for the first
quarter but subsequently the losses are reversed due to demand in the product. The company is expecting
further rise in the NRV in the third as well as fourth quarter. Can Practice Ltd. reverse the loss of ` 2,00,000
in next quarters?
Solution: As per para 29 of AS-25 when losses are provided in one interim period and recoveries takes place
in subsequent interim period, then losses once provided can be reversed in subsequent interim period. In the
present case Practice Ltd can reverse ` 2,00,000 on account of demand in the product.
Problem 7: Your client is a listed company is in the process of preparing first quarter accounts as required
by its stock exchange. Advise your client on its following position.

Note: Problem No.: 7 and 8 are master problems in AS-25 don’t forget to revise it.
AS-25: Interim Financial Reporting 241

(a) Training expenses incurred in the first quarter will be allocated equally over the four quarters
because the benefit is spread over the entire year
(b) Training expenses expected to be incurred in the last quarter will be estimated and equally allocated
to all the four quarters
(c) A donation of ` 5 million is expected to be made in the second quarter; provision will be made in the
first quarter.
(d) Since historically there has been an immaterial variance between budgets and actual, depreciation
charge each quarter will be determined by the budgeted figures.
(e) Incentives are provided to customers if they purchase 1 million kg of urea on an annual basis, it is
expected that atleast 30 customers would be able to achieve this target before the end of the third
quarter. No provision is made for the incentive in the first quarter, since the client believes that the
provision has not yet fructified
(f) 70% of the clients revenue comes in the second quarter. The client wants to spread this revenue to
all the four quarters, else the quarterly accounts will fluctuate significantly.
(g) A major repair is planned of the plant in the fourth quarter, which is absolutely certain. The estimated
repair expenditure will be accounted for in the first quarter itself.
(h) Over the years the client has been unfailingly giving bonuses to staff in the third quarter. This has
become its constructive obligation. The client does not wish to charge a proportionate amount of
bonus in the current quarter.
Solution: Answers for all the questions are = False i.e. the company is wrong in its contention for each of the
cases for preparation of IFR.
Problem 8: Antarbarti Limited reported a Profit Before Tax (PBT) of ` 4 Lakhs for the third quarter ending
30th September 2011. On enquiry you observe the following. Give the treatment required under AS–25.
(i) Dividend Income of ` 4 Lakhs received during the quarter has been recognised to the extent of ` 1
Lakh only.
(ii) 80% of Sales Promotion Expenses ` 15 Lakhs incurred in the third quarter has been deferred to the
fourth quarter as the Sales in the last quarter is high.
(iii) In the third quarter, the Company changed depreciation method from WDV to SLM, which resulted in
excess depreciation of ` 12 Lakhs. The entire amount has been debited in the third quarter, though
the share of the third is only ` 3 Lakhs.
(iv) ` 2 Lakhs Extraordinary Gain received in third quarter was allocated equally to the third and fourth
quarter.
(v) Cumulative loss resulting from change in method of Inventory Valuation was recognized in the third
quarter of ` 3 Lakhs. Out of this loss ` 1 Lakh relates to previous quarters.
(vi) Sale of investment in the first quarter resulted in a gain of ` 20 Lakhs. The Company had apportioned
this equally to the four quarters.
Prepare the adjusted Profit before tax for the third quarter. (C.A. Final Nov 2012) (5 Marks)
Solution:
Statement of adjusted PBT
PBT (given) 4,00,000
+ Dividend income not recorded 3,00,000
- Sales promotion expenses deferred (1500000 x 80%) (12,00,000)
+ Excess depreciation charged 9,00,000
+ Extraordinary income c/f now recorded 1,00,000
+ Excess loss on inventory write down eliminated 1,00,000
- Gain on sale of investments not related to this quarter (5,00,000)
Adjusted P B T 1,00,000
242 Accounting Standards

Problem 9: Pfizer Cosmo Limited has provided for ` 80,000 as warranty expenses for the first quarter
30/6/2009. The actual warranty claims amounted to ` 85,000. Also the warranty expenses for the second
quarter 30/9/2009 estimated at ` 70,000. Calculate the expenditure of warranty charged to profit/loss
account for the second quarter.
Also give the year to date expenditure with breakup of first and second quarter provision charged to
profit/loss a/c.
Solution: Total expenditure of warranty charged to profit/loss account for the second quarter = ` 85,000.
(includes 80,000 for second quarter and ` 5,000 change in estimate).
Year to date expenditure (cumulative) = ` 1,55,000 out of which ` 80,000 is already is charged to the first
quarter. This means 1,55,000 – 80,000 i.e. 75,000 will be net provision in the second quarter (including ‘
5,000 change in estimates)
Problem 10: A Company which has presented its quarterly results for a Limited Review Report, has
represented that expenditure incurred on heavy repairs carried in that quarter are being spread over the
entire year, since it would otherwise, distort the quarterly results Comment.
(CA Final June 09 Audit) (Marks 5)
Solution: Costs that is incurred unevenly during an enterprise’s financial year should be anticipated or def
erred for interim reporting purposes. This is to be anticipated if, and only if, it is also appropriate to anticipate
or defer that type of cost at the end of the financial year.
Conclusion: Costs that are incurred unevenly during an enterprise’s financial year should be anticipated
or deferred for interim reporting purposes, if and only if, it is also appropriate to anticipate or defer that
type of cost at the end of the financial year. But, cost of major planned periodic maintenance or overhaul is
not anticipated for interim reporting purpose, unless any event has caused the enterprise have a present
obligation.
Problem 11: An enterprise reports quarterly, estimates as annual income of `10 lakhs. Assume tax rates
on 1st `5,00,000 at 30% and on the balance income at 40%. The estimated quarterly income are `75,000,
`2,50,000, `3,75,000 and `3,00,000. Calculate the tax expenses to be recognized in each quarter.
(CA final May 2011 Accounts) (Marks 4)
Solution: As per para 29(c) – Treatment of Taxes: Income Tax Expense is recognized in each interim period
based on the best estimate of the weighted average annual income tax rate expected for the full financial year.
(a) Estimated Total Income for the year: 75000 + 250000 + 375000 + 300000 = `10,00,000
(b) Estimated Total Tax Liability: Upto `5 Lakhs = 30% × `5,00,000 = `1,50,000; Next `5,00,000 = 40%
× `5,00,000 = `2,00,000. Total Liability = 1,50,000 + 2,00,000 = `3,50,000.
(c) Effective Annual Tax Rate: Total Tax Liability ÷ Total Income = `3,50,000 ÷ `10,00,000 = 35%
(d) Quarterly Tax Expense: Therefore, tax expense for each quarter —Q1, Q2, Q3, Q4 = ` 75,000 × 35% =
`26,250, ` 2,50,000 × 35% = ` 87,500, ` 3,75,000 × 35% = ` 1,31,250, ` 3,00,000 × 35% = ` 1,05,000.
Problem 12: On 30-6-2011, X Limited incurred ` 3,00,000 net loss from disposal of a business segment.
Also on 31-7-2011, the company paid ` 80.000 for Property taxes assessed for the calendar year 2011. How
should the above transactions be included in determination of net income of X Limited for the six months
interim period ended on 30-9-2011? (CA Final Nov. 2008 Marks 5) + CA Final A/Cs May 2012 Marks 5)
Solution:
Disposal of Business Segment: Net Loss of from disposal of a business segment of ` 3,00,000 should be
reported in full as the loss was incurred during the half year period.
Property Taxes: Cost charged as Expense should be allocated to interim period on accrual basis. Hence,
Property Taxes of ` 80,000 for the whole year should be apportioned as follows:
1/1/2011 – 31/3/2011 – related to last year’s prepaid ` 20,000
1/4/2011 – 30/9/2011 – 6 months quarter 1+2 ` 40,000
1/10/2011 – 31/12/2011 – prepaid for next quarter ` 20,000
CHAPTER 24
AS-26:
Intangible Assets

Chapter Outline
vv Objective and Scope vv Amortization Treatment
vv What is an Intangible Asset vv Treatment of Sale/Disposal of Intangible Asset
vv Recognition Principle of Intangible Asset vv Disclosures

Objective and Scope


The objective of this standard is to prescribe the recognition principles of intangible assets. But such intangible
assets can only be recorded when they meet asset recognition criteria. In addition to the recognition principles
of intangible assets the standard also discusses amortization and disclosure aspects.
AS – 26 is applicable to the following cases:
1) Copy writes, 2) Patents, 3) Technical know – how, 4) Plays, 5) manuscripts, 6) Designs, 7) Stock
exchange card holders (right to trade + right to become a shareholder), 8) Import quotas, 9) lease hold rights,
10) Marketing rights, 11) Purchased magazines titles, 12) R & D expenses, 13) Franchises rights or any other
intangible assets which meets the conditions for intangible assets.
The Standard does not apply to those intangible assets covered by other Standards, such as
◘◘ Intangible assets held for sale in the ordinary course of business (AS2/IAS 2)
◘◘ Deferred tax assets (AS 22/IAS 12)
◘◘ Leases within the scope of (AS 19/IAS 17)
◘◘ Assets arising from employee benefit plans (AS 15/IAS 19)
◘◘ Financial assets covered by (AS 21,23,27,30, 31, 32/IAS 39, IAS 27, IAS 28, or IAS 31)
◘◘ Goodwill acquired in a business combination (AS 14/IFRS 3)
◘◘ Deferred acquisition costs and intangible assets arising from insurance contracts (IFRS 4)
◘◘ Noncurrent intangible assets classified as held for sale in accordance with (AS-24/IFRS 5).

DO YOU KNOW
With the introduction of AS-26 the concept of deferral (Deferred Revenue Expenditure) has been abolished…..
para 56.
244 Accounting Standards

What is an Intangible Asset


As per AS – 26 intangible assets is one which satisfies all the following conditions:
i) It should be an asset, ii) It should be a non – monetary asset, iii) Identifiable, iv) Without physical substance, v) Held for
used in production, administrative purposes, fetching rent etc.

i) Asset
An asset is an economic resource, it should be controlled by an enterprise, as a result of some past event, from
which economic benefits should flow to the enterprise.
◘◘ An economic resource - Asset should be an economic resource which gives some economic benefit.
◘◘ It should be controlled by an enterprise – Control is said to be placed only when the economic gains
flow only to the enterprise and not to others. The enterprise can restrict the access to others. There
are various ways in which case control can be exercised – by legal rights, by using passwords, or by a
legal duty on employees to maintain confidentiality.
◘◘ As a result of some past event – the asset may be either purchased or generated i.e. a past event.
◘◘ From which economic benefits should flow to the enterprise – The company should be able to reap
economic benefits from the assets. Benefits include proceeds from sale of goods, cost savings ex: use
of intellectual property reduces cost of process.

ii) Non-Monetary Asset


Monetary assets are those assets whose value is fixed and determinable For example: Cash balance, Debtors.
Non – Monetary assets are those assets which are other than monetary assets. For example: Plant and
machinery, Short term investments etc. Debtors are assets without physical substance, yet being monetary
assets, the trade debtors are not regarded as intangible assets. Intangible Asset should be Non-monetary
asset.

iii) Identifiable
Generally an asset is said to be identifiable if it’s separable from other assets and if enterprise could rent, sell,
exchange or distribute the specific future benefits of this assets used in revenue earning activity. For example:
If a company has got 3 brands of soap and if the company sells one of the brands without selling the business
then such sale of brand name is identifiable.
However separability is not necessary condition for identifiability, since an enterprise may be able to
identify an asset in some other way. Even if the asset is able to generate income in combination with other
assets it is enough to identify such asset as an intangible asset. For example: Where an enterprise acquires a
travel agent business and apart from paying consideration for the net assets of the business, pays ` 1 crore as
non – competing fees. Such non – compete fees can be said to be identifiable.

iv) Without physical substance


Intangible assets have no physical substance, however some intangible assets may have physical substance
but the value of tangible asset is much less than the value of intangible asset. For example: Motion picture in
a CD, legal documentation in case of Patents. Books with highly technical content can be said to be intangible.

v) It is held for used in production, administrative purposes, fetching rent


If intangible assets are held as investments then AS - 13 is applicable. If an intangible asset is kept for resale
then it is covered by AS - 2. Intangible assets should either be used for production or for administrative
purpose. Example for administrative purpose may be computer web –site used by staff for administrative
work.
AS-26: Intangible Assets 245

DO YOU KNOW
Unlike Old Schedule VI, the New Schedule III has a speical classification for Intangible Asset under the Sub –
Head Non – Current Asset as a Fixed Asset.

Recognition Principle of Intangible Asset


As per AS - 26 intangible asset should be recognised if and only if:
•  It is probable that the future economic benefits from the asset flows to the enterprise,
•  Cost of the intangible asset can be measured reliably.

An intangible asset should be initially measured at Cost.


There are two ways in which intangible asset can come into existence:
i) Purchased one, ii) internally generated I.A.

Purchased Intangible Asset


Intangible asset can be acquired by various means:
(i) Acquisition by paying cash: Costs consists of purchase price, legal fees, payment of duties and non-
refundable taxes and other expenses directly attributable to the acquisition of intangible asset.
(ii) Exchange of asset: Fair value of the asset given up if it more evident.
(iii) Issue of shares and other securities: Fair value of shares and securities issued.
(iv) Intangibles in case of amalgamation (a special case):
ww In case of amalgamation, if the purchase consideration is determined/paid by net payments
method – and amalgamation is in the nature of purchase the intangible asset (ex.: patents, rights
& not goodwill) has to be recorded at fair – value if such intangible asset meets the recognition
criteria.
ww If the fair – value if such intangible asset cannot meets the recognition criteria then it is to be
included in the goodwill arising on amalgamation.
ww If there is no active market for an intangible asset acquired in an amalgamation in the nature
of purchase, the cost initially recognized for the intangible asset is restricted to an amount that
does not create or increase any capital reserve arising at the date of the amalgamation.
(v) Acquisition of intangible asset through government grants: Sometimes rights are received through
government as a grant. For example Right to operate Radio station. In such case one has to record
such IA at concessional price and if free of cost then nominal value.

Internally Generated Intangible Asset


(i) Internally generated goodwill: Internally generated goodwill is generated within the business slowly
during its lifetime. For example: Reliance has created an internal goodwill of billions and billions
of Rupees which cannot be estimated. As cost of such goodwill cannot be determined hence self-
generated goodwill is not recognized in the books of accounts…….. para 35.
(ii) Cost of internally generated brands names/publishing titles/mastheads: As the cost cannot be
measured reliably, the self-generated brands/publishing titles etc. are not recognized in the financial
statements/books of accounts. This is because such publishing titles etc cannot be separated from
the cost of separating the business.
(iii) Cost of internally generated intangible assets other than referred to (i) & (ii) above: There may be
other intangible assets like computer software created or any formula invented. But all expenses
246 Accounting Standards

incurred towards generating such intangible assets cannot be capitalized. Cost incurred is divided
into two phases:
(a) Research phase: Research Phase means and includes expenses incurred to create or to invent
some new product. Research expense should not be recognized as cost because the expenses
cannot be translated into future economic benefits. As per AS – 26 expenses incurred during the
Research Phase should be expensed.
(b) Development phase: It is the activity which converts the research product into a marketable
commodity. Development cost can be capitalized if the product meets asset criteria. Otherwise
such costs are also to be expensed. The company should be able to demonstrate that the expenses
incurred towards development phase as an asset.
Expenditure on research (or the research phase of an internal project) is to be written off as an expense as
and when incurred, as it is not possible to demonstrate that an asset exists that will generate future economic
benefit. Examples include
◘◘ Activities aimed at obtaining new knowledge
◘◘ The search for, evaluation, and selection of applications of research findings or knowledge
◘◘ The search for alternatives for materials, devices, products, systems, or processes
◘◘ The formulation, design, evaluation, and selection of possible alternatives for new or improved mate-
rials, devices, products, systems, or processes
Development expenditure may be recognized as an intangible asset when, and only when, all of the
following can be demonstrated: (Para 44)
◘◘ The technical feasibility of completing the asset so that it will be available for use or sale
◘◘ The intention to complete the asset and use or sell it
◘◘ The ability to use or sell the asset
◘◘ How the asset will generate probable future economic benefit, including demonstrating a market for
the asset’s output, or for the asset itself, or the asset’s usefulness
◘◘ The availability of sufficient technical, financial, and other resources to complete the development and
to use or sell the asset
◘◘ The ability to reliably measure the expenditure attributable to the asset during its development.

Determination of Cost of Internally Generated Intangible Asset


It includes all the expenses incurred during development phase.
Cost of materials and services used or consumed
+ Salaries, wages and other employment related cost of personnel engaged in the developing the intangible
asset
+ Any expenditure directly attributable to the generation of intangible assets
+ Overheads that are necessary for the generation of the intangible asset and that can be apportioned on
reasonable basis
Costs not to be included
a. Selling, administrative and other general overheads not directly attributable
b. Inefficiencies and initial operating losses and other cost before the intangible asset at the recognition
criteria
c. Expenditure on staff training.

Accounting for Subsequent Expenditure in Case of Intangible Asset


Subsequent expenses on intangible assets are recognized as an expense in the same period in which it is
incurred.
AS-26: Intangible Assets 247

However, the expenses may be capitalized if,


◘◘ Subsequent expenses increases the future economic benefits
◘◘ Such expenses can be attributable to the intangible assets
◘◘ Such expenses can be measured reliably
Subsequent expenses on internally generated goodwill brands etc.
Subsequent expenditure on brands, mastheads, publishing titles, customer lists and items similar in
substance (whether externally purchased or internally generated) is always recognized as an expense to
avoid the recognition of internally generated Goodwill/Intangible Asset.
Illustrations on Internally Generated Intangible Assets:
(i) A pharmaceutical entity is developing a vaccine for HIV that has successfully completed Phases I and
II of clinical testing. The drug is now in Phase III of clinical testing. Management still has significant
concerns about securing regulatory approval and has not started manufacturing or marketing the
vaccine. Should management start capitalising development costs at this point?
Solution: As technical feasibility of the project is not proven the management should not capitalise the costs.
(ii) A pharmaceutical entity is developing a vaccine for HIV that has successfully completed Phases I
and II of clinical testing. The drug is now in the late stages of Phase III testing. It is structurally
similar to drugs the entity has successfully developed in the past with very low levels of side effects,
and management believes it will be favourably treated by the regulatory authority because it meets
currently unmet clinical need. The entity has also started producing inventory.
Solution: Yes the management should capitalise the development costs because the project has met the
criteria as stated in para 44 of AS-26.
(iii) A pharmaceutical entity has obtained scientific regulatory approval for a new respiratory drug in
Country Japan. It is now progressing through the additional development procedures and clinical
trials necessary to gain approval in Country India. Management believes that achieving regulatory
approval in this secondary market is a formality. Mutual recognition treaties and past experience
show that India’s authorities rarely refuse approval for a new drug that has been approved in Japan.
Solution: Yes the management should capitalise the development costs because the registration is highly
probable and there are likely to be low barriers to obtain regulatory approvals.
(iv) A pharmaceutical entity is developing a generic version of a painkiller that has been sold in the
market by another company for many years. The technical feasibility of the asset has already been
established because it is a generic version of a product that has already been approved, and its
chemical equivalence has been demonstrated. The lawyers advising the entity do not anticipate that
any significant difficulties will delay the process of obtaining commercial regulatory approval. (The
scenario assumes that the other conditions in AS-26 para 44 can be satisfied.)
Solution: Yes the management should capitalise the development costs because the commercial regulatory
approval will be received. If as per the judgement uncertainty prevails about the commercial success of the
product then the development costs is to be expensed.
(v) Pharmaceutical entity Glaxo Ltd has obtained scientific regulatory approval for a new respiratory
drug and is now incurring expenditure to educate its sales force and perform market research.
Solution: Glaxo Ltd should expense sales and marketing costs (training expenses, performing market
research). This type of expenditure does not create or prepare the asset for any intended use. Expenditure of
training, S & D should not be capitalised............para 54 of AS-26.
248 Accounting Standards

Amortization Treatment
i) Amortization
It is the process of allocating cost of intangible asset over its useful life. Depreciable amount of intangible
asset should be allocated on a systematic basis over the best estimates of its useful life.

ii) Depreciable amount


Depreciation amount = (Cost of intangible asset less its (estimated) residual value)/Useful life of the asset.

iii) Useful life


In the absence of any evidence, the useful life is assumed to be 10 years.
If any shorter/longer life is estimated, it can be amortized over such shorter/longer period. If life is
estimated more/less than ten years, then enterprise must justify such longer/shorter period. When legal
rights protects intangible asset, the useful life is the period for which such legal rights is available.

iv) Method of amortization


Amortization method:
◘◘ If the enterprise can reasonably estimate the pattern of revenues or benefits, the cost of intangible
should be amortized on the same pattern.
◘◘ If it is not possible to estimate such pattern, then straight-line method [SLM] is to be followed.
Amortization period:
◘◘ As far as the period for amortization is concerned the life of intangible is based on the best estimated
useful life of the asset. Sometimes control over the future economic benefits can be achieved through
legal rights for a finite period.
◘◘ If it is not possible to estimate such useful life or asset is not acquired under legal right then 10 years
can be taken as the period of amortization.
◘◘ Usually amortized amount is recognized as an expense and debited to the statement of profit and loss.
Sometimes the intangible asset may be used for producing another asset, in such case; amortization
amount should be added to the cost of such another asset.
Example for the pattern of cash flow for amortization:
Acquired distribution rights comprising home video, internet rights are capitalized as intangible assets.
These rights may be amortised over a period in the ratio of 75%, 15% and 10%. The accelerated rate in the
first year represents the market situation where the market for these products diminishes rapidly after the
initial period of release.
Some examples of amortization:
(1) Stock Exchange Cards: In this case the right to trade at BSE/NSE is for an indefinite period hence a
longer period (more than 10 years) is appropriate for amortization.
(2) Mining rights: Amortization will be as a percentage of resources get exhausted from the mine.
(3) Music rights: Significant portion of cost of music rights should be amortized within a short period of
time and balance over a period of time.

v) Residual value
Residual value is usually taken as zero. Except when active market exists for intangible assets or third party
is willing to buy at the end of its economic life.

vi) Change in the amortization method


AS-26: Intangible Assets 249

The amount to be amortized may change due to change in expected useful life of intangible asset, change in
the pattern of revenues/benefits.

vii) Carrying amount


In financial statements, intangible asset should be carried at its cost less accumulated amortization less
impairment losses.

Treatment of Sale/Disposal of Intangible Asset


When the intangible asset is disposed off or no future economic benefits are expected from the use of
intangible asset it will be deleted from the financial statements. Intangible asset which is retired from active
use has to be written – off. Gains or losses arising on disposal are generally recognized in the statement of
profit and loss in the same year in which asset is disposed off.

Disclosures
The financial statements should disclose the following for each class of intangible assets:
1. The useful lives or the amortization rates used
2. The amortization methods used
3. The gross carrying amount and the accumulated amortization (aggregated with accumulated
impairment losses) at the beginning and end of the period.
4. A reconciliation of the carrying amount at the beginning and end of the period showing:
(i) Additions, indicating separately those from internal development and through amalgamation
(ii) Retirements and disposals
(iii) Impairment losses recognized in the statement of profit and loss during the period, if any
(iv) Impairment losses reversed in the statement of profit and loss during the period, if any
(v) Amortization recognized during the period
(vi) Other change in the carrying amount during the period.

Increase Your Knowledge


Asset/Expense Intangible Asset
Self generated marketing rights Not
Self generated franchise rights Not
Purchased marketing rights Yes
Special Computer Software Yes
Advance license Fees Yes
Duty drawback Yes
Sales Tax exemption Not
Share issue expenses Not
Internally generated brands Not
Purchased brands Yes
Right to use land Yes
Feasibility study and start up costs Not
250 Accounting Standards

PROBLEMS AND SOLUTIONS

Problem 1: Rotex Pharma comes up with a formulation that destroys pests immediately. It is in the form
of a gel/paste, which is highly effective. The company does not have a patent for this product but it keeps
the ingredients a secret. These ingredients are not printed on the package of the product. Can the company
regard the knowhow as an intangible asset?
Solution: Yes the know – how is an intangible asset because the company has got control on the use of
intangible asset. The company can control the future economic benefits from this know-how even without
taking the patent because the ingredients of the content of the gel/paste are not printed.
Problem 2: An enterprise has a portfolio of customers and a market share and expects that, due to its efforts
in building customer relationships and loyalty, the customers will continue to trade with the enterprise. Can it
recognise portfolio customers, market shares, customer relation-ships, customer loyalty as intangible assets?
Solution: Market share, customers lists are not intangible assets because a company cannot control the
future economic benefits arising from the loyalty of customers, market share. There are enough competitors
who can share the market economic benefits. And no one can have any control on the geographical area
(market share).
Problem 3: Logifirms Pvt. Limited is developing a new production process. During the year 2006,
expenditure incurred was ` 10 lakhs, of which ` 9 lakhs was incurred before 1st December, 2006 and 1 lakh
was incurred between 1st December, 2006 and 31st December, 2006. The enterprise is able to demonstrate
that, at 1st December, 2006, the production process met the criteria for recognition as an intangible asset.
Comment? (C.A. Final May 2008)
Solution: As per AS-26 expenses incurred for R & D has to be expensed. And expenses incurred at the
development phase should be capitalised only if one is able to demonstrate that the development stage has
met the criteria for recognition of an intangible asset. In the present case only ` 1,00,000 can be capitalised
as costs of intangible and ` 9,00,000 has to be expensed.
Problem 4: Cipla Ltd. has a patent for a particular product. A case has been filed against the company
by another party claiming that it is the rightful owner to that patent and that the company has copied and
infringed its patent. The company has incurred heavy legal expenditure to the tune of ` 15 lakhs to defend the
suit the company eventually won the suit. The company has capitalized this expenditure as part of the cost of
the patent. Do you agree with this accounting treatment?
Solution: As per AS-26 subsequent expenditure incurred on intangible asset can be capitalised only when
such expenditure increases the efficiency or life of the asset. In the present case the company has incurred
` 15,00,000 to defend the suit and not to improve the earning capacity of the asset. Hence ` 15,00,000 has to
be expensed immediately.
Problem 5: An enterprise has purchased an exclusive right to generate hydro-electric power for 40 years.
The costs of generating hydro-electric power are much lower than the costs of obtaining power from
alternative sources. It is expected that the geographical area surrounding the power station will demand a
significant amount of power from the power station for at least 40 years.
Solution: As per AS-26 the life of the intangible asset is to be assumed as 10 years. But any longer or shorter
life can be justified. In the present case the company the life of the asset is taken as 60 years because there
is significant demand in the next 40 years for the electricity. Hence, amortization of cost of right to generate
electricity is to be taken as 40 years.
P roblem 6: A company with a turnover of ` 250 crores and an annual advertising budget of ` 2 crore had
taken up the marketing of a new product. It was estimated that the company would have a turnover of ` 25
AS-26: Intangible Assets 251

crores from the new product. The company had debited to its Profit and Loss account the total expenditure of
` 2 crore incurred on extensive special initial advertisement campaign for the new product. Is the procedure
adopted by the company correct? (C.A. Final Nov 2003 & Nov. 2006) (4 Marks)
Solution: Advertisement expense is sales promotion expense. Such expenditure should not be capitalized
as an be capitalized as cost of asset as per AS-26. Sales expenses are to be excluded from costs of Intangible
Assets. Advertisement expense may be deferred or expensed but cannot be capitalized. But even the concept
of deferral (deferred revenue exp.) has been abolished now …..para 56.
Problem 7: B Pvt. Ltd., implements a Voluntary Retirement
st
Scheme (VRS) for its employees. It follows a
policy of amortising the expenditure over 10 years. As at 1 April, 2004, the unamortised VRS expenditure
was ` 15 lakhs. During the year 2004-05, it incurred further ` 12 lakhs as VRS. For the year ended 31st March,
2005, the company proposes to revise the period of amortisation to 5 years. It also proposes to follow the
revised period for the opening balance. (C.A. Final May 2005) (5 Marks)
Solution:
Expenditure on Voluntary Retirement Scheme: Termination benefits are employee benefits payable as a
result of either an enterprises’ decision to terminate an employee’s employment before the normal retirement
date; or an employee’s decision to accept voluntary redundancy in exchange for those benefits (voluntary
retirement). Accordingly, Voluntary Retirement Scheme (VRS) is one form of termination benefit and hence
AS 26 principles are not applicable. The same has to be, therefore, accounted as per AS-15 (Rev). But both
standards i.e. AS-26 and AS-15 (Rev) now wants VRS expenditure should be written off immediately. When
to record VRS expense is governed by AS-29.
Problem 8: Camlin Ltd. acquired assets and liabilities of Carlin Ltd. for a consideration of ` 6,00,000,
discharged in the form of 40,000 equity share ` 10 each at ` 15 per share. Following is the carrying amount
and fair value business Assets & Liabilities is as below:
Assets/Liabilities Carrying amount Fair Value
(1) Fixed Assets 8,97,000 8,90,000
(2) Current Assets 2,95,000 2,80,000
(3) Loan 5,50,000 5,50,000
(4) Creditors 50,000 50,000
Carlin Ltd. has a Patent right. The patent was not recognised because the company did not have the
resources necessary to make use of it. Record Amalgamation entries in the books of Camlin Ltd. in each of the
following 4 cases:
(A) Patent’s Fair Value = ` 25,000
(B) Fair Value of patent is not determinable.
(C) Fair Value patent is 35,000 but no active market for such patent exist.
(D) Neither Fair Value is determined nor active market exists.
Solution:
As it is a case of amalgamation PC = 40,000 × 15 = ` 6,00,000
Case A: In case of amalgamation, if the P.C. is determined/paid by net payments method – and amalgamation
is in the nature of purchase the intangible asset (ex.: patents, rights & not goodwill) has to be recorded at
fair – value if such intangible asset meets the recognition criteria.
252 Accounting Standards

Journals:
(1) Business Purchase Dr. 6,00,000
To Liquidator of Carlin Ltd. 6,00,000
(2) Fixed Assets Dr. 8,90,000
Current Assets Dr. 2,80,000
Patents Dr. 25,000
Goodwill Dr. (b.f.) 5,000
To Loan 5,50,000
To Creditors 50,000
To Business Purchase 6,00,000
(3) Liquidator of Carlin Ltd. Dr. 6,00,000
To Equity Share Capital 4,00,000
To Securities Premium 2,00,000
Case B: If the fair – value of the intangible asset cannot meet the recognition criteria then it is to be included
in the goodwill arising on amalgamation.
Journals: (Entry no. 1 and 3 remains same)
In Entry no 2 Goodwill be the balancing figure i.e. ` 30,000 and patent is a part of G/W.
Case C: If there is no active market for an intangible asset acquired in an amalgamation in the nature of
purchase, the cost initially recognized for the intangible asset is restricted to an amount that does not create
or increase any capital reserve arising at the date of the amalgamation.
Journals: (Entry no. 1 and 3 remains same)
In Entry no 2 Patent will be recorded maximum ` 30,000 to the extent it does not create Capital Reserves.
Case D: No patent will exist at all.
Problem 9: Infopark Ltd. acquired for its internal use a software on 28.01.2012 from USA for US $1,00,000.
The exchange rate on that date was ` 50 per US $. The seller allowed trade discount of 5%. The other expenditure
were: (i) Import duty : 20%, (ii) Purchase tax : 10%, (iii) Entry tax 5% (refundable), (iv) Installation expenses:
` 25,000, (v) Professional fees for Clearance from Customs: ` 20,000. Compute the cost of software as per
AS-26. (C.A. Final RTP and IPCC Nov 2012 Marks 4)
Solution:
Purchase cost $1,00,000
(–) Trade discount ($5,000)
95,000
Rupee equivalent value ($ 95,000 × ` 52) 47,50,000
(+) Import duty @20% 9,50,000
57,00,000
(+) Purchase tax 10% 5,70,000
(+) Installation expenses 25,000
(+) Professional fees for clearence 20,000
Cost of Software Capitalised (`) 63,15,000

Note: Refundable taxes are not considered as cost.


Problem 10: Swift Ltd. acquired a patent at a cost of ` 80,00,000 for a period of 5 years and the product
life-cycle is also 5 years. The company capitalised the cost and started amortizing the asset at ` 10,00,000
per annum. After two years it was found that the product life-cycle may continue for another 5 years from
AS-26: Intangible Assets 253

then. The net cash flows from the product during these 5 years were expected to be ` 36,00,000, ` 46,00,000,
` 44,00,000 ` 40,00,000 and ` 34,00,000. Find out the amortization cost of the patent for each of the years.
(C.A. Final May 2006) (4 Marks)
Solution:
The whole problem is divided into three parts:
(i) Transitional provisions
(ii) Amortization period
(iii) Amortization method
(i) Transitional provisions:
Position of patents at the end of second year:
Total Cost amortized (as per the companies policy) = 10,00,000 × 2 = ` 20,00,000
Total Cost to be amortized (as per AS-26) = 16,00,000 × 2 = ` 32,00,000
Transitional adjustment = ` 12,00,000
Opening Reserves A/c. Dr. 12,00,000
To Cost of Patents 12,00,000
(ii) Amortization period: It is clearly given in the problem that the revised life of the intangible asset is yet
another 5 years. This means the costs of patents will be written off in the next 5 years.
(iii) Amortization method: If the enterprise can reasonably estimate the pattern of revenues or benefits, the
cost of intangible should be amortized on the same pattern.
Following is the apportionment of Costs of Patents:

Year % (Pattern) Amortization


1 18% ` 8,64,000
2 23% ` 11,04,000
3 22% ` 10,56,000
4 20% ` 9,60,000
5 17% ` 8,16,000
100%

Problem 11: Pfizer is developing a hepatitis vaccine compound. Sun Pharma is developing a measles
vaccine compound. Pfizer and Sun enter into an agreement to swap these two products. Under the terms
of the agreement, Pfizer will retain the marketing rights to its drug for all Asian countries. The fair value of
Pfizer’s compound has been assessed as 3 million, including 0.2 million relating to the Asian marketing rights.
The carrying value of the compound is 0.5 million. Pass the journal Entry for exchange of intangible assets.
Solution:
Pfizer’s management should recognise the compound received at the far value of the compound given up.
Even gain has to be recorded on sale of hepatitis vaccine compound.
Journal Entry:
Rights for Measles Vaccine Compound Dr 2.8 million
To Rights for Hepatitis Vaccine Compound a/c 0.5 million
To Profit on sale a/c 2.3 million
Profit on sale = 2.3 million = (2.8 – [0.2/3] x 0.5)
254 Accounting Standards

Problem 12: Carnival Limited pays ` 5,00,000 as club membership for 12 years membership. The
membership fees is non-refundable. Please advise for the accounting treatment of Club membership Fees?
Solution: Club membership fees is an intangible asset. Benefits will be received over the 12 years period.
Also the fees paid is not refundable hence the cost of intangible asset can be written off over the period of
12 years either by any pattern of services ratio or SLM basis.
Problem 13: Sun Pharma Limited is engaged in research on a new process design for its product. It had
incurred an expenditure of ` 530 lakhs on research upto 31st March, 2015.
The development of the process began on 1st April, 2015 and Development phase expenditure was ` 360
lakhs up to 31st March, 2016 which meets assets recognition criteria.
From 1st April, 2016, the company will implement the new process design which will result in after tax
saving of ` 80 lakhs per annum for the next five years.
The cost of capital is 10%
Explain:
(i) Accounting treatment for research expenses.
(ii) The cost of internally generated intangible asset as per AS 26.
(iii) The amount of amortization of the assets (The present value of annuity factor of ` 1 for 5 years
@ 10% = 3.7908)
Solution:
Research expenses: As per AS-26 the expenditure on research of new process design for its products
530 lakhs is to be charged to P/L accounts. ………..para 41.
Cost of internally generated intangible asset: As per the question the development costs meet the asset
recognition criteria on 31/3/2016. As per AS-26 for measurement purpose such internally generated
intangible asset can be capitalized by discounting the cash flows:
PV of savings after tax = 80 lakhs × 3.7908 = ` 303.26 lakhs; But the cost incurred on development phase
is ` 360 lakhs. Therefore the difference of (360 – 303.26) = ` 56.74 lakhs will be charged to P/L account. After
capitalizing the cost of asset ` 303.26, the company has to amortize this cost for the next 5 years. Annual
amortization will be ` 60.65 lakhs.
CHAPTER 25
AS-28:
Impairment of Assets

Chapter Outline
vv Scope and Objective vv Corporate Assets
vv Meaning of Impairment vv Reversal of Impairment Loss
vv Meaning of Some Terms Relating to Impairment vv Reversal of Impairment Loss of an Individual Asset
vv Impairment Loss vv Reversal of Impairment Loss of a CGU
vv Symptoms of Impairment of Assets vv Transitional Provisions
vv Impairment of Individual Asset vv Disclosures
vv Impairment of Cash Generating Unit

Scope and Objective


Indian Companies have built up huge capacities over the period of time, though the actual capacity is less
ex: Capitalising pre-operative expenses costs instead of charging to P & L etc. This is nothing but WINDOW
DRESSING adopted by the companies. And of course Accounting Standards 1-29 has been introduced to
show true and fair and transparent position of financial accounts. So as to remove such and other types of
overvaluation of Assets especially AS-28 has been inserted with effect from 01.04.2005.
Capital Intensive companies be alert. Specially, Textiles industries, Automobiles companies, will become
the victim of AS-28. These are generally capital-intensive companies. But AS-28 has less relevance to the
companies using light capital doses (Service offering companies) because most of them do not hold significant
assets.

Scope
Impairment of Assets applies to which type of Assets?
Impairment of Assets specially applies to “Operating Fixed Assets”
Fixed Assets can be “Tangible/Intangible Assets”
Fixed assets can be carried at historical cost/Revalued figure

Exclusions: Certain assets are not covered by the Standard, including


• Inventories (AS 2/IAS 2)
• Assets arising from construction contracts (AS 7/IAS 11)
• Deferred tax assets (AS 22/IAS 12)
• Assets arising from employee benefits (AS 15/IAS 19)
256 Accounting Standards

• Financial assets (AS 30, 31, 32/IAS 39)


• Investment property carried at fair value (No corresponding AS/IAS 40)
• Biological assets carried at fair value (No corresponding AS/IAS 41)
• Assets arising from insurance contracts (No corresponding AS/IFRS 4)
• Assets that are held for sale (AS 24/IFRS 5)

DO YOU KNOW
Intangibles gets impaired faster as compared to Tangible assets during recession.

Meaning of Impairment
Impairment of Assets means decrease in the carrying amount of the assets. It is provided only when carrying
value of asset exceeds recoverable amount. It is debited to P & L Account as a normal expense. It is a provision
of loss in addition to normal depreciation. However, one should not get confused between Depreciation and
Impairment. Following is the differentiation:

Points Depreciation Impairment


(i) Why? It is due to wear and tear or passage of time or It is due to carrying value of asset exceeds its
obsolescence. recoverable amount.
(ii) Sequence For every newly purchased asset depreciation One cannot charge impairment on newly
is charged first and not impairment. acquired asset. Thus, Impairment follows
Depreciation.
(iii) Methods SLM, WDV, Sinking Fund etc. No different methods yet because it is a newly
born baby.
(iv) Governed by AS- 6 AS – 28
(v) Reversal Reversal of Depreciation provision is not Impairment can be reversed.
allowed
(vi) No. of Assets Depreciation is charged on single asset Impairment is charged on a single or group of
assets.
(vii) Type of loss Its like a Normal loss Its like an Abnormal loss
(viii) Taxability Depreciation is a tax deductible expenditure It is not a tax-deductible expense.

Meaning of Some Terms Relating to Impairment


(a) Carrying Amount: It is the amount at which the asset is carried in the Balance Sheet.
Thus, before AS-28
Carrying Amount = Cost – Depreciation or Amortization

But, after AS-28


Carrying Amount = Cost – Depreciation – Amortization – Impairment

◘◘ Carrying Amount & Liability attached to an Asset:


If a customer takes over a liability attached to an asset then carrying amount will be notionally
adjusted.
◘◘ Recoverable Amount: It is higher of {Net Selling Price OR Value in use}
Recoverable Amount = Higher of (NSP or VIU)
AS-28: Impairment of Assets 257

◘◘ Net Selling Price (NSP): NSP is the amount obtained from selling an asset less its costs of disposal. It
excludes finance cost and income tax. Examples of costs of disposal are legal costs, costs of removal of
assets. Termination benefits related to reorganizing a business following the disposal of asset are not
direct costs related to the disposal of assets, hence not to be included.
IAS 36 sets out how an entity should determine the fair value less costs to sell. The Standard sets out
these examples:
• Where there is a buying and selling agreement, the price in that agreement less the costs to sell can
be used.
• The price in an active market less the cost of disposal can be used.
• The fair value less costs to sell can be based on the best information available which reflects the
proceeds that could be obtained from the disposal of the asset in an arm’s-length transaction.
• As per AS-28/IAS-36 best evidence is the price in a binding sale agreement in an arm’s length
transaction adjusted for the costs of disposal
NSP = Sale Value – Cost of disposal

◘◘ Value in Use (VIU): Value in use is the sum of the present value of future cash flows from continuing
use of the asset and its residual value at the end of its useful life.
VIU = PV of Assets (Future Cash flows + Residual value)

Following points need consideration for determining VIU:

Composition of future Cash Flows


Includes what?
◘◘ Cash inflows from continuing use of assets.
◘◘ Cash outflows to generate cash inflows.
◘◘ Cash flows from Foreign currency transactions.
◘◘ Cash inflows from disposal of asset.
◘◘ Cash outflows on account of costs of disposal.
Excludes what?
◘◘ Cash inflows/outflows from financing activities.
◘◘ Income tax – Receipts and Payments.
◘◘ Additions made to the concerned asset.
◘◘ Cash inflows from other assets whose inflows are independent of inflows of asset under review.
◘◘ Cash outflows of already recorded liabilities like pensions, provisions.
◘◘ Benefits from future restructuring plans (Cash flows should be calculated for the asset in the current
condition).

How to measure cash flows


• Cash flows should be based on the best estimates made by the management.
• Greater weight should be given to the external information
• Cash flow projections should be based upon the most recent financial budgets/forecasts that have
been approved by the management.
• Use of exploration (declining or accelerating growth) should be considered for cash flows beyond
the period of budgets.
258 Accounting Standards

Period to be reckoned for the cash flows


Projections should be based on the most recent budgets/forecasts. Generally cash flows of 5 years are
considered unless any longer period is justified.
Discount Rate: For determining Value in Use discount rate plays an important role. Discount rate is same as
Cut-off rate as applicable to Capital Budgeting Decisions. Discount rate should:
Be a Pre – Tax rate.
Reflect current market assessment of time value of money and the risks specific to the asset.
Reflect risks attached to that particular asset.
Weighted Average Cost of Capital or Incremental borrowing rate or required @ of return applied to
that specific asset.
Consideration is given to country risk, currency risks, price risks, and cash flow risks.
Is it necessary to determine both VIU and NSP:
If NSP exceeds Carrying amount of the asset, then it is enough, i.e. VIU need not be calculated.

Impairment Loss
Asset is said to be impaired when recoverable amount of the asset is less than its carrying amount. It means
that if recoverable amount of the asset is more than its carrying amount then there is no impairment.
Impairment loss = Carrying Amount – Recoverable Amount

Accounting Entry for Impairment Loss:


Impairment loss is a nominal account. It is debited to P & L Account as a normal expense
Impairment loss A/c Dr. xxx
To Asset A/c xxx
Profit and loss A/c Dr. xxx
To Impairment loss xxx
Effects of Impairment Loss: If an Asset is impaired following are the effects:
(1) Profit and loss will be debited by an amount equal to Impairment loss.
(2) Creation of Deferred Tax Asset.
(3) Future deprecation amount per year will be prospectively adjusted.
(4) Carrying Amount of the Asset is to be adjusted by the amount of Impairment loss.

DO YOU KNOW
Impairment loss is a timing difference as per AS-22.

Symptoms of Impairment of Assets


(a) External factors: * Market Value of the asset may have declined due to efflux of time or due to normal
use. For example: Invention of Colour T.V. has made Black & White almost extinct.
ww Changes in technology, market conditions, legal restrictions. Taxing highly or Restricting
GUTKHA and Tobacco within some cities have impaired the concerned companies.
ww Increase in rates of return to investors, which increases the discount rate. And if discount rate
increases net cash inflows available with the company will be less.
ww Carrying amount of net assets is more than the market capitalization.
AS-28: Impairment of Assets 259

◘◘ Internal factors: *Obsolescence or physical damage of an asset. For example: Devastating Fire damages
the major portion of the assets.
ww The asset is not used in the manner it is expected to be used. For example: Benches and desks
used in schools gets impaired fast than used in Offices and Halls.
ww Continued Cash losses or Budgeted losses etc.

DO YOU KNOW
Impairment loss is an exceptional item and not extra-ordinary item for AS-5

Impairment of Individual Asset


◘◘ If Asset is carried at historical cost: The Loss is debited to P & L Account as any other normal expense.
◘◘ If Asset is carried at revalued figure: As far as calculation of Impairment loss is concerned the formula
remains same as discussed above. But the treatment differs in the following sense.
If impairment loss = Revaluation Reserve, then Impairment loss to be written off against Revaluation
Reserve.
If impairment loss > Revaluation Reserve, then Impairment loss to be written off against Revaluation
Reserve to make Revaluation Reserve nil and the balance impairment loss is to be debited to profit
and loss account.

Impairment of Cash Generating Unit


◘◘ Meaning of Cash Generating Unit: Sometimes it becomes difficult to estimate NSP or VIU of a single
asset. Can we calculate Recoverable Amount of individual asset?
The answer to the above question is NO. We cannot calculate Recoverable Amount of a single asset
separately. In such a case, we have to see the group to which the asset belongs. Such a “group of
assets” in totality is called Cash Generating Asset (CGU).
“Cash Generating Asset is defined as the smallest group of assets that generates cash inflows from
continuing use that are likely to be independent of the cash inflows from other assets or group of assets”.
Features of Cash Generating Unit:
• CGU is the smallest unit or segment of assets.
• It generates income independent of other assets.
• A single asset as such has no identity unless it takes shelter under CGU.
• Assets forming part of CGU should “come under one umbrella” for computing recoverable amount.
• CGU may be comprised of a single asset or a group of assets.
• There may be as many as CGU’s as many group of assets.
• CGU may include direct assets and even an allocable portion of asset/assets.
• CGU may be a product line/a segment/a plant/a unit/a business operation etc.
Example: A manufacturing entity owns several vehicles. The vehicles are several years old and could be sold
only for scrap value. They do not generate cash independently from the entity. How will the recoverable value
of the vehicles be determined?
Solution:
• The entity cannot estimate the recoverable amount of the vehicles because their value-in-use
cannot be determined separately, and it will be different from the scrap value. Therefore, the entity
260 Accounting Standards

would incorporate the vehicles into the cash-generating unit to which they belong and estimate the
recoverable amount of that cash-generating unit.
Example: Cosy and comforts is in the hotel business. They have installed Sony LED in each room for elite
customers. The initial costs was ` 2,50,000 for each unit. After 1 year the carrying value is ` 2,00,000. The
market value has fallen down due to competitors coming out with similar product at a much lower price.
Hence here impairment is obvious. One has to calculate NSP of the LED’s for impairment loss.
After understanding the meaning of CGU lets see how to calculate Impairment loss for a CGU. Recognition
of Impairment loss for a CGU will depends upon whether Goodwill is allocable or unallocable. Following are
some of the situations:
Situation 1: No Goodwill stands in the books of accounts.
If a company has no Goodwill, Impairment loss of a CGU is similar to what we have seen in case of single asset.
The impairment loss of the total CGU is to be allocated between the individual assets on a pro-rata basis.
Situation 2: Goodwill stands in the books of accounts and allocable to CGU.
In this second situation where Goodwill can be allocated to CGU, Carrying Amount of Tangible Assets is to
be increased by allocating GW. Now comparing Total Carrying Amount of CGU with Recoverable Amount of
CGU will give us Impairment loss. Wipe off the Impairment loss first against GW then the balance apportion
against Tangible Assets. This approach is known as bottom-up test.
Word of caution: Ensure that; after allocating Impairment loss to individual assets the Carrying Amount of
such individual asset should not be less than the higher of {NSP, VIU, 0}.
Situation 3: Goodwill stands in the books of accounts but not allocable to CGU.
In this third situation it finds improper or impossible to allocate Goodwill to CGU. Then we have to go
for the next larger CGU in such a way that GW can be allocated. If still GW is not allocable, the search for the
next larger CGU should continue. Finally a position will come where the whole enterprise will be treated as
a CGU and GW will be totally allocable to the whole enterprise. Now the total carrying amount (CGU+GW)
will be compared to the Recoverable value. First adopt bottom up test without goodwill on all the CGU’s and
then calculate impairment at total level which includes Assets + G/W (which is unallocated). If total carrying
amount exceeds recoverable amount then impairment should be written off against unallocated goodwill.
As such CGU’s will not be impaired as they are already impaired by bottom up test (refer special problem on
Narang Ltd).

DO YOU KNOW
If a particular CGU is not impaired (i.e. Recoverable Amount > Carrying Amount), then the assets within the
CGU are also not impaired even though any one of the asset is showing impairment signs. It means provision
for impairment loss depends upon the CGU in totality and not the individual asset included in the CGU.

Corporate Assets
Corporate Assets include group or divisional assets such as the building of a headquarters, office premises,
division of an enterprise, EDP equipment or a research centre.
Features of Corporate Assets:
◘◘ They contribute towards the cash flows of CGU under review and even other CGU’s.
◘◘ Their cash flows are largely dependent on other assets or group of assets.
◘◘ Their Carrying Amount cannot be fully attributed towards the CGU under review.
◘◘ Corporate Assets do not generate separate cash inflows; the recoverable amount of an individual
corporate asset cannot be determinable unless management has decided to dispose of the asset.
AS-28: Impairment of Assets 261

Impairment loss for a Corporate asset is not same as Goodwill. In case of Corporate assets the loss will be
shared between the CGU and Corporate assets on pro rata basis. Yes both bottom up and top down approach
is applicable on Corporate assets also (refer special problem on Rahul Ltd).

Reversal of Impairment Loss


Assessment for Impairment is an annual exercise. It may be possible that some of the assets which were
impaired previously are now showing good signs i.e. the previously recognized impairment loss cease to exist.
The symptoms may in the form of increase in Cash inflows, decrease in Cash outflows, decrease in discount
rates etc. However, following are some of the factors which contribute towards Reversal of Impairment loss:

External Factors
◘◘ Market Value of the asset may have increased. For example: The Congress Government introduces
once again the Khadi system in 2004. Definitely Khadi Industries/Machines will be in high demand.
Helmet was mandatory in 2007 which encourages the production of helmet.
◘◘ Decrease in market expectation rates of return which increases the discount rate.

Internal Factors
◘◘ Evidence that economic performance of the asset is better than expected.
◘◘ Use of the asset in a manner, which maintains the current condition of the asset in a proper manner.

Reversal of Impairment Loss of an Individual Asset


Reversal of Impairment loss = Recoverable Amount - Carrying Amount but restricted to: On the date of Reversal à
(Deemed Carrying Amount had no impairment loss charged previously - Carrying Amount) (refer special problem)

One should remember the following points for the Reversal of Impairment loss of individual asset:

For Assets not Carried at Revalued Amount


(1) Reversal of impairment loss should be restricted to loss recognised previously. In other words the
increase in carrying amount so increased after reversal should not be more than the carrying amount
that would have been determined (net of depreciation or amortization) had no impairment loss been
recognised in some previous years.
(2) Reversal amount is to be credited to profit and loss account (treat it as a income).

For Assets Carried at Revalued Amount


(1) The increase in carrying amount so increased after reversal should not be more than the carrying
amount that would have been determined (net of depreciation or amortization) had no impairment
loss been recognised as a reduction from revaluation reserves.
(2) Reversal amount is to be credited to Revaluation Surplus Account.
(3) However, if previously impairment loss is adjusted partly from revaluation surplus (Dr.) and partly
from profit and loss account (Dr.) then reversal of loss is to be proportionately distributed between
revaluation reserves (Cr.) and accordingly profit and loss account (Cr.).
Depreciation/Amortization in future years is to be prospectively adjusted after considering the scrap
value and the remaining life of the asset.
262 Accounting Standards

DO YOU KNOW
Restrictions placed on carrying amounts not to be increased as previously stated is because any increase in
carrying amounts more than the restrictions placed will give rise to recognition of unrealized gains. → Concept
of Prudence.

Reversal of Impairment Loss of a CGU


Firstly, all the points as discussed above is to be considered. Additional conditions relating to the Cash
Generating Units is enumerated below:
(1) The Reversal of loss is first to be applied towards the assets under CGU’s on pro – rata basis. The
balance if any will be left to create goodwill.
(2) After allocating the reversal of loss towards the carrying amounts of the assets under CGU, the
carrying amount should not be increased above the lesser of: {Recoverable Amount or Carrying
Amount if the impairment loss has not been recognised previously}.
◘◘ Reversal of Impairment loss for Goodwill:
Firstly, all the points as discussed in the preceding parais to be considered. Additional conditions
relating to Goodwill is enumerated below:
(1) Reversal of loss after allocating to the assets under CGU is to be left to create Goodwill.
(2) Further restriction placed is that reversal is to be allocated to create Goodwill only if the following
factors cumulatively gets satisfied:
(i) Impairment loss was caused by specific external non –recurring event, which is not expected
to arise in future.
(ii) Subsequent events have reversed the effect of impairment loss as in (i).
It means factors other than above (i + ii) if prevails to write off Goodwill previously are not to be considered
for Reversal of loss towards Goodwill. Other factors may be change in discount rate, increase in future cash
inflows etc.

Transitional Provisions
As on the date As-28 becomes mandatory every company (if AS-28 is applicable) should assess its value of
assets for impairment. If impairment loss exists then following treatment is to be given:

Assets carried at Effect


Assets carried at Historical Cost Debited to opening revenue reserves
Assets carried at Revalued Amount • If revaluation reserves are sufficient to wipe off impairment loss → Debited
to revaluation reserves
• If revaluation reserves are not sufficient to wipe off impairment loss →
Debited to revaluation reserves and balance loss debited to opening
revenue reserves

Disclosures
Disclosures are categorized into four forms:
(a) Disclosures for each class of Assets:
(i) Amount of impairment loss debited to profit and loss account.
(ii) Amount of impairment loss reversed back (credited) to profit and loss account.
AS-28: Impairment of Assets 263

(iii) Amount of impairment loss adjusted against revaluation reserves.


(iv) Amount of impairment loss reversed back (credited) to revaluation reserves.
◘◘ Disclosures for CGU:
Where the amount to be disclosed is material for reporting purposes then following is to disclosed:
(i) Events and circumstances, which led to impairment loss and its reversal.
(ii) Amount of impairment loss recognized or reversed.
(iii) Nature of individual asset and reportable segment to which it belongs.
(iv) Description of CGU in terms of: whether it is a product line, segment, a unit, or a plant etc.
(v) Amount of impairment loss recognized or reversed by reportable segment as per enterprise’s
primary format (as defined in AS-17).
(vi) Change in Composition of CGU.
◘◘ Disclosures for both individual asset as well as Cash Generating Unit:
(i) Recoverable Amount taken is Net Selling Price or Value in Use.
(ii) If Recoverable Amount taken is as per Net Selling Price then sources thereof.
(iii) If Recoverable Amount taken is Value in Use then discount rate used.

Impairment No
No Review
condition exist

Yes

Identity asset/CGU (NRA = Net Recoverable amount)

Higher of Compute : NRA

No
NSP VIU CA > NRA No Impairment

Yes

Cash Discount
Impairment =
Flows rate
[CA – NRV]
Binding Sale Active
agreement Markets

Follow up every year


264 Accounting Standards

PROBLEMS AND SOLUTIONS

Problem 1: Venus Limited has an asset which is carried in the B/S on 31-03-2005 at ` 500 lakhs. As at the
date the value in use is ` 400 lakhs and the NSP `375 lakhs. From the above data: (a) Calculate Impairement
loss, (b) Pass Journals.
(C. A. Final Nov. 2005)
Solution:
Value in Use = ` 400 lacs, NSP = ` 375 lacs.
Recoverable amount = Higher of VIU or NSP = ` 400 lacs.
Carrying amount of asset = ` 500 lacs.
Hence Impairment loss = Carrying amount - Recoverable amount
= 500 - 400 =` 100 lacs.
Journal Entry
Impairment loss Dr. 100 lacs
To Asset 100 lacs

Profit and loss Dr. 100 lacs


To Impairment loss 100 lacs
Problem 2: Max Ltd. enters into a binding sale agreement to sell an asset to Bakra Ltd. at ` 1,20,000 having
carrying amount of ` 1,30,000. But there is a mortgage on the asset of Bank loan ` 40,000. Bakra Ltd. also
decided to take over the liability. Compute Adjusted Carrying amount.
Solution: Adjusted Carrying amount = 1,30,000 – 40,000 (liability t/over) =`90,000. Further there is no
impairment loss. Recoverable amount 1,20,000 > Adj carrying amt. 90,000
Problem 3: An asset does not meet the requirements of environmental laws which have been recently
enacted. The asset is required to be destroyed as per the law. The asset is carried in the books at `. 6,00,000.
The estimated cost of destroying the asset is `. 70,000. How the asset is to be accounted for? Also calculate
the Liability. (CA Final Nov 2010 Financial Reporting Marks 5)
Solution:
Impairment loss = (Carrying amount - Recoverable amount) = (6,00,000– 0) =` 6,00,000.
Recoverable amount = higher of -70,000 or 0 = 0
A liability wil be created as per AS-29. It is not an asset related loss. It’s a loss which is over and above the
value of asset i.e. ` . 70,000 (as per law).
Journal Entry
Impairment loss Dr. 6,00,000
To Asset 6,00,000
Profit and loss Dr. 6,70,000
To Impairment loss 6,00,000
To Provision for disposal of asset 70,000
Problem 4: From the following cases identify the CGU:
(i) Sellobell Ltd. undertakes various projects all over India. It has installed a big Plant “X” in some remote
hilly area. It has also provided housing accommodations to its employees at companies cost.
AS-28: Impairment of Assets 265

(ii) State Government of Mumbai has given Colombo Institute of Management to run Management School
in 5 different areas of Mumbai. As per the contract Colombo Institute cannot dispose off/close down
any school in isolation. Out of the 5 Schools 2 are running in loss.
(iii) A huge multiplex has a bowling alley, theatres, hotel, shopping mall, etc; since each of these facilities
is not capable of generating revenues independently the entire hotel and not the individual facilities
would be treated as a CGU.
(iv) An enterprise has three plants A, B and C. All three produce same product. Production and delivery
from each plant depends on factors like availability capacity, nearness to customers etc.
Solution:
(i) Housing complex on its own has no value in use hence no recoverable amount. Therefore, CGU for the
housing complex is the Plant ”X” as a whole.
(ii) In this case, though 5 different institutes are generating independent cash inflows they are CGU in
itself, but not a single unit can be disposed off separately so they cannot be treated as different CGU’s
but a single CGU.
(iii) Yes the whole multiplex is the CGU. The individual facilities are not capable of generating revenues.
The multiplex in combination with other assets can generate revenues.
(iv) Since the cash flows from the plants are inter-dependant, the enterprise itself is the smallest unit of
group of assets generating independent cash flows.
Problem 5: A publisher owns 150 magazine titles of which 70 were purchased and 80 were selfcreated.
The price paid for a purchased magazine title is recognised as an intangible asset. The costs of creating
magazine titles and maintaining the existing titles are recognised as an expense when incurred. Cash inflows
from direct sales and advertising are identifiable for each magazine title. Titles are managed by customer
segments. The level of advertising income for a magazine title depends on the range of titles in the customer
segment to which the magazine title relates. Management has a policy to abandon old titles before the end of
their economic lives and replace them immediately with new titles for the same customer segment.
What is the cash-generating unit?
Solution: It is likely that the recoverable amount of an individual magazine title can be assessed. Even though
the level of advertising income for a title is influenced, to a certain extent, by the other titles in the customer
segment, cash inflows from direct sales and advertising are identifiable for each title. In addition, although
titles are managed by customer segments, decisions to abandon titles are made on an individual title basis.
Therefore, it is likely that individual magazine titles generate cash inflows that are largely independent
one from another and that each magazine title is a separate cash-generating unit.
Problem 6: G Ltd. acquired a machine on 1st April 2005 for ` 7 crore that had an estimated useful lifest
of 7 years. The machine is depreciated on straight line basis and does not carry any residual value. On 1
April 2009 the carrying value of the machine was re-assessed at `5.10 crore and the surplus arising out
of the revaluation being credited to revaluation reserve. For the Y.E. March 2011 conditions indicating an
impairment of the machine existed and the amount recoverable ascertained to be only ` 79 lakhs. You are
required to calculate the loss on impairment of the machine and show how this loss is to be treated in the
books of G Ltd. G Ltd. had followed the policy of writing down the revaluation surplus by the increased charge
of depreciation resulting from the revaluation.
(CA final Nov 2011 A/C Marks 4, Similar problem June 2009 Marks 5,
Similar qt in May 2013 Marks 5)
266 Accounting Standards

Solution:
` crores
Cost of the Asset on 1/4/2005 7
(–) Depreciation for 4 years (7/7 × 4) (4)
Carrying amount on 1/4/2009 3
Revaluation of asset 5.10
Revaluation reserve 2.1
Now, Carrying amount of asset on 31/03/2011 [A] 1.70
(5.10 – [5.10/3] × 2)
Recoverable Amount (RA) [B] 0.79
Impairment loss [A – B] 0.91
Less: Revaluation reserve balance left out (2.1 – [2.1/3] × 2) (0.70)
Net charged to P/L 0.21

Problem 7: Radar Ltd. enters into an agreement to dispose one of its plant, whose carrying amount is
` 60,000 at ` 70,000. Radar Ltd. is expected to incur the following costs to effect the sale:
Repairs ` 1,500, Legal charges ` 3,000, Freight & Carriages ` 200. Capital gains tax ` 1,000. Compute NSP.
Solution:
NSP = Sale Value – *Cost of disposal = 70,000 – 4,700 = `65,300
*Cost of disposal =1,500+3,000+200 = ` 4,700. Cost of disposal includes all direct costs incurred to effect
sale except Tax expense.
P roblem 8: Goosers Limited let out land on rental basis as a parking space as a result of which there is
impairment. However, the landowner argues that an impairment provision is not required since the land
will be soon converted into a bowling alley, a highly profitable business? Is that argument acceptable for not
making an impairment provision?
Solution: As per para 36 of AS-28, future cash flows for Impairment Loss is to be calculated for the asset
under current condition. Future restructuring costs are not to be considered for calculating cash flows. Any
future use of the asset is not to be considered unless that change is fully committed to. Therefore impairment
loss is to be provided is required to be provided.
P roblem 9: United Ink and Varnish Ltd. has an ink processing machine costing ` 20,00,000 (01.04.2006). As
on 31.03.2008 the company went for impairment of the machine. The recoverable amount as on 31.03.2008 is
` 9,00,000. Depreciation rate 25%. The Company goes for SLM method of Depreciation but as per Income Tax
WDV method is allowed. Corporate Tax @35%. Calculate Impairment Loss, DTL/DTA, Future Depreciation
per annum for United Ink and Varnish Ltd.
(CA Final May 2012) (5 Marks)
Solution:
Workings:
(a) Calculation of Carrying Amount as on 31.03.2008:

As per
Books Income Tax
Cost 20,00,000 20,00,000
Less: Depreciation for 2006-2007 – 5,00,000 – 5,00,000
WDV as on 31.03.2007 15,00,000 15,00,000
Less: Depreciation for 2007-2008 – 5,00,000 – 3,75,000
WDV as on 31.03.2008 10,00,000 11,25,000
AS-28: Impairment of Assets 267

(1) Calculation of Impairment Loss: IL = 10,00,000 – 9,00,000 = ` 1,00,000


(2) Calculation of DTA/DTL as per AS-22:
As Impairment Loss is not an allowable expenditure as per Income Tax, therefore whatever is
the Impairment Loss is itself a timing difference (DTA).
Also timing difference for difference in Depreciation is = (5,00,000 – 3,75,000) = ` 1,25,000
Total Timing difference = As per Impairment Loss 1,00,000 + As per Depreciation 1,25,000
Total Deferred Tax Asset = 2,25,000 x 35% = ` 78,750 on an assumption that in future sufficient
taxable income is available.
(3) Future adjusted Depreciation per annum (prospectively adjusted):
Revised Cost of the Asset or WDV as on 31.03.2008 = 10,00,000 – 1,00,000 = ` 9,00,000
Remaining useful life = 2 years.

Depreciation = = (9,00,000 – 0)/2 =

= ` 4,50,000 p.a.
Problem 10: VARGHESE WORKS PVT. LTD. has 3 Assets are included in a CGU under review NSP or VIU of
individual assets cannot be determined. GW in the company’s books is zero. Value of assets is ` 50,000, `
20,000 and ` 30,000 respectively. Recoverable Amount is ` 90,000. Compute Impairment Loss for individual
asset.
Solution:
Steps OR Checklist Answer to Checklist
Is NSP or VIU ascertainable on individual asset No
Then find Carrying Amount of the CGU to which the asset Carrying Amt. = (50,000+20,000+30,000) = `1,00,000
belongs
Any Goodwill No
Allocable or Unallocable Not Applicable
Total Carrying Amount of CGU (Tangible Assets + GW) 1,00,000 + 0 = ` 1,00,000
Total Recoverable Amount of CGU 90,000
Impairment loss of CGU = CA – RA 10,000
Impairment loss of individual Asset Ratio: 5:2:3
Impairment loss for individual Asset
= 5,000, 2,000, 3,000.
Revised Carrying Amount of Assets: 45,000, 18,000, 27,000

Problem 11: NARANG PVT. LTD. has 3 UNITS (CGUs) A, B, C. Goodwill appearing in the books of Narang Ltd.
is ` 40 crores. Compute Impairment Loss for CGU:
(i) Assuming goodwill is allocable
(ii) Goodwill is not allocable.

Assets A B C Narang Ltd


Carrying amount 150 100 150 400
Recoverable amount 115 107 178 400
Goodwill 40
268 Accounting Standards

Solution:
(i) This method is known as bottom-up test . The total impairment loss is first apportioned towards
write off G/W then balance towards CGU/asset.

Asset CA G/W Total CA RV IL After impairment


Carrying amt
G/W Asset
A 150 15 165 115 50 0 115
B 100 10 110 107 3 7 100
C 150 15 165 178 - 15 150

(ii) As the GW is not allocable we have to apply Top down approach. But first apply bottom-up test
without allocating GW.

Asset CA RV IL After impairment


Asset
A 150 115 35 115
B 100 107 - 100
C 150 178 - 150

Carrying amount = 405 (Including G/W) and R.V. = 400


Impairment of ` 5 is applicable to G/W only.
Problem 12: UK International Ltd. is developing a new production process. During the financial year ending
31st March 2007, the total expenditure incurred was ` 50 Lakhs. This process met the criteria for recognition
as an intangible asset on 1st Dec 2006. Expenditure incurred till this date was ` 22 Lakhs. Further expenditure
incurred on the process for the financial year ending 31st March 2008 was ` 80 Lakhs. As at 31st March 2008,
the Recoverable Amount of Know-how embodied in the process is estimated to be ` 72 Lakhs. This includes
estimates of future cash outflows as well as inflows. Calculate the amount to be charged to Profit and Loss A/c
and Carrying Value of Intangibles as on the closing date in respect of financial year ending - (a) 31st March
2007, and (b) 31stMarch 2008. Ignore depreciation. (CA Final May 2008)
Solution: Expenditure charged to P&L A/c for 2006-07:`22 lakhs will be recognized as an expense because
the recognition criteria were not met until 1st December 2006. This expenditure will not form part of the
cost of the production process recognized in the balance sheet. Carrying value of intangible as on 31.03.2007.
`28 lakhs.
Expenditure charged to P&L A/c for 2007-08:

Particulars `lakhs
Book value on 31.3.2008 = carrying amt on 31.3.2007 + expenditure in 07-08 = (28 + 80) 108
Less: Recoverable amount 72
Impairment loss to be charged to P & L A/c 36

Carrying value of intangibles as on 31.03.2008: the production process will be shown at book value `108
lakhs, or recoverable amount `72 lakhs, whichever is less, i.e. `72 lakhs. Amount charged to P/L will be
36lakhs
Problem 13: Rahul Industries Ltd. has three cash-generating units A, B and C. The carrying amount of assets
as on 31.3.2005 of A, B and C is ` 400 lacs, ` 600 lacs and` 800 lacs respectively. Due to adverse technological
changes effecting activities of the company, the company conducts an impairment test of each of its cash-
generating units on 31.3.2005.
AS-28: Impairment of Assets 269

The Company has corporate assets comprising Head Office building of ` 600 lacs and Research and
Development (R&D) Centre of ` 200 lacs. The carrying amount of building can be allocated on reasonable
basis but R&D centre cannot be allocated on reasonable basis to the cash-generating units. The remaining
useful life of CGU “A” is 10 years and B and C is 20 years. The Head Office assets are depreciated on straight-
line basis.
The Net selling price of each cash-generating unit is not determinable. Hence recoverable amount is based
on “value in use”; 15% discount rate is to be considered.
Calculate impairment loss to be recognised in financial statement for the year ended 31.3.2005 and
allocation of impairment loss. Also calculate the revised carrying amount of assets of all cash-generating
units.
Future cash flow for A, B and C and for the company as a whole at the end of 31.3.2005 is as follows :
` in lacs

Year A B C (Rahul Ltd. as a whole)


1- 5 140 each yr. 100 each yr. 120 each yr. 370 each yr.
6 – 10 200 each yr. 130 each yr. 240 each yr. 650 each yr.
11- 20 82 each yr. 230 each yr. 446 each yr.

Solution:
Note: The carrying amount of Head Office building can be allocated on reasonable and consistent basis to the
cash-generating units under review and hence a ‘bottom up’ test is necessary.
The carrying amount of R & D Centre cannot be allocated on a reasonable basis to the individual cash-
generating units therefore a ‘top down’ test will be applied in addition to ‘bottom up’ test.
Step 1: Allocation of carrying amount of the H.O. building ` in lacs

Year end 31.3.2005 A B C Total


a. Carrying amount 400 600 800 1800
b. Useful life (yrs.) 10 20 20
c. Weight based on useful life and carrying amount 4000 12000 16000 32000
d. Weighted carrying amount proportion 12.5% 37.5% 50%
e. Allocation of carrying amount of the building on the basis of 75 225 300 600
weighted carrying amount
f. Carrying amount (after allocation of the H.O. building) (a+e) 475 825 1100 2400

Step 2: Calculation of value in use ` in lacs

Year Future cash flow Annuity factor @ 15% Discounted cash flow
For CGU “A”
1 to 5 140 3.352 469
6 to 10 200 1.666 333
Value in use 802
For CGU “B”
1 to 5 100 3.352 335
6 to 10 130 1.666 215
11 to 20 82 1.239 102
Value in use 652
For CGU “C”
270 Accounting Standards

1 to 5 120 3.352 402


6 to 10 240 1.666 400
11 to 20 230 1.239 286
Value in use 1088
For Rahul Ltd as a whole
1 to 5 370 3.352 1240
6 to 10 650 1.666 1083
11 to 20 446 1.239 553
Value in use 2876
Note: Discounted rates are mentioned in total for 1 to 5 years as annuity factor (0.870 + 0.756 + 0.657 + 0.572 + 0.497 = 3.352)
Step 3: Impairment test - Bottom up

Year-end 31.3.2005 A B C
(a) Carrying amount (After allocation of building) 475 825 1100
(b) Recoverable amount (value in use; NSP not available) 802 652 1088
(c) Impairment loss 0 (173) (12)

Step 4: Allocation of impairment loss to cash-generating units B and C only ( ` in lacs)

CGU “B” CGU “C”


To H.O. building 173 x (225/825) = 47 12 x (300/1100) = 3
To assets of cash-generating units 173 x (600/825) = 126 12 x (800/1100) = 9
Total 173 12

Step 5: As the R & D Centre is not allocable on reasonable basis to GGU’s A, B and C, the top down test for
impairment is done by considering the carrying amount of larger cash-generating unit to which R & D Centre
can be allocated i.e. Company as a whole. (` in lacs)

Year 31.3.2005 A B C Building R&D Co. as a


whole
a. Carrying amount 400 600 800 600 200 2600
b. Impairment loss arising out of ‘bottom up’ test - (126) (9) (50) - (185)
c. Revised carrying amount 400 474 791 550 200 2415
After bottom up test
d. Recoverable amount of company on a whole. 2876
e. The impairment loss arising from “top down Nil
test’ (as recoverable amount of ` 2876 lacs is
more than carrying amount `2415 lacs)

Note:
The impairment loss of only ` 185 lacs is to be recognised in financial statement and allocated to B, C and
Building of `126, 9 and 50 respectively, resulting in revised carrying amount of
(i) CGU “A” - ` 400 lacs (iv) Building - ` 550 lacs
(ii) CGU “B” - ` 474 lacs (v) R & D - ` 200 lacs
(iii) CGU “C” - `791 lacs (vi) Total - ` 2415 lacs
AS-28: Impairment of Assets 271

Problem 14: JUBILEE LIMITED acquired business of LIFTINGS LIMITED as on 01.01.1999 by issuing
10,000 Equity Shares of ` 100 each. Net Assets acquired is ` 9,50,000.
On 31.12.2002 due to government reforms, which badly affected their production line, it had to carry out
impairment programme. The manufacturing plant is the only cash-generating unit for the company. Its useful
life is 10 years. The plant is so specialized that its NSP could not be determined. For Recoverable Amount, its
Value in Use is calculated @15%. Estimated VIU is as follows:

Year Future Cash Inflows


2003 1,70,000
2004 1,60,000
2005 1,30,000
2006 1,30,000
2007 1,00,000
2008 75,000

But, in 2004 the government withdrawn the restrictions on the contrary some incentives and rebates
were provided. JUBILEE LIMITED re-estimated the recoverable amount in 2004, which was estimated at
follows:
Year Future Cash Inflows
2005 1,00,000
2006 1,60,000
2007 1,70,000
2008 1,20,000

You are required to:


(a) Compute impairment loss in 2002.
(b) Carrying Amount as on 2004 after reversal of loss.
Solution:
(a) Computation of Impairment loss in 2002.

Net Tangible Assets Goodwill1 Total


Cost of the assets acquired 9,50,000 50,000 10,00,000
Less: Depreciation/Amortization (1999 – 2002) (3,80,000) (40,000) (4,20,000)
(9,50,000/10) x 4 (50,000/5) x 4
Carrying Amount as on 2002 5,70,000 10,000 5,80,000
Less: Recoverable Amount2 5,10,756
Impairment loss3 (59,244) (10,000) 69,244
Carrying Amount as on 2002 5,10,756 0
(after impairment loss)
1. Goodwill will be amortized over 5 years as per AS-14.
2. Value in Use (Recoverable Amount) is calculated @15% :
Present Value of Future Cash Inflows @15% (2003 till 2008) =
(1,47,826+1,20,976+85,475+74,334+49,720+32,423) = ` 5,06,456.
3. Impairment loss will be first absorbed by Goodwill. Balance will be attributed towards Tangible
Assets.
(b) Carrying Amount as on 2004 after reversal of loss.
(i) Amount of Reversal available:
272 Accounting Standards

Net Tangible assets Goodwill 1 Total


Carrying Amount as on 2002 5,10,756 0 5,10,756
(after impairment loss)
Less: Depreciation/Amortization (2003-2004) (1,70,252) 0 (1,70,252)
(5,10,756/6) x 2
Carrying Amount as on 2004 3,40,504 0 3,40,504
Less: Recoverable Amount4 3,88,297
Gross Reversal available: 47,793
4. Value in Use (Recoverable Amount) is calculated @15% :
Present Value of Future Cash Inflows @15% (2005 till 2008) =
(86,957 + 1,20,960 + 1,11,860 + 68,520) = ` 3,88,297
(ii) Amount of Reversal allowed:
Deemed Carrying Amount as if no impairment charged ` 3,80,000
(Cost 9,50,000 – Deemed Dep. till 2004 95,0000 x 6/10)
Less: Actual Carrying Amount as on 2004 ` 3,40,504
Maximum Reversal allowed: ` 39,496
Carrying Amount as on 2004 after reversal ` 3,80,000

Problem 15: A plant was acquired 15 years ago at a cost of ` 5 crores. Its accumulated depreciation as at
31st March, 2011 was ` 4.15 crores. Depreciation estimated for the Financial year 2011-12 is ` 25 lakhs.
Estimated Net Selling Price as on 31st March, 2011 was ` 30 lakhs, which is expected to decline by 20 per cent
by the end of the next financial year.
Its value in use has been computed at ` 35 lakhs as on 1st April, 2011, which is expected to decrease by 30
per cent by the end of the Financial year.
(i) Assuming that other condition for applicability of the impairment Accounting Standard is satisfied,
what should be the carrying amount of this plant as at 31st March, 2012?
(ii) How much will be the amount of write off for the financial year ended 31st March, 2012?
(iii) If the plant had been revalued ten years ago and the current revaluation reserves against this plant
were to be ` 12 lakhs, how would you answer to questions (i) and (ii) above?
(iv) If the value in use was zero and the enterprise were to incur a cost of ` 20 lakhs to dispose of the
Plant, what would be your response to question (i) and (ii) above? (May 2010 OS) (8 Marks)
Solution:
(i) As on 31/3/2012: Carrying amount b/d = 500 lakhs – 415 lakhs – 25 lakhs = `60 lakhs; VIU = 24.5
lakhs (70% of 35); NSP = 24 lakhs (30 × 0.8). Impairment loss = CA – RV = 60 – 24.50 = `35.50.
Carrying amount after impairment loss = ` 24.5 lakhs.
(ii) Total w-off will be ` 35.50 i.e. impairment loss.
(iii) The entire impairment loss would be first adjusted against revaluation reserves `12 lakhs and the
balance loss will be charged to P/L = 35.5 – 12 = ` 23.5 lakhs.
(iv) In that case the carrying amount will be Nil. The entire value of the asset will be written – off.
CHAPTER 26
AS-29: Provisions, Contingent
Liabilities and Contingent
Assets

Chapter Outline
vv Objective vv How Should Future Operating Losses Be Treated?
vv Recognition of Provisions vv Onerous Contract
vv How to Measure the Amount of Provisions? vv Restructuring Costs
vv Recognition of Contingent Liabilities vv Disclosure Requirements Under AS 29
vv Recognition of Contingent Assets

Objective
(i) Ensure that appropriate recognition criteria and measurement bases are applied to provisions and
contingent liabilities.

Scope of the Standard


AS 29 is to be applied in accounting for provisions and contingent liabilities and in dealing with contingent
assets except those:
(i) resulting from financial instruments that are carried at fair value;
(ii) resulting from executory contracts;
(iii) arising in insurance enterprises from contracts with policy-holders; and
(iv) covered by another accounting standard. If another accounting standard deals with a specific type of
provision, contingent liability or contingent asset, an enterprise should apply that Standard instead
of AS 29, e.g., construction contracts are specifically dealt with by AS 7 (Revised). Hence provisions,
contingent liabilities, etc. relating to construction contracts should be treated as specified in AS 7
(Revised).
Note: AS 29 also applies to provisions for restructuring (including discontinuing operations). Where a
restructuring meets the definition of a discontinuing operation, additional disclosures as required by AS 24
should also be made.

Key Terms that have been Defined in AS 29


i) Provision, ii) Liability, iii) Contingent Liability, iv) Contingent Asset, v) Possible obligation, vi) Obligating
Event, vii) Present Obligation
274 Accounting Standards

Are ‘Provisions’ same as ‘Liabilities’


No, provisions are different from liabilities. Provisions are liabilities that require substantial degree of
estimation. Although, even in case of liabilities it is sometimes necessary to estimate the amount of accruals,
the degree of estimation is generally much less than that for provisions,

The Recognition Criteria Under AS 29


(a) Recognition of provisions
An enterprise should recognise a provision only when all of the following conditions are met:
(i) there is a present obligation as a result of a past event;
(ii) it is probable that an outflow of resources embodying economic benefits will be required to settle the
obligation; and
(iii) reliable estimate can be made of the amount of the obligation.
i) Present obligation
Present obligation means on the B/S date based on the evidence available its existence is considered
probable i.e. more likely than not.
An obligation always involves another party to whom the obligation is owed. But it is not necessary
to know the identity of the party to whom the obligation is owed — indeed the obligation may be to
the public at large.
ii) Past event
Only the obligations arising from past events existing independently of an enterprise’s future
actions (i.e. the future conduct of its business) should be recognised as provisions. Examples of
such obligations are penalties or clean-up costs for unlawful environmental damage, both of which
would lead to an outflow of resources embodying economic benefits in settlement regardless of
the future actions of the enterprise. Similarly, an enterprise should recognise a provision for the
decommissioning costs of an oil installation to the extent that the enterprise is obliged to rectify
damage already caused.
Past events give rise to obligations.
Obligation arises not only due to legal enforcement/binding contract/statutory requirement, but also
due to some customs/practice or a desire to maintain good business relations. Ex: A company may
provide for bonus because there is a tradition to pay bonus every year right from the incorporation
of the company to its employees
iii) Probable outflow of resources embodying economic benefits
An outflow of resources or other event is regarded as probable if the probability that the event will
occur is greater than the probability that it will not. Where it is not probable that a present obligation
exists, an enterprise should disclose a contingent liability.
Where there are a number of similar obligations (e.g. product warranties or similar contracts) the
probability that an outflow will be required in settlement should be determined by the considering
the class of obligations as a whole.
iv) Reliable estimate of the obligation
By their very nature, provisions require a greater degree of estimation than most other items. The
use of estimates does not undermine the reliability of financial statements. In the extremely rare
case, where a liability exists but no reliable estimate can be made, that liability should be disclosed
as a contingent liability (refer definition of contingent liability)
AS-29: Provisions, Contingent Liabilities and Contingent Assets 275

How to Measure the Amount of Provisions?


Following points are worth helpful for recognition of provisions:
1. Best estimate: The amount of provision should be based on the best estimate of the expenditure
required to settle the present obligation at the balance sheet date. For best estimate one has to take
lessons from past experience, judgment, management representation, events occurring after the B/S
date.
2. Taxes: The amount should be measured before tax.
3. Discounting: Besides, the amount should not be discounted to its present value. But in IFRS, Provision
is discounted if the time value of money is relevant.
4. Risk and uncertainty: In making the best estimate of a provision, the related risks and uncertainties
should also be considered. A risk adjustment may increase the amount at which a liability is measured.
Caution is required in making judgments under conditions of uncertainty, so that income or assets
are not overstated and expenses or liabilities are not understated. However uncertainty does not
justify the creation of excessive provisions or a deliberate overstatement of liabilities.
5. Future events that may affect the amount required to settle an obligation should also be reflected
in the amount of a provision. Example Future clean up costs may be incurred by applying newer
technology which will definitely affect provisions.
6. Gains from the expected disposal of assets: Gains from the expected disposal of assets should not
be taken into account as it amounts to recording of unrealised gains.
7. Reimbursement of expenses: Sometimes, some or all of the expenditure is expected to be reimbursed
by another party, e.g., through indemnity clauses, suppliers warranties, insurance contracts, etc. (i) If
the enterprise is not liable to pay for the costs, in question, there is no need to include such costs in
the amount of provisions. (ii) However, if the enterprise remains liable for the whole of the amount
in question so that the enterprise would have to settle the full amount if the third party failed to pay
for any reason, provision should be recognised for the full amount of the liability. A separate asset
should be recognised for the expected reimbursement when it is virtually certain that reimbursement
will be received if the enterprise settles the liability. In the statement of profit and loss, the expense
relating to a provision may be presented net of the amount recognised for a reimbursement. (iii)
If reimbursement is no certain then ignore the reimbursement and provision is to be made to its
full value.

Should Provisions be Reviewed?


Yes, provisions should be reviewed at each balance sheet date and adjusted to reflect the current best estimate
till they become actual liability.

Recognition of Contingent Liabilities


Definition 1: A Contingent liability is a possible obligation arises out of the past events and the existence of
which will be confirmed only by the occurrence or non – occurrence of one or more uncertain future events
not wholly within the control of the enterprise;
or
Definition 2: A present obligation that arises as a result of past events but is not recorded because;
(i) it is not probable that an outflow of resources embodying economic benefits will be required to
settle the obligation; and
(iii) a reliable estimate of the amount of the obligation cannot be made.
276 Accounting Standards

Example:
1) A contracting company executes several road building and bridges contract. Most of these contracts
have liquidated damages clause built in them involving complex formula for computing the
compensation. The past experience of the company is that very often the liquidated damage clause is
not invoked, partly because the fault also lies with the contractee, e.g., payments not made on time,
clear contract site not made available to contractor, etc. and unforeseen situations like bad weather,
lack of cooperation from local authorities, etc. The company does not make any provision since a
reliable estimate cannot be made.
2) A contingent liability does not require recognition, it only requires a disclosure. Where an enterprise
is jointly and severally liable for an obligation, the part of the obligation that is expected to be met by
other parties is treated as a contingent liability.
3) Contingent liabilities are assessed continually to determine whether an outflow of resources
embodying economic benefits has become probable. If it becomes probable that an outflow of future
economic benefits will be required for an item previously dealt with as a contingent liability, it should
be recognized as a provision.

Recognition of Contingent Assets


1) A Contingent asset is a possible asset that arises as a result of past events the existence of which will
be confirmed only by the occurrence or non – occurrence of more or more uncertain future events
not wholly within the control of the enterprise;
2) Generally, contingent assets arise from unplanned or other unexpected events that give rise to the
possibility of an inflow of economic benefits to the enterprise, e.g., a claim that an enterprise is
pursuing through legal processes, where the outcome is uncertain.
3) Contingent assets are not recognised in financial statements since this may result in the recognition
of income that may never be realised. However, when the realization of income is virtually certain,
then the related asset is not a contingent asset and its recognition is appropriate.
4) Contingent asset is not required to be disclosed in the financial statements.

How Should Future Operating Losses Be Treated?


Future operating losses neither meet the definition of a liability nor the recognition criteria of a provision.
Hence, provisions should not be recognised for future operating losses. However, an expectation of future
operating losses is an indication that certain assets of the operation may have been impaired. Hence, an
enterprise should test these assets for impairment under AS 28.

Onerous Contract
An onerous contract is a contract where the unavoidable costs of meeting the obligations > expected benefits.
As per ASI-30 such losses are to be provided for. Ex: In case of AS-7 if the total Costs of construction > total
revenue then provision is created for the expected loss and such contract is known as onerous contract. (Also
refer problem no.: 13)
Example: An entity is bound under the terms of a franchise agreement for a local brand that it has marketed
for years. Based on market survey and a cost-benefit study, the entity decided to stop marketing the local
brand and entered into a new agreement to market an international brand. Although the entity does not
derive any economic benefit from the franchise agreement for the local brand, there is an obligation to pay a
lump-sum amount to the franchiser under the non cancellable franchise agreement for a period of two more
years. Thus the entity would need to make a provision for the commitment under the franchise agreement
(since it is an onerous contract).
AS-29: Provisions, Contingent Liabilities and Contingent Assets 277

Restructuring Costs
A restructuring is the programme that is planned and controlled by the management, and materially changes
either the scope of the business or the manner in which the business is conducted Example Demerger costs.
To provide guidance on this issue, IAS 37 provides these examples of events that may qualify as
restructuring:
◘◘ Sale or termination of a line of business
◘◘ Closure of business locations in a region or relocation of business activities from one location to
another
◘◘ Changes in management structure, such as elimination of a layer of management
◘◘ Fundamental reorganization of the entity such that it has a material and a significant impact on its
operations.

Disclosure Requirements Under AS 29


i) For each class of provision:
(a) the carrying amount at the beginning and end of the period;
(b) additional provisions made in the period, including increases to existing provisions;
(c) amounts used (i.e. incurred and charged against the provision) during the period; and
(d) unused amounts reversed during the period.
(e) a brief description of the nature of the obligation and the expected timing of any resulting
outflows of economic benefits;
(f) an indication of the uncertainties about those outflows, an enterprise should also disclose
the major assumptions made concerning future events, if it is necessary to provide adequate
information; and
(g) the amount of any expected reimbursement, stating the amount of any asset that has been
recognised for that expected reimbursement.
ii) For each class of contingent liabilities
ww Nature
ww Estimate of financial effect
ww Indication of uncertainties relating to any outflow
ww Possibility of any reimbursement
yy If it is not practicable to disclose the required information, state the fact
yy If disclosure expected to prejudice seriously the position of an enterprise in a dispute with
other parties, don't disclose. But even then, the general nature of dispute, together with the
fact that, and reasons why, the information has not been disclosed should be stated.

Practical disclosure of Provision (Dummy figures)


The movements in the above provisions are summarised below:

Warranties Liquidated Loss orders Contingencies


damages
Balance as at 1 April, 2014 1,248,631 1,574,211 1,099,514 1,002,476
Provisions:
 Created 1,096,676 4,575,933 692,148 841,697
 Utilised (103,141) (182,918) (838,809) (47,877)
 Reversed (261,634) (315,906) (61,239) (157,916)
Balance as at 31 March 2015 1,980,532 5,651,320 891,614 1,638,380
278 Accounting Standards

PROBLEMS AND SOLUTIONS

Problem 1: A furnace has a lining that needs to be replaced every five years for technical reasons. At the
balance sheet date, the lining has been in use for three years. Is a provision required to be recognised?
Solution: There is no present obligation as at the balance sheet date. Because, at the balance sheet date, no
obligation to replace the lining exists independently of the company’s future actions. Even the intention to
incur the expenditure depends on the company deciding to continue operating the furnace or to replace the
lining. In view of this, no provision for replacement is required.
Problem 2: An airline is required by law to overhaul its aircraft once every three years. This is the third year
of use. Is a provision required to be recognised? (CA Final Nov 2010 OS)
Solution: Provision for costs of overhauling aircraft are not recognised for the same reasons that the cost
of replacing the lining is not recognised as a provision as above. Even a legal requirement to overhaul does
not make the costs of overhaul a liability, because no present obligation exists to overhaul the aircraft
independently of the enterprise’s future actions. The enterprise could avoid the future expenditure by its
future actions, for example by selling the aircraft. In view of this, no provision for overhauling costs is required.
Problem 3: A manufacturer gives warranties at the time of sale to purchasers of its product. Under the terms
of the contract for sale the manufacturer undertakes to make good, by repair or replacement, manufacturing
defects that become apparent within three years from the date of sale. On past experience, it is probable
(i.e. more likely than not) that there will be some claims under the warranties. Is a provision required to be
recognised?
or
V Ltd. sold 1 Lakh vacuum pumps during the year 2006-07 with a condition to make good by repair/
replacement any manufacturing defects reported within 6 months from the date sale. Past experience in this
regard showed that there were no replacements carried out, but minor/major repairs were necessitated to
the extent of 10%/5% respectively of the units sold. The cost of such minor/major repairs would amount
to `1,000/` 6,000 respectively. While finalizing the accounts for the year, the Company does not reflect any
provision, in this regard. Comment on the above situation. (CA Final May 2008)
Solution: The obligating event is the sale of the product with a warranty. An outflow of resources embodying
economic benefits in settlement is probable for the warranties as a whole. Therefore, provision for the best
estimate of warranty costs should be made.
Problem 4: An enterprise in the oil industry causes contamination but does not clean up because there is no
legislation requiring cleaning up, and the enterprise has been contaminating land for several years. At 31st
March 2012 it is virtually certain that a law requiring a clean-up of land already contaminated will be enacted
shortly after the year end. Is a provision required to be recognised? (CA Final Nov 2012)
Solution: The obligating event is the contamination of the land because of the virtual certainty of legislation
requiring cleaning up. An outflow of resources embodying economic benefits in settlement is Probable.
Hence, a provision is recognised for the best estimate of the costs of the clean-up.
Problem 5: An enterprise operates an offshore oilfield where its licensing agreement requires it to remove
the oil rig at the end of production and restore the seabed. Ninety per cent of the eventual costs relate to
the removal of the oil rig and restoration of damage caused by building it, and ten per cent rise through the
extraction of oil. At the balance sheet date, the rig has been constructed but no oil has been extracted. Is a
provision required to be recognised?
Solution: The construction of the oil rig creates an obligation under the terms of the licence to remove the
rig and restore the seabed and is thus an obligating event. At the balance sheet date, however, there is no
AS-29: Provisions, Contingent Liabilities and Contingent Assets 279

obligation to rectify the damage that will be caused by extraction of the oil since extraction of oil has not
commenced as of the balance sheet date. An outflow of resources embodying economic benefits in settlement
is probable. A provision is recognised for the best estimate of ninety per cent of the eventual costs that relate
to the removal of the oil rig and restoration of damage caused by building it. These costs are included as part
of the cost of the oil rig. The ten per cent of costs that arise through the extraction of oil are recognised as a
liability when the oil is extracted.
Problem 6: A retail store has a policy of refunding purchases by dissatisfied customers, even though it is
under no legal obligation to do so. Its policy of making refunds is generally known. Is a provision required to
be recognised?
Solution: It is Present obligation as a result of a past obligating event. The obligating event is the sale of
the product, which gives rise to an obligation because obligations also arise from normal business practice,
custom and a desire to maintain good business relations or act in an equitable manner. An outflow of resources
embodying economic benefits in settlement is probable because a proportion of goods are returned for
refund.
A provision should be recognised for the best estimate of the costs of refunds since recognition criteria in
AS-29 is satisfied.
Problem 7: Samsung Limited has ` 4.50 crores in sales tax demand relating to previous year against which
the company has gone into appeal for the year 2013-14. The ground of appeal dealt with points covering
` 3.50 crores of demand. Show how the matter will have to be dealt with the final accounts for the year
2008-09. (C.A. Final Nov. 2009 OS Accounts, May 2010 OS Audit)
Solution: The company has gone for an appeal consisting of amount ` 3.50 i.e. there is no present obligation
but possible obligation. Therefore disputed amount of ` 3.50 crores where appeal is preferred will be shown
as a contingent liability. The balance 1 crore should be provided as actual liability (company accepts the
liability)
Problem 8: As on 31-3-2012, there was a claim for damage from one of the customers against the company
in selling of accounting software for an alleged failure to provide after-sales services in relation to the software
purchased form it. Before finalization of accounts for the year ended 31-3-2012 (the accounts were finalized
on 14th June 2012), the company won the case and had no liability whatsoever in this regard. The company
has made a provision for this contingent liability in its accounts for the year ended 31-3-2012, which it says,
will be reserved in the next year.
Solution: The case was outstanding on the balance sheet date and the case disposed off after the year ending
hence it attracts AS-4 Events Occurring after the B/S date. Accounting effect has to be given for such event
took place after the year end. The company had provided for the loss as per AS-29 which now, should be
reversed for the previous year. This is because the company is not liable to pay any amount.
Problem 9: XYZ Ltd. pays bonus every year as per the minimum bonus act. Bonus is payable in the month of
November. Is bonus required to be provided for the year ended 31/3/2015 for the rendering of the services
by that date?
Solution: The obligating event is the rendering of service by the employee, which legally entitles the
employee to bonus. Thus provision is recognized. Obligation can arise either due to legal enforcement, statute,
agreement or to maintain good business relations or to act in an equitable manner.
Problem 10: After a wedding in 2013-14, ten people died, possibly as a result of food poisoning from
products sold by the enterprise. Legal proceedings are started seeking damages from the enterprise but
it disputes liability. Up to the date of approval of the financial statements for the year 31 March, 2009 the
enterprise’s lawyers advise that it is probable that the enterprise will not be found liable. However, when the
enterprise prepares the financial statements for the year 31 March, 2015 its lawyers advise that, owing to
developments in the case, it is probable that the enterprise will be found liable. How this should be dealt with
in financial statement?
280 Accounting Standards

Solution:
(a) At 31 March, 2014
On the basis of the evidence available when the financial statements were approved, there is no
present obligation as a result of past events.
Hence no provision is recognised. The matter is disclosed as a contingent liability unless the
possibility of any outflow is regarded as remote.
(b) At 31 March, 2015
Present obligation as a result of a past obligating event - On the basis of the evidence available, there
is a present obligation.
An outflow of resources embodying economic benefits in settlement is probable. A provision is
recognised for the best estimate of the amount to settle the obligation.
Problem 11: During 2009-10, Enterprise X gives a guarantee of certain borrowings of Enterprise Y, whose
financial condition at that time is sound. During 2010-11, the financial condition of Enterprise Y deteriorates
and on 30 September, 2010 Enterprise Y goes into liquidation. How it will he dealt with by ‘X’?
Solution:
(a) At 31 March, 2010
Guarantee given by Co X, gives rise to a possible obligation.
No outflow of benefits is probable at 31 March, 2010 since financial position of Y is sound.
Hence, no provision is recognised. The guarantee is disclosed as a contingent liability unless the
probability of any outflow is regarded as remote.
(b) At 31 March, 2011
The obligating event is the giving of the guarantee, which gives rise to a legal obligation to make good
enterprise Y’s defaults.
At 31 March 2011, it is probable that an outflow of resources embodying economic benefits will be
required to settle the obligation, since enterprise Y has gone into liquidation.
A provision should be recognised for the best estimate of the obligation.
Problem 12: Gopal Company has at its financial year ended 31st March, 2004 fifteen law suits outstanding,
none of which has been settled by the time the accounts are approved by the directors. The directors have
estimated that the possible outcomes as below:
Result Probability Amount of loss
For first ten cases:
Win 0.6 —
Lose-low damages 0.3 90,000
Lose-high damages 0.1 1,60,000
For remaining five cases:
Win 0.5 —
Lose-low damages 0.3 60,000
Lose-high damages 0.2 95,000
The directors believe that the outcome of each case is independent of the outcome of all the others.
Estimate the amount of contingent loss and state the accounting treatment of such contingent loss.
(C.A. Final May 2004, May 2013)
Solution: Please refer May 2013 paper - Question 9
Problem 13: Mini Ltd. took a factory premises on lease on 01.04.07 for ` 2,00,000 per month. The lease is
operating lease. During March 2008, Mini Ltd. relocates its operation to a new factory building. The lease on
AS-29: Provisions, Contingent Liabilities and Contingent Assets 281

the old factory premises continues to be live upto 31.12.2010. The Lease cannot be cancelled and cannot be
sub-let to another user. The auditor insists that lease rent of balance 33 months upto 31.12.2010 should be
provided in the accounts for the year ending 31.3.2008. Mini Ltd seeks your advice. (CA Final May 2008)
Solution:
1. “Onerous Contract” is a contract in which the unavoidable cost of meeting the obligation under the
contract exceeds the economic benefit expected under it.
2. In the given case , the operating lease contract has become onerous, as the economic benefit of lease
contract for next 33 months upto 31.12.2010 will be nil
3. Lessee, Mini Ltd has to pay lease rent of `66,00,000 (2,00,000 p.m for next 33 months). Therefore,
provision on account of `66,00,000 is to be made in the account for the year ending 31.03.08,in
accordance with AS-29 requirements.
Total period = 45 months → 12 months current yr and 33 months future period. Do you know 12 months
period rent is expenditure but 33 months future rent is a provision (loss).
Problem 14: Sun Ltd. has entered into a sale contract of ` 5 crore with X Ltd during 2013-14 financial year.
The profit on this transaction is 1 crore. The delivery of goods to take place during the first month of 2014-15.
In case of failure to deliver goods within the schedule, a compensation of ` 1.5 crore is payable to X Ltd. Sun
Ltd decided to manufacture the goods during the last month of 2013-14. As on 31/3/2014 Sun Ltd has not
manufactured the goods and it is unlikely that Sun Ltd will be in a position to meet the contractual obligation.
(i) Should Sun Ltd provide for contingency as per AS-29?
(ii) Should provision be measured as an excess of compensation to be paid over the profit?
(CA Final May 2010)
Solution:
(i) Yes, Sun Ltd has to make a provision as per AS-29. Past event is non – production of goods.
Present obligation is to pay compensation. It is probable that there will be outflow of resources as
compensation payable.
(ii) As on 31/3/2014 the goods are not yet manufactured compensation is required to be payable. Hence
the whole compensation of ` 1.5crores is to be provided. Provision should not be measured as an
excess of compensation to be paid over the profit.
Basic condition for any provision (Check list): V V Imp
(i) there is a present obligation as a result of a past event;
(ii) it is probable that an outflow of resources embodying economic benefits will be required to settle the
obligation; and
(iii) a reliable estimate can be made of the amount of the obligation.
Problem 15: An engineering goods company provides after sales warranty for 2 years to its customers.
Based on past experience, the company has been following policy for making provision for warranties on the
invoice amount, on the remaining balance warranty period:
Less than 1 year : 2% provision
More than 1 year : 3% provision
The company has raised invoices as under:

Invoice Date Amount (`)


19th January, 2013 40,000
29th January, 2014 25,000
15th October, 2014 90,000

Calculate the provision to be made for warranty under Accounting Standard 29 as at 31st March, 2014
and 31st March, 2015. Also compute amount to be debited to Profit and Loss Account for the year ended 31st
March, 2015.
282 Accounting Standards

Solution: Provision to be made for warranty under AS 29 ‘Provisions, Contingent Liabilities and
Contingent Assets’
As at 31st March, 2012 = ` 40,000 × .02 + ` 25,000 × .03
= ` 800 + ` 750
= ` 1,550
As at 31st March, 2013 = ` 25,000 × .02 + ` 90,000 × .03
= ` 500 + ` 2,700
= ` 3,200
Amount debited to Profit and Loss Account for year ended 31st March, 2015

`
Balance of provision required as on 31.03.2015 3,200
Less: Opening Balance as on 1.4.2014 (1,550)
Amount debited to profit and loss account 1,650

Problem 16: A company, incorporated under Section 8 of the Companies Act, 2013, have main objective to
promote the trade by organizing trade fairs / exhibitions. When company was organizing the trade fair and
exhibitions it decided to charge 5% contingency charges for the participants/outside agencies on the income
received from them by the company, while in the case of fairs organized by outside agencies, 5% contingency
charges are levied separately in the invoice, the contingency charges in respect of fairs organized by the
company itself are inbuilt in the space rent charged from the participants. Both are credited to Income and
Expenditure Account of the company.
The intention of levying these charges is to meet any unforeseen liability, which may arise in future.
The instances of such unforeseen liabilities could be on account of injury/loss of life to visitors/ exhibitors,
etc., due to fire, terrorist attack, stampede, natural calamities and other public and third party liability. The
chances of occurrence of these events are high because of large crowds visit the fair. The decision to levy 5%
contingency charges was based on assessment only as actual liability on this account cannot be estimated.
The following accounting treatment and disclosure was made by the company in its financial statements:
1. 5% contingency charges are treated as income and matching provision for the same is also being
made in accounts.
2. A suitable disclosure to this effect is also made in the notes forming part of accounts.
Required: (i) Whether creation of provision for contingencies under the facts and circumstances of the case
is in conformity with AS 29. (ii) If the answer of (i) is “No” then what should be the treatment of the provision
which is already created in the balance sheet.
Solution:
(i) Para 14 of AS 29 “Provisions, Contingent Liabilities and Contingent Assets” states that a provision
should be recognised when (a) An enterprise has a present obligation as a result of a past event
and (b) It is probable that an outflow of resources embodying economic benefits will be required to
settle the obligation and (c) A reliable estimate can be made of the amount of the obligation. If these
conditions are not met, no provision should be recognised.
From the above, it is clear that in the contingencies considered by the company, neither a present
obligation exists as a result of past event, nor a reliable estimate can be made of the amount of the
obligation. Accordingly, a provision cannot be recognised for such contingencies under the facts and
circumstances of the case.
(ii) “Provision” is the amount retained by the way of providing for any known liability. Since the
contingencies stipulated by the company are not known at the balance sheet date, the provision in
this regard cannot be created. Therefore, the provision so created by the company shall be treated as
a ‘Reserve’.
CHAPTER 27
AS-21: Consolidated Financial
Statements (Holding Company
Vs Subsidiary)

Chapter Outline
vv Applicability vv Effective Date of Consolidation
vv Consolidation Procedures vv Losses Exceeding Equity for Minority
vv Intragroup Transactions vv Disclosure

Applicability
This Statement should be applied in the preparation and presentation of consolidated financial statement for
a group of enterprises under the control of a parent
This statement should also be applied in accounting for investments in subsidiaries in the separate
financial statement of a parent.

DEFINITIONS
(i) Control
ww The ownership, directly or indirectly through subsidiary (ies), of more than one-half of the voting
power of an enterprise; or
ww Control of the composition of the board of directors in the case of a company or of the composition of
the corresponding governing body in case of any other enterprise so as to obtain economic benefits
from its activities.
(ii) Minority interest is that part of net result of operation and the net assets of a subsidiary attributable to
interest which are not owned, directly through subsidiary (ies), by the parent.
Consolidated financial statement normally include consolidated balance sheet, consolidated statement of
profit and loss, and notes, other statements and explanatory material that from an integral part thereof,
consolidated cash flow statement is presented in case a parent presents its own cash flow statements.
The consolidated financial statements are presented, to the extent possible, in the same format as that
adopted by the parent for its separate financial statement.

Exclusions
An enterprises may control the composition of the governing bodies of entities such as gratuity trust provident
fund trust etc. since the objective of control over such entities is not to obtain economic benefits from their
activities, these are not considered for the purpose of preparation of consolidate d financial statements.
284 Accounting Standards

A subsidiary should be excluded from consolidation when:


a) control is intended to be temporary because the subsidiary is acquired and held exclusively with a
view to its subsequent disposal in the near future; or
b) It operates under server long-term restrictions which significantly impair its ability to transfer funds
to the parent.

Consolidation Procedures
In preparing consolidated financial statements, the financial statement of the parent and its subsidiaries
should be combined on a line by line basis by adding together like items of assets, liabilities, income arc
expanses. In order that the consolidated financial statement presents financial information about the group
as that of a single enterprise, the following steps should be taken.
a) the cost to the parent of its investment in each subsidiary and the parent’s portion of equity of each
subsidiary at the date on which investment in each subsidiary is made, should be eliminated;
b) any excess of the cost to the parent of its investment in a subsidiary over the parent’s portion of equity
of the subsidiary, at the date on which investment in the subsidiary is made, should be described as
goodwill to be recognized as an asset in the consolidated financial statements;
c) when the cost to the patent of its investment in a subsidiary is less than the parent’s portion of equity
of the subsidiary, at the date on which Investment in the subsidiary is made, the difference should be
treated as a capital reserve in the consolidated financial statement;
d) Minority interest in the net income of consolidated subsidiaries for the reporting period should
be Identified and adjusted against the income of the group in order to arrive at the net income
attributable to owners of the parent; and
e) Minority interests in the net assets of consolidated subsidiaries should be identified and presented in
the consolidated balance sheet separately from liabilities and the equity of the parent’s shareholders.

Different Dates of Financial Statements


If the financial statement of a subsidiary, as on the date of investment, are not available and if it is impracticable
to draw the financial statement of the subsidiary as on that date, financial statement of the Subsidiary for
the immediately preceding period are used as a basis for consolidation. Adjustments are made in financial
statement for the effect of significant transactions or other events that occur between the date of such
financial statement and the date of investment in the subsidiary. In any case difference between reporting
dates should not exceed six months.

Investments on Different Dates


If an enterprise makes two or more investment in another enterprise at different dates and eventually obtains
control of the other, enterprise, the consolidated financial statement are presented only from the date on
which holding-subsidiary relationships comes into existence. If two or more investment are made over a
period of time, the equity of the subsidiary at the date of investment, is generally determined on a step-by-
step basis however, if small investment are made over a period of time and then an investment is made that
result in control, ‘the date of the latest investment, as a practicable measure, may be considered as the date
of investment.

Intragroup Transactions
Intragroup balance and intragroup transitions, including sales, expenses and dividend, are eliminated in full.
Unrealized profits resulting from intragroup transactions that are included in the carrying amount of assets,
unrealized losses resulting from intragroup transitions that are deducted in arriving at the carrying amount
of assets are also eliminated unless cost cannot be recovered.
AS-21: Consolidated Financial Statements (Holding Company Vs Subsidiary) 285

Consolidated financial statements should be prepared using uniform accounting policies for like
transactions and other events in similar circumstances. If is not practicable to use uniform accounting policies
in preparing the consolidated financial statement, that fact should be disclosed together with the proportions
of the item in the consolidated financial statements to which the different accounting policies have been
applied.

Effective Date of Consolidation


The result of operation of a subsidiary is included in the consolidated financial statement as from the date
on which parent-subsidiary relationship came in existence. The result of operations of a subsidiary with
which parent-subsidiary relationship ceases to exist are included in the consolidated statement of profit and
loss until the date of cessation of the relationship. The difference between the proceeds from the disposal
of investment in a subsidiary and the carrying amount of its assets less liabilities as of the date of disposal is
recognized in the consolidated statement of profit and loss as the profit or loss on the disposal of the investment
in the subsidiary. In order to ensure the comparability of the financial statement from one accounting period
to the next, supplementary information is often provided about the effect of the acquisition and disposal of
subsidiaries on the financial position at the reporting date and the result for the reporting period and on the
corresponding amounts for the preceding period.
An investment in an enterprise should be accounted for in accordance with “Accounting Standard (AS)-13
Accounting for Investment, from the date that the enterprise-ceases to be a subsidiary and does not become
an associate.
The carrying amount of the investment at the date that-it ceases to be a subsidiary is regarded as cost
thereafter.

Losses Exceeding Equity for Minority


The losses applicable to the minority in a consolidated subsidiary may exceed the minority interest in equity
of the subsidiary. The excess, and any further losses applicable to the minority, are adjusted against the
majority interest except to the extent that the minority has a binding obligation to, and is able to, make good
the losses.

Accounting in Separate Financial Statements of Parents


In a parent’s separate financial statement, investment in subsidiaries should be accounted for in accordance
with Accounting Standard (AS) 13, Accounting for Investment.

Disclosure
Following disclosure should be made:
(a) In consolidated financial statement a list of all subsidiaries including the name, country of
incorporation or residence, proportion of ownership interest and, if different, proportion of voting
power held;
(b) In consolidated financial statement, where applicable:
(i) The nature of the relationship between the parent and a subsidiary, if the parent does not
own directly or indirectly through subsidiaries, more than one-half of the voting power of the
subsidiaries;
(ii) The effect of the acquisition and disposal of subsidiaries on the financial position at the reporting
date the results for the reporting period and on the corresponding amounts for the preceding
period; and
(iii) The names of the subsidiary (ies) of which reporting date (s) is/are different from that of the
parent and the difference in reporting dates.
286 Accounting Standards

General Clarification Issued by the ICAI


One subsidiary has two parents if an enterprise is controlled by two enterprises- one controls by virtue of
majority voting power and the other controls by virtue of controlling of the composition of the Board of
Directors. In this case there will be two parent/holding enterprises of one subsidiary. The ICAI has clarified
that both the enterprises (Holding) should consolidate the financial statements of the same subsidiary
enterprise.

Tax Expense (GC-18/2002)


It is clarified by the ICAI that Tax expense appearing in the’ separate financial statements of parent and
subsidiaries do not require any adjustments for the purpose of consolidation and therefore simply aggregated
in consolidated financial statements.

Voting power shares are held as stock In trade (GC-17/2002)


It is clarified that where an enterprise owns majority of voting power by virtue of ownership of the share of
another enterprises and all the shares held as stock in trade with a view to their subsequent disposal in the
near future then control will be considered temporary and therefore no consolidation of financial statement
are required to be prepared.

AUTHOR’S NOTES
For Problems on AS-21 Refer Advance Account Book.
VERY VERY IMPORTANT FOR CA FINAL STUDENTS
Even if you know holding company please refer this for better practice
FOLLOWING ARE SOME CONCLUDED POINTS IN HOLDING SUBSIDIARY CONSOLIDATION
All Assets and liabilities: Just add it subject to adjustments.
Share Capital (SC) of Holding: Consolidated Financial Statements.
Reserves of Holding: Consolidated P/L as a working note. Post reserves of S will be included in reserves of
H and then consolidated reserves will be transferred to Balance sheet.
Investments in ESC by H in S: Cost of Control working note.
SC and Reserves of S Co.: Analysis of Net worth of S Co. working note.
Number of working notes: (i) Consolidated B/S, (ii) Consolidated Reserves/P/L of Holding co. (iii) Minority
interest, (iv) Cost of Control, (v) Analysis of Net worth of S Co.
1) Appropriation of Net Worth of S Co.: Net Worth of S Co is distributed into the following 3 parts (i)
Pre Net Worth, (ii) Post Net Worth, (iii) Minority Interest.
2) Can Minority Interest go negative: No
3) Mutual Owing: Between H – S: Full Cancellation, Between H – S2: Full Cancellation, Between S1-S2:
Full Cancellation. (Assume S1 is Subsidiary and S2 is sub subsidiary.)
4) Stock Reserve (Unrealised profit):Between H – S: Full Cancellation, Between H – S2: Full
Cancellation, Between S1-S2: Full Cancellation. (Assume S1 is Subsidiary and S2 is sub subsidiary.)
Entry for cancellation: Consolidated P/L A/c Dr.
 To Stock Reserve A/c
5) Unrealised profit on Fixed Asset: Full Cancellation.
Entry for cancellation: Consolidated P/L A/c Dr.
 To Unrealised Profit from Fixed Asset Reserve a/c
AS-21: Consolidated Financial Statements (Holding Company Vs Subsidiary) 287

Depreciation saved on above if any: (from the date of sale)


Fixed asset A/c Dr.
 To Consolidated P/L A/c
Some authors appropriate the unrealized profit/loss on sale of stock by S to H between H’s Consolidated
P/L and Minority.
6) Revaluation of Fixed Asset: 1) It is the Revaluation of FA of S Co., 2) Revaluation gain/loss =
[Revalued FA on the date of acquisition of interest – Book Value as on that date], 3) Revaluation is
always a Pre P/L, 4) Depreciation on Revaluation is always a Post P/L., 5) If Revaluation is gain then
Depreciation will be loss and wise versa.
7) Current A/c balances between Group: It is not Debtors/Crs or B/Rec or B/Payable. It is Current
A/c balance between group.
• If Current A/c balance tally then just Cancel mutual balance
• If Loan A/c balance does not tally then Provide for the interest and then cancel mutual balance
• If Current A/c balance does not tally (Asset > liability) then It is Cash in transit. Adjust it and then
cancel mutual balance
Recipient will pass the entry:
• Cash in transit Dr.
 To Debtors balance
• If Current A/c balance does not tally (Asset < liability) Keep the net balance as unreconciled amount
in the CFS.
• If BR does not tally with BP (BP > BR) then it indicates BR is discounted or endorsed. Mutual Owings
in that case = [Total Bills drawn among the group – BR discounted or endorsed outside the group].
8) Dividend by S Co.:
i) If S Co has paid Dividend: If H has correctly recorded the dividend then ignore it. If H has not
correctly recorded (ex: pre dividend wrongly recorded in post): then rectify the dividend as less
from Cost of Control (COC) and less from Consolidated P/L.
ii) If S Co has proposed dividend: If H has not recorded dividend receivable: then consider such
proposed dividend by S co in Analysis of Net worth i.e. proposed dividend is just like undistributed
reserve. If H has recorded dividend receivable: then cancel the dividend receivable with proposed
dividend as mutual cancellation and transfer the balance proposed dividend to minority interest
a/c.
iii) If S Co is yet to declare dividend: Ignore it.
iv) Dividend entry for rectifying pre to post:
Debit Investment
To P/L.
But if cancellation of dividend related to S1 receiving from S2:
Debit Investment (Receipt by S1 from S2)
To P/L (Holdings share)
To Minority int.
9) Investment in Preference Share Capital: If H Co has investments in PSC in S Co. following are some
important points:
(i) PSC Face Value and Cost of investments in PS should be treated in the same manner as we treat E
Shares. But the analysis of PS should be separate from analysis from ES. This is because PS and
ES holdings are altogether different.
288 Accounting Standards

(ii) PSH do not share any reserves or profit/loss of S Co.


(iii) Treatment for Dividend paid and proposed is same as ES. But if S Co is yet to declare preference
dividend then it is not same as equity dividend. In case of PS Dividend is yet to be declared by S
Co then H Co should provide for the Preference Dividend i.e. deduct dividend from profits of S
Co and then the balance profits should be distributed to equity shareholders.
(iv) If Preference Dividends are not provided for many years let’s say 4-5 then provide such dividends
from reserves and the balance reserves will be held for H and Minority for distribution.
(v) If whole PSC is held in S Co. by outside party and not by H Co. then the whole PSC will be Minority
Interest.
10) Appropriations of Reserves: Dividend declared by S Co and General Reserve appropriated from
profits should always take colour from its nature of P/L. If Shares are acquired during the year (say
September for March ending) then General Reserve/Dividend appropriated from profits should be
½ pre and ½ post.
11) Investment in Debentures: If H Co has investments in Debentures of S Co. then just cancel as mutual
debt. But if Face value >/< then Cost then treat the difference as Goodwill/Capital Reserve.
12) Investment in Preference Share Capital/Debentures by S Co.: If S Co has investments in PSC/
debn in H Co. then cancel as Mutual Owings. It may so happen that Face value and Cost of investments
may be different then such difference should be treated gain/loss (P/L) and should be distributed
between H Co and Minority interest.
13) Share acquired more than one time: If Lot 1 is small lot and subsequent controlling interest
Lot 2 Analyze pre and post from the date of Lot 2. If Lot1 is controlling interest subsequent small
Lot 2 Analyze pre and post separately for Lot 1 and Lot 2. But remember Dividend received will be
analysed based on each Lot separately governed by AS-13.
14) Chain holding: There are 3 types of Chain holding:
(i) H having interest in 2 S → Consolidation of fellow subsidiaries
(ii) H having interest in S1 and S1 further having interest in S2 → Consolidation of Sub - subsidiaries
(iii) H having interest in S1 as well as in S2 and S1 also holds interest in S2 → H and S together holding
interest in another subsidiary.
Steps to Consolidate Chain Holding:
In case of H having interest in 2 subsidiaries, then consolidation of S Co is as usual i.e. two separate
Subsidiaries.
H having S1 and S1 further having S2, → In case of Consolidation of Sub – subsidiaries, first we have
to analyse the ultimate subsidiary. Analysis of net worth of the ultimate subsidiary is as follows:

NET WORTH OF S2 PRE POST


Holding Co share Cost of Control Consolidated P/L
S1 share Cost of Control Further c/f in analysis of net worth of S1
Minority Interest (Pre + Post) × Minority stake %

DO YOU KNOW
Under the New Schedule VI Minority Interest is a new sub-head within “Equity”.
CHAPTER 28
AS-23: Accounting for
Investment in Associates

Chapter Outline
vv Introduction vv Disclosures
vv Meaning of Associate and Significant Influence vv Interpretations Issued by the ICAI
vv Exception to Equity Method vv Transitional Provision
vv Equity Method Accounting: Para 6

Introduction
Scope: Accounting Standard 23 set out principles and procedures for recognizing effect of the investment in
associates on the financial position and operating results in consolidated financial statements prepared and
presented by an investor.
It is not applicable for separate financial statement prepared and presented by, an investor. (AS-13 applies
for separate financial statements)

Meaning of Associate and Significant Influence


Meaning of Associate: An associate is an enterprise in which the investor has significant influence and which
is neither a subsidiary nor a joint venture of the investor.
Meaning of Significant influence: Significant influence is the power to participate in the financial and\or
operating policy decisions of the investee but not control over those policies. If control exists then AS-21
would apply. The definition of control under AS-21 and AS-23 are similar.
The existence of significant influence by an investor is usually evidenced in one or more of the following
ways:
(a) Representation on the board of directors or corresponding governing body of the investee;
(b) participation in policy making processes;
(c) material transactions between the investor and the investee;
(d) interchange of managerial personnel; or
(e) provision of essential technical information.
For the purpose of this Statement, significant influence does not extends to the power to govern the
financial and/or operating policies of an enterprise.
How Significant influence is gained: Significant influence may be gained by share ownership, statute or
agreement. As regards share ownership if an investor holds, directly or indirectly through subsidiary (ies),
290 Accounting Standards

20% or more of the voting power of the investee, it is presumed that the investor has significant influence,
unless it can be clearly demonstrated that this is not the case. Conversely, if the investor holds, directly or
indirectly through subsidiary(ies), less than 20% of the voting power of the investee, it is presumed that
the investor does not have significant influence, unless such influence can be demonstrated. A substantial
or majority ownership by another investor does not necessarily preclude an investor for having significant
influence.
It is clarified by ICAI that for the purpose of considering 20% of voting power, the potential equity shares
of the investee held by the investor will not be taken into account for determining the voting power of the
investor. Example of Potential equity shares are convertible debentures, convertible preference shares, stock
options.

Exception to Equity Method


Investments in an associate should be accounted for under the equity method except:
(i) Investment is acquired and held for subsequent disposal in the near future.
(ii) Associate operates under severe long-term restriction that significantly impairs its ability to transfer
funds to the investor.
(iii) It has no investments in associates;
(iv) It has investments in associates but has no subsidiaries and therefore consolidation is not applicable;
(v) It is not a listed enterprise and therefore consolidation is not mandatory and it has not chosen to
adopt consolidation voluntarily.
Accounting under equity method should discontinue from the date when it ceases to have significant
influence. In such situation accounting should be done as per AS-13 and the same should be disclosed.

Equity Method Accounting: Para 6 (AS-23)


Equity is the residual interest in the assets of an enterprise after deducting all its liabilities. It means that
equity would include (Share capital + Net reserves).
While drawing CFS a holding company is required to absorb its Associate as per Equity Method. The
following procedure should be followed:—
◘◘ The investment is initially recorded at cost (price paid for) → at the CFS level itself.
◘◘ Identify if any goodwill/capital reserve (not to record goodwill/capital reserve) at the time of
acquisition of investment.
◘◘ Goodwill/Capital Reserve identified at the time of acquisition of investment should be included in the
carrying amount investment in associate but should be disclosed separately.
◘◘ Profits earned or losses suffered by the associate after the date of acquisition should be adjusted in
the Carrying amount of the investments.
◘◘ Distributions received from associate should be reduced from the carrying amount.
◘◘ Adjustment to the carrying amount should be made for alterations in the investor’s proportionate
interest in the associate arising from changes in the associate’s equity that have not been included in
the profit and loss account. Such as changes arising from the revaluation of fixed assets and invest-
ment, from foreign exchange transaction etc.
◘◘ Unrealized profits and losses resulting from transactions between investor and the associate should
be eliminated to the extent of the investor’s interest in the associate. Unrealized losses should not
be eliminated, if the cost of the asset could not be realized (refer special problem).
◘◘ Investor’s share in associates profits or losses should be computed after adjusting dividend on cumu-
lative preference shares held outside the group, whether or not dividend has been declared.
AS-23: Accounting for Investment in Associates 291

◘◘ When the financial statements of associate and investor have different reporting date adjustment
should be made for the effects of any significant event or transaction between the investor and
associate, which occurs between the reporting dates.
◘◘ When the financial statements of associate and investor have different accounting policies for like
transactions and events adjustment should be made, if it is not practicable to do, the fact is disclosed
along with a brief description of the difference between the accounting policies.

Stock Reserve (Unsold Goods × Margin % × Holding’s Share)


For goods sold by A → H: Profit/Loss A/C Dr. xxx
To Stock Reserve xxx
For goods sold by H → A: Profit/Loss A/C Dr. xxx
To Investments A/C xxx

Formulae for Equity Method:


Cost of investments (cash paid) xxx
Less: Pre acquisition dividend (xxx)
Net adjusted cost xxx
Add: Share of Post acquisition increase in net worth
of Associate company due to P/L + revln reserves; etc. xxx
Less: Dividend paid out of the above profits (xxx)
Less: Stock reserve (when H sells goods to A) (xxx)
Less: Preference dividends provided uptill date (xxx)
Carrrying amount of investments xxx

Treatment of Goodwill/Capital Reserves


No treatment of goodwill/capital reserves after its recognition has been specified under the standard. Some
companies amortize goodwill, others test it for impairment.

Carrying Amount of Investment in Associate


If there is permanent decrease in the value of investment in associate, the carrying amount of investment in
associate should be reduced by the amount of permanent reduction as per AS-13.
If investor’s share of losses in associates equals or exceeds the carrying amount of investment, the investor
discontinues recognizing its share of further losses and investment is reported at nil value in CFS as per
AS-23.
Consolidated financial statements: Where an associate presents consolidated financial statements, the
results and net assets to be taken into account are those reported in that associate’s consolidated financial
statements.

Contingencies
In the consolidated financial statement of investor following facts should be disclosed:
◘◘ Its share of the contingencies and capital commitments of an associate for which it is also contingently
liable.
◘◘ Those contingencies that arise because the investor is severally liable for the liabilities of the associate.
292 Accounting Standards

Disclosures
Investor should disclose in its consolidated financial statement the following:—
◘◘ Description of associate including the proportion of ownership interest should be disclosed.
◘◘ Investment in associates accounted for using the equity method should be classified as long-term
investments.
◘◘ Difference in reporting dates of financial statements of associates and of the investor should be
disclosed.
◘◘ In case an associate uses accounting policies other than those adopted for the consolidated financial
statements for like transactions and events in similar circumstances and it is not practicable to make
appropriate adjustments to the associate's financial statements, the fact should be disclosed along
with a brief description of the differences in the accounting policies.

Interpretations Issued by the ICAI


The ICAI has issued following clarifications —

Proposed Dividend by the Associate ASI-16 (V.Imp.)


It is clarified that in case the associate has made a provision for proposed dividend in its financial statements,
the investor’s share of the results of operations of the associate should be computed without taking into
consideration the proposed dividend (refer problem asked in May 2010)(also asked in Audit Nov. 2013).

Changes in Associate’s Equity ASI-17


It is clarified that adjustments to the carrying amount of investment in an associate arising from changes
in the associate’s equity that have not been included in the statement of profit or loss should be directly
adjusted in the carrying amount of investment without routing it through the consolidated statement of
profit and loss. The corresponding debit/credit should be made in the relevant head of the equity interest
in the consolidated balance sheet. For example, in case the adjustment arises because of revaluation of fixed
assets by the associate then the a/c treatment will be:
Investments Dr. xxxx
To Consolidated Revaluation reserves xxxx

Potential Equity Shares and Voting Power ASI-18


It is clarified that, the potential equity shares of the investee held by the investor will not be taken into account
for determining the voting power of the investor. In other words, the voting power should be determined
on the basis of the current outstanding securities with voting rights. For ex: If H Ltd holds 30% stake in its
Associate company A Ltd. H further holds 50,000, 7% Conv Debentures of ` 100 each which will entitle H
Ltd so many shares which will increase the stake by 6%. As per ASI-18 voting power will be determined by
considering only 30% stake and not 36%.

Transitional Provision
On the first occasion when investment in an associate is accounted for in consolidated financial statements
in associate with this statement, the carrying amount of investment in the associate should be brought to the
amount that would have resulted had the equity method of accounting been followed as per this statement
since the acquisition of the associate. The corresponding adjustment in this regard should be made in the
retained earnings in the consolidated financial statements.
AS-23: Accounting for Investment in Associates 293

Accounting Entries for AS-13 and AS-23 (Equity Consolidation) on Comparative Basis
Particulars Separate financial statements Consolidated financial statements
Investments in Associates Investments xx Not required (disclose at cost)
To Bank / Securities xx
Goodwill / Capital Reserves NA Just identify
Stock Reserves NA Consolidated P/L xx
To Stock Reserve xx (A->H)
To Investments xx (H->A)
Pre dividend received Bank xx No entry required if correct entry is done
To Investments xx in SFS
Share of post profits NA Investments xx
(for losses entry will be To Consolidated P/L xx
reversed)
Share of any other reserve NA Investments xx
To Concerned Reserve xx
Dividend distributed from Bank xx Consolidated P/L xx
post profits To P/L xx To Investments xx
Dividend provided Not to be recorded as governed by AS-9 Just ignore it as per ASI -16
(i.e. recorded in the year when AGM will be
held)
Investments w-off as per Diminution in the value xx NA
AS-13 To Investments xx
(charged to P/L)

PROBLEMS AND SOLUTIONS

Problem 1: Consolidated Balance Sheet of H Ltd. group as on 31/3/2012 before adjustment for equity
method for investment in associate A Limited is given below:

Liabilities ` Assets `
Share Capital (` 10) 8,00,000 Sundry Assets 18,50,000
Capital Reserves 20,000 Investments in A Ltd. 1,50,000
Profit/Loss A/c 4,45,000
Minority Interest 1,35,000
Sundry Liabilities 6,00,000
20,00,000 20,00,000

H Limited acquired 25% of equity shares of A Ltd. 01/04/2010 for ` 1,50,000.


Share Capital of A Limited as on 1.4.2010 was ` 4,00,000, P & L A/C. of A Limited as on 1.4.2010 was
` 1,80,000.
Show equity adjustment and Consolidated Balance Sheet of the group, if profits earned for the 2 years was
totalled ` 60,000.
Solution:
Steps for Consolidated as per AS-23 (Equity Method):
294 Accounting Standards

1) Initially recognise investments @Cost.


2) Identify any Goodwill/Capital Reserve if any.
3) Increase the Value of investments once recorded by further profit/loss share in associates.
4) Dividends received is to be deducted from the Carrying amount.
Costs of Investments and G/W/CR:
Cash paid = 150000 (Cost) → Step 1
Less: Share in Net Assets 145000 (400000+180000) 25%
Goodwill 5000 → Step 2
Disclosure in the CFS (B/S) as on 1.4.2010
ASSETS SIDE
Investments in A Limited: 1,45,000
Goodwill (Identify) 5,000
Cost = 1,50,000

Consolidated Balance Sheet of H Ltd. and its group as on 31/3/2012


Particulars Note No. ` in lacs
I. Equity and Liabilities
(1) Shareholders’ Funds
(a) Share capital 8,00,000
(b) Reserves and Surplus 4,80,000
(2) Minority Interest 1,35,000
(3) Current Liabilities 6,00,000
Total 20,15,000
II. Assets
(1) Non-Current assets
(a) Fixed Assets
Tangible Assets 18,50,000
(b) Non-Current Investments (150000 + 15000) 1,65,000
Total 20,15,000

1. Reserves and Surplus


Retained Earnings (4,45,000 + 15,000) = 4,60,000
Capital Reserves 20,000
4,80,000
Problem 2: Consolidated Balance Sheet of H Ltd. group as on 31/3/2012 before adjustment for equity
method for investment in associate A Limited is given below: (` in 000)

Liabilities Group A Ltd. Assets Group A Ltd.


Share Capital (` 10) 800 300 Goodwill 25 —
Revaluation Reserves — 25 Sundry Assets 1,700 465
Profit/Loss A/c 400 140 Investments in A Ltd. 75 —
Minority Interest 125 —
Sundry Liabilities 475 —
1,800 465 1,800 465
AS-23: Accounting for Investment in Associates 295

H Limited acquired 20% of equity shares of A Ltd. 01.04.2010 for ` 75,000. P & L A/c of A Limited as on
1.4.2010 was ` 1,00,000. The assets of A Ltd. were revalued on 31.03.2011.
Show equity adjustment and Consolidated Balance Sheet of the group as on 31.03.2012.
Solution:
Costs of Investments and G/W/CR:
Cash paid = 75000 (Cost) → Step 1
Less: Share in Net Assets 80000 (300000+100000) 20%
Capital Reserves 5000 → Step 2
Disclosure in the CFS (B/S) as on 1.4.2010
ASSETS SIDE
Investments in A Limited:
80000
Capital Reserves (5000) 75000 (Cost) (identify)

Consolidated Balance Sheet of H Ltd. and its group as on 31/3/2012


Particulars Note No.
I. Equity and Liabilities
(1) Shareholders’ Funds
(a) Share capital 8,00,000
(b) Reserves and Surplus 4,13,000
(2) Minority Interest 1,25,000
(3) Current Liabilities 4,75,000
Total 18,13,000
II. Assets
(1) Non-Current assets
(a) Fixed Assets
 Tangible Assets 17,00,000
 Goodwill 25,000
(b) Non-Current Investments # (includes Capital Reserves ` 5,000) 88,000
Total 18,13,000

1. Reserves and Surplus


Retained Earnings (400000 + 8000) = 4,08,000
Revaluation Reserves 5,000
4,13,000
# Investments in A = 75000 + 40000 x 20% + 25000 x 20% = ` 88,000
Problem 3: Grasim Ltd. purchased 25% equity shares of Helium Ltd. on 01.04.2012 at a cost of ` 10 lakhs.
On that date net worth of Helium Ltd. was as under:
Amount (`)
Equity share capital 30 lakhs
Reserves & Surplus 6 lakhs
During the years 2012-13 and 2013-14 Helium Ltd. incurred a loss of ` 25 lakhs and ` 40 lakhs respectively.
Grasim Ltd. has a subsidiary and is required to prepare consolidated financial statements, how the investments
in associates will be shown in consolidated financial statements of Grasim Ltd.?
296 Accounting Standards

Solution:
Costs of Investments and G/W/CR:
Cash paid = 1000000 (Cost) → Step 1
Less: Share in Net Assets 900000 (3000000 + 600000) 25%
Goodwill 100000 → Step 2
Disclosure in the CFS (B/S) as on 1.4.2012
ASSETS SIDE
Investments in Helium Limited: 1000000
Goodwill (identify) = [100000]
Disclosure in the CFS (B/S) as on 31.3.2013
ASSETS SIDE
Investments in Helium Limited: 1000000
Goodwill (identify) [100000]
Less: Share of losses (625000)
(2500000 × 25%) 3,75,000
Disclosure in the CFS (B/S) as on 31.3.2014
ASSETS SIDE
Investments in Helium Limited: Nil (375000 – 4000000 × 25%)
If investors share of losses in associates equals or exceeds the carrying amount of investment, then the investor
discontinues recognizing its share of losses and investments is reported at nil value.
Problem 4: A Ltd. acquired 25% of shares in B Ltd. as on 31.3.2002 for ` 3 lakhs. The Balance Sheet of B Ltd.
as on 31.3.2002 is given below: `
Share Capital 5,00,000
Reserves and Surplus 5,00,000
10,00,000
Fixed Assets 5,00,000
Investments 2,00,000
Current Assets 3,00,000
10,00,000
During the year ended 31.3.2003 the following are the additional information available:
i. A Ltd. received dividend from B Ltd., for the year ended 31.3.2002 at 40% from the Reserves.
ii. B Ltd., made a profit after tax of ` 7 lakhs for the year ended 31.3.2003.
iii. B Ltd., declared a dividend @ 50% for the year ended 31.3.2003 on 30.4.2003.
A Ltd. is preparing Consolidated Financial Statements in accordance with AS-21 for its various subsidiaries.
Calculate:
i. Goodwill if any on acquisition of B Ltd.’s shares.
ii. How A Ltd., will reflect the value of investment in B Ltd., in the Consolidated Financial Statements?
iii. How the dividend received from B Ltd. will be shown in the Consolidated Financial Statements?
(C. A. Final May 2003) (6 Marks)
Solution: In terms of AS 23 B Ltd. will be considered as an associated company of A Ltd. as shares acquired
are more than 20%.
i. Calculation of Goodwill ` in lakhs
Cost of investment 3.00
Less: Share in the value of Equity of B Ltd. as at the date of investment
AS-23: Accounting for Investment in Associates 297

[25% of ` 10 lakhs (` 5 lakhs + ` 5 lakhs)] 2.50


Goodwill 0.50
ii. A Ltd. Consolidated Profit and Loss Account for the year ended 31st March, 2003 ` in lakhs
By Share of profits in B Ltd. 1.75
By Dividend received from B Ltd. 0.50
Transfer to investment account (pre) 0.50
Nil 
iii. A Ltd. Consolidated Balance Sheet as on 31.3.2003
Cost of investments (3,00,000 - 50,000) = 2,50,000
Add: share of profit for 2002-03
Carrying amt of investments 1,75,000
4,25,000
Problem 5: Ranbaxy Ltd. invests in 30% equity in Biotech Ltd. at ` 3 crores when the net assets of the
investee were ` 15 crores. Profit earned by B Ltd during the year is ` 2 crores. By the year end ` 5 lakhs goods
are remaining unsold which was previously sold by R Ltd to B Ltd. Profit margin on stock is 20% on sales.
Compute the carrying amount of investments in standalone a/cs as well as CFS?
Solution:
Carrying amount on standalone basis investments = ` 3crores
In CFS: Investments = 3cr + 0.60 cr – 0.01cr x 30% = ` 35970000
Problem 6: Maruti Udyog acquired 10% of shares in Tata Motors as on 31.3.2010 for ` 200,000. Net assets
as on 31.3.2010 was ` 1500,000. Again it acquired 10% shares when the net assets was 2500,000 paying
` 400,000. Compute the G/W/Capital Reserve.
Solution:
Costs of Investments and G/W/CR: total lot (provisions are similar to AS-21)
Total cash paid = (cost of both the lots) 6,00,000 (Cost)
Less: Share in Net Assets 5,00,000 (2500,000 × 20%)
Goodwill (identify) 1,00,000
Problem 7: Bright Limited acquired 30% of shares of East India for ` 2,00,000 in 01/06/2009. During
the financial year ending on 31/3/2009 East India has earned profits ` 80,000 and declared a dividend of
` 50,000 on 12/08/2009. East India has earned profits ` 3,00,000 and declared a dividend of ` 60,000 on
12/06/2010 for the year ending 31/3/2010.
Calculate the carrying amount of investments.
i) Separate FS of B Ltd as on 31/3/2010.
ii) Consolidated Financial Statement (CFS) of B Ltd as on 31/3/2010.
iii) Carrying amount as on 30/6/2010 in CFS. (CA Final Nov 2010) 8 Marks
Solution:
i) Carrying amt of investments in separate financial statements as on 31.3.2010
Cash paid ` 2,00,000 – Pre dividend 15,000 = 1,85,000 as per AS-13
ii) Carrying amt of investments (CFS) as on 31.3.2010
Cost of investments = 1,85,000
Add: Share of post reserves 75,000 (300000 × 30%) 10/12
2,60,000
iii) Carrying amt of investments (CFS) as on 31.6.2010
298 Accounting Standards

Cost of investments 1,85,000


- Pre Dividend received (3,000) (60,000 × 30%) 2/12
Net cost as per AS-13 1,82,000
Add: Share of post reserves # 60,000
2,42,000
# Profits of 2008-09 = Full Pre
Profits of 2009-10 = (300000 – 60000) 30% x 10/12 = ` 60,000
In this problem we cannot calculate G/W or Capital Reserve on the date of acquisition because
complete net worth of Associate company is not given.
P roblem 8: Valuation Limited acquired 20% of shares of Weiging India for ` 1,00,000 in 01/04/2005. During
the financial year ending on 31/3/2005 Weiging India has Share Capital of ` 2,00,000 with accumulated
reserves of ` 2,00,000. Net profit after tax earned during various years is as follows:

Year NPAT Dividends


2005-06 40,000 10%
2006-07 50,000 10%
2007-08 90,000 15%
2008-09 (10,000) 30%
2009-10 40,000 0
2010-11 50,000 10%

On 1.4.2010 8% Debentures were issued with face value ` 2,00,000 and collected Securities Premium
` 30,000. On 1.6.2010 12% Preference Shares were issued at a face value of ` 1,00,000 and collected Securities
Premium ` 10,000.
Calculate Carrying amount of investments for every year till 31.3.2011 as per AS-23. Assume that the final
dividends were declared in the month of September every year pertaining to previous year. Ex: 10% dividend
rate in 2005-06 pertains to appropriations from 2004-05. Also calculate G/W or Capital reserve as per AS-23.
Solution:
Carrying amt 31/3/2006 31/3/2007 31/3/2008
Opening Bal 1,00,000 1,04,000 1,10,000
(–) Pre Dividend (4,000) Nil Nil
Opening Carrying amt (a) 96,000 1,04,000 1,10,000
+ Profits/Reserves i 40,000 50,000 90,000
Less: Dividend Distributions ii Nil (20,000) (30,000)
i + ii 40,000 30000 60000
Share 20% (b) 8,000 6,000 12,000
Closing Carrying amt (a + b) 1,04,000 1,10,000 1,22,000
Carrying amt 31/3/2009 31/3/2010 31/3/2011
Opening Bal 1,22,000 1,08,000 1,16,000
(–) Pre Dividend Nil Nil Nil
Opening Carrying amt (a) 1,22,000 1,08,000 1,16,000
+ Profits/Reserves i (10,000) 40,000 *80,000
Less: Dividend Distributions ii (60,000) Nil (20,000)
i + ii (70,000) 40,000 60000
Share 20% (b) (14,000) 8,000 12,000
Closing Carrying amt (a + b) 1,08,000 1,16,000 1,28,000
AS-23: Accounting for Investment in Associates 299

50,000 Profits earned – Pref Dividend 10,000 + 40,000 Securities Premium.


Problem 9: BPL has made 25% investments in CPL. CPL is an associate of BPL. CPL sold an equipment of
` 105,000 to BPL Ltd. at ` 82,000. Assuming BPL is required to prepare the consolidated financial statement,
therefore AS-23 is applicable and equipment is still with BPL. Show the adjustment of unrealised losses if the
recoverable value of the asset is (i) 1,10,000, (ii) 75,000, (iii) 90,000.
Solution: There is an unrealized loss of `23,000 as per AS-23. Unrealized profit/loss should be eliminated to
the extent of 25% of 23,000 = 5,750, but it should be eliminated only when recoverable amount of equipment
is equal to ` 1,05,000 or more. If recoverable amount of equipment is less than 1,05,000, loss should not be
eliminated. The meaning of recoverable amount is as per AS-28, Impairment of asset.

If recoverable Amt is → ` 1,10,000 ` 75,000 ` 90,000


Journal Entry for recognsing unrealised losses Equipment Dr  5,750 No entry Equipment Dr  2,000
  To P/L 5,750   To P/L 2,000
(90000 –82000) 25%

P roblem 10: A Ltd. has a share capital of 50,000 shares @ `100 per share. H Ltd. Acquired 15% shares in
A Ltd. on 1.4.2013. It also acquired all the 5,000, 12% convertible debentures of `100 each of A Ltd. These
debentures will be converted at par into equity shares of A Ltd. after 3 years. State whether A Ltd. is an
Associate of H Ltd. or not with reasons?
Solution: As per para 3 of AS 23 ‘Accounting for Investments in Associates in Consolidated Financial
Statements’, an associate is an enterprise in which the investor has significant influence and which is neither
a subsidiary nor a joint venture of the investor.
Standard further explains in para 4 that as regards share ownership, if an investor holds, directly or
indirectly through subsidiary (ies), 20% or more of the voting power of the investee, it is presumed that the
investor has significant influence, unless it can be clearly demonstrated that this is not the case. Conversely,
if the investor holds, directly or indirectly through subsidiary (ies), less than 20% of the voting power of the
investee, it is presumed that the investor does not have significant influence, unless such influence can be
clearly demonstrated.
Further as per an explanation to para 4 of the standard, for the purpose of classification of associate, the
potential equity shares of the investee held by the investor will not be taken into account for determining
the voting power of the investor. In other words the voting power should be determined on the basis of the
current outstanding securities with voting rights.
As per the information given in the question, H Ltd. presently holds indirectly 22.7% shares (with and
without voting rights) (Refer W.N.) in A Ltd. However, the current outstanding securities with voting rights in
A Ltd. is only 15% and the remaining holding is on account of potential equity shares. Since potential equity
shares do not have voting rights they will not be taken into consideration while determining the significant
influence of H Ltd. on A Ltd. Hence, A Ltd. is not an associate of H Ltd.
Working Note:
Calculation of percentage of holding of shares after conversion `
Current holding is 15% i.e. 7,500 shares of `100 each 7,50,000
Potential equity shares i.e. 5,000 shares of `100 each 5,00,000
12,50,000
Total share capital of A Ltd. after conversion of debentures into equity shares will be
 = `50,00,000 + `5,00,000 = `55,00,000
Percentage of holding = `(12,50,000/55,00,000) × 100 = 22.7% approx.
CHAPTER 29
AS-27: Financial Interest
in Joint Ventures

Chapter Outline
vv Detailed Discussion on the Elements of Joint Venture vv Transactions between Venturer and Joint Venture
vv Forms of Joint Venture vv Distinction between JCO/JCA and JCE
vv Jointly Controlled Operations (JCO) vv Operators Fee
vv Jointly Controlled Assets (JCA) vv Disclosures Requirements
vv Jointly Controlled Entity (JCE)

SOME DEFINITIONS
(i) Joint Venture: Joint Venture is a contract between two or more parties to undertake an economic activity,
under joint control. Thus it is a commercial undertaking by two or more persons, which relates to disposition
of goods or completion of single project. Joint Venture are in 3 forms: (i) Jointly Controlled Operations, (ii)
Jointly Controlled Assets, (iii) Jointly Controlled Entities.
Features:
ww Agreement between two or more parties.
ww The agreement is made to execute a specific venture
ww Parties can be partnership firm, individuals, limited companies, AOP’s etc.
ww The main purpose of setting JV is to undertake business activity and earn profit.
ww Formation of JV can be same as venturers or a totally separate entity.
ww JV’s are mostly entered for business projects like chartering of ship, engineering projects, building
dams/bridges etc. JV entered between NMDC (State Owned Enterprise) and Tata Steel for extracting,
exploring and developing mines.
(ii) Venturer: Venturer is a party to the Joint Venture having joint control over the Joint Venture. A Venturer
should record investments in JV as per AS-27.
(iii) Investor: Even investor is a party to a Joint Venture but does not have joint control. An investor should
record investments in JV as per AS-13/A-S21/AS-23.
AS-27: Financial Interest in Joint Ventures 301

Detailed Discussion on the Elements of Joint Venture


Joint Control
It is a strong element in the definition of Joint Venture.
◘◘ Joint Control is the power to direct the financial and operating policies of the Venture mutually to reap
profits.
◘◘ Joint Control requires that all the venturer’s has to agree on every important decision.
◘◘ No single venturer should be in a position to control the complete activities of the Venture.
◘◘ ‘Control’ is different from ‘influence’ as defined in AS-18 and AS-23.
◘◘ Operator is one of the venturer having no joint control. He is paid remuneration for his services.
◘◘ Ratio of shareholding in JV is irrelevant, only the inclusion of the word “Joint Control” in the contrac-
tual agreement is important.
◘◘ Another test for existence of joint control is whether the agreement provides for protective rights or
participative rights. Existence of participating rights is joint control. Protective rights means right to
safeguard interests in the venture against adverse conditions. Participative rights means participating
in the day to day financial and operating policies of the venture.

Nature of Contract
The nature of contractual arrangement entered between the venturers is enumerated below:
◘◘ First of all Contracts should be in black and white.
◘◘ It is the contract entered between venturers which shows existence of Joint Control.
◘◘ Contract can be in the form of general agreement, Articles, Bye-laws.
◘◘ Contract may contain all or some of the following matters:
ww Nature of Activity, Period and reporting obligations of JV.
ww Appointment of BOD or any other similar governing body.
ww Capital Contributions by Venturers.
ww Voting rights of Venturers.
ww Sharing of output, income, expenses or results of the joint venture.

Forms of Joint Venture


Joint Venture can be in 3 forms: (i) Jointly Controlled Operations, (ii) Jointly Controlled Assets, (iii) Jointly
Controlled Entities.
All the three forms of JV have the following features in common:
◘◘ Two or more parties.
◘◘ Bounded by contractual arrangement.
◘◘ Existence of Joint Control.
However forms of JV explained separately is as follows:

Jointly Controlled Operations (JCO)


◘◘ JV in the form of Jointly Controlled Operations involves the employment of assets, resources, exper-
tise, time etc. to manufacture or market a particular product.
◘◘ Formation of JV is not separate from its venturers. In other words existence of JV is not a separate legal
entity like company, firm etc.
◘◘ Venturer carries out joint ventures activities along with their own business activities.
302 Accounting Standards

◘◘ Venturer Contributes out its own Assets/inventories etc. Even it raises liabilities on their own.
◘◘ No separate accounting records are required to be maintained for the JV by the Venturer except a need
arises for internal reporting purposes.
◘◘ Agreement between the venturers explains the profit sharing norms, sharing of output, sharing of
expenses etc.
Example of JCO: A & B are engaged in construction business. C is having his own advertising agency. A, B,
and C decided to combine their assets, operations, resources and expertise to manufacture a Steamer. A & B
goes for manufacturing the Steamer. C goes for marketing and distributing activities. Each venturer incur its
own expenses and share profits as per the agreement entered into.

Accounting for Jointly Controlled Operations (JCO)


JV in the form of Jointly Controlled Operations has to record the following in its separate financial statements
or Consolidated financial statements (CFS) as the case may be:
◘◘ Assets that it holds.
◘◘ Liabilities it incurs
◘◘ Expenses it incurs.
◘◘ Share of revenue from JCO.
If once accounting effect of transactions relating to JCO is done in separate financial statement no further
adjustment is required in CFS.

Jointly Controlled Assets (JCA)


◘◘ JV in the form of Jointly Control Assets involves the joint control & ownership of assets.
◘◘ Share of income in the form of Rent/other revenue is derived from the joint assets.
◘◘ Common expenses are shared as per the contract.
◘◘ Formation of JV is not separate from its venturers. In other words existence of JV is not a separate legal
entity like company, firm etc.
Example of JCA: P Ltd. & S Ltd. are oil producing companies. They jointly own Oil pipeline (Asset). Oil
pipeline is jointly owned by them and hence they will incur common expenses related to the pipeline.
Example of JCA: A Ltd. & B Ltd. acquired a Machine of ` 50 crores. They share rental income as per the
contract entered between them.

Accounting for Jointly Controlled Assets (JCA)


JV in the form of Jointly Controlled Operations has to record the following in its separate financial statements
or Consolidated financial statements (CFS) as the case may be:
◘◘ Share of assets that it owns/control.
◘◘ Liabilities it incurs separately.
◘◘ Expenses it incurs separately.
◘◘ Expenses it incurs in common.
◘◘ Income from sale of share of output.

Jointly Controlled Entity (JCE)


◘◘ Unlike JCO & JCA, Joint Venture in the form of Jointly Control Entity enjoys a separate taxable and legal
entity. Separate entity may be in the form of Corporation, Firm etc.
◘◘ JCE itself controls the Assets of the JV, incurs liabilities and expenses and earns income.
AS-27: Financial Interest in Joint Ventures 303

◘◘ Even JCE on their own enters into contracts with outsiders.


◘◘ JCE maintains separate accounting records.
◘◘ Sharing of income or expenses between the venturers does not arise because it forms part of JCE’s
Incomes & Expenses.
◘◘ Investors have only joint interest in the venture.
Example of JCE: Mr. X and Mr. Y decided to construct a tunnel for which they form a new entity M/S. XY &
Co. Assets contributed by them is in the ratio of 70:30. Profit Sharing ratio is 60:40.

Accounting for Jointly Controlled Entity (JCE)


◘◘ In case of separate financial statements:
In the books of venturer resources invested in the JV business will be accounted as an “Investment”
AS-13. Profit/Loss earned by the JV will not form part of investment of venturers, but will be trans-
ferred to their individual profit and loss account.
◘◘ In case of Consolidated financial statements :
If the venturer is required to prepare CFS, then interest in a JCE should be reported as per “Proportionate
Consolidated Method.”
In some exceptional cases, an enterprise (Say H Ltd.) by a contractual arrangement may establish joint
control over an entity (Say S Ltd.) which is a subsidiary of that enterprise (i.e. H Ltd.) within the meaning
of Accounting Standard (AS) 21, Consolidated Financial Statements. In such cases, the entity (i.e. S Ltd.)
is consolidated under AS 21 by the said enterprise (i.e. H Ltd.), and is not treated as a joint venture. The
consolidation of such an entity does not necessarily preclude other venture (s) treating such an entity as a
joint venture. (Limited Revision; Paragraph 6)

What is Proportionate Consolidated Method (PCM)?


It applies to the entities having subsidiary. While preparing consolidated accounts, share of income and
expenses from JCE will be consolidated in the books of venturer. Similarly, share of assets and liabilities in
JCE will be consolidated in the books of venturer.
Other points regarding Proportionate Consolidated Method (PCM):
PCM reflects the actual economic reality or substance rather than the form of joint venture. This is a good
example of “substance over form”.
◘◘ Specialty of PCM is that share of each asset, liability, income and expense is clearly reflected in the
financial accounts of the venturer.
◘◘ Consolidation procedures are similar as prescribed in AS-21 (including different reporting dates,
different accounting policies etc.). But the difference in AS-21 and PCM is in case of PCM only propor-
tionate consolidation is possible and not full consolidation unlike AS-21.
◘◘ If Net Assets share in venturer is > then its cost of investment, Capital Reserve emerges, where If Net
Assets share of venturer is < then its cost of investment, Goodwill emerges.
◘◘ Netting off of Assets with liabilities or income with expenses is not allowed under PCM unless a legal
right of set-off exists.
◘◘ Any share of losses exceeding the interest in the equity by the venturer should be recognized. But if
JCE in future starts making profits the losses previously absorbed by the venturers has to be adjusted
first then the balance is to be shared among the venturers…..para 38

Exceptions to Proportionate Consolidated Method (PCM)


A venturer should discontinue the use of PCM from the date:
(i) Venturer ceases to have interest in the JV but retains, either in full or partly its interest in the JCE.
304 Accounting Standards

(ii) JCE operates under severe long term restrictions that significantly impair its ability to transfer funds
to the venturer. Ex. A Ltd. has a Venture in Iraq. From a long time flow of funds is not possible. In such
a case venturer should not go for PCM.
How interest in JCE is to be accounted after discontinuance of PCM:

Situation Accounting treatment


Unilateral/Complete control over the JCE and becomes parent As per AS – 21
Investors relation is of associate As per AS – 23
In any other case As per AS – 13

Transactions between Venturer and Joint Venture


In case of JCE
In separate financial statements it is a normal sale or purchase. But in case of CFS following provisions will
be applicable.
◘◘ Sale of Asset: Conditions for Sale of Asset are:
(i) There is a Sale of asset by the venturer to JCO or JCA. (ii) Sale of asset can be of Current Assets or
Fixed Assets. (iii) The venturer should transfer all the significant risks and rewards of ownership to
the JV. (iv) The JV retains the Asset.

Accounting for Sale of Asset by Venturer to JCE: para 41 of AS-27


Possibility Effect
1) Profit on sale of asset Venturer should recognize profit only to the extent of
other venturers share (i.e. own share of profit should be
eliminated).
2) Loss on sale of asset Venturer should recognize loss only to the extent of other
venturers share (i.e. own share of loss should be eliminated).
3) Loss on sale of asset; if sale evidence reduction in NRV of Venturer should recognize “full” loss on the transaction.
stock or impairment in case of fixed assets

Accounting for Purchase of Asset by Venturer from the Joint Venture:


◘◘ Purchase of Asset: Conditions for Purchase of Asset are:
i) There is a Purchase of asset by the venturer from JCO or JCA. ii) Purchase of asset can be of Current
Assets or Fixed Assets. iii) The JV should transfer all the significant risks and rewards of ownership to
the venturer. (iv) TheVenturer retains the Asset.

Accounting for Sale of Asset by JCE to Venture: para 42 of AS-27


JV records it as a sale of asset in normal manner. But the sale of asset gives rise to profit or loss to the Joint
Venture. Also the profit or loss will be further shared between the venturers. How a venturer records profit
or loss on the transaction is tabulated below:

Possibility Effect
1) Profit to JV Venturer should recognize profit only to the extent of
other venturers share (i.e. own share of profit should be
eliminated).
2) Loss to JV Venturer should recognize loss only to the extent of other
venturersshare(i.e. own share of loss should be eliminated).
AS-27: Financial Interest in Joint Ventures 305

3) Loss to JV; if sale evidence reduction in NRV of current Venturer should recognize his share of loss immediately.
assets or impairment in case of fixed assets

Note: Provisions of para 41 and 42 are applicable only in case of CFS and not stand alone financial statements.

DO YOU KNOW
Accounting treatment of Venturer and JCE is similar to Holding and subsidiary. The difference is that in case of
H & S we adopt full consolidation and in case of H & JCE we adopt proportionate consolidation.

Distinction between JCO/JCA and JCE


JCO/JCA JCE
i) Existence of JCO/JCA is same as of the venturers i) Altogether JCE is a separate legal entity. JCO can stand
on its own legs.
ii) JCO/JCA is totally dependent upon the venturers for ii) JCE can purchase its own assets, incur expenses. Even it
Assets and liabilities. can enter into contracts in its own name. Thus, JCO can
stand on its own legs.
iii) Separate books of accounts of Venturer and JV is iii) Venturer and JV prepares separate books of accounts.
optional.
iv) No proportional consolidated method is applicable. iv) Proportional consolidated method is applicable.

Operators Fee
Joint Venture may appoint any outside person or one among the venturers as the Operator or Manager. Fee is
paid for such services rendered. Accounting is as follows:
In the Books of the Operator: Operator Fee receipt for a operator is a income to be recognized as per AS-9
Revenue Recognition.
In the Books of Joint Venture: For JV Operator Fee is just like any other expense.
In the Books of Joint Venture: Para 36: Intercompany transaction will not be eliminated. Offset is possible
only when there is a legal right to offset the asset and liability.

Disclosures Requirements
Applicability of Disclosures
i) Disclosures enumerated below are to be reflected in the books of the Venturer.
ii) Disclosures are common for both separate financial statements and CFS.
iii) Disclosures are common for whether it is JCO/JCA/JCE.

Disclosures
◘◘ List of Joint Ventures.
◘◘ Description of interest in JV.
◘◘ Proportion of interest in JV.
◘◘ In case of JCE only, the proportion of ownership interest.
◘◘ A venturer should disclose the contingent liabilities, separately from other contingent liabilities.
i) Venturers share of own interest in contingent liabilities.
306 Accounting Standards

ii) Share of contingent liabilities incurred jointly with other co-venturers.


iii) Contingent liabilities in case of contingency i.e. in case of other venturers share.
◘◘ A venturer should disclose the capital commitments incurred.
i) Venturers share of own capital commitments pending.
ii) Share of capital commitment pending jointly with other co-venturers.
iii) Capital commitment pending for the JV as a whole and venutrers share thereof.

PROBLEMS AND SOLUTIONS

Problem 1:
Case 1) H Ltd does not have any subsidiary but have a JV @40% share.
Case 2) H Ltd have its subsidiary as well as a JV J Ltd @60% share.
Case 3) H Ltd have its subsidiary as well as a JV J Ltd @40% share.
Case 4) H Ltd have its subsidiary as well as a JV J Ltd @50% share.
Solution: Proportionate consolidate method (PCM) will be applied only when a company has its separate JV
with a different identity. But such company should have its subsidiary too. Further if share in JV is more than
50% then full consolidation will be applicable for JV as per AS-21.
Case 1) PCM is not applicable as H Ltd does not have its subsidiary.
Case 2) If H Ltd has a voting power more than 50% in JV then the holding company should adopt Full
consolidation. Here H Ltd has invested more than 50% in JV hence Full Consolidation is applicable.
Case 3) Here PCM will be applicable.
Case 4) Here PCM will be applicable.
Problem 2: Following is the Trial Balance of Monty Limited for the year ended 31/3/2013 is given below:
Dr. Balance ` in lakhs Cr. Balance ` In lakhs
Opening Stock 160 Sales 1,300
Purchases 1,040 Share Capital (` 10) 1,000
Expenses 150 Profit & Loss (31/3/2012) 80
Depreciation 70 Suspense A/C 20
Fixed Assets 630
Debtors 250
Bank 100
2,400 2,400

During the year 2012-13, Monty Ltd. entered into a JV with Ponty Ltd to buy and sell used machines. The
profits & losses of the venture are to be shared equally. The balance of Suspense a/c represents transactions
of Monty Ltd for the venture as per details given below:

` Lakhs ` Lakhs
Sold 16 machines for the venture @ 5 lakh 80
Purchased 14 machines for the venture @ 4 lakh 56
Expenses incurred on behalf of JV 4
Balance of Suspense a/c 20
AS-27: Financial Interest in Joint Ventures 307

Ponty Ltd purchased 18 machines @4 lakh per machine and sold 10 machines @5 lakh. Expenses incurred
on behalf of JV ` 6 lakh.
Value of closing stock of Monty Ltd excluding stock of JV was ` 168 lakhs.
Prepare profit & loss a/c for the year ended 31/3/2013 and its Balance sheet as at that date. If Monty Ltd
has a Subsidiary/Holding Co. then what additional/special entries are required to be passed for CFS.
Solution:
IT’S A CASE OF JCO: (Books of Monty)
1) Share of Monty Ltd in JV purchase = 50% (32 machines x 4 lakh) = ` 64,00,000
2) Share of Monty Ltd in JV sales = 50% (26 machines x 5 lakh) = ` 65,00,000
3) Share of Monty Ltd in JV expenses = 50% (4 lakh + 6 lakh) = ` 5,00,000
4) Share of Monty Ltd in JV stock = 50% (4 lakh x 6) = ` 12,00,000
5) Adjusting entry:
Purchase A/C. Dr. 64,00,000
Expenses A/C. Dr. 5,00,000
Suspense A/C. Dr. 20,00,000
   To Sales A/C. 65,00,000
   To Ponty Ltd (bal fig) 24,00,000
5) Total value of Closing stock of Monty Ltd = ` 168 lakhs + 12 lakhs = ` 180 lakhs
Profit and Loss Statement 31/3/2013 (` in lakhs)
`
Sales 1365
Less: Expenditure
Purchases Cost of goods (64+1040) (1104)
Changes in inventories (180-160) 20
Depreciation (70 + 0) (70)
Other expenses (150 + 5) (155)
Net Profit 56

Consolidated Balance Sheet of Monty Ltd. group as on 31/3/2013 (` in lakhs)


Particulars ` in lacs
I. Equity and Liabilities
(1) Shareholders’ Funds
(a) Share capital 1000
(b) Reserves and Surplus (P/L) 80+56 136
(3) Current Liabilities (Ponty Ltd) 24
Total 1,160
II. Assets
(1) Non-Current assets
Fixed Tangible Assets
(2) Current assets 630
(a) Inventories 180
(b) Trade receivables 250
(c) Bank 100
Total 1,160
308 Accounting Standards

Once Monty Ltd merges the transactions of JV with its own business then there is no need to once again
pass any separate entries for CFS with group accounts subsequently applicable to JCO.
Problem 3: On 1/4/2012 Salvi Ltd and Dalvi Ltd entered into a JV to purchase Volvo Buses and to let out
on rental basis. Cost of the total fleet of assets amounted to ` 100 lakhs. Stamp duty, Registration charges etc
amounts to 2%. Salvi and Dalvi decided to record the Jointly Controlled Asset in equal ratio. Salvi raised a
loan of ` 70lakhs and the balance cost was financed by Dalvi @12% p.a. Yearly hire charges collections made
by Salvi was ` 9 lakhs and Dalvi ` 21 lakhs. Expenses incurred out of pocket for JV (including AMC): Salvi `
2 lakhs and Dalvi ` 5 lakhs. Sharing of revenue and expenses is decided as 3:2 between Salvi and Dalvi. For
your further information interest on loan was outstanding and unpaid by the year end. Life of the Buses were
estimated to be 10 years. Compute the profit/loss of each coventurer for the year 2012-13. Also pass journals
in the books of accounts of each venturer.
Solution:

Calculation of Profit/Loss on JCA (` In lakhs)


Particulars Total Ratio Salvi Ltd. Dalvi Ltd.
Income (Collections) 30 3:2 18 12
(–) Expenditure
Expenses (7) 3:2 (4.2) (2.8)
Depreciation (10.2) 1:1 (5.1) (5.1)
Interest (12.24) — (8.4) (3.84)
Net Profit 0.56 0.30 0.26

Interest is calculated in the ratio of Loans and Depreciation in the ratio of share of assets.

Journals in the books of Co-venturers (` In lakhs)


Books of Salvi Books of Dalvi
Asset a/c Dr. 51 Asset a/c Dr. 51
Dalvi a/c Dr. 19 To Loan 32
To Loan 70 To Salvi 19
Cash a/c Dr. 9 Cash a/c Dr. 21
Dalvi a/c Dr. 9 To Income 12
To Income 18 To Salvi 9
Expenses a/c Dr. 4.2 Expenses a/c Dr. 2.8
To Cash 2 Salvi a/c Dr. 2.2
To Dalvi 2.2 To Cash 5
Depreciation a/c Dr. 5.1 Depreciation a/c Dr. 5.1
To Assets a/c. 5.1 To Assets a/c 5.1
Interest a/c Dr. 8.4 Interest a/c Dr. 3.84
To O/S Interest 8.4 To O/S Interest 3.84

The above entries get absorbed in the separate financial statements of respective Venturer.
AS-27: Financial Interest in Joint Ventures 309

Problem 4:
Mapro Ltd. JCE Ltd. JCE Ltd.
closing B/S Opening B/S
Share Capital 1,00,000 50,000 50,000
Reserve & Surplus 20,000 30,000 10,000
Loan and other liabilities 30,000 10,000 10.000
Total 150,000 90,000 70,000
Fixed Assets Net 1,10,000 60,000 60,000
Investment in JCE 40% 38,000 — —
Inventory 2,000 30,000 10.000
Total 1,50,000 90,000 70,000

1. Above is the Mapro Ltd closing consolidated B/S without JCE.


2. Mapro Ltd obtained Joint control on the opening b/s date by acquiring 40% equity.
3. During the year Mapro Ltd sold goods worth ` 5,000 @ 10% margin to JCE Ltd. which in turn was
sold to a third party
4. During the year Mapro Ltd. sold goods at ` 4,000 @ 25% margin on sale to JCE Ltd. all of which was
lying in stock.
Prepare Consolidated financial statement of Mapro Ltd. including JCE.
Solution:
Consolidated Balance Sheet of Mapro Ltd as on _______
Applying Proportionate Consolidation for JCE

Particulars ` in lacs
I. Equity and Liabilities
(1) Shareholders’ Funds
(a) Share capital 1,00,000
(b) Reserves and Surplus (20000 + 40% (30000 – 10000) – 400 27,600
(3) Non – Current Liabilities: Loans etc (30000 + 10000 × 40%) 34,000
Total 1,61,600
II. Assets
(1) Non-Current assets
(a) Fixed Tangible Assets (110000 + 40% × 60000)
(b) Goodwill 38000- 40% of (50000+10000) 1,34,000
(2) Current assets 14,000
(a) Inventories (2,000 + 40%x30,000 – 400 unrealised profit) 13,600
Total 1,61,600

Problem 5: Optimix Ltd has a share of 40% in the Share Capital of Joint Venture in the form of jointly
controlled entity. As per the agreement O Ltd does not have any joint control but can only be mere an investor.
Comment whether AS-27 is applicable to O Ltd.
Solution: Optimix Ltd has a share of 40% in the Share Capital of Joint Venture in the form of jointly controlled
entity. As per the agreement O Ltd does not have any joint control but can only be mere an investor. Here
AS-13 for standalone a/cs and AS-23 for CFS (equity method) will be applied for O Ltd. But AS-27 PCM will
not be applicable.
CHAPTER 30
Comprehensive Problems on CFS
(AS-21 + AS-23 + AS-27) Group
Consolidation

Problem 1: P Ltd. owns 80% of S, 40% of J and 40% of A. J is jointly controlled entity and A is an associate
Balance Sheet of four companies as on 31.3.2009 are: (Rs in lakhs)

Assets P Ltd. S J A
Investment in S 800 -
Investment in J 600
Investment in A 600
Fixed assets 1000 800 1400 1000
Current assets 2200 3300 3250 3650
Total 5200 4100 4650 4650

Liabilities
Share capital `1 equity share 1000 400 800 800
Retained earnings 4000 3400 3600 3600
Creditors 200 300 250 250
-
Total 5200 4100 4650 4650

P Ltd. acquired shares in S many years ago when S retained earnings was ` 520 lakhs. P Ltd. acquired
shares in J at the beginning of the year when J retained earnings was ` 400 lakhs. P Ltd. acquired shares in A
on 01.04.2008 when A retained earnings was ` 400 lakhs.
The Balance of goodwill relating to S had been written off 3 years ago. The value of goodwill in
J remains unchanged.
Prepare the consolidated Balance Sheet of P Ltd. as on 31.03.2009 as per AS-21, AS-23 and AS-27.
(CA Final Nov 2009) Marks 16
Solution:
Consolidated Balance Sheet of H Co as on 31.3.2009
Particulars ` in lacs
I. Equity and Liabilities
(1) Shareholders’ Funds
(a) Share capital 1,000
(b) Reserves and Surplus 8,800
(2) Minority Interest 760
(3) Current Liabilities
Trade Payables (200 + 300 + 40% of 250) 600
Total 11,160
Comprehensive Problems on CFS (AS-21 + AS-23 + AS-27) Group Consolidation 311

II. Assets
(1) Non-Current assets
(a) Fixed Assets
Tangible Assets [1,000 + 800 + 560 (1400 × 40%)] 2,360
Intangible Assets 120
(b) Non-Current Investments 1,880
(2) Current Assets [2,200 + 3,300 + 1,300 (3,250 × 40%)] 6,800
Total 11,160

Consolidated P/L of H
Bal of H Co 4000
+ Share from S 2304
(-) Goodwill woff -64
+ Share from JV (3600-400) × 40% 1280
+ Share from Associates (3600-400) × 40% 1280
8800
Disclosure of Investm as per Equity Method
Cost of Invesmtents in A Ltd.:
Share in Net Assets of A Ltd. 480
Goodwill (identified) 120 600 (Cash paid)
+ Share in further profits 1280
(3200 × 40%) 1880
Analysis of Networth of Subsidiary (80%)
PRE POST
SC 400
P/L 520 2880
920 3800 2880
MI(20%) = 760
H (80%)
Pre = 736
Post = 2,304
Calculation of Cost of Control
Investments (@Cost) 800
(-) Pre Share -736
Godwill 64
Less: W-off -64
Balance 0
FOR JV
Analysis of Networth OFJV (40%)
PRE POST
SC 800
P/L 400 3200
1200 4400 3200
H (40%)
312 Accounting Standards

PRE POST
480 1280
Note: O nly proportionate consolidation and not full.
No Minority interest
Calculation of Cost of Joint Control
Investments (@Cost) 600
(-) Pre Share -480
Godwill 120
Analysis of Networth of Associates (40%)
PRE POST
SC 800
P/L 400 3200
1200 4400 3200
H (40%)
PRE POST
480 1280
Note: Equity method
Calculation of G/W or C/R
Investments (@Cost) 600
(-) Pre Share -480
Goodwill 120
Problem 2: The draft Balance Sheet of three companies, W, H, O, as at 31.3.2010 is as under:
` in thousands
Assets W. H. O.
Fixed assets 697 648 349
Investments 1,60,000 shares in H 562 -- --
80,000 shares in O 184 -- --
Cash at bank 101 95 80
Trade receivables 386 321 251
Inventory 495 389 287
Total 2,425 1,453 967
Liabilities
Share Capital 600 200 200
(Nominal Value `1 per share)
Reserves 1,050 850 478
Trade payables 375 253 189
De bentures 400 150 100
Total 2,425 1,453 967
You are given the following information:
(a) W purchased the shares in H on 13.10.2005 when the balance in reserves was ` 500 thousands.
(b) The shares in O were purchased on 11.5.2005 when the balance in reserves was ` 242 thousands.
(c) The following dividend have been declared but not accounted for before the accounting year end.
W ` 65 thousands
H ` 30 thousands
O ` 15 thousands
Comprehensive Problems on CFS (AS-21 + AS-23 + AS-27) Group Consolidation 313

(d) Included in inventory figure of O is inventory valued at ` 20 thousands which had been purchased
from W at cost plus 25%.
(e) Goodwill in respect of the acquisition of H has been fully written off.
(f) On 31.3.2010 H made bonus issue of one share for every share held. This had not been accounted in
the balance sheet as on 31.3.2010.
(g) Included in trade payables of W is` 18 thousands to O, which is included in trade receivables of O.
Prepare Consolidated Balance Sheet of W as at 31.3.2010.
(CA Final New Course May 2010)(16 marks)
Solution: Consolidated Balance Sheet of W and its subsidiary as at 31.3.2010
Particulars ` in lacs
I. Equity and Liabilities
(1) Shareholders’ Funds
(a) Share capital 600.00
(b) Reserves and Surplus 1,355.80
(2) Minority Interest 210.00
(3) Non-current Liabilities
(a) Long term borrowings [Debentures] [400 + 150] 550.00
(4) Current Liabilities
(a) Trade Payables (375 + 253) 628.00
(b) Other Current Liabilities [Proposed Dividend] 65.00
Total 3,408.80
II. Assets
(1) Non-Current assets
(a) Fixed Assets (697 + 648) 1,345.00
(b) Non-Current Investment (G/W of O `7,200) 276.80
(2) Current Assets
(a) Inventories (495 + 389) 884.00
(b) Trade Receivable (386 + 321) 707.00
(c) Cash and Cash Equivalents [101 + 95] 196.00
Total 3,408.80

Analysis of profit of H as on 313.3.2010 (Rupees in thousands)


Pre-acquisition Post-acquisition
Reserves 500 350
Dividend — (30)
Bonus (200) —
300 320
Minority Interest 20% 60 64
‘W’ shares 240 256
314 Accounting Standards

Calculation of Cost Control or Goodwill (Rupees in thousands)


Cost of investments 562
Less:
Paid up value of shares 160
Bonus shares 160
Pre-acquisition profit 240 560
Goodwill 2

Minority Interest (Rupees in thousands)


H
Share Capital (200 + 200 Bonus) 20% 80
Pre-acquisition profit 60
Post-acquisition profit 64
+ Dividend from H 6
210

Interest in Associates ‘O’ as on 31-3-2010 (Rupees in thousands)


Particulars Amount
Cost at acquisition 184.00
Post acquisition profit (478-242) * 40% 94.40
278.40
Less: Unrealized profit on inventory [20 * (25/125) * 40%] 1.60
276.80

Consolidated Revenue Reserve


Balance of Reserve in Balance Sheet 1050.00
Share of reserve from H 256.00
Dividend receivable from H (30*80%) 24.00
Post acquisition from O associates 94.40
1424.40
Less: Goodwill written off on consolidation 2.00
Less: Dividend payable 65.00
Less: Unrealized profit on inventory of Associates ‘O’ 1.60
Balance as on 31-3-2010 1355.80

Note: As Associates ‘O’ is not being consolidated. Therefore, trade payable of W of ` 18 thousand which is
trade receivable of O are not cancelled.

Proposed dividend by the Associate ASI-16


It is clarified that in case the associate has made a provision for proposed dividend in its financial statements,
the investor’s share of the results of operations of the associate should be computed without taking into
consideration the proposed dividend.
Comprehensive Problems on CFS (AS-21 + AS-23 + AS-27) Group Consolidation 315

Problem 3: Air Ltd., a listed company, entered into an expansion programme on 1st October, 2009. On that
date the company purchased from Bag Ltd. Its investments in two Private Limited Companies. The purchase
was of
(a) the entire share capital of Cold Ltd.
and
(b) 50% of the share capital of Dry Ltd.
Both the investments were previously owned by Bag Ltd. after acquisition by Air Limited, Dry Ltd. was to
be run by Air Ltd. and Bag Ltd. as a jointly controlled entity.
Air Ltd. makes its financial statements upto 30th September each year. The terms of acquisition were :

Cold Ltd.
The total consideration was based on price earnings ratio (P/E) of 12 applied to the reported profit of ` 20
lakhs of Cold Ltd. for the year 30 September, 2009. The consideration was settled by Air Ltd. Issuing 8%
debentures for ` 140 lakhs (at par) and the balance by a new issue of ` 1 equity shares, based on its market
value of ` 2.50 each.

Dry Ltd.
The market value of Dry Ltd. on first October, 2009 was mutually agreed as ` 375 lakhs. Air Ltd. satisfied its
share of 50% of this amount by issuing 75 lakhs ` 1 per equity shares (market value ` 2.50 each) to Bag Ltd.
Air Ltd. has not recorded in its books the acquisition of the above investments or the discharge of the
consideration.
The summarized statements of financial position of the three entities at 30th September, 1010 are :
(` in Thousands)

Air Ltd. Cold Ltd. Dry Ltd.


Assets
Tangible Assets 34,260 27,000 21,060
Inventories 9,640 7,200 18,640
Debtors 11,200 5,060 4,620
Cash - 3,410 40
Total Assets 55,100 42,670 44,360
Liabilities
Equity capital :
` 1 each 10,000 20,000 25,000
Retained earnings 20,800 15,000 4,500
Trade and other payables 17,120 5,270 14,100
Overdraft 1,540 - -
Provision for taxes 5,640 2,400 760
Total Liabilities 55,100 42,670 44,360

The following information is relevant.


(a) The book values of the net assets of Cold Ltd. and Dry Ltd. on the date of acquisition were considered
to be a reasonable approximation to their fair values.
(b) The current profits of Cold Ltd. and Dry Ltd. For the year ended 30th September, 2010 were ` 80 lakhs
and ` 20 lakhs respectively. No dividends were paid by any of the companies during the year.
316 Accounting Standards

(c) Dry Ltd., the jointly controlled entity, is to be accounted for using proportional consolidation, in
accordance with AS-27 “Interests in joint venture”.
(d) Goodwill in respect of the acquisition of Dry Ltd. has been impaired by ` 10 lakhs at 30th September,
2010. Gain on acquisition, if any, will be separately accounted.
Prepare the consolidated Balance Sheet of Air Ltd. And its subsidiaries as at 30th September, 2010.
(CA Final New Course Nov. 2010) (16 marks)
Solution: (All are ` in 000)
(a) Calculation of Business consideration to Cold Lid.
PC = Earnings × P/E = 20 lakhs × 12 = ` 24,000
To be paid as under—
8% Debenture 14,000
Equity Shares (1,00 lakhs/2.5 = 40 lakhs shares) 10,000
24,000
(b) Consideration paid to Dry Ltd.
Total consideration as given 37,500
Share 50% - issued equity shares 18,750
Numbers of shares @ ` 2.50 per share (thousands) 7,500
(c) Analysis of Profit of Cold Ltd. as on 30-09-2010
Pre-acquisition Post-acquisition
Retained Earnings 7,000 8,000
Air Ltd. share 100% 7,000 8,000
(d) Calculation of goodwill or Cost of control
Amount paid 24,000
Less:
Paid up value of share 20,000
Pre-acquisition profit 7,000 27,000
Capital Reserve 3,000
(e) Calculation of Goodwill of Dry Ltd.
Amount paid 18,750
Less: Share Capital 25,000
Pre-acquisition profit + 2,500
27,500 × 50% 13,750
Goodwill 5,000
(f) Retained Earnings
Air Ltd. Own balance 20,800
Share of Cold Ltd.(100%) 8,000
Share of JV (50% post) 1,000
29,800
Less: Goodwill w/off (1,000)
28,800
(g) Consolidation of assets and liabilities - Air Ltd. plus 100% of Cold Ltd. plus 50% of Dry Ltd
Comprehensive Problems on CFS (AS-21 + AS-23 + AS-27) Group Consolidation 317

Consolidated Balance Sheet of Air Ltd. and its subsidiary and Joint Venture as on 30/09/2010
Particulars Note No. ` in 000
I. Equity and Liabilities
(1) Shareholders’ Funds
(a) Share capital 1 21,500
(b) Reserves and Surplus 2 49,050
(2) Non current liabilities
Long Term Borrowings
8% Debentures 14,000
(3) Current Liabilities
(a) Short term borrowings (O/D) 1,540
(b) Trade payables (17120+5270+50% × 14100) 29,440
(c) Short term provisions (tax) (5640+2400+760 × 50%) 8,420
Total 1,23,950
II. Assets
(1) Non-Current assets
Fixed Tangible Assets (34260+27000+50% × 21060) 71,790
Intangible assets (G/W on consolidation) 4,000
(2) Current assets
(a) Inventories (9640+7200+50% × 18640) 26,160
(b) Trade receivables (11200+5060+50% × 4620) 18,570
(c) Cash/Cash equivalents (3410+50% × 40) 3,430
Total 1,23,950

1) Share Capital:
Issued Subscribed paid up:
21,500 thousand equity shares of `1/- = 21500 thousands
(of the above, 11500 thousand equity shares have been issued for consideration other than cash)
2) Reserves and Surplus:
Issued Subscribed paid up:
Capital Reserves 3,000 (on consolidation)
Securities Premium 17,250
Surplus 28,800
49,050
Minority will not be calculated in the problem because Cold Ltd is wholly owned subsidiary and Dry
Ltd is a JCE.
Problem 4: Eagle Ltd had acquired 51% in Sparrow Ltd for `75.80 lakhs on 01/04/2010. On the date of
acquisition Sparrow’s Assets stood at `196 lakhs and liabilities at `16 lakhs The Net asset position of Sparrow
Ltd as on 31/3/2011 & 30/09/2011 were `280 lakhs & `395 lakhs respectively, the increase resulting from
profits earned during the period. On 01/10/2011 25.5% holdings were sold for `125 lakhs.
You are required to (i) Explain the nature of the relationship between the two companies on the relevant
dates, (ii) The goodwill/capital reserve that arises on change in nature of the investment, (iii) Carrying
amount of unsold investments, (iv) The profit arising on part sale of investment in consolidated as well as
separate financial statements. (CA Final Nov 2011)(Marks 8)
318 Accounting Standards

Solution:
(i) On the date of acquisition, i.e. 01/04/2010, Sparrow Ltd is a subsidiary of Eagle Ltd as it holds 51%
shares. On 1/10/2011, half of the investments was sold due to this now the relationship between
Sparrow Ltd and Eagle Ltd is of associates. Eagle Ltd. will now account for the investment in Sparrow
Ltd as per equity method in its consolidated financial statements.
Working: (` in Lakhs)
Cost of investments on 1/4/2010 75.80
(-) Share of net worth (pre) [196-16] × 51% Total net assets of (91.80)
Capital Reserves : 16

Calculation of carrying amount of investments of associate company (` in Lakhs)


Cost of investments on 1/4/2010 75.80
+ Share of post increase in net worth [395-180] × 51% 109.65
Carrying amount as on 1/10/2011 (total) 185.45
Carrying amount as on 1/10/2011 for ½ sold (a) 92.725
Proceeds from disposal of shares of subsidiary Sparrow Ltd. (b) 125
Profit on disposal in consolidated Profit and Loss A/c (a-b) 32.27
(ii) G/W / Capital Reserves on 1/4/2010 = `16 lakhs and on 1/10/2011 = `8 lakhs (50%).
(iii) Carrying amount as on 1/10/2011 for unsold portion = `92.725 lakhs
(iv) Profit on sale in CFS = `32.27 lakhs and Profit on sale in separate financial statements = Sale proceeds
– Cost of shares = 125 – 75.80 × 50% = `87.10 lakhs.
Problem 5: AS-21+AS-23+AS-27+AS-11
A holding company ‘H’ has two subsidiaries a local subsidiary ‘S-L’ and a foreign subsidiary ‘S-F’. It also has an
Associate ‘A’ and Jointly controlled entity ‘J’.
The separate Financial/Statement and other details of each of them are given below:

Balance sheet as on 31.3.2011


Name H S-L S-F A J
Currency ` ` $ ` `
Liabilities:
Share capital equity 500 100 10 50 200
Reserve-pre-acquisition 200 15 2 10 20
P&L 100 10 2 8 10
Loans 150 10 2 5 20
Creditors 200 20 4 7 30
1150 155 20 80 280
Assets:
Fixed assets 400 70 10 30 120
Investment in group 436 - - - -
Other 60 10 1 5 10
Stock 120 30 4 20 80
Debtor 114 40 4 20 60
Cash bank 20 5 1 5 10
1150 155 20 80 280
Comprehensive Problems on CFS (AS-21 + AS-23 + AS-27) Group Consolidation 319

Profit and loss a/c for the year ended 31.3.2011


Sales 1500 200 60 200 400
Cost of goods sold 1150 160 40 160 300
Expenses 200 22 16 27 75
Depreciation 50 8 2 5 15
Total expenditure 1400 190 58 192 390
Profit 100 10 2 8 10

Other details:
1. Details of inter-group investment is as follows:

Company % of equity held by ‘H’ Cost


S-L 80 100
S-F 60 245 ($7)
A 25 16
J 40 75
Total 436
2. Exchange Rates : (a) At the time of acquisition and applicable to share capital, fixed asset, investment
and pre-acquisition reserves is ` 35 per $ (b) Rate at the beginning of the year ` 37 at the end of the
year ` 40 and average for the year ` 38 (c) Assume that all transaction of sale, purchase, expenses
took place at average rate of ` 38.
3. Foreign subsidiary S-F is an integral operation as defined in AS-11 and be accordingly consolidated.
4. ‘H’ sold goods at 20% profit on sale to group companies as per following details:

Sale to Sale value Balance due on 31-3-2011 Stock with them


S-L 20 10 7.5
A 15 5 10.00
J 40 30 25.00

5. Cost of goods sold of S-F is $6 opening stock + $ 38 purchases (-) $ 4 closing stock = $40.
Solution:
Important points: S-F Subsidiary Foreign, A – associates, J – Joint Venture
1. First S-F financial statement is converted into rupees by using AS-11 then it is consolidated as per
AS-21,
2. `A’ will be considered as per AS-23 which requires identification of goodwill/capital reserve
on the date of acquisition but not recognition and thereafter applying equity method. Wherein
proportionate share in profits will be recognized. No assets, liabilities, expense income of associate
will be consolidated. Stock reserve for unrealized profit will be calculated to the extent of our holding.
3. `J’ will be consolidated by proportionate consolidation method wherein proportionate part of asset,
liabilities, expense, and incomes shall be consolidated. Hence no question of minority interest arises.
Similarly inter co. debt and unrealized profit will be eliminated proportionately only.
Conversion of Financial Statement of S-F AS-11 (Rev)
Balance sheet $ Rate ` P&L $ Rate `
Equity 10 35 350 Sales 60 38 2280
Pre-acq reserve 2 35 70 Cost of goods sold
P&L 2 — 85 Opening stock 6 37 222
320 Accounting Standards

Loan 2 40 80 + Purchases 38 38 1444


Creditor 4 40 160 – Closing stock 4 40 160
Total 20 — 745 40 — 1506
Fixed assets 10 35 350 Expenses 16 38 608
Investment 1 35 35 Depreciation 2 35 70
Stock 4 40 160 Total expenses 58 — 2184
Debtor 4 40 160 Exchange loss — — 11
Cash 1 40 40 Profit 2 — 85
20 — 745 — 60 — 2280
Note: Exchange gain/loss can be ascertained as balancing figure after converting trial balance (i.e. all items)
Closing stock is a non – monetary item and carried at historical rate. As historical rate is not given we
should consider FIFO method and apply closing rate i.e. stock purchase at recent prices.
Calculation of Goodwill/Capital Reserve
Company S-L (80%) S-F (60%) A (25%) J (40%)
` `
` `
Share in equity capital 80 210 12.5 80
+ Pre-acquired profits 12 42 2.5 8
Intrinsic value 92 252 15.0 88
Cost of investment 100 245 16.0 75
Goodwill 8 00 1.0 00
Capital reserve 00 7 0.0 13

Note:
1. Goodwill 8 and capital reserve 20 can be netted in consolidated balance sheet with details in notes
or can be shown separately.
2. Goodwill in case of associate will not be recognized separately but will be shown with investment
only.

Minority interest S-L (20%) S-F (40%) Total


Share capital held 20 140 160
Pre-acquisition reserves 3 28 31
P&L 2 34 36
25 202 227

Consolidated Profit & Loss A/c


Net profit earned by H 100
Opening P/L of H b/d 200
Share of post net profits from:
Associates 8 × 25% 2
JV 10 × 40% 4
S-L 10 × 80% 8
S-F # 85 × 60% 51
Less: Unrealised profits on stock (4)
361

# Sales `2280 – Cost of goods `1506 – Depn `70 – Expenses `608 – Exchange loss `11 = `85
Comprehensive Problems on CFS (AS-21 + AS-23 + AS-27) Group Consolidation 321

Consolidated Balance sheet 31/12/2009 Note `


Equity and Liabilities
(1) SHF
(a) Share Capital 1 500.00
(b) Reserves & Surplus 2 373.00
(2) Minority Interest 227.00
(3) Non Current liabilities 3
(a) Long Term borrowings (Loans) 248
(4) Current Liabilities
(a) Trade Payables 3 370
Total 1,718.00
Assets
(1) Non-Current Assets
(a) Fixed Assets
Tangible Assets 3 868
Intangible Assets
(b) Non-current Investments 3 126.50
(2) Current Assets
(a) Inventories 3 338.50
(b) Trade Receivable 3 316.00
(c) Cash/Cash equivalents 3 69.00
Total 1,718.00

Notes to A/cs ` in lakhs


1. Equity Share Capital `10 (50,00,000 Shares) 500.00
2. Reserves & Surplus
Profit / Loss A/c (161+200) 361.00
Capital Reserves on consolidation 12.00
373.00

3. Consolidated Fixed Assets Invs in A (AS 23) Other Invs Inventories Trade Rec C/CE Loans
Balances
H Co 400.00 16.00 60.00 120.00 114.00 20.00 150.00
S – local 70.00 10.00 30.00 40.00 5.00 10.00
S Foreign 350.00 35.00 160.00 160.00 40.00 80.00
JV (*) 48.00 4.00 32.00 24.00 4.00 8.00
Total 868.00 16.00 109.00 342.00 338.00 69.00 248.00
(-) Stock reserve (0.50) (3.50)
(-) Mutual Debts
H and S (10.00)
H and JV (12)
Share of Profit 2.00
868.00 17.50 109 338.50 316.00 69.00 248.00
Trade crs = 200+20+160+12+-10-12 = 370
JV* taken proportionate assets and liabilities
Stock Reserves Unsold Goods Margin Elimination% Product Dr. Cr.
S-L 7.50 0.20 1.00 1.50 P/L Stock Res
A 10.00 0.20 0.25 0.50 P/L Investments
JV 25.00 0.20 0.40 2.00 P/L Stock Res
4.00
CHAPTER 31
AS-30, 31, 32 Financial
Instruments Corresponding to
IAS-32, 39, IFRS-7

Chapter Outline
vv Objective of Standard vv Initial Measurement of FA/FL
vv Financial instruments vv Impairment
vv Contract to Buy/Sell Non-financial Instruments vv Derecognition/Retirement/Removal of FA/FL
vv Financial Assets vv When to Record FA/FL
vv Financial Liability vv Reclassification of Financial Instruments
vv Equity Instrument vv Compound Financial Instruments
vv Expenses Incurred on Equity vv Embedded Derivatives
vv Derivatives vv Treasury Shares/Buy Back
vv Classification of Financial Assets vv Hedging
and Financial Liabilities

Abbreviations: FL (Financial liabilities), FA (Financial Assets), FI (Financial Instruments)

DO YOU KNOW
Indian GAAPS and IAS are on similar lines on Financial Instruments

Objective of the Standard:


The standard gives the principle of measurement and recording of financial assets and financial liabilities.

Financial Instruments (AS-31)


◘◘ It is a legal document entered between two parties.
◘◘ It is a legal enforcement to receive financial assets and repay financial liabilities (It is always under a
contract).
◘◘ One party has the right to receive payment of money/liquid asset and another party has the right to
pay the money/liquid asset. For one party financial instrument is financial asset and for other it is
financial liability or equity.
Primary financial instruments: receivables, payables, loans and advances, derivative instruments like
options, futures, swap etc.
AS-30, 31, 32 Financial Instruments Corresponding to IAS-32, 39, IFRS-7 323

Contract to Buy/Sell Non-financial Instruments


Contract to buy/sell non financial items are not FI. Ex: Contact to buy fixed asset, goods. But if goods are
sold and money is payable then the liability to pay becomes financial instrument because then it is a right to
receive or pay cash or any other financial asset.

Examples of FA/FL
Financial assets within the scope of AS 31/IAS 39 include
◘◘ Cash
◘◘ Deposits in other entities
◘◘ Receivables (e.g., trade receivables)
◘◘ Loans to other entities
◘◘ Investments in bonds and other debt instruments issued by other entities
◘◘ Investments in shares and other equity instruments issued by other entities
Financial liabilities within the scope of AS 31/IAS 39 include
◘◘ Deposit liabilities
◘◘ Payables (e.g., trade payables)
◘◘ Loans from other entities
◘◘ Bonds and other debt instruments issued by the entity

Scope (Exception)
AS 30/IAS 39 applies in the accounting for all the financial instruments except for those financial instruments
specifically exempted.
Employee benefit plans Employee Benefits (AS15)
Interests in subsidiaries CFS (AS21)
Interests in associates Investments in Associates (AS23)
Interests in joint ventures Interests in Joint Ventures (AS27)
Share-based payment transactions, Share-Based Payment (GN)
Insurance contracts Insurance Contracts (IRDA ACT)

Financial Assets: AS-30


It is any asset that is:
◘◘ Cash;
◘◘ Equity instrument of another entity.
◘◘ A contractual right - to receive cash/financial assets
◘◘ To exchange financial assets/financial liabilities with another entity under conditions that is poten-
tially favorable to the entity.
◘◘ Contracts that will or may be settled in entities own equity instruments that is:
(i) a non – derivative instrument where the entity is obliged to receive variable number of entity’s
own equity instruments.
OR
(ii) a derivative instrument that will or may be settled other than by the exchange of a fixed amount
of cash/FA for a fixed number of entity’s own equity instruments.
324 Accounting Standards

Analysis of the Definition Financial Assets


1. Cash: It is the medium of exchange. It is a specific mention in AS-30 as a FA.
2. Equity instrument of another entity: Means equity shares of another company, capital in a
partnership firm.
3. A contractual right to receive cash/FA – (Ex: Trade receivables, Bank balance, Loans, investments
in bonds)
4. To exchange financial assets/financial liabilities with another entity for terms which are potentially
favorable to the entity.
Example: Berlin Crafts entered into Futures February 2015 contract to buy 125MT of steel parts at an agreed
price of ` 6500 per MT. Also there is a practice of settling transactions on net basis for commodity futures.
This is now 31st Dec 2014 where Berlin Crafts finalizes accounts. As on that date actual price for steel parts
is `6600 per MT.
Solution: On 31st Dec 2014 Berlin Crafts will show the future contract (derivative) as:
Financial asset Dr 12500 (6600 – 6500) × 125
 To Income A/c 12500
(It is a potentially favorable contract [derivative contract] on 31/1/2013)
5. Contracts which require settlement in (receipt of variable number of) entities own equity
instruments:

In case of Non – derivative instruments


Example: Mistry Ltd entered into a contract to issue 10% Debentures ` 6,00,000 to Jamshed Ltd. against
receipt of entities own equity shares at the price prevailing after 3 months. Suppose the price of the share
after 3 months is ` 1200. Then Mistry Ltd. has to receive 500 own equity shares. The shares received are
variable and not fixed. Also it is not a derivative contract.
OR

In case of Derivative instruments:


Example: R. Com Ltd. entered into a contract to receive its own equity shares for 1000 liters of oil @ 52
forward rate. On 31/3 the forward rate at the Commodity Exchange is ` 47 and on settlement date is ` 44.
Market price of equity shares of R Com is ` 37, 34 and 32 on respective dates.
As the number of shares to be received by R Com Ltd will depend upon the value of oil at the Commodity
Exchange and even the price of the shares of R Com. The number of equity shares are variable hence we
should record a Financial Asset.
31/3: (Contract potentially favorable)
Financial asset Dr ` 5,000
To Profit on Derivative contract ` 5,000 {(52-47)1,000}
Settlement date:
Financial asset Dr ` 3,000
To Profit on Derivative contract ` 3,000 {(47-44)1,000}
Equity Shares (own) Dr 8,000
To Financial Asset 8,000 (No of own shares = 8,000/32 = 250)
Important Note: If a company entered into a contract to receive 5,000 (fixed) number of own equity shares
for fixed cash say ` 2,00,000 then it is an equity instrument (also known as fixed for fixed contracts).
AS-30, 31, 32 Financial Instruments Corresponding to IAS-32, 39, IFRS-7 325

DO YOU KNOW
Equity is a head and not an a/c under the new Companies Act. It means if we say equity instrument it should
go to the head EQUITY and if it is FL then it should go to Non – Current/Current liability.

Financial Liability (AS-30)


Any liability that is:
◘◘ A contractual obligation to deliver cash or another financial asset to another entity; (already covered
in FA).
◘◘ or to exchange financial assets or financial liabilities with another entity under conditions that are
potentially unfavourable to the entity; (already covered in FA)
◘◘ or A contract that will or may be settled (delivery to be made) in the entity’s own equity instruments
and is a non derivative instrument. Also the entity is obliged to issue variable number of equity instru-
ments of the entity. (already covered in FA)
◘◘ or a derivative instrument that will be settled other than by exchange of a fixed amount of cash/FA
against fixed number of entity’s own equity instrument (already covered in FA).
Examples of liabilities that meet the definition of financial liabilities are
◘◘ Payables (e.g., trade payables), Loans from other entities, Issued bonds and other debt instruments
issued by the entity, Derivative financial liabilities, Obligations to deliver own shares worth a fixed
amount of cash.

Comprehensive Illustrations on Financial Instruments


Particulars Financial Reasons
Instrument
Trade receivable Y Right to receive cash/fixed monetary value
Trade creditors Y Right to pay cash/fixed monetary value
Contract for right to sell goods/Fixed N Delivery of non financial instrument
asset
Advance paid to supplier of goods N Promise to supply goods
Advance received from customers N Promise to receive goods
Prepaid exp/rent etc. N Promise to render service/house accommodation
Gold bullion N Gold is like cash/liquid. But there is no contractual right to
receive cash. So it is considered as a commodity and not FI
Cash Y Amount will be recd – It is the basis on which all FI are based.
Bank borrowings Y Obligation to pay cash
Derivative Y Contractual right/obligation to receive/pay cash (except
some contracts which are settled by physical delivery)
Intangible Assets N Non monetary.
Operating lease contract N Non cash benefit.
Lease income/rent O/S under Y Lease payable/receivable are FI as cash will be recd/paid in
Operating lease contract. future which is fixed under a contract.
Finance lease contract Y At the inception of signing lease contract the lessor records
the PV of lease receivables (in future cash will be received).
So also for lessee.
326 Accounting Standards

Advance tax/VAT/TDS/Excise duty etc N Statutory liability and not under a contract
Bank deposit Y Right to receive cash
Gold Bonds Y Right to receive cash (investments)
Commodity contracts N As they are held for sale/usage requirement
Commodity contracts: Y It’s a derivative contract as well as FI as it is entered for earning
i. net settled in cash/financial profit and not for further sale/usage.
instrument.
or
ii. practice of making short term profit
or
iii. readily converted into cash
Issued equity capital Y It is an equity instrument.

Equity Instrument (AS-30 + AS-31)


Equity represents a share in the residual assets after deducting the liabilities. Note: From issuer point of
A financial instrument is an equity instrument if all the conditions are view and not investor point
satisfied: of view
(i) There should not be a contractual liability/obligation to pay any
cash/FA by the issuer i.e. only when an instrument does not gives rise
to a contractual obligation on the part of the issuer then only it is Equity. It means there should not
be a liability to redeem at a fixed date for a fixed amount Ex: Preference Share carries an obligation
to pay hence it is a liability.
Preference is liability as there is a force to redeem it. In case of share warrant or ESOP’s fixed shares
are issued whenever such option is exercised. For example: If ESOP o/s is `6,00,000 @40 exercise
price the company has to issue 600000 (fixed shares) at a fix price @40. If options are lapsed then
the lapsed portion is again transferred to Equity Account.
(ii) If the contract is settled by issuers own equity instruments (fix shares for fix amount).
(iii) There is no such obligation to exchange financial assets or financial liabilities with another entity
under conditions that are potentially unfavourable to the entity.

Why Equity Share Capital is an equity instrument?


In case of ordinary shares declaration of dividend is not mandatory also redemption is not mandatory hence
it is an equity instrument. When the issuer is obliged to deliver cash or any other financial asset then it is a
financial liability. This point makes the difference between equity and liability.
Classification of Debt and Equity instruments: If the issuer has the contractual obligation to settle the
contract in cash, financial asset or to issue its own variable equity shares then such instruments are financial
liabilities. The existence of obligation is necessary. Such classification between equity and debt is made right
at the inception only and it is not to be revised subsequently.
Folllowing are some equity instruments as per AS-31:
◘◘ Equity Share Capital of Issuer Company (No force to redeem the shares)
◘◘ ESOP’s (Contract to deliver fix shares at a fixed exercise price)
◘◘ Share Warrants (Contract to deliver fix shares at a fixed exercise price)
◘◘ Some of the Preference Shares (only if they have the features of equity)
◘◘ Written call option which allows the holder of the instrument to purchase fixed number of shares for a
fixed cash payment in the issuer company.
AS-30, 31, 32 Financial Instruments Corresponding to IAS-32, 39, IFRS-7 327

◘◘ Non-puttable shares/instruments that cannot be put back to the issuer by the holder (all equity shares
are non puttable shares).
Always remember: if an investor invest in a company in shares then for the investor it is always financial
asset but for the company it can be either equity or financial liability depends upon the applicability of the
above definition.

DO YOU KNOW

Debentures Equity Preference Conv Debn


For an Investor FA FA FA FA
For an Issuer FL Equity FL/Equity PV of Cash flows is FL and residual component Equity

Expenses Incurred on Equity: AS-30


The financial/transaction costs relating to the issue of financial instruments like printing expenses, regulatory
fees, stamp duties, registration fees etc are to be properly accounted.
Expenses related to equity are to be deducted from equity.
Expenses which could have been abandoned or avoided are to be expensed.

Derivatives (AS-30) (asked in Nov. 2014 Marks 4)


Derivatives are contracts such as options, forwards, futures, and swaps. Because they are often entered into
at no cost, many times derivatives were not recognized in financial statements prior to IAS 39/AS-30. But
now the accounting is done as soon as the contract for derivative is entered into. Even MTM valuation is done
on year ending.
For an instrument to be eligible for derivative all the combined conditions should be satisfied:
1) Value of contract is based on current value of underlying asset. Ex: Interest rate in case of Swap
agreements, forward contract may be based upon the forex rate.
2) It requires low or no initial investments. In case of Swaps/forward zero investments are required, in
case of options premium is required to be paid. The initial investments are smaller as compared to
other contract to have similar response to change in market factor.
3) Settlement takes place at a future date.
Example
Derivative financial instruments within the scope of AS-30/IAS 39 include
◘◘ A purchased call option to purchase (call) a financial asset at a fixed price at a future date.
◘◘ A forward contract for the purchase (or sale) of a financial asset at a fixed price at a future date

DO YOU KNOW
Margins are not initial investments; they are collateral security for provision of losses. They are to be accounted
separately.

Example 1: 6 months future contract of gold entered at ` 12000, it is currently trading at ` 10000 by paying
20% margin.
Analysis: Value of contract is based on current value of underlying asset = The future contract price 12000
is based on current price 10000.
328 Accounting Standards

Low initial investments = 20% margin


Future settlement. i.e. after 6 months. Yes it is a derivative contract.

Underlying variable of derivatives contract:


List of contracts Underlying variable
Interest rate swap Interest rate
Credit swap Credit rating
Stock index futures/options Benchmark index like Sensex/Nifty
Stock futures/options Particular equity share
Currency swap/future/forward/options Forex rate
Interest rate futures/options Interest rate

DO YOU KNOW
Derivative contracts end up in three ways: (i) Net settlement in cash, (ii) Net settlement in shares/goods,
(iii) Actual Delivery based.

Financial Guarantee
A Financial guarantee is a contract that requires the issuer to make specified payments to reimburse the
holder for the loss arising out of the default made by the debtor relating to a debt instrument.

Contractual Right and Contractual Obligation


While defining the term FI we used the term Contractual right and contractual obligation. It can be absolute or
contingent. For Ex: Financial guarantee is a contractual right of the lender to receive cash from the guarantor.
But Advance tax paid arises out of statutory right.

Loan Commitment
Loan commitment is actually a Contingent liability as per AS-29.
But Loan commitment can be covered under AS-30 in any of the conditions mentioned below:
(i) If Loan commitment is held for trading (earning short term profit).
(ii) Loan commitment in the nature of derivatives.

RECOGNITION AND MEASUREMENT OF FINANCIAL INSTRUMENTS (AS-30)

Classification of Financial Assets and Financial Liabilities


Financial assets are of 4 four types:
1. Financial assets at fair value through profit or loss (FVTPL): Such Financial assets are held for trading
(HFT) i.e. held as inventory, of course not covered by AS-2. A FA is said to be held for trading when
the purpose of acquiring the investments is just to earn profit. Trading assets includes debt, equity,
loans, receivables etc where the intention is to make short term profit.
Financial assets that are either (1) held for trading or (2) electively designated into this category are
FVTPL. Derivative assets are always FVTPL unless they are accounted for as hedges.
2. Held-to-maturity (HTM) investments are investments for which the entity has an intent and ability to
hold to maturity. Ex: Fix Deposits kept with Bank.
AS-30, 31, 32 Financial Instruments Corresponding to IAS-32, 39, IFRS-7 329

Important points for HTM:


ww HTM are non – derivative financial assets
ww HTM has a fixed maturity period and fixed maturity amount
ww This category excludes loans and advances (excludes originated loans).
ww Equity cannot be classified as HTM because they do not have fixed maturity.
ww Debt securities/redeemable preference shares with fixed maturity period and amount is
included.
If a portion of HTM investments is sold before maturity then the balance portion is treated/classified
as AFS. If the reclassification takes place for more 2-3 years then the investor cannot classify any of
the financial assets as HTM.
3. Loans and receivables
ww L & R are non – derivative unquoted financial assets
ww L & R has fixed and determinable amounts.
ww The intention is neither to sell it immediately nor held for trading.
They typically arises when an entity provides money, goods or services directly to a debtor with no
intention of trading the receivable.
Example: Accounts receivable, notes receivable, loan assets, and investments in unquoted debt securities. An
unquoted preference share treated as liability but not equity as classified by AS-30 as L & R.
Investments in equity shares cannot be L & R/HTM.
4. Available-for-sale (AFS): AFS assets are those financial assets that do not fall into any other category.
Financial assets that are either (1) electively designated into the category or (2) do not fall into any
other category. If a manager actively buys and sells the instruments to earn short term profit then it
is HFT, but if buying and selling the instruments is for managing the risk then it is classified as AFS.
Ex: Investments in debt and equity securities that do not fall into any other category are classified as
AFS.

Classification of Financial Liabilities


FL at FVTPL and Other FL:
(i) Financial liabilities are classified under FVTPL only if any one condition is satisfied: (1) Such FL
are held for trading or (2) electively designated into the category ex: Derivative liabilities and other
trading liabilities.
(ii) It is a residual head. Financial liabilities at invoice value and even carried at amortized cost are well
included in this category. All financial liabilities other than those at fair value through profit or loss
are FL at amortised cost/at original invoice value ex: Accounts payable, notes payable, customer
deposits, Loans etc.

DO YOU KNOW
There are 4 classification of FA and 2 classification of FL.

Initial Measurement of FA/FL


When a financial asset or financial liability is recognized initially in the balance sheet, the asset or liability
is measured at fair value (plus transaction costs in some cases). Fair value is the amount for which an
asset could be exchanged, or a liability settled, between knowledgeable, willing parties in an arm’s-length
transaction. In other words, fair value is an actual or estimated market transaction price on the reporting
330 Accounting Standards

date for a transaction taking place between unrelated parties that have adequate information about the asset
or liability being measured.
Transaction costs may arise in the acquisition, issuance, or disposal of a financial instrument. Transaction
costs are incremental costs, such as fees and commissions paid to agents, advisers, brokers and dealers;
charges levy by regulatory agencies and securities exchanges; and taxes.
Initial Recognition:
FVTPL → Initial recognition is at Fair value and transaction costs should be separately recorded.
AFS → Initial recognition is at Fair value inclusive of transaction costs.
HTM → Initial recognition is at Fair value inclusive of transaction costs.
Loans and Advances →
(i) Short term receivables with no stated interest rate: Original invoice value.
(ii) Other items: @ Fair value inclusive of transaction costs.
Example on initial recognition:
◘◘ Entity A purchases 100 shares of Entity B with a quoted price of `124 for a total consideration of
`12,400. In addition, Entity A incurs transaction costs in the form of broker fees of `100 to acquire the
shares. Entity A classifies the shares as at fair value through profit or loss. In this case, Entity A would
make these journal entries on initial recognition:
FVTPL Dr. 12,400
Fee expense Dr. 100
 (it will be trf to P/L a/c at the year end)
 To Cash 12,500
(To recognize acquisition of 100 shares at fair value of ` 12,400)
◘◘ If Entity A had classified the shares of Entity B as available for sale (i.e., a category for which changes
in fair value is not recognized in profit or loss), the transaction costs would have been included in the
initial measurement of the financial asset:
Available-for-sale financial asset Dr. 12,500 (12400+100)
To Cash 12,500
(To recognize acquisition of 100 shares at fair value plus transaction costs of `12,500)
◘◘ The same requirements apply to financial liabilities. For instance, if Entity A issues bonds for total
proceeds of `18,100 and incurs transaction costs of `400 in issuing the bonds:
Bonds Dr. 17,700 (net)
 To Cash 17,700
(To recognize issuance of bonds for net proceeds of `17,700)

Valuation of FA/FL on Balance Sheet Date


Subsequent to initial recognition, financial assets and financial liabilities are measured using one of these
three measurement attributes:
(i) Cost
(ii) Amortized cost
(iii) Fair value
Whether a financial asset or financial liability is measured at cost, amortized cost, or fair value depends
on its classification into one of the four categories of financial assets or two categories of financial liabilities
defined by IAS 39/AS – 30.
AS-30, 31, 32 Financial Instruments Corresponding to IAS-32, 39, IFRS-7 331

Valuation @ Cost
Cost is the amount for which an asset was acquired or a liability incurred, including transaction costs (i.e.,
fees or commissions paid).
Example: Subsequent to initial recognition, only one type of financial instrument is measured at cost under
IAS 39/AS 30: investments in unquoted equity instruments that cannot be reliably measured at fair value
For instance, an entity may conclude that fair value is not reliably measurable for an investment in a non
public entity (“private equity” investment). In that case, the entity is required to measure the investment at
cost.
Example: Entity Appu Ltd purchases a 10% holding of the ordinary shares in a non public, start-up entity for
a total cost of $5,000 paid in cash.
Thus, on initial recognition, it debits financial assets $5,000 and credits cash $5,000.
Financial asset Dr. 5,000
 To Cash 5,000
Entity Appu should continue to measure the investment at its cost of $5,000.

Valuation @ Amortized Cost


Amortized cost is the cost of an asset or liability as adjusted necessary, to achieve a constant effective interest
rate over the life of the asset or liability (i.e., constant interest income or constant interest expense as a
percentage of the carrying amount of the financial asset or financial liability).
Subsequent to initial measurement, these categories of financial assets and financial liabilities are
measured at amortized cost in the balance sheet:
◘◘ Held-to-maturity investments
◘◘ Loans and receivables
◘◘ Financial liabilities not measured at fair value through profit or loss
It is not possible to compute amortized cost for instruments that do not have fixed or determinable
payments, such as for equity instruments.

Valuation @ Fair Value


Subsequent to initial measurement, these categories of financial assets and financial liabilities are measured
at Fair Value in the balance sheet:
◘◘ FVTPL
◘◘ AFS (only applicable for FA because FL is not classified as AFS)
Note: Fair Value (MTM) difference in case of FVTPL is transferred to profit/loss a/c. In case of AFS such
difference is accumulated in the equity till it is disposed off/impaired.
Financial Assets at Fair Value through Profit or Loss
Assume Entity Urban on March 15, 2014, acquires 2,000 shares in Entity Rural at a share price of `55 for
a total of `1,10,000 and classifies them as at fair value through profit or loss. On December 31, 2014, the
quoted price of Entity Rural increases to `62, such that the fair value of all shares held in Entity Rural now
equals `1,24,000.
In this case, the journal entries would be
March 15, 2014
FVTPL Dr. 1,10,000
 To Cash 1,10,000
December 31, 2014
FVTPL Dr. 14,000
 To Profit or loss 14,000
332 Accounting Standards

Available-for-Sale Financial Assets


In this case, the journal entries would be
March 15, 2014
AFS – Financial assets Dr. 1,10,000
 To Cash 1,10,000
December 31, 2014
AFS – financial assets Dr. 14,000
 To Investments Allowance Reserve/Equity 14,000

DO YOU KNOW
FA and FL both are valued but equity is never valued on the B/S date.

Impairment (AS-30)
IAS 39/AS – 30 requires an entity to assess at each balance sheet date whether there is any objective evidence
that a financial asset or group of financial assets are impaired. Objective evidence of impairment that a
financial asset or group of financial assets is impaired includes observable data about these loss events:
(a) Significant financial difficulty of the issuer or obligor
(b) A breach of contract, such as a default or delinquency in interest or principal payments
(c) A troubled debt restructuring
(d) It becomes probable that the borrower will enter bankruptcy or other financial reorganization
(e) The disappearance of an active market for that financial asset because of financial difficulties
Comparative rules for impairment
Particulars Calculation of impairment
FA carried at original invoice value without any stated (Carrying amt – undiscounted value of cash flows)
interest rate
FA carried @ Cost (i.e. unquoted equity investments) (Cost – present value of cash flows for a similar financial
asset) as its own fair value is not possible
FA carried @ amortised cost (i.e. HTM, Loans investments) (Carrying amt – present value of remaining cash flows at
original discounting rate)
AFS – Financial assets 1. Value AFS @ FV by adjusting in FV reserve every year.
2. Compute Impairment Loss in the year of symptoms of
impairment.
3. Utilise FV reserve (Cr bal) for writing off IL.
4. If FV reserves are insufficient then charge the IL to P/L
A/C.
5. But if FV Reserve is Dr balance then both FVR and IL will
trf to P/L A/C.
FVTPL – Financial assets Mark to Market valuation takes care of impairment loss

Reversal of impairment loss


Particulars Reversal Rule
FA carried at original invoice value without any stated Allowed (Maximum original amount)
interest rate
FA carried @Cost (i.e. unquoted equity investments) Not allowed
AS-30, 31, 32 Financial Instruments Corresponding to IAS-32, 39, IFRS-7 333

FA carried @ amortised cost (i.e. HTM, Loans investments) Allowed (Maximum unamortised amount had been no
impairment loss)

AFS – Financial assets • For equity: Not allowed


(reasons for reversal mix up with other reasons of
increase in fair value)
• For debt: Yes allowed
FVTPL – Financial assets Not applicable

Derecognition/Retirement/Removal of FA/FL (AS-30/IAS-39)


The term “derecognition” refers to when an entity should remove an asset or liability from its balance sheet.
The derecognition requirements in IAS 39/AS – 30 set out the conditions that must be met in order to
derecognize a financial asset or financial liability and the computation of any gain or loss on derecognition.
Derecognition is the end of the life cycle of any FA/FL which starts with recognition. There are separate
derecognition requirements for financial assets and financial liabilities.

Derecognition of Financial Assets


Derecognition of a financial asset is appropriate if either one of these two criteria is met:
(1) The contractual rights to the cash flows of the financial asset have expired, or
(2) The financial asset has been transferred (e.g., sold) and the transfer qualifies for derecognition.
AS-30/IAS-39 further prescribe the following possibilities for derecognition for a FA:
(1) The entity has retained substantially all risks and rewards of ownership of the transferred asset.
(2) The entity has transferred substantially all risks and rewards of ownership of the transferred asset.
(3) The entity has neither retained nor transferred substantially all risks and rewards of ownership of
the transferred asset (i.e., cases that fall between situations (1) and (2) above).
CASE I: If an entity transfers substantially all risks and rewards of ownership of a transferred in financial
asset—situation (2) above—the entity derecognizes the financial asset in its entirety.
Example: If the carrying amount of a financial asset is ` 7,400 and the entity sells it for cash of ` 7,500 in a
transfer that qualifies for derecognition, an entity makes these entries:
Cash Dr. 7,500
 To Asset 7,400
 To Gain on sale 100
Examples of transactions where an entity has transferred substantially all risks and rewards of
Ownership:
◘◘ A sale of a financial asset where the seller (transferor) does not retain any rights or obligations (e.g.,
an option or guarantee) associated with the sold asset
◘◘ A sale of a financial asset where the transferor retains a call option to repurchase the transferred
asset, at the transferor’s option, but that option is deep-out-of-the-money (i.e., it is not probable that
the option will be exercised)
CASE II: If an entity transfers a financial asset but retains substantially all risks and rewards of ownership
of the financial asset—situation (1) above—AS 30/IAS 39 requires the entity to continue to recognize
the financial asset in its entirety. No gain or loss is recognized as a result of the transfer. This situation is
sometimes referred to as a failed sale. (It is similar to Repo agreements referred in AS-9)
334 Accounting Standards

Example
◘◘ A sale of a financial asset where the asset will be returned to the transferor for a fixed price at a future
date (e.g., a sale and repurchase [repo] transaction)
◘◘ A securities lending transaction
◘◘ A sale of a financial asset where the transferor retains a call option to repurchase the transferred
asset, at the transferor’s option, where the option is deep-in-the-money (i.e., it is highly probable that
the option will be exercised)
For example, if an entity sells a financial asset for $14,300 in cash and at the same time enters into an
agreement with the buyer to repurchase the asset in three months for $14,500, the sale would not qualify for
derecognition. The asset would continue to be recognized, and the seller would instead recognize a borrowing
from the buyer, as follows:
Cash Dr. 18,000
 To Borrowing 18,000
In the period between the sale and repurchase of the financial asset, the entity would accrue interest
expense on the borrowing for the difference between the sale price (` 18,000) and repurchase price (` 20,000):
On the date of the repurchase, the entity would record the repurchase as follows:
Borrowing Dr. 18,000
Finance cost Dr. 2,000
 To Cash 20,000
CASE III: If the transferor has retained control over the transferred asset, the entity continues to recognize
the asset to the extent of its continuing involvement. The continuing involvement is determined based on the
extent to which the entity continues to be exposed to changes in amounts and timing of the net cash flows
of the transferred asset (i.e., based on its nominal or maximum exposure to changes in net cash flows of the
transferred asset). (For problem; Please refer Q 3(b) of Financial Reporting Nov 2013 paper solution).

Derecognition of Financial Liabilities


The derecognition requirements for financial liabilities are different from those for financial assets. There is
no requirement to assess the extent to which the entity has retained risks and rewards in order to derecognize
a financial liability. Instead, the derecognition requirements for financial liabilities focus on whether the
financial liability has been extinguished. This means that derecognition of a financial liability is appropriate
when the obligation specified in the contract is discharged or is cancelled or expires.
Examples:
(a) Billard Fox Ltd has written a put option which expires.
Derecognition is appropriate because the option liability has expired. Therefore, the entity no longer
has an obligation and the liability has been extinguished.
(b) Cat Ltd owes Dog Ltd ` 4,00,000 and has set aside that amount in a special trust that it will not use
for any purpose other than to pay Dog Ltd
Derecognition is not appropriate because Cat Ltd still owes Dog Ltd ` 4,00,000. It has not obtained
legal release from paying of this amount.

When to Record FA/FL


As per AS-30 an entity should record a FA/FL only when they become parties to the contract. Becoming
parties to a contract means committing to purchase a security or committed to write a derivative option. Only
promises/intent to do something is not important.
AS-30, 31, 32 Financial Instruments Corresponding to IAS-32, 39, IFRS-7 335

Timing of recording FA/FL also depends upon the trade date and settlement date.
Recording as per trade date and settlement date is similar to regular purchase or sale transaction.
◘◘ In case of trade date transaction is recorded on the date when trade takes place.
◘◘ In case of settlement method the transaction is recorded when financial instrument is received or
delivered by an entity.
Example: Restless Ltd purchased a Financial asset as on 29/3/2011 for ` 10 lakhs. The fair value of the asset
as on 31/3/2011 (year end) ` 10.5 lakhs and on 02/4/2011 (settlement date) ` 10.30.
Assuming the purchase of FA is recorded as per trade date.
FVTPL AFS
29/3/2011
FA (Financial Asset) Dr. 10 10
To FL 10 10
31/3/2011
FA Dr. 0.5
To P/L 0.5
FA Dr. 0.5
To FA reserve 0.5
2/4/2011
P/L Dr. 0.2
To FA 0.2
FA reserve Dr. 0.2
To FA 0.2
FL Dr. 10 10
To Cash/Bk 10 10

Note: Assets recorded by both FVTPL as well as by AFS stands for 10.30 because as per AS-30 such assets are
recorded @Fair Value not at Cost.
Assuming the purchase of FA is recorded as per settlement date.
FVTPL AFS
29/3/2011 No entry No entry
31/3/2011
FA Dr. 0.5
To P/L 0.5
FA Dr. 0.5
To FA reserve 0.5
2/4/2011
FA Dr. 9.8
P/L Dr. 0.2
To FL/Cash 10
FA Dr. 9.8
FA reserve Dr. 0.2
To FL/Cash 10
336 Accounting Standards

Reclassification of Financial Instruments


a) From FVTPL to any classification.
b) From HTM to AFS as well as AFS to HTM is permitted as a result of change in the intention or ability.
c) From L & R to FVTPL if the objective to hold the investments has changed.
d) From AFS to FVTPL if there is an evidence of short term profit making.

Compound Financial Instruments (AS-30, AS-31)


Sometimes a non derivative financial instruments issued contains both liability and equity elements. In other
words, one component of the instrument meets the definition of a financial liability and another component
of the instrument meets the definition of an equity instrument. Such instruments are referred to as compound
instruments. The approach of accounting for compound financial instruments is to apply split accounting,
that is, to present the liability and equity elements separately. First apply proceeds towards the present
value/fair value of liability component and then the balancing figure will be Equity.

Embedded Derivatives
It refers to a non derivative (Host) contract with a derivative element included. Sometimes derivatives are
embedded in other types of contracts. This is an arrangement where derivative element is implicit. For
instance, one or more derivative features may be embedded in a loan, bond, share, lease, insurance contract, or
purchase or sale contract. When a derivative feature is embedded in a non derivative contract, the derivative
is referred to as an embedded derivative and the contract in which it is embedded is referred to as a host
contract.
Example: An entity may issue a bond with interest or principal payments that are indexed to the price of
gold (e.g., the interest payments increase and decrease with the price of gold). Such a bond is a contract that
combines a host debt instrument and an embedded derivative on the price of gold. In the above example
(compound instrument para 19) for the investor it is an embedded derivative. Conversion option is the
derivative option and debt instrument raised is a host contract.

Accounting for Embedded Derivatives


The question is whether embedded derivative should be split and separately recorded from its host contract
or recording the host contract is itself enough. If all the following conditions given below is satisfied then split
accounting is necessary:
(1) On a stand-alone basis, the embedded feature meets the definition of a derivative.
(2) The hybrid contract is not measured at fair value.
(3) The economic characteristics and risks of the embedded feature are not closely related to the
economic characteristics and risks of the host contract. (Refer problems on embedded derivatives)

Treasury Shares/Buy Back (In India)


An entity acquires its own shares which are known as buy back in India. In USA it is known as Treasury
shares. In India treasury shares are cancelled, but in other countries it is shown as a deduction from equity.

Accounting Treatment as per IFRS or AS-31


When an entity reacquires an outstanding share or other equity instrument, the consideration paid is
deducted from equity. No gain or loss is recognized in profit or loss even if the reacquisition price differs from
the amount at which the equity instrument was originally issued. Similarly, if the entity subsequently resells
the treasury share, no gain or loss is recognized in profit or loss even if the proceeds at reissuance differ from
AS-30, 31, 32 Financial Instruments Corresponding to IAS-32, 39, IFRS-7 337

the consideration paid when the treasury shares were reacquired previously. The amount of treasury shares
is disclosed separately either in the notes or on the face of the balance sheet.
Example: On February 15, 2015, Poshak Ltd issues 100 shares at a price of ` 50 per share, resulting in total
proceeds of ` 5,000. It makes this journal entry:
Cash Dr. 5,000
 To Equity 5,000
On September 12, 2015, Poshak Ltd reacquires 20 of the shares at a price of ` 100 per share, resulting in
a total price paid of ` 2,000. It makes this journal entry:
Equity Dr. 2,000
 To Cash 2,000

Hedging
Forward exchange contract for trading or speculative purposes: At present it is covered by AS-11 till AS-30
becomes mandatory.

DO YOU KNOW
Accounting for forward exchange contract for trading or speculative purposes as per AS-11 and AS-30 is ex-
actly same. In case of AS-30 it is FVTPL. Solving FVTPL for speculative purpose is as good as solving problems
of AS-11 for speculative purposes.

DO YOU KNOW
Accounting for hedging is not same as per AS-11 and AS-30. Please refer the provisions given below.

Hedging as per AS-11 (Rev)


Forward exchange contract for hedging purposes (risk management) :
1) It deals with the sole purpose of risk management associated with the fluctuations in exchange rates.
2) In forward contracts the premium paid or discount earned is to be equally spread over the life of the
contact.
3) Premium paid or discount earned = [Exchange rate on the date of entering into contract - Forward
exchange rate entered into].
4) The notional gain/loss arising on the settling date is to be absorbed by profit & loss account.
5) If forward exchange contract is cancelled the profit or loss is to be transferred to profit and loss
statement.
6) Similarly any profit or loss on account of renewal of contract is to be transferred to profit & loss
statement.
The sole purpose of hedging is to hedge the risk.

Hedging as per AS-30


The purpose of hedging is to eliminate fluctuation risk by locking at a fix rate. By entering into derivatives
contracts (for hedging) one can offset loss in one contract by the gain in another contract.
338 Accounting Standards

i) Components of hedging
1) A hedged item: A hedged item is an asset, liability, firm commitment, highly probable forecast
transaction, or net investment in a foreign operation. To be designated as a hedged item, the
designated hedged item should expose the entity to risk of changes in fair value or future cash flows.
2) A hedging instrument: A hedging instrument is a financial instrument, expected to offset changes
in the fair value or cash flows of the hedged item. In addition, the hedging instrument must be with
an external party.
Example: Arvind Exporters entered into a forward contract for receiving a fixed amount ` 5,00,000 after 3
months. Here the hedging item is amount receivable (asset) and hedging instrument is the forward contract.
A hedging instrument should be effective in managing the risk of the hedged item.

ii) Hedging relationships


AS-30/IAS 39 identify three types of hedging relationships:
(1) Fair value hedges – which hedges (manages) the exposure relating to changes in fair value of an
item or transaction. FVH provides protection against changes in the FV of recognised asset/liability
or an unrecognised firm commitment which could affect the income account. Ex: Protecting fixed
income security against falling interest rates where present value of the investments comes down.
(2) Cash flow hedges - which hedges (manages) the exposure relating to the expected cash flows. In
other words it hedges the exposure on account of cash that are attributable to a particular risk
associated with (i) recognised asset or liability ex: interest receipt/payment of a recognised floating
@bond and not fixed @bond because fixed rate bond has fixed income hence no hedging required.
(ii) Probable forecasted sale or purchase commitment ex: Sale going to take place in future, raw
materials yet to be purchased in future.
Some important distinctions between FVH and CFH
Distinction with example:
In case of FVH we hedge the changes in interest rates fix interest on fixed interest loan but in case
of CFH we hedge the future interest receipts/payments. In case of FVH ex: a tea manufacturing
company fixes/hedges the value of tea inventory (asset) on the other hand for CFH the same tea
manufacturing company can also fixes the sale proceeds of tea sold.

Fair value hedge (FVH) Cash flow hedge (CFH)


Protects the asset/liability Protects the future cash flows
Gain/loss trf to profit/loss a/c Effective portion trf to equity and ineffective portion to profit/loss a/c

(3) Hedges of a net investment in a foreign operation (AS- 1) - which hedges (manages) the exposure
relating to foreign operations.

iii) Hedge effectiveness


A hedging instrument should be effective in managing the risk of the hedged item. The effectiveness of a
hedge is measured to the by which risk of a hedging item is hedged by a hedging instrument.
Effective hedging in AS-30 ranges between 80% – 125%.
Formula:
(hedging item/hedging instrument) × 100 or
(hedging instrument/hedging item) × 100
AS-30, 31, 32 Financial Instruments Corresponding to IAS-32, 39, IFRS-7 339

iv) Hedge accounting


(1) Fair value hedges: Hedge accounting is used normally to avoid mismatches in the recognition of
gains and losses. Hedging instrument should be re-measured at fair value, with the gain or loss
transferred to income account.
Fair value hedge accounting involves this accounting:
ww The hedging instrument is measured at fair value with changes in fair value recognized in profit
or loss.
ww If the hedged item is otherwise measured at cost or amortized cost (e.g., because it is classified
as a loan or receivable), the measurement of the hedged item is adjusted for changes in its fair
value attributable to the hedged risk. These changes are recognized in profit or loss.
ww If the hedged item is an available-for-sale financial asset, changes in fair value that would other-
wise have been included in equity are recognized in profit or loss.
Under fair value hedge accounting, changes in the fair value of the hedging instrument and of the
hedged item are recognized in profit or loss at the same time. The result is that there will be no
(net) impact on profit or loss of the hedging instrument and the hedged item if the hedge is fully
effective, because changes in fair value will offset each other. If the hedge is not 100% effective (i.e.,
the changes in fair value do not fully offset), such ineffectiveness is automatically reflected in profit
or loss.
Some other examples of FVH:
Fair value hedges include
ww A hedge of the exposure to changes in the fair value of an available-for-sale investment
ww A hedge of the exposure to changes in the fair value of a nonfinancial asset (e.g., inventory)
ww A hedge of the exposure to changes in the fair value of a firm commitment to purchase or sell a
nonfinancial item (e.g., a contract to purchase or sell gold for a fixed price on a future date)
(2) Cash Flow Hedge: C ash flow hedge accounting involves this accounting: Hedging instrument should
be split between effective portion and ineffective portion. Effective portion transferred to equity and
ineffective transferred to income statement.
If a hedge of a forecast transaction subsequently results in the recognition of a nonfinancial asset
or nonfinancial liability (or becomes a firm commitment for which fair value hedge accounting is
applied), the entity has an accounting policy choice of whether to keep deferred gains and losses in
equity or remove them from equity and include them in the initial carrying amount of the recognized
asset, liability, or firm commitment.
Examples: Cash flow hedges include: A hedge of the exposure to variable interest cash flows on a
bond that pays floating interest payments
ww A hedge of the cash flows from a forecast sale of an asset.
ww A hedge of the foreign currency exposure associated with a firm commitment to purchase or sell
a nonfinancial item
(3) Hedges of a net investment in a foreign operation (AS-11) – same as cash flow hedge.

Case Study on Cash Flow Hedge


Kriplani Works is a producer of steel plates used in auto sector. To hedge the risk of decline in the price of
100 units that it expects to sell on December 31, 2013, Kriplani on January 1, 2012, enters into a forward
contract on 100 units for delivery on December 31, 2013. During 2012, the change in the fair value of the
forward contract is a decrease of ` 8,00,000. During 2013, the change in the fair value of the forward contract
is an increase of ` 2,00,000. On December 31, 2013, Kriplani Works settles the forward contract by paying
` 6,00,000. At the same time, it sells 100 steel plates to customers for ` 93,00,000.
Required: Prepare the appropriate journal entries in the year 2012 and 2013. Assume that all conditions for
hedge accounting are met and that the hedging relationship is fully effective (100%).
340 Accounting Standards

Solution:
January 1, 2012
No entry required as the FV = 0.
December 31, 2012
Dr Equity 8,00,000
Cr Derivative liability 8,00,000
(To record the decrease in fair value of the hedging instrument)
December 31, 2013
Dr Derivative liability 2,00,000
Cr Equity 2,00,000
(To record the increase in fair value of the hedging instrument)
Dr Derivative liability 6,00,000
Cr Cash 6,00,000
(To record the settlement of the hedging instrument)
Dr Cash 93,00,000
Cr Equity 6,00,000
(releasing equity in revenue a/c.)
Cr Sales revenue 87,00,000
(To record the sale and the associated amount deferred in equity related to the hedge of the sale)

DO YOU KNOW
AS-30/31/32 is not yet notified by Govt. Hence at present it is not applicable to companies. Till AS-30... is not
notified, companies are required to follow AS-13 (area of investment valuation), AS-11 (Rev) relevant portions.

Note: At present AS-30,31,32 is not applicable in India hence we covered only Recognition, Measurement
and Presentation principles only. Disclosure principles are quite lengthy and at present it will be too much
for the students.

PROBLEMS AND SOLUTIONS


Problem 1: Classification of financial assets and financial liabilities:
Duck Financing is considering how to classify these financial assets and financial liabilities:
(a) An accounts receivable that is not held for trading
(b) An investment in an equity instrument quoted in an active market that is not held for trading
(c) An investment in an equity instrument that is not held for trading and does not have a quoted price,
and whose fair value cannot be reliably measured
(d) A purchased debt instrument quoted in an active market that Entity plans to hold to maturity. If
market interest rates fall sufficiently, Entity will consider selling the debt instrument to realize the
associated gain.
(e) An investment in a financial asset that is held for trading
Required: Indicate into which category or categories each item can be classified. Please note that some of the
items can be classified into more than one category.
Solution:
(a) An accounts receivable that is not held for trading should be classified into the category of loans and
receivables, unless the entity elects to designate it as either at fair value through profit or loss or
available for sale.
AS-30, 31, 32 Financial Instruments Corresponding to IAS-32, 39, IFRS-7 341

(b) An investment in an equity instrument that has a quoted price and that is not held for trading should
be classified as an available-for-sale financial asset, unless the entity elects to designate it as at fair
value through profit or loss.
(c) An investment in an equity instrument that is not held for trading and does not have a quoted price,
and whose fair value cannot be reliably measured, should be classified as an available for- sale
financial asset.
(d) This purchased debt instrument should be classified as available for sale unless the entity elects
to designate it as at fair value through profit or loss. Even though the debt instrument is quoted in
an active market and Entity plans to hold it to maturity, Entity cannot classify it as held to maturity
because Entity will consider selling the debt instrument if market interest rates fall sufficiently.
(e) An investment in a financial asset that is held for trading should be classified into the category of
financial asset at fair value through profit or loss.
Problem 2: Expenses on equity:
Data Cables Limited issues 1,00,000 equity shares of ` 10 at ` 12. The company incurred ` 30,000 printing
and other charges related to the issue. Out of the expenses ` 5,000 could have been abandoned. You are
required to show the correct position of equity.
Solution:
Equity share capital = 10,00,000
Share Premium 2,00,000
Less: Expenses on issue –25,000 1,75,000
Equity = 11,75,000
` 5,000 charged to income account.
Problem 3: Classification of financial instruments into FL or equity.
Particulars (Remember it is books of issuer company) E/FL Reasons
Preference shares redeemable at a future date for specified amount
Preference shares redeemable by virtue of put option exercisable by the holders
Preference shares which do not provide mandatory redemption
Preference shares will be redeemed subject to profits/government approval, availability of
cash
Preference shares will be redeemed at the option of the issuer
Compulsory payment of dividend
X Ltd issues 10% PS redeemable after 5 years
X Ltd issues 10% PS which is perpetual
X Ltd issues 10% PS which is perpetual and the dividend will be decided by the Board
10% Loan semi-annual interest but no repayment clause.

Solution:
Particulars E/FL Reasons
(Remember it is books of issuer company)
Obligation to pay dividend FL Obligation to pay cash
Preference shares redeemable at a future date for FL Contractual obligation to pay cash
specified amount
Preference shares redeemable by virtue of put option FL Contractual obligation to pay cash
exercisable by the holders
342 Accounting Standards

Preference shares which do not provide mandatory E Company has an unconditional right to refuse
redemption redemption
No contractual obligation to pay

Preference shares will be redeemed subject to profits/ FL No unconditional right to avoid delivering cash hence
government approval, availability of cash FL. Potential redemption inability does not negate the
obligation
Preference shares will be redeemed at the option of E Company has an unconditional right to refuse
the issuer redemption
No contractual obligation to pay
Compulsory payment of dividend FL Distribution of dividend is a liab. to pay
X Ltd issues 10% PS redeemable after 5 years FL Contains redemption obligations
X Ltd issues 10% PS which is perpetual E Contains no redemption obligations But if dividends
are cumulative (to pay compulsory anytime)then it is
FL.
Div are non cumulative then it is Equity.
X Ltd issues 10% PS which is perpetual and the E Contains no redemption obligations. Also dividends
dividend will be decided by the Board are like non cumulative.
10% Loan semi-annual interest but no repayment FL Obligation to pay interest is enough to treat the
clause. instrument as FL

Problem 4: Forward Contracts – Speculation (applies to both AS-11 and AS-30)


A party enters into forward contract for trading or speculation. Contract is to sell $ 100000 due on 30.06
@ ` 47.50. Today (1st January) spot @ is `47. On 31st March (yearend) the forward contract for 3 months (i.e.
remaining maturity period upto 30.06) to sell $ is available @ `47.55. Rate on 30.06 is `47.60. Journalise the
entries as per AS- 11 [R]. Assume both transaction and settlement method of accounting.
Solution:
Journal Entries in the books of Speculator:(assuming transaction date method)
Date Particulars Dr. (`) Cr. (`)
(assuming trade date method)
1.1. Foreign Contract entered into
Financial asset A/C. Dr. (due from Bk) 47,50,000
To Financial liability A/C. 47,50,000
$1,00,000 × 47.50
31.3. Mark – to – Market the Contract
Profit/loss A/C. Dr. 5,000
To Financial liability A/C. 5,000
(Forward Contract is valued on MTM @47.55 forward @ available for 3months)
30.6 Mark – to – Market at final rate
Profit/loss A/C. Dr. 5,000
To Financial liability A/C. 5,000
30.6 Final Square – off of the Contract
Financial liability A/C. Dr. 47,60,000
To Financial asset A/C. (due from Bank) 47,50,000
To Bank A/C. 10,000
AS-30, 31, 32 Financial Instruments Corresponding to IAS-32, 39, IFRS-7 343

(assuming settlement date method)


31.3. Mark – to – Market the Contract
Profit/loss A/C. Dr. 5,000
To Financial liability A/C. 5,000
(Forward Contract is valued on MTM @47.55 forward @ available for 3months)
30.6 Mark – to – Market at final rate
Profit/loss A/C. Dr. 5,000
To Financial liability A/C. 5,000
30.6 Final Square – off of the Contract
Financial liability A/C. Dr. 10,000
To Bank A/C. 10,000

Problem 5: Basic accounting – Financial assets:


On 30 March 2011 fair value per share of Sherry Limited was ` 45. On this date Brave Limited committed to
buy 10,000 of these shares @Fair value. Brave Ltd paid the price and accepted the delivery of these shares on
April 3, 2011. Brave Ltd closes its annual accounts on 31 March 2011. Fair value of Sherry ltd shares was `
45.40 on March 31, 2011 and ` 45.30 on April 3, 2011.
Show journals in the following 3 cases:
(i) Brave Ltd classifies the investments in Sherry Ltd as FA HTM.
(ii) Brave Ltd classifies the investments in Sherry Ltd as FA AFS.
(iii) Brave Ltd classifies the investments in Sherry Ltd as FA FVTPL.
Solution:
Date Particulars Dr. ` Cr. `
30.3.2011 Investment in S Ltd Dr. 4,50,000
To payable a/c (10000 × 45) 4,50,000
(Commitment as per AS-30)
31.3.2011 Brave Ltd classifies the invt. in Sherry Ltd as FA - HTM.
Equity investments cannot be HTM.
31.3.2011 Brave Ltd classifies the invt. in Sherry Ltd as FA - AFS.
Investments in S Ltd Dr. (45.4-45) × 10000 4,000
To Equity a/c 4,000
31.3.2011 Brave Ltd classifies the invt. in Sherry Ltd as FA – FVTPL.
Investments in S Ltd Dr. (45.4-45) × 10000 4,000
To P/L a/c 4,000
3.4.2011 Brave Ltd classifies the invt. in Sherry Ltd as FA - AFS.
Equity Dr. (45.4-45.30) × 10000 Dr. 1,000
To Investments in S Ltd 1,000
Payable Dr. 4,50,000
To Cash a/c 4,50,000
3.4.2011 Brave Ltd classifies the invt. in Sherry Ltd as FA - FVTPL.
P/L a/c (45.4-45.30) × 10000 Dr. 1,000
To Investments in S Ltd 1,000
Payable Dr. 4,50,000
To Cash a/c 4,50,000
344 Accounting Standards

Problem 6: Option accounting in the books of Buyer:


On 1/1/2015 Appu Ltd purchased 4000 call options of Gammon Ltd at a strike price of `700. Premium
paid `40 per unit. The option will expire on 1/5/2015. Pass journal entries if the options are settled in net
cash assuming:

Year ended 31/3/2015 Settlement date 1/5/2015


Case 1) Market price `690 `720
Case 2) Market price `630 `695
Case 3) Market price `730 `770

Solution: Journal Entries in the books of Appu Ltd.


Case 1 Case 2 Case 3
1/1/2015 1/1/2015 1/1/2015
Financial Asset Dr 160000 Financial Asset Dr 160000 Financial Asset Dr 160000
To Cash 160000 To Cash 160000 To Cash 160000
31/1/2015 31/1/2015 31/1/2015
P/L Dr 160000 P/L A/C 160000 P/L A/C 40000
To Prov for loss 160000 To Prov for loss 160000 To Prov for loss 40000
[(730 – 700) × 4000] – 160000
1/5/2015 1/5/2015 1/5/2015
Prov for loss Dr 8000 Prov for loss Dr 160000 Prov for loss Dr 40000
To P/L 8000 To FA 160000 To P/L 40000
Prov for loss Dr 152000 Cash Dr 280000
To FA 152000 (770 – 700) 4000
Cash Dr 8000 To FA 160000
To Financial Asset 8000 To P/L 120000
(720 – 700) 4000

Problem 7: Accounting of Equity Index Futures


Vidhwaan purchases the following units of Equity Index Futures (EIF).

Date of Purchase Name of Future Expiry Date/ Contract Price Contract Multiplier
Series Per Unit (No. of units)
28th March, 2011 EF1 May, 2011 4120 200
29th March, 2011 EF2 June, 2011 2280 50
29th March, 2011 EF1 May, 2011 4116 200

Daily Settlement Prices of the above units of Equity Index Futures were as follows:

Date EF 1 May Series (`) EF 2 June Series (`)


28/03/2011 4110 —
29/03/2011 4128 2300
30/03/2011 4135 2270
31/03/2011 4107 2290
01/04/2011 4115 2250
02/04/2011 4130 —
03/04/2011 4142 —
AS-30, 31, 32 Financial Instruments Corresponding to IAS-32, 39, IFRS-7 345

The above contracts were settled on the following dates:-


◘◘ EF 2 June Series on 1st April, 2011 for ` 2295.
◘◘ A contract of 200 units of EF 1 May Series on 2nd April, 2011 for 4133
◘◘ The other contracts of EF 1 May Series on 3rd April, 2011 for 4141.
Give journal Entries.
Solution:
Calculation of Mark to Market (MTM)
Date EF1 EF2 Total
28.3.2011 (2000) — (2000)
[200 × (4110 – 4120)]
29.3.2011 6000 1000 7000
[200 × 18 + 200 x 12) 50 x (2300 – 2280)
30.3.2011 2800 (1500) 1300
[400 × (4135 – 4128) 50 × (2270 – 2300)
31.3.2011 (11200) 1000 (10200)
400 × (4107 – 4135) 50 × (2290 – 2270)
1.4.2011 3200 250 3450
400 × (4115 – 4107) 50 × (2295 – 2290)
2.4.2011 6600 — 6600
299 × (4130 – 4115) + 200 × (4133 – 4115)
3.4.2011 2200 — 2200
200 × (4141 – 4130)

Books of Vidhwaan
Journal entries
Date Particulars Dr. (`) Cr. (`)
28.3.2011 Market to Market Dr. 2,000
To Bank 2,000
(Being amount paid to broker as MTM Margin)
29.3.2011 Bank A/c Dr. 7,000
To Market to Market 7,000
(Being MTM Margin recd from broker)
30.3.2011 Bank A/c Dr. 1,300
To Market to Market 1,300
(Being MTM Margin received from broker)
31.3.2011 Market to Market Dr. 10,200
To Bank 10,200
(Being amount paid to broker as MTM Margin)
31.3.2011 P & L A/c Dr. 3,900
To MTM 3,900
(Provision for loss) [-4120+4107] 200+ [-4116+4107] 200+[-2280+2290] 50
1.4.2011 Bank A/c Dr. 3,450
To Market to Market 3,450
(Being MTM margin recd)
346 Accounting Standards

1.4.2011 MTM A/c Dr. 250


To P & L A/c 250
(Being profit earned)
2.4.2011 Bank A/c Dr. 6,600
To Market to Market 6,600
(Being MTM margin recd)
2.4.2011 MTM A/c Dr. 5,200
To P & L A/c 5,200
3.4.2011 Bank A/c Dr. 2,000
To Market to Market 2,000
(Being MTM margin recd)
3.4.2011 MTM A/c Dr. 6,800
To P & L A/c 6,800
(Being profit earned)

Problem 8: Embedded Derivatives:


HT Ltd holds 500 6% 8 years Debenture of ` 1000 acquired at 99%. Debenture grants a put option to
the holder at the end of 5 years at which point the holder can redeem them at 102%. The Debentures are
repayable at 102.55% on maturity. Comment whether HT Ltd can classify the investment as held till maturity?
Solution: No a put option cannot be classified as HTM. The holder has paid premium for the put option. It
means the holder can violate the rule of HTM by selling it before maturity. Also the put feature should not be
separately recorded as the put option has the same interest rate risk as the host contract carries.
Problem 9: Forward Contract: Speculation (CA Final May 2010)
Mr. A bought a forward contract for 3 months of US$ 1,00,000 on 1st December at US$ 1= `47.10, as
against the spot rate of `47.02. On 31st Dec the exchange rate was 1US$ = `47.15. On 31 – January he decided
to sell the contract at `47.18/$. Compute the profit from the contract.
Solution: Mr A is a speculator and hence difference between spot rate and forward rate is not relevant. The
contract rate and sale rate is relevant.
Total profit = US$ 1,00,000 × (47.18 – 47.10) = ` 8,000
Cash Dr.  8,000
 To Profit on cancellation 8,000
Problem 10: Definition of derivative: (CA Final Nov 2010, Nov 2011)
M/s TS Ltd has entered into a contract by which it has the option to sell its specified asset to NB Ltd for
`100 Lakhs after 3 years whereas the current market price is `150 Lakhs. Company always settles account
by delivery. What type of option is this? Is it a Financial Instrument? Explain with reference to the relevant
Accounting Standard.
Solution: TS Ltd has an “Option to Sell” the underlying asset. Hence, it is a Put Option Contract. But as the
past practice is to settle the option is by delivery hence it is not a financial instrument. But if the past practice
is to settle on cash basis or to sell the commodity immediately at a profit then it is a FI. Put/Call option is a
derivative instrument but may or may not be a Financial Instruments.
Problem 11: Accounting for Financial Instruments – Forward Contracts
On 1st February 2009, Future Ltd entered into a contract with Son Ltd, to receive the Fair Value of 1000
Future Ltd.’s Own Equity Shares Outstanding as on 31-01-2010 in exchange for payment of `1,04,000 in cash,
i.e. ` 104 per Share. The contract will be settled in net cash on 31-01-2010.
AS-30, 31, 32 Financial Instruments Corresponding to IAS-32, 39, IFRS-7 347

The Fair Values of this forward contract on the different dates were:
Fair Value of forward on 01-02-2009 Nil
Fair Value of forward on 31-12-2009 ` 6,300,
Fair Value of forward on 31-01-2010 ` 2,000
Presuming that Future Ltd, closes its books on 31st December each year, pass entries – If net settled in
cash and If net is settled by Son Ltd, by delivering Shares of Future Ltd. (CA Final Nov 2010 Marks 8)
Solution: Contracts which will be settled in entities own equity instruments:
But it should be a non – derivative instrument where the entity is obliged to receive variable number of
the entity’s own equity instruments.
OR
A derivative instrument that will be settled other than by exchange of a fixed amount of cash/FA against
fixed number of entity’s own equity instrument.
1. Journal Entries (Cash Settlement)
01.02.2009 Entering into Forward Contract – No Journal Entry since fair value of the derivative
instrument is “` NIL” and no cash is paid or received.
31.12.2009 Year end
Forward Contract A/c Dr. 6,300
To Profit and Loss A/c 6,300
(Being Increase in Fair Value of Forward Contract)
31.1.2010.
Profit and Loss A/c Dr. 4,300
To Forward Contract A/c 4,300
(Being Decrease in value of Forward Contract recorded)
31.1.2010 Settlement of Net Receivable by Cash
Bank A/c Dr. 2,000
To Forward Contract A/c 2,000
(Being Settlement of Forward Contract by Cash)
2. Journal Entries (Settlement of “net” amount by shares)
First three journal entries are the same as above for Cash Settlement. The final entry will be as
follows —
31.1.2010 : Settlement - by delivery of Shares of Future Ltd.
Equity Shares of Future Ltd (Own Shares) A/c Dr. 2,000
To Forward Contract A/c 2,000
(Settlement of Forward Contract by receipt of Shares of Son Ltd own equity shares)
No of shares = 18.87 = 2000/106. Todays market price = (104000+2000)/1000 = 106
Problem 12: Impairment of Financial Instruments
A Ltd. purchases equity shares of B Ltd. The fair value of the asset was ` 20 Lacs on initial recognition. A
Ltd classifies them as available for sale financial asset. On end of year1 the fair value of shares was ` 18 lacs.
On end of year 2, the fair market value was 12 lacs. However, by applying valuation model it assesses that
fair value of shares should be ` 14 lacs. The market has over reacted to the bad news. As per market report,
B Ltd. may recover from there as well. Accordingly A Ltd. has decided to charge ` 4 lacs as impairment loss.
At end of year 3 the fair market value was ` 8 lacs because of management fraud and heavy loss incurred by
B Ltd. B Ltd. is highly likely to dissolve and shareholders will get only the residual value. A Ltd has estimated
that based on residual value the financial asset should be valued at ` 9 lacs. Journalize the above transitions
in the books of A Ltd.
348 Accounting Standards

Solution:
Books of A Ltd. Journal Entries
Date Particulars Dr. (`) Cr. (`)
Yr1 Financial Asset A/c Dr. 20
To Bank 20
(on acquisition)
— Fair Value reserve A/c Dr. 2
To Financial Asset 2
(Valuation at fair value)
Yr2
— Impairment Charges Dr . 4
To Financial Asset 4
(provision for impairment)
— P/L A/c 6
To Fair Value reserve A/c 2
To Impairment Charges 4
(balance in fair value reserve & IL trf to P/L)
Yr3
— Impairment Charges Dr . 5
To Financial Asset 5
(provision for impairment)
P/L A/c. 5
To Impairment Charges 5
(IL trf to P/L)

Problem 13: Amortised cost:


(i) A debt security of Gappi Listings has a stated principal amount of ` 100,000, which will be repaid by
the issuer at maturity in five years, and a stated coupon interest rate of 6% per year payable annually
at the end of each year until maturity (i.e., ` 6,000 per year).
Angel finances purchases the debt security in the market on January 1, 2011, for ` 93,400 (including
transaction costs of ` 100), that is, at a discount. Angel Fin classifies the debt security as held to
maturity.
Show journals for each year. (IRR – 7.64%)
(ii) At the end of the 4th year Gappi Listings has faced a lower credit rating due to global adverse conditions
and symptoms of impairment were visible. The cash flows expected by the investee company were
75% of the entire installments remaining for 4th and 5th year each. Calculate the impairment loss in
the 4th year in the books of investee company.
Solution:
(i) First we have to calculate IRR for applying effective interest rate principal.
Yr cash flow
0 –93400
1 6000
2 6000
3 6000
4 6000
5 106000
IRR @7.64%
AS-30, 31, 32 Financial Instruments Corresponding to IAS-32, 39, IFRS-7 349

Schedule of cash flows with effective interest calculations:


Year HTM invs Cash int Effective Amortization Closing
Interest of discount HTM
0 93400 93400
1 93400 6000 7136 1136 94536
2 94536 6000 7225 1225 95761
3 95761 6000 7316 1316 97077
4 97077 6000 7417 1417 98494
5 98494 6000 7525 1525 100000
2011
Beginning
Investm in HTM Dr. 93400
To Cash 93400
End 2011 2012 2013 2014
Cash Dr. 6000 6000 6000 6000
Investments in HTM Dr. 1136 1225 1316 1417
To Int income 7136 7225 7316 7417
2015
Cash Dr. 6000
Investments in HTM Dr. 1525
To Int income 7525
Cash Dr. 100000
To Investments in HTM 100000
(ii) If installments were recd to the extent of 75% then Impairement loss is calculated as below:
6000 × 75% × 0.929 + 106000 × 75% × 0.863 = ` 72790 carrying value based on present value of
cash flows.
But actual carrying amt = 97,077 Impairement loss = 97,077 – 72790 = ` 24,287
Accounitng:
P/L (Impairement loss) Dr. 24,287
To Investments in HTM 24,287
Problem 14: Amortised cost: - Zero Coupon Bond
Mahan Securities Ltd acquires 4 years 5,000 ZCB of ` 100 each. The issue price is ` 74. Pass journals for
all the 4 years assuming M Securities held the investments till maturity.
Solution:
First lets calculate IRR
Yr cash flow
0 –74
1 0
2 0
3 0
4 100
IRR @7.8%
7.8182%
350 Accounting Standards

Year HTM invs cash int Effective Amortiz Closing


Interest of discount HTM
0 370000 370000
1 370000 0 28934 28934 398934
2 398934 0 31197 31197 430131
3 430131 0 33636 33636 463767
4 463767 0 36234 36234 500000
Yr 1
Beginning
Investments in ZCB Dr. 370000
 To Cash 370000
End Yr1 Yr2 Yr3
Cash Dr. 0 0 0
Investments in ZCB Dr. 28934 31197 33636
 To Int income 28934 31197 33636
Yr4
Cash Dr. 0
Investments in ZCB Dr. 36234
 To Int income 36234
Cash Dr. 500000
 To Investments in ZCB Dr. 500000
Problem 15: Interest rate swap
Company A has outstanding debt on which it currently pays fixed rate of interest at 9.5%. The company
intends to refinance the debt with a floating rate interest. The best floating rate it can obtain is LIBOR + 2%.
However it does not want to pay more than LIBOR.
Another Company B is looking for a loan at a fixed rate of interest to finance its exports. The best rate it
can obtain 13.5%, but it cannot afford to pay more than 12%. However, one bank has agreed to offer finance
at a floating rate of LIBOR + 2%. Citi Bank is in the process of arranging an interest rate swap between these
two companies.
(a) With a schematic diagram, show how the swap deal can be structured.
(b) What are the interest savings by each company?
(c) How much would Citi Bank receive?
Pass Journals in the book of A, B and Citi Bank?
Solution:
Fixed Rate Floating Rate Preferred
Company A 9.5% L + 2% Floating
Company B 13.5% L + 2% Fixed

Gross Gain = Preferred Option – Other Option = (L + 2% + 13.5) – (L + 2% + 9.5) = 4%.


Sharing of gains will be: A = 40% of 4 = 1.6%, B = 40% of 4 = 1.6% and Citi Bank 0.80%.
Effective interest for A = L+2-1.6 = L + 0.4 ; for B = 13.5 – 1.6 = 11.90%.
AS-30, 31, 32 Financial Instruments Corresponding to IAS-32, 39, IFRS-7 351

Assuming LIBOR = 10%, and loan amount `100 crores (notional) following are the journal entries:

Books of A Books of B Citi Bank


Interest Dr 9.5 Interest Dr 12 (L+2)
To Cash 9.5 To Cash 12
(paid interest on own finance) (paid interest on own finance)
Citibank Dr 9.5 Citibank Dr 12 Interest Dr 21.5
To Interest 9.5 To Interest 12 To A 9.5
(Int reimbursed from swap bank) (Int reimbursed from swap bank) To B 12.0
Interest Dr 10.40 (10+0.4) Interest Dr 11.90 A Dr 10.40
To Citibank 10.40 To Citibank 11.90 B Dr 11.90
(int payable under swap) (int payable under swap) To Interest 22.30
Citibank Dr 0.90 Cash Dr 0.10 Cash Dr 0.90
To Cash 0.90 To Citibank 0.10 To A 0.90
(net settlement) (net settlement) B Dr 0.10
To Cash 0.10

Problem 16: Hedging:


Morris Canny Co. has entered into hedging relationship. Year end assessment reveals the following gains/
losses:
Hedging item – gain $ 1,000
Hedging instrument – loss $ 1,200
Compute hedge effectiveness.
Solution: The effectiveness of a hedge is measured by which the risk of a hedging item is hedged by a hedging
instrument.
Effective hedging in AS-30 ranges between 80% – 125%.
Formula:
(hedging item/hedging instrument) × 100 or
(hedging instrument/hedging item) × 100 = 1200/1000 × 100 = 120% (highly effective)
Problem 17: Hedging:
Sun life Pharma has hedged future cash flows on a financial liability using interest related derivatives. The
hedge effectiveness is computed at 90%. The additional interest cost relating to the derivative instrument is
` 40,000. Account for the cash flow hedge.
Solution:
(2) Cash flow hedges - hedging instrument should be split between effective portion and ineffective portion.
Effective portion transferred to equity and ineffective transferred to income statement.
The hedge effectiveness is 90%, therefore ` 36,000 (40,000 × 90%) should go to equity account and
remaining ` 4,000 should go to profit and loss account.
Problem 18: Forward Contracts – Hedging : (C. A. Final May 2006 & June 2009) (8 Marks)
On 24th January, 2006 Chinnaswamy of Chennai sold goods to Watson of Washington, U.S.A. for an invoice
price of $ 40,000 when the spot market rate was ` 44.20 per US $. Payment was to be received after three
months on 24th April, 2006. To mitigate the risk of loss from decline in the exchange rate on the date of
receipt of payment, Chinnaswamy immediately acquired a forward contract to sell on the APRIL, 2006 US $
40,000 @ ` 43.70. Chinnaswamy closed his books of accounts on 31st March. 2006 when the spot rate was
`43.20 per US $. On 24th April, 2006, the date of receipt of money be Chinnaswamy, the spot rate was `42.70
per US $.Pass journal entries in the books of Chinnaswamy to record the effect of all the above mentioned
events. Apply AS-11 as per the existing provisions.
352 Accounting Standards

Solution:
The problem has been solved as per provisions of AS-11
Analysis of the problem:
Financial asset = Fixed $40,000 receivable from bank.
Financial liability = Value of contract – payable
Spot rate on 24 Jan = 44.20 per $
Forward rate = 43.70 per $
Contract for the period = 24 Jan to 24 April
Spot rate on 31 March = 43.20 per $ (Year ending)
Spot rate on 24 April / Ultimate settlement rate = 42.70 per $
As per AS-11 (Rev) the discount / premium on initial contract for forward contract should be spread/
amortised over the period of contract.
Journal Entries in the books of Chinaswamy: BY APPLYING AS – 11 PROVISIONS
Date Particulars Dr. ` Cr. `
24.1.2006 Sales of Goods – Initial Recognition
Watson (USA) A/C Dr. 17,68,000
To Sales A/C. 17,68,000
$40,000 x 44.20
24.1.2006 Recording of Premium on contract
Financial Asset Dr. (@43.7) 17,48,000
Premium on forward contract Dr. 20,000
To Financial liability A/C. (@ 44.2) 17,68,000
(FA is fixed at Forward @ & FL at spot @)
31.3.2006 Valuation of Drs and financial liabilities:
Financial liability A/C. Dr 40,000
To P/L 40,000
P/L Dr 40,000
To Watson USA A/C. 40,000
$40,000 x (44.2 – 43.2)
31.3.2006 Notional Premium amortised
Profit / loss A/C. Dr. 14,667
To Premium on FC A/C. 14,667
20,000 x 66 days / 90 days
24.4.2006 Valuation of Drs and financial liabilities:
Financial liability A/C. Dr 20,000
To P/L 20,000
P/L Dr 20,000
To Watson USA A/C. 20,000
$40,000 x (43.20 – 42.70)
31.3.2007 Balance Notional Premium amortised
Profit / loss A/C. Dr. 5,333
To Premium on FC A/C. 5,333
31.3.2007 Final settlement of Drs:
Cash / Bk A/C. Dr 17,08,000
To Watson USA A/C. 17,08,000
$40,000 x 42.70
31.3.2007 Squaring off financial contract:
Financial liability A/C. Dr 17,08,000
Cash A/C. Dr. (bal. fig) 40,000
To Financial Asset A/C. 17,48,000
AS-30, 31, 32 Financial Instruments Corresponding to IAS-32, 39, IFRS-7 353

Problem 19: Hedging:


Aakshaya Ltd. has given a 12.50% fixed rate loan to its subsidiary Shaya Ltd. Aakshaya Ltd. measures this
loan at an amortised cost of `2,50,000. Aakshaya Ltd. has plans to hive off the receivable at a later stage and
as a measure to safeguard against fall in value of its due enters into a pay-fixed, received floating interest rate
swap to convert the fixed interest receipts into floating rate receipts, Aakshaya Ltd. designates the swap as a
hedging instrument in a fair value hedge of the loan asset.
Over the following months, market interest rates increase and Aakshaya Ltd. earn interest income
of `25,000 on the loan and `1,000 as net interest payments on the swap. The fair value of the Loan Asset
decreases by ` 5,000 while that of the interest rate swap increases by `5,000. You are informed that all
conditions required for the Hedge Accounting are satisfied. You are required to pass journal entries, with
suitable narrations, in the books of Aakshaya Ltd. to record the above transactions. (CA Final May 2010 OS)
Solution:
S. No. Particulars Dr. ` Cr. `
(1) Cash A/c Dr. 25,000
To Interest A/c 25,000
(Being interest recd on loan asset)
(2) Derivative A/c Dr. 5,000
To Hedging gain A/c 5,000
(Being increase in interest rate swap)
(3) Hedging loss A/c Dr. 5,000
To Loan to Shaya A/c 5,000
(Being decrease in fair value of loan)
(4) Cash A/c Dr. 1,000
To Interest A/c 1,000
(Being interest settlement on increase in swap)

Problem 20: Loans and receivables amortization:


(CA final May 2010 NS 12 marks + OS 8 marks, May 2012 marks 8)
ABC Ltd grants loan ` 10,00,000 to its employees on January 1, 2009 at a concessional interest rate of 4%
p.a. loan will be repaid in 5 equal annual principal installments along with interest. Market rate of interest
for such loan is 10% p.a. Pass journals for first 2 years.
Solution:
Calculation of initial recognition amount of loan - that will bediscounted present value of future cash flows
from the re-payment of the loan.

Year end Cash in flows Total Discount factor Discounted


at 10% Value
Principal Interest
2009 2,00,000 40,000 2,40,000 0.9090 2,18,160
2010 2,00,000 32,000 2,32,000 0.8263 1,91.702
2011 2,00,000 24,000 2,24,000 0.7512 1,68,268
2012 2,00,000 16,000 2,16,000 0.6829 1,47,506
2013 2,00,000 8,000 2,08,000 0.6208 1,29,126
Present Value or Fair Value 8,54,762
354 Accounting Standards

Journal Entries
1-1-2009
Staff Loan A/c Dr. 8,54,762
Staff Cost Dr. 1,45,268
To Bank 10,00,000
(As the fair value of loan is ` 8,54,762 it will be initially recognized at this value balance amount debited to
staff cost account)
Calculation of amortized cost at the end of each year
Year Balance Interest to be Re-payment Amortised cost
recognized (10%) (including interest)
2009 8,54,762 85,476 2,40,000 7,00,238
2010 7,00,233 70,024 2,32,000 5,38,262
2011 5,38,262 53,826 2,24,000 3,68,088
2012 3,68,088 36,809 2,16,000 1,88,897
2013 1,88,897 19.103* 2,08,000 NIL
*Balancing figure (rounding off )

Entry for 2009


Staff Loan A/c Dr. 85,476
To Interest on Staff Loan 85,476
Bank A/c Dr. 2,40,000
To Staff Loan 2,40,000
Interest on Staff Loan Dr. 85,476
To Profit & Loss A/c 85,476
Problem 21: Securitisation of Loans:
Arul Ltd holds `50,000 of loans yielding 18% interest p.a. for their estimated lives of 10 years. The company
securitizes the principal component and the right to receive interest at 14% to B Ltd a SPV at a Fair Value.
Out of the balance interest of 4%, it is stipulated that half of such balance interest namely 2%, will be due to
Arul as fees for continuing to service the loans. The expenses are estimated to be ` 400 as a collection fee. The
remaining half of the interest is due to Arul as an interest strip receivable..Give a/c treatment assuming IRR
as 13%. (CA final June 2009 10 Marks and Similar problem asked in Nov 2013 Marks 6)
Solution: As part of the loan is sold we have to split up the loan into various strips at Fair Value. For this
purpose we have to calculate PV of loan (cash flows):

Principal 14% Interest 2% Interest 2% Interest Total


transfer transfer (service fee) (right to retain
int retained)
Cash Flows 50,000 7,000 600 (1000-400) 1,000
PVF @ 13% 0.295 5.426 5.426 5.426
Fair Value 14,750 37,982 3,256 5,426 61,414
Ratio based on Fair Value 24% 62% 5.30% 8.70% 100%

Bank a/c 52,732 (14750 + 37982)


To Loan 43,000 (50000 × 86%)
To Profit on securitization 9,732
(Loan sold @Fair Value and credited @carrying amount)
AS-30, 31, 32 Financial Instruments Corresponding to IAS-32, 39, IFRS-7 355

Interest receivable 2,650 (50,000 × 5.3%)


Service Fee receivable 4,350 (50,000 × 8.75%)
To Loan 7,000
(Interest and fee receivable is restated at carrying amount in the ratio of fair value. It is a case of
derecognition of loan and recognition of Interest and fee receivable)
Problem 22: Compound Financial Instrument
(CA Final Nov 2009, Nov 2010 new syll. marks 8, May 2011 marks 5, Nov 2013 marks 4, Nov. 2014 marks 6,
Nov. 2015 Marks 8)
On 1st April 2008 Delta Ltd issued `30,00,000 6% Convertible debentures of ` 100 per debenture @par.
The debentures will be redeemed on 31/3/2012 10% premium or given an option to the holder to convert it
into equity shares. The interest rate without conversion option will be 10% for similar debentures. Split the
compound financial instruments into equity and debenture.
Solution:
PV of Redemption amount (3300000 × 0.68) 22,53,944
PV of interest payments (180000 × 3.17) 5,70,600
PV of liability 28,24,544
Less: Cash received 30,00,000
Equity Component 1,75,456
Journal Entry:
Cash Dr. 30,00,000
 To 6% Debt 28,24,544
 To Equity 1,75,456
Problem 23: Multiple Embedded Derivative Contracts
Certain callable convertible debentures are issued at `60. The value of similar debentures without call or
equity conversion option is `57. The value of call as determined using Black and Scholes model for option
pricing is `2. Determine the value of liability and equity component.
Solution:
Entry:
Cash a/c Dr. 60 (Without call option the cash would have been ` 62)
Call Option a/c Dr. 2 (Financial asset for exercising the right to redeem)
To Debentures 57 (pure liability component free from Conversion and Call option)
To Equity (bal fig) 5 (residual value)
Problem 24: PV of loans: (CA Final Nov 2011 Marks 5)
FEE Ltd. borrows a sum of ` 20 crore from COFEE Ltd. repayable as a single bullet payment at the end of
5 years. The interest thereon @ 5% is payable at yearly rests. Since the market rate is 8% FEE Ltd. paid an
origination fee of ` 2.40 crores to COFEE Ltd. for the lower rate of interest. Apart from the above, there are no
other transactions between the two parties. You are required to show the value at which COFEE Ltd. would
recognise the loan and the annual interest thereon.
Solution:
Borrowings = 2000 lacs
Market interest @8%, Actual interest @5%,
Origination fees = 240 lacs (All ` in lacs)
Lets check IRR = 8%
356 Accounting Standards

yr cash flows
0 1760 (2000-240)
1 –100
2 –100
3 –100
4 –100
5 –2100
8% (Yes market rate = IRR)
Effective interest Cash Interest Amort. O/S
0 1760.00
1 140.80 100.00 40.80 1800.80
2 144.06 100.00 44.06 1844.86
3 147.59 100.00 47.59 1892.45
4 151.40 100.00 51.40 1943.85
5 155.51 100.00 55.51 2000.00
(a) Since, the Origination Fee is attributable to the Loan, the said cash flow is also adjusted against the
Loan Receivable = 20 crores – 2.4 crores = 17.60 crores.
(b) Loan Receivable from FEE Ltd is in the nature of a Financial Asset in the books of COFEE Ltd. therefore,
it should be carried in the Financial Statements at Fair Value determined using the Effective Interest
Rate.
Accounting entry: Books of COFFEE LTD.
On loan given to FEE LTD:
Loan to Fee Ltd (Financial Asset) Dr. 20 crores
To Bank 20 crores
On receipt of origination fee:
Bank Dr 2.4 crores
To Loan to Fee Ltd (Financial Asset) 2.4 crores
Problem 25: Hedging - FVH
On April 1, 2013, Omega Ltd. borrowed ` 10 lakh at annual fixed interest rate of 7% payable half-yearly.
The life of the loan is 4 years with no pre-payment permitted. The company expected the interest rate to fall
and on the same day, it entered into an interest rate swap arrangement, whereby the company would pay
6-month LIBOR and would receive annual fixed interest of 7% every half-year. The swap effectively converted
the company’s fixed rate obligation to floating rate obligation. The following value of swap and debt are
available
Value of swap (` in lakh) Value of debt (` in lakh)
April 1, 2013 + 0.2 10.2
March 31, 2014 – 0.1 9.9
Six-month LIBOR on April 1, 2013 was 6% and that on October 1, 2013 was 8%.
Show important accounting entries in respect of the swap arrangement.
Solution: The interest rate swap is used to hedge fair value of fixed-rate debt. This is a case of fair value
hedge.
AS-30, 31, 32 Financial Instruments Corresponding to IAS-32, 39, IFRS-7 357

In the books of Omega Ltd. (Journal Entries)


Particulars Dr. Cr.
Interest A/c Dr. (10 x 7% × 6/12) 0.35
To Cash A/c 0.35
(Being interest on fund borrowed for first half-year 2013-14)
Loss on valuation of debt A/c Dr. (10.2-10) 0.20
To Loan A/c 0.20
(Being increase in value of debt recognised)
Swap Hedge A/c Dr. 0.20
To Gain on Swap Hedge A/c 0.20
(Being increase in value of swap recognised)
Cash A/c Dr. A/c Dr. (10 – [7%-6%] ) 6/12 0.05
To Interest A/c 0.05
(Being swap settlement received for first half-year 2013-14)
Interest A/c Dr. 10 x 7% x 6/12 0.35
To Cash A/c 0.35
(Being interest on fund borrowed for second half-year 2013-14)
Loan A/c Dr. 10.2 – 9.9 0.30
To Gain on valuation of debt 0.30
(Being decrease in value of debt recognised)
Loss on Swap Hedge A/c Dr. 0.2 – (-0.1) 0.30
To Swap Hedge A/c 0.30
(Being cumulative loss on swap recognised)
Interest A/c Dr. (10 [8% - 7% ) 6/12 0.05
To Cash A/c 0.05
(Being swap settlement paid for second half-year 2013-14)

Problem 26: Hedging


Samta Ltd has investments in equity shares of Jalpa Ltd @3,00,000 classified as available for sale. S Ltd
expects a decline in the value of investments and hence enter into a put option to sell the shares for ` 3,00,000.
If the fair value of the shares falls to ` 2,85,000 and value of option to be ` 15,000 assuming perfect hedge,
account the above transactions considering hedging and state whether it is Fair Value Hedge or Cash Flow
Hedge. What would be your answer if hedge accounting is not adopted by S Ltd?
Solution: It is a FVH as put option is entered to protect the value of investments (asset)
Accounting if hedge a/c is adopted:
P/L Dr. 15,000
To Investments in J Ltd 15,000
Derivative asset Dr. 15,000
To P/L 15,000
Accounting if hedge a/c is not adopted:
Equity Dr. 15,000
(As per AS-30 changes in FA → is retained in equity)
To Investments in J Ltd 15,000
Derivative asset Dr. 15,000
To P/L 15,000
358 Accounting Standards

This means if Hedge A/c is not followed then:


(i) Investments in J Ltd. will be accounted as per “AFS” purely.
(ii) Put option will be accounted as per “FVTPL” purely.
Problem 27: Embedded Derivatives
On 1 April 2013 Imagica Limited issued 70 lakhs 8% Convertible Debentures of face value ` 100 per
debenture at 6% premium. The Debentures will be compulsorily converted at the end of 4 years. Debentures
without conversion rights will pay an interest of 9% p.a. Conversion ratio will be 6 equity shares for every
1 debenture held. Face value of equity share is ` 10. Split the component of Debt and Equity. Pass journals for
2013-14 and 2016-17.
Solution: (` in lakhs)

Year Cash flows PVAF9% PVACF


4 years 560 3.239 1813.84
Redpt. 0 0 0

Journals
2013-14 (1st year) Opening bal = 1813.84
Cash Dr. 7,420 + Interest 9% = 163.25
To 8% Debentures 1,813.84 (–) Cash paid = (560)
To Equity suspense 5,606.16 Amortised Cost = 1417.09
Finance expenses Dr. 163.25 + Interest 9% = 127.54

Amortization
To Debentures 163.25 (–) Cash paid = (560)
Debentures Dr. 560 Amortised Cost = 984.62
To Cash Dr. 560 + Interest 9% = 88.60

table
2016-17 (last year) (–) Cash paid = (560)
Finance expenses Dr. 46.76 Amortised Cost = 513.24
To Debentures 46.76 + Interest 9% = 46.76 (round off )
Debentures Dr. 560 (–) Cash paid = (560)
To Cash Dr. 560 Amortised Cost = Nil
Equity Suspense Dr. 5606.16
To Equity Share Capital 4200 ** ** Equity is an embedded derivative but cannot be revalued
To Securities Premium 1406.16 as it is equity.
(` 5606.16 is the effective consideration already received) At the end of derecognition the company issued fixed
shares for fixed price.
No of equity shares = 70 lakhs × 6 = 420 lakhs

Problem 28: Embedded Derivatives in the books of Investor


On 1 April 2013 Vatika Investcomp purchases 70 lakhs 8% Convertible Debentures of face value ` 100
per debenture at 6% premium. Redemption of the debenture will be at par. Debentures without conversion
rights will pay an interest of 9% p.a. Conversion ratio will be 2 equity shares for every 1 debenture held.
Face value of equity share is ` 10. The fair value of the conversion option of the equity in 2013-14 ` 650.44.
(` in lakhs) as valued by BSM – Model.
Split the component of Investments in Debt and Embedded derivative. Pass journals for 2013-14 only.
Vatika Inestcomp classifies the investments as held to maturity.
AS-30, 31, 32 Financial Instruments Corresponding to IAS-32, 39, IFRS-7 359

Solution: (` in lakhs)

Year Cash flows PVAF9% PVACF


4 years 560 3.239 1813.84
Redpt. 7000 0.708 4958.98
Total: 6772.82

Journals
(` in lakhs)
2013-14 Opening bal = 6772.82
Investments in Debn Dr. 6772.82 + Interest 9% = 609.55
Option Asset Dr. 647.18 (–) Cash recd = (560)
To Cash 7,420 Amortised Cost = 6822.37
(remember for an investor embedded derivative can be + Interest 9% = 614.01
revalued because it is not an equity unlike issuer. It is just (–) Cash recd = (560)
like a Put Option and Vatika is a holder of the Option. Amortised Cost = 6876.39
Cash Dr. 560 + Interest 9% = 618.88
Investments in Debn Dr. 49.55 (–) Cash recd = (560)
To Interest income 609.55 Amortised Cost = 6935
Option Asset Dr. 3.26 + Interest 9% = 625 (round off )
To Profit on derivative 3.26 (–) Cash recd = (560)
(Profit on revaluation of put option/embedded derivative) Amortised Cost = 7000

Problem 29: Embedded Derivatives master problem


On 1 April 2014 Ikra Feast Limited purchases a 5,000 gold index linked ZCB of L & T of Face Value @ ` 100
at ` 103. The bonds carry a maturity of 3 years. The maturity value is computed as equal to the face value
plus % increase in gold index at the maturity of bond. For ex: Gold Index on the issue date is 30000 and on
the maturity date it is 33000, the redemption of the bond will be ` 3000/30000 i.e. 10% higher = ` 110 per
unit. Ikra Feast Limited wishes to classify the financial asset as HTM. Pass Journals for all the years.
The value of Call Option is: 1/4/2014 ` 26.47 ; 1/4/2015 ` 27.33; 1/4/2016 ` 29.34. The final spot index
of the gold on 31/3/2017 : 39300.
Solution:
Ikra Feast Limited: The Company is a holder of the call option.
Computation of Call premium = Cash paid – PV of ZCB. Suppose the IRR for a standalone ZCB is 7.5% then
the Option value = 103 – 100/1.075 3 = 103 – 80.5 = ` 22.50. Alternatively Call premium can be calculated
by Option Valuation Formula like BSM, Risk Neutral Theory etc. Let’s say such valuation gives us the value as
` 26.47. In that case Cash flow attributable to the ZCB will be = 103 – 26.47 = ` 76.53.
Call option is an embedded derivative included in the host contract Investments in Bonds. The Call option
has a risk attached with gold different from bond hence call option should be recorded separately from the
host contract. Had the Investments in Bond recorded at FVTPL then the call option would have been clubbed
with the host contract.
IRR of the Bond:
76.53 = 0/(1+r)1 + 0/(1+r)2 + 100/(1+r)3 = 9.33%
360 Accounting Standards

Journal Entries in the books of Ikra Limited

Date Particulars Dr. Cr.


1/4/2014 ZCB (HTM) Dr. (5000 × 76.53) 3,82,650
Gold Index Option Dr. (5000 × 26.47) 1,32,350
To Cash 5,15,000
31/3/2015 ZCB (HTM) Dr. (76.53 × 9.33%) 5000 35,701
To Interest income (trf to P/L) 35,701
Gold Index Option Dr. (27.33 – 26.47) 5000 4,300
To P & L 4,300
31/3/2016 ZCB (HTM) Dr. (382650 + 35701) × 9.33% 39,032
To Interest income (trf to P/L) 39,032
Gold Index Option Dr. (29.34 – 27.33) 5000 10,050
To P & L 10,050
31/3/2017 ZCB (HTM) Dr. 42,617
To Interest income (trf to P/L) 42,617
Gold Index Option Dr. (31-29.34) 5000 8,300
To P & L 8,300
Cash Dr. 5,00,000
To ZCB (HTM) 5,00,000
Cash Dr. 1,55,000
To Gold Index Option (5000 × 31) 1,55,000

Interest for the year 31/3/2017 = (382650+35701+39032) × 9.33% = 42,674 -57 roff = 42617
Additional payment on index = 39300 – 30000/30000 = 31% i.e. 100 + 31 = ` 131.

Case Study on Fair Value Hedge


On January 1, 2015, GT purchases a five-year bond that has a principal amount of $100,000 and pays annually
fixed interest rate of 5% per year (i.e., $5,000 per year). GT classifies the bond as an available-for-sale financial
asset. Current market interest rates for similar five-year bonds are also 5% such that the fair value of the bond
and the carrying amount of the bond on the acquisition date is equal to its principal amount of $100,000.
Because the interest rate is fixed, GT is exposed to the risk of declines in fair value of the bond. If market
interest rates increase above 5%, for example, the fair value of the bond will decrease below $100,000. This
is because the bond would pay a lower fixed interest rate than equivalent alternative investments available
in the market (i.e., the present value of the principal and interest cash flows discounted using market interest
rates would be less than the principal amount of the bond). To eliminate the risk of declines in fair value due
to increases in market interest rates, GT enters into a derivative to hedge (offset) this risk. More specifically,
on January 1, 2015, Entity A enters into an interest rate swap to exchange the fixed interest rate payments
it receives on the bond for floating interest rate payments. If the derivative hedging instrument is effective,
any declines in the fair value of the bond should offset by opposite increases in the fair value of the derivative
instrument. GT designates and documents the swap as a hedging instrument of the bond. On entering into the
swap on January 1, 2015, the swap has a net fair value of zero. (In practice, swaps usually are entered into at a
zero fair value). Therefore, no journal entry is required on this date. At the end of 2015, the bond has accrued
interest of $5,000.
GT makes this journal entry:
Interest receivable Dr 5,000
To Interest income 5,000
In addition, market interest rates have increased to 6%, such that the fair value of the bond has decreased
to $96,535. Because the bond is classified as available for sale, the decrease in fair value would normally have
been recorded directly in equity rather than in profit or loss. However, since the bond is classified as a hedged
AS-30, 31, 32 Financial Instruments Corresponding to IAS-32, 39, IFRS-7 361

item in a fair value hedge of the exposure to interest rate risk, this change in fair value of the bond is instead
recognized in profit or loss:
Hedging loss (hedged item) 3,465
To Available-for-sale financial asset 3,465
At the same time, GT determines that the fair value of the swap has increased by $3,465 to $3,465. Since
the swap is a derivative, it is measured at fair value with changes in fair value recognized in profit or loss.
Therefore, GT makes this journal entry:
Swap asset Dr 3,465
To Hedging gain (hedging instrument) 3,465
Since the changes in fair value of the hedged item and the hedging instrument exactly offset, the hedge is
100% effective, and the net effect on profit or loss is zero.

Case Study on Cash Flow Hedge


At the beginning of 2010, Berry Ltd. issues a 10-year liability with a principal amount of $100,000 for
$100,000 (i.e., at par). The bond pays floating interest that resets each year as market interest rates change.
Entity A measures the liability at amortized cost ($100,000). Because the interest rate regularly resets to
market interest rates, the fair value of the liability remains approximately constant irrespective of how market
interest rates change. However, Berry wishes to convert the floating rate payments to fixed rate payments in
order to hedge its exposure to changes in cash flows due to changes in market interest rates over the life of
the liability.
To hedge the exposure, Berry enters into a five-year interest rate swap under which the entity pays
fixed rate payments (5%) and in return receives floating rate payments that exactly offset the floating rate
payments it makes on the liability. Entity B designates and documents the swap as a cash flow hedge of its
exposure to variable interest payments on the bond. On entering into the interest rate swap, it has a fair value
of zero. The effect of that interest rate swap is to offset the exposure to changes in interest cash flows to be
paid on the liability. In effect, the interest rate swap converts the liability’s floating rate payments into fixed
rate payments, thereby eliminating the entity’s exposure to changes in cash flows attributable to changes in
interest rates resulting from the liability.
At the end of 2015, the bond has accrued interest of $6,000. Berry Ltd makes this journal entry:
Interest expense Dr 6,000
To Bond interest payable 6,000
At the same time, a net interest payment of $1,000 has accrued under the swap for the year. Therefore;:
Swap interest receivable 1,000
To Interest expense 1,000
The net effect on profit or loss is fixed net interest expense of $5,000 (= 6,000 – 1,000). Because the swap
is a derivative, it is measured at fair value. Berry Ltd determines that the fair value of the swap (excluding
accrued interest) has increased by $5,200. As the swap is designated as a hedging instrument in a cash flow
hedge, the change in fair value is not recognized in profit or loss but as a separate component of equity to the
extent the swap is effective. In this case, Entity A determines that the swap is 100% effective.
Journal entry:
Swap asset Dr 5,200
To Equity (hedging reserve) 5,200
Because the fair value of the swap will converge to zero by its maturity, the hedging reserve for the swap
will also converge to zero by its maturity to the extent the hedge remains in place and is effective.
EXTRA PRACTICAL PROBLEMS FROM PAST YEAR PAPERS
AND RTPs OF FINANCIAL REPORTING AND AUDIT

AS1
Problem 1: As an auditor your opinion on the following. A public Charitable Institution registered under
section 25 of the Companies Act, 1956 is running a printing press and transport system as feeder of funds to
carry on charitable activities. The accounting policies by way of notes to accounts are stated as follows:
Accounts are maintained on cash basis. Accounting Standards are not being followed as they are considered
to be inapplicable to charitable entities.
Solution: AS is applicable to every entity whose objective is to run business on commercial / industrial basis.
Even if suppose a part of the activities of such societies or institutions is profit oriented then AS has to be
applied in totality. AS has to be applied by the Charitable Institution as the Institution is running a printing
press and transport system. The Auditor has to qualify the Audit Report.
Problem 2: XYZ Company is engaged in the business of financial services and is undergoing tight liquidity
position, since most of the assets of the company are blocked in various claims/petitions in a Special Court.
XYZ has accepted Inter-Corporate Deposits (ICDs) and, it is making its best efforts to settle the dues. There
were claims at varied rates of interest, from lenders, from the due date of ICDs to the date of repayment. The
company has provided interest, as per the terms of the contract till the due date and a note for non-provision
of interest on the due date to the date of repayment was affected in the financial statements. On account of
uncertainties existing regarding the determination of the amount and in the absence of any specific legal
obligation at present as per the terms of contracts, the company considers that these claims are in the nature
of “claims against the company not acknowledged as debt”, and the same has been disclosed by way of a note
in the accounts instead of making a provision in the profit and loss accounts. State whether the treatment
done by the Company is correct or not.
Solution: AS 1 “Disclosure of Accounting Policies” recognises prudence as one of the major considerations
governing the selection and application of accounting policies. In view of the uncertainty attached to future
events, profits are not anticipated but recognized only when realized though not necessarily in cash. Provision
is made for all known liabilities and losses even though the amount cannot be determined with certainty and
represents only a best estimate in the light of available information.
Further, ‘accrual’ is one of the fundamental accounting assumptions as per AS 1. Irrespective of the terms
of the contract, so long as the principal amount of a loan is not repaid, the lender cannot be placed in a
disadvantageous position for non-payment of interest in respect of claim for interest from the due date to
date of repayment of loan.
From the aforesaid, it is apparent that the company should provide for the liability (since it is not waived
by the lenders) at an amount estimated or on reasonable basis based on facts and circumstances of each
case. Non-provision of the interest from the due date to the date of repayment of loan amounts to violation of
accrual basis of accounting.
Problem 3: A company was classified as Non-SMC in 2011-12. In 2012-13 it has been classified as SMC.
The management desires to avail the exemption or relaxations available for SMCs in 2012-13. However, the
accountant of the company does not agree with the same. Comment.
Solution: As per Rule 5 of the Companies (Accounting Standards) Rules, 2006, an existing company, which
was previously not an SMC and subsequently becomes an SMC, shall not be qualified for exemption or
relaxation in respect of accounting standards available to an SMC until the company remains an SMC for two
Extra Practical Problems from Past Year Papers and RTPs of Financial Reporting and Audit 363

consecutive accounting periods. Therefore, the management of the company cannot avail the exemptions
available with the SMCs for the year ended 31st March, 2013.

AS2
Problem 4: On 31.03.2013 a Business Firm finds that Cost of a partly finished unit on that date is `530.
The unit can be finished in 2013– 2014 by an additional expenditure of `310. The finished unit can be sold
for `750 subject to payment of 4% brokerage on Selling Price. The firm seeks your advice regarding : 1) The
amount at which the unfinished unit should be valued as at 31.03.2013 for preparation of Final Accounts and
2) The desirability or otherwise of producing the finished unit.
Solution:
1) As per AS–2, WIP should be valued at actual Cost or NRV whichever is less.
NRV = 750 – 30 (brokerage) – 310 = ` 410 or Cost ` 530 whichever is less i.e. Valuation as on
31/3/2013 = ` 410.
2) Desirability of Further Production –
Particulars Amount
Expenditure incurred till date 530
Add: Additional Expenses 310
Total Cost of the Product (A) 840
Sale Value (NRV) 750
Less: Brokerage at 4% 30
Net Sale Proceeds (B) 720
Profit/(Loss) on further Production (B – A) (120)
The company should not manufacture such product.
Problem 5: Walia Products had the following transactions in 2012-13.
(a) Purchased 1,000 units of certain raw material at `50 per unit. Excise duty `5 per unit included in
purchase price is available as CENVAT credit.
(b) Consumed 800 units of the material to make 750 units of finished product.
(c) Cost of conversion was`18,000, selling expenses 2,000
(d) Excise duty on the final product was`8 per unit.
(e) Sold 700 units at`100 per unit.
Show accounting entries in books of the company in respect of above. Present the Profit & Loss A/c as per
new Schedule III. Also present the disclosure as per ASI-14.
Solution:
Journal Entries Revenue Statement
Purchase Dr 45,000 Revenue from operations
Cenvat Credit Rec Dr 5,000 Sales and Services 70000
To Suppliers 50,000 Less: Excise Duty (5600)
Raw Materials Cons 36000 (ASI – 14) Net Sales 64400
Closing Stock 9000 Less: Expenditure:
Conversion cost Dr 18000 Raw materials cons 36000
To Bank a/c 18000 Difference of stock (4000)
Selling / dist exp Dr 2000 Selling / dist. 2000
To Bank a/c 2000 Excise on closing stock 400
Labour / Overheads 18000 52400
Excise Duty Dr 6000
To Provision for ED 6000 Net Profit 12000
364 Accounting Standards

Debtors / Cash Dr 70000


To Sales 70000
Provision for Excise Dr 5600
To Cenvat Credit Rec 5000
To Cash 600

WN: Calculation of FG:


Cost of goods = 36000 + 18000 + 6000 = 60000
Production = 750 units; Cost per unit = 60000/750 = 80.
Closing Stock = 60000 – 700 × 80 = 4000
Problem 6: As an auditor state your views on the following situations: Included under Current Assets of
XYZ Ltd. is inventory aggregating to`20 crores. A part of the said inventory manufactured for export had to
be sold earlier at a discounted price offshore due to moisture content present at the time of delivery. Similar
inventory is included in`20 crores.
Solution: Valuation of inventories should be at the lower of cost or NRV. In the present case the inventories
are sold at a discount due to the moisture content, it has to be valued at NRV on the B/S date.
Problem 7: Comment of the following: F Limited included in the Inventory, those items of fixed assets which
have not been in active use and held for disposal, as inventory item.
Solution: As per AS-10 any asset held out of active use should be valued at the lower of book value or NRV. But
still they are disclosed under the head fixed assets. Such FA should be disclosed in current assets only when
they are held for sale. Yes the company is correct in its contention of disclosing the asset held for disposal
under the head Current assets. But it should not be mixed up with inventories. Under the New Companies Act
Schedule III, it should be disclosed under other CA.
Problem 8: The closing inventory at cost of a Company amounted to `9,56,700. The following items were
included at cost in the total:
(i) 350 Shirts, which had cost `380 each and normally sold for `750 each. Owing to a defect in
manufacture, they were all sold after the balance sheet date at 50% of their normal price. Selling
expenses amounted to 5% of the proceeds.
(ii) 700 Trousers, which had cost `520 each. These too were found to be defective. Selling expenses for
the batch totaled `3,800. They were sold for `950 each.
What should be the closing inventory value (to the nearest rupee), after considering the above items?
Solution: Calculation of value of closing inventory:
`
Value of closing inventory (given) 956700
Less: Adjustment to bring the stock of shirts at net realizable value (W.N.1) (8313)
Revised value of closing inventory as per AS 2 948387
WN:
1) Valuation of Shirts as per AS-2
Cost price (per shirt) 380
Net realizable value per shirt :
Selling price of a shirt ` 750 × 50% = 375.00
Less: Selling expense (5% of ` 375) = (18.75)
NRV of a shirt 356.25
As per AS 2, inventories are valued at cost or NRV whichever is less. 356.25
Difference of cost and NRV 23.75
Extra Practical Problems from Past Year Papers and RTPs of Financial Reporting and Audit 365

Therefore, value of inventory of shirts is to be reduced by ` 8,313


(` 23.75 × 350 shirts)
2) Valuation of Trousers
Cost price (per trouser) 520
NRV per trouser:
Selling price of a trouser ` 950.00
Less: Selling expense of a trouser (` 3,800/700) `(5.43)
NRV of a trouser ` 944.57
As per AS 2, inventories are valued at cost or NRV whichever is less.
Since, inventory of trousers is already carried at cost, no further adjustment
is required in the total value of closing inventories. 520

AS3
Problem 9: Garden Ltd. acquired fixed assets viz. plant and machinery for `20 lakhs. During the same year
it sold its furniture and fixtures for `5 lakhs. Can the company disclose, net cash outflow towards purchase of
fixed assets in the cash flow statement as per AS-3?
Solution: According to Para 21 of AS 3 (Revised) ‘Cash Flow Statements’, an enterprise should report
separately major classes of gross cash receipts and gross cash payments arising from investing and financing
activities, except to the extent that cash flows described in paragraphs 22 and 24 are reported on a net
basis. Acquisition and disposal of fixed assets is not prescribed in para 22 and 24 of the standard. Hence,
the company cannot disclose net cash flow in respect of acquisition of plant and machinery and disposal of
furniture and fixtures.

AS4
Problem 10: A claim for damages of `10 lakhs for breach of patents and copyrights had been served on
the company in January, 2011. The Directors sought competent legal advice on the eligibility of the claim
and were advised that the claim was highly frivolous, without any bases and would not survive even in the
first trial court. The company however anticipates a long drawn legal battle and huge legal costs. Advise the
company regarding how this should be treated in the accounts.
Solution:
Previously this problem was covered by AS-4, but now it is covered by AS-29.
Provision: No provision is required as there is no probability of outflow of resources. The company is strong
in its contention about the case. The case is not likely to survive in the first trial court.
Contingent liability: Even no disclosure is required for the case because the settlement of the liability is very
remote.
But the company has to disclose the fact that huge expenses will be incurred on legal cases. But the
company cannot record the same because it is a future expenses.

AS5
Problem 11: Tiger Motor Car Limited signed an agreement with its employees union for revision of wages
on 01.07.2011. The revision of wages is with retrospective effect from 01.04.2008. The arrears of wages
up to 31.3.2011 amounts to `40,00,000 and that for the period from 01.04.2011 to 01.07.2011 amount to
`3,50,000. In view of the provisions of AS 5 “Net Profit or Loss for the period, Prior Period Items and Changes
in Accounting Policies”, decide whether a separate disclosure of arrears of wages is required while preparing
financial statements for the year ending 31.3.2012.
366 Accounting Standards

Solution: Wages related to the current year 2011 – 12 ` 7,50,000 is a normal wages under the head Employees
Cost. But wages related to past years ` 40,00,000 is an exceptional item which requires a separate disclosure.
P roblem 12: Comment on the following: A Co. Ltd has not included in the Balance Sheet as on 31-3-2010
a sum of `1.50 crores being amount in the arrears of salaries and wages payable to the staff for the last
2 years as a result of successful negotiations which were going on during the last 18 months and concluded
on 30-4-2010. The auditor wants to sign the said Balance Sheet and give the audit report on 31-5-2010. The
auditor came to know the result of the negotiations on 15/5/2010.
Solution: The negotiations were concluded on 30/4/2010 which is an event occurring after the balance
sheet date. Yes it is an adjusting event for the year 2009 – 2010. The condition for such event was present on
the balance sheet date. Also as per AS-5 it is to be separately disclosed as an exceptional item to explain the
performance of the company ion a better way.
P roblem 13: Discuss with reference to the Accounting Standards the nature of classification and disclosure
requirements in the statement of profit and loss of an entity in the following cases:
(a) Losses sustained as a result of enemy action.
(b) Disposal of long-term Investments of a trading entity.
(c) Setting up of group gratuity scheme where there was none.
(d) Payment of arrears of bonus for the earlier year as a result of settlement with the workers in the
current year.
Solution:
(a) Extra ordinary activity – being the event is beyond the control of the management.
(b) Exceptional item – It is necessary to explain such transaction within ordinary.
(c) Prior period item – Non-compliance of AS -15.
(d) If we assume the provision was not necessary i.e. remote liability then now a separate disclosure is
required. But if Provisions was required then it is a prior period item.

AS6
Problem 14: A plant was depreciated under two different methods as under –
Straight line method Diminishing Balance
1st year 3.90 10.69
2nd year 3.90 7.90
3rd year 3.90 5.84
4th year 3.90 4.32
15.60 28.75
5th year 3.90 3.19

Required: (a) If the company followed diminishing balance for first four years and decides to switch over to
SLM, what would be the amount of resultant surplus/deficiency? (b) If the company followed SLM for first
four years and decides to switch over to diminishing balance what would be amount of resultant surplus/
deficiency?
Solution: Change in the method of depreciation is change in accounting policy. Change in AP is governed by
AS-1 and AS-5. As per AS-5 any change in AP is required to be disclosed separately. Also change in AP always
has a retrospective effect.
Extra Practical Problems from Past Year Papers and RTPs of Financial Reporting and Audit 367

In the present problem the retrospective effect in the books is as follows:


Asset A/C Dr 13.15
To Asset A/C 13.15
Also in the current year the depreciation will be charged as per the new method SLM ` 3.90.
Problem 15: In the Trial Balance of M/s. Sun Ltd. as on 31-3-2011, balance of machinery appears `5,60,000.
The company follows rate of depreciation on machinery @ 10% p.a. on Straight Line Method*. On scrutiny it
was found that a machine appearing in the books on 1-4-2010 at `1,60,000 was disposed of on 30-9-2010 at
`1,35,000 in part exchange of a new machine costing `1,50,000.
Required: Calculate:
(i) Total depreciation to be charged in the Profit and Loss Account.
(ii) Loss on exchange of machine.
(iii) Book value of machinery in the Balance Sheet as on 31.3.2011.
Solution:
(i) Total Depreciation to be charged in the Profit and Loss Account
Depreciation on old machinery in use [10% of (5,60,000 – 1,60,000)] ` 40,000
Add: Depreciation on new machine @ 10% for six months 7500
(1,50,000 × 10% × 6/12)
Total depreciation on machinery in use 47500
Add: Depreciation on machine disposed of (10% for 6 months) 8000
(1,60,000 × 10% × 6/12)
So, total depreciation to be charged in Profit and Loss A/c 55,500
(ii) Loss on Exchange of Machine
Book value of machine as on 1.4.2011 160000
Less: Depreciation for 6 months @ 10% (8000)
Written Down Value as on 30.9.2011 1,52,000
Less: Exchange value 1,35,000
Loss on exchange of machine 17,000
Book Value of Machinery in the Balance Sheet as on 31.03.2012
Balance as per trial balance 560000
Less: Book value of machine sold (160000)
400000
Add: Purchase of new machine 150000
550000
Less: Depreciation on machinery in use (47500)
502500

AS7
Problem 16: Comment of the following: B Co. Ltd is engaged in the business of developing mass scale
housing projects including development of small commercial complexes. The flats/ commercial spaces are
booked by the public and are allotted by way of allotments letter to each allottee. Major construction activities
pertaining to buildings are undertaken after allotment is over. After completing the construction, possession
of flats/commercial spaces is given to allottees by executing legal document. The CEO of the B Co. Ltd. says
that AS 7 is not applicable to the company.
368 Accounting Standards

Solution: The contention of the CEO of B Ltd is correct. Any builder who constructs property on his own
and under a contract should follow AS-9 for Revenue Recognition and AS-2 for Valuation if Inventories.
Flats / premises are inventories unsold on 31/3. In 2012 ICAI came with a Guidance note on “Real Estate
Developers”, which is applicable to all the builders who construct properties on their own. The GN is based
on AS-2 and AS-9 provisions.
Problem 17: Contractors Ltd. have recognized contract revenue on a contract awarded in the financial year
2009-10. The target date of completion is 5 years. The contract provides for incentives for early completion
at the rate of`1,000 per day subject to a maximum of `3,00,000. The company has included this amount in
contract revenue (in the first year of contract) on the ground that based on the previous experience in similar
contracts, it is confident of completing the contract in 4 years. The company’s past track record shows that
company was able to complete such contracts well in time and earn incentives. Comment on the company’s
accounting policies.
Solution: The Company’s accounting policy is not in accordance with AS 7 (Revised) “Construction Contracts”.
Past track record is not the criteria for recognition of incentive payments receivable for early completion of
contract. According to AS 7 (Revised) incentives payments can be included in contract revenue only when
- the contract is sufficiently advanced that it is probable that the specified performance standards will be
met or exceeded; and - the amount of the incentive payment can be measured reliably. The contract is not
sufficiently advanced as it is in the first year and its normal time is 4-5 years. Hence, the recognition criteria
are not met and it is inappropriate to include incentive payments receivable in the current year is part of
contract revenue.
Problem 18: An amount of `9,90,000 was incurred on a contract work upto 31-03-2010. Certificates have
been received to date to the value of `12,00,000 against which `10,80,000 has been received in cash. The cost
of work done but not certified amounted to `22,500. It is estimated that by spending an additional amount
of`60,000 (including provision for contingencies) the work can be completed in all respects in another two
months. The agreed contract price of work is `12,50,000. Compute an estimate of the profit to be taken to the
Profit and Loss Account as per AS 7.
Solution:
Computation of estimate of profit as per AS 7
`
Expenditure incurred upto 31.3.2010 9,90,000
Estimated additional expenses 60,000
(including provision for contingency)
Estimated cost (A) 10,50,000
Contract price (B) 12,50,000
Total estimated profit [(B – A)] 2,00,000
Percentage of completion (9,90,000/10,50,000) × 100 = 94.29%
Computation of estimate of the profit to be taken to Profit and Loss Account:
Total estimated profit
× Expenses incurred till 31.3.2010 = ` 188571 (app)
Total estimated cost
According to para 21 of AS 7 ‘Construction Contracts’, when the outcome of a construction contract can be
estimated reliably, contract revenue and contract costs associated with the construction contract should be
recognized as revenue and expenses respectively by reference to stage of completion of the contract activity
at the reporting date. Thus, estimated profit amounting `1,88,571 should be recognized as revenue in the
statement of profit and loss.
Extra Practical Problems from Past Year Papers and RTPs of Financial Reporting and Audit 369

AS9
Problem 19: Goods worth `5,00,000 were destroyed due to flood in September, 2006. A claim was lodged
with insurance company. But no entry was passed in the books for insurance claim in the financial year 2006-
07. In March, 2008, the claim was passed and the company received a payment of `3,50,000 against the claim.
Explain the treatment of such receipt in final accounts for the year ended 31st March, 2008.
Solution: The company has not recorded the revenue in 2006-07 when the loss took place because the claim
was not certain to receive. The company was correct by not recording the claim. Now in March 2008 the
company has received the claim now the company can record the receipt. As per AS–5 even this receipt is not
a PPI. It is an exceptional item which requires separate disclosure.
Problem 20: M/s Moon Ltd sold Goods worth `6,50,000 to Mr. Star. Mr. Star asked for a Trade Discount
amounting to `53,000 and the same was dispatched. On receipt of Goods, Mr. Star found that Goods worth
`67,000 are defective. Mr. Star returned defective goods to M/s Moon Ltd and made payment due amounting
to `5,30,000. The Accountant of M/s Moon Ltd booked the Sale for `5,30,000. Discuss the contention of the
Accountant with reference to AS–9.
Solution: As per Para 4.1 of AS 9 “Revenue Recognition”, revenue is the gross inflow of cash, receivables or
other consideration arising in the course of the ordinary activities of an enterprise from the sale of goods,
from the rendering of services, and from the use by others of enterprise resources yielding interest, royalties
and dividends. In the given case, Moon Ltd. should record the sales `6,50,000 less Discount of `53,000 in
price and goods returned worth `67,000 are to be adjusted by suitable provisions.
Therefore the revenue to be recorded will be `5,97,000. Suitable provision should be made for sales
return.
Problem 21: An infrastructure company has constructed a mall and entered into agreement with tenants
towards license fees (monthly rental) and variable license fees, a percentage on the turnover of the tenant
(on an annual basis). Chief Finance Officer wants to account/recognize license fee as income for 12 months
during current year under audit and variable license fees as income during next year, since invoice is raised
in the subsequent year. As an auditor, how would you deal and state in the statement of Accounting policies?
Solution: AS-9 deals with the timing of recognition of Revenue. Revenue should be recorded on accrual basis
if there is no uncertainty prevailing over revenue.
Both the fixed rentals and variable licence fees on turnover should be recorded in the same accounting
year. The year of raising invoices is immaterial. The contention of the CEO of the company is incorrect. The
Auditor should qualify the report under SA 705 indicating the understatement of income .
Problem 22: As a Statutory Auditor, how would you deal with the following?
LM Ltd. has 2 divisions L and M. The finished products of division L are transferred to division M where
further processing is carried out before sale to customers. To achieve transparency and accountability
between the divisions, division L raises an invoice on division M at cost plus normal margins. At the year
end the unrealized profits on inter-division stocks are eliminated. However, the transfers are recorded at the
invoice value as sales and purchases in the respective division for the purpose of preparing the Profit and
Loss Account. Suitable disclosure, for this are given in the ‘Notes to Accounts’.
Solution: The risks and rewards related to the ownership is not transferred by the company to another party
.i.e. the risk and rewards remains with the company only. Therefore the company cannot record the gain on
account of inter dept. transfer. It is the sales and purchases at departmental level and not at the company’s
level.
Problem 23: In the course of audit of T Ltd. you observed that export incentives are not accounted on
accrual basis. The company’s management contended that these would be accounted on cash basis citing the
uncertainty about its receipts as they are not admitted as due by the customs authorities. Comment.
370 Accounting Standards

Solution: When the ability to access the ultimate collection with reasonable certainty is lacking at the time
of raising any claim: e.g., for escalation price, export in incentives, revenue recognition is postponed to the
extent of uncertainty involved.
In this case, as the condition regarding reasonable certainty as to ultimate collection is lacking as govt.
has not admitted the incentives as due, the company is justified in postponing revenue recognition till govt.
admits these incentives.
Problem 24: As an auditor state your views on the following situations: during the year ended 31-10-
2009 Long Ltd. sent an application to the excise authorities for refund of duty and grant of cash assistance
amounting to `50 lakhs in pursuance of a scheme yet to be finally approved by the authorities. The Chief
Accountant of the company would like to include the said amount of `50 lakhs in the income statement for
the year ended 31-10-2009.
Solution: Same as problem 27.
Problem 25: A company is engaged in the manufacture of electronic products and systems. As per Chief
Accountant a prototype system was installed at one of the customer’s locations in June 2010 for getting
acceptance on the performance of the system. The Chief Accountant has stated that as the ownership of
the system installed for field trials was vested with the company, for accounting & control purposes, the
prototype system installed at customer’s location in 2010 was capitalized in the accounts for the year 2010-
11 at its bought-out cost. State whether the accounting treatment adopted by the company is correct or not?
Solution: As per para 3 of AS-2 ‘Valuation of Inventories’, inventories mean assets held for sale in the ordinary
course of business, or in the process of production for such sale, or for consumption in the production of
goods or services for sale, including maintenance supplies and consumables other than machinery spares.
And as per para 6.1 of AS 10 ‘Accounting for fixed Assets’, fixed asset is an asset held with the intention of
being used for the purpose of producing or providing goods or services and is not held for sale in the normal
course of business.
Accordingly, the system installed by the company at customer’s site for his acceptance, based on the field
trials of the system, is an item of inventory, and is not a fixed asset. Installation of such prototype system at
customers’ sites for their acceptance is akin to sale of goods on approval basis. Therefore, the capitalization
of such prototype system at its bought-out-cost is not correct.
At the time of installation of such systems at the customers’ site, value of the same should be transferred
to a separate account such as ‘Goods sent for approval account’.
As it does not qualify the definition of fixed asset, therefore, no depreciation can be charged on it.
Problem 26: A company is engaged in the business of ship building and ship repair. On completion of
the repair work, a work completion certificate is prepared and countersigned by ship owner (customer).
Subsequently, invoice is prepared based on the work completion certificate describing the nature of work
done together with the rate and the amount. Customer scrutinizes the invoice and any variation is informed
to the company. Negotiations take place between the company and the customer. Negotiations may result in
a deduction being allowed from the invoiced amount either as a lumpsum or as a percentage of the invoiced
amount. The accounting treatment followed by the company is as follows:
(i) When the invoice is raised, the customer’s account is debited and ship repair income account is
credited with the invoiced amount.
(ii) Deduction, if any, arrived after negotiation is treated as trade discount by debiting the ship repair
income account.
(iii) At the close of the year, negotiation in respect of certain invoices had not been completed. In such
cases, based on past experience, a provision for anticipated loss is created by debiting the Profit and
Loss account. The provision is disclosed in Balance Sheet. Following two aspects are settled in the
negotiations:
Extra Practical Problems from Past Year Papers and RTPs of Financial Reporting and Audit 371

(a) Errors in billing arising on account of variation between the quantities as per work completion
certificate and invoice and other clerical errors in preparing the invoice.
(b) Disagreement between the company and customer about the rate/cost on which prior agreement
has not been reached between them.

Comment:
(i) Whether the accounting treatment of deduction as trade discount is correct? If not, state the correct
accounting treatment.
(ii) Whether the disclosure of the provision for anticipated loss in Balance Sheet is correct; if not, state
the correct accounting treatment. (CA Final June 09)
Solution:
(i) As per AS 9 “Revenue Recognition”, revenue is recognized at the time when the invoice is raised to
the customers; however the treatment of deduction as trade discount is not in accordance with AS 9.
Such deductions should be shown separately. The description of the difference as ‘trade discount’ is
not appropriate.
(ii) In respect of unsettled invoices on the B/S date provisions have to be created based on the past
experience. In the present problem company has passed accounting entry as:
Profit/loss A/c.  Dr xxxx
To Provision for doubtful debts xxxx
When the debtors are unable to pay or cash discount is incurred in that case the company should charge
the loss to Profit and loss account. This is because it is a loss post sales.

Correct entry: (For Reversal of invoices unsettled)


Revenue Dr. xxxx
To Debtors xxxx
Companies a/c treatment is not correct for the loss on account of invoices which were not settled on the
year end.

AS10
Problem 27: A new Plant X was acquired in exchange of old Plant B and on payment of `20,000. The carrying
amount of the old Plant B was `1,75,000 (Historical cost less depreciation). The fair value of the Plant B on
the date of exchange was `1,50,000.
Suggest the accounting entry in the books of the enterprise.
Solution: In case of exchange of assets the cost of the asset acquired will be the fair value of the asset given
up:
Plant X A/C Dr 170000
(150000 + 20000)
Loss on sale of Plant B 25000
To Cash 20000
To Plant B 175000
(Being Plant A purchased in consideration of Plant B and Cash)
Problem 28: X Ltd., paid `25 lakhs as advance to Y Ltd. towards the purchase of a printing machinery on
15.01.14 with delivery instructions to deliver the same in the last week of June, 08. Further on 02.02.14 X
Ltd. purchased two diesel generator sets from Y Ltd. for `30 lakhs on 90 days Credit term. In the accounts for
372 Accounting Standards

2013-14, X Ltd. intends to adjust the advance paid against Credit purchase and show the net amount of `5
lakhs as due from them. As the statutory auditor, how would you deal with this?
Solution: X Ltd. has paid advance amount to the supplier of machinery to be used in the project, such
advance amount should be grouped under the head ‘Capital Advances’ under the line item Long term loans
and advances. It is as per requirement of New Schedule III to the New Companies Act 2013. Short term credit
for 2 generators should be disclosed under Other Current liabilities. The proposal of X Ltd., to show the net
balance in the personal account of Y Ltd., is not correct. Such proposal will cancel the two material items in
the balance sheet – one, expenditure towards capital asset and the other current liability for purchase of the
generator set. Hence, the auditor should advise X Ltd., to show these two items separately. If X Ltd. does not
accept the advice, the auditor should qualify his report with suitable quantification of amount involved.

AS11
Problem 29: Option Ltd. is engaged in the manufacturing of steel. For its steel plant, it required machineries
of latest technology. It usually resorts to Long Term Foreign Currency Borrowings for its fund requirements.
On 1st April, 2011, it borrowed US $1 million from International Funding Agency, USA when exchange rate
was 1 $ = `52. The funds were used for acquiring machineries on the same date to be used in three different
steel plants. The useful life of the machineries is 10 years and their residual value is `20,00,000.
Earlier also the company used to purchase machineries out of foreign borrowings. The exchange differences
arising on such borrowings were charged to profit and loss account and were not capitalized even though the
company had an option to capitalize it as per notified AS 11 (notification issued by the MCA in 2009).
Now for this new purchase of machinery, Option Ltd, is interested to avail the option of capitalizing the
same to the cost of asset. Exchange rate on 31st March, 2012 is 1 US $ = `51. Assume that on 31st March, 2012,
Option Ltd. is not having any old Long term foreign currency borrowings except for the amount borrowed for
machinery purchased on 1st April, 2011.
Can Option Ltd. capitalize the exchange difference to the cost of asset on 31st March, 2012? If yes, then
calculate the depreciation amount on machineries as on 31st March, 2012.
Would your answer differ, if Option Ltd. was not a company and was a LLP?
Solution: Ministry of Corporate Affairs, Government of India, inserted paragraph 46A in notified AS 11 by
Notification dated 29th December, 2011, which is relevant for companies. It states that in respect of accounting
periods commencing on or after 1st April, 2011, for an enterprise which had earlier exercised the option
under paragraph 46 or not (such option to be irrevocable and to be applied to all such foreign currency
monetary items), the exchange differences arising on reporting of long term foreign currency monetary items
at rates different from those at which they were initially recorded during the period, or reported in previous
financial statements, in so far as they relate to the acquisition of a depreciable capital asset, can be added to
or deducted from the cost of the asset and shall be depreciated over the balance life of the asset.
Accordingly, though Option Ltd. had not earlier exercised the option as given by the notification on AS 11,
issued in 2009, yet it can avail the option to capitalise the exchange difference to the cost of machinery by
virtue of para 46A inserted in the notified AS 11 in December, 2011.

Exchange difference to be capitalised:


Cost of the asset in $ $ 10 lakhs
Exchange rate on 1st April, 2011 `52 = 1$
Cost of the asset in ` ($10 lakhs x 52) 520 lakhs
Less: Exchange differences as on 31st March, 2012 (52-51) x $ 1 million Gain (10 lakhs)
510 lakhs
Less: Depreciation for 2011-12 (510-20)/10 49 lakhs
461 lakhs
Extra Practical Problems from Past Year Papers and RTPs of Financial Reporting and Audit 373

Notification number G.S.R 914(E), dated 29th December, 2011, is relevant only for companies. If Option
Ltd. is a LLP, then this notification is not applicable and it cannot capitalise the exchange difference to the cost
of the machinery. The amount recognised to Profit and Loss account would be `10 lakhs.
Problem 30: XYZ Ltd. purchased an equipment at a price of $ 1,00,000 on 2-4-2011 upon terms of credit
the price should be settled within six months. The company capitalized the asset in terms of Indian `at a
rate of exchange prevailing as on date of purchase. When the liability is settled as per terms on 10.9.2011, it
incurs an additional amount of `3,50,000 due to exchange fluctuation the date of settlement. The said sum is
charged off to profit and loss account.
Solution: The accounting treatment is in consonance with AS-11 (Rev). All the exchange difference is
transferred to P/L account.
Problem 31: The draft Balance Sheets of A Ltd. and its American subsidiary B incorporated. As at 31st
December, 2014 are:

A (`) B (`)
Fixed Assets 18,00,000 20,000
Investments in B
Incorporated at cost 16,00,000 —
Stocks 12,00,000 30,000
Debtors 24,00,000 60,000
Cash and Bank 8,00,000 10,000
78,00,000 1,20,000
Represented by
Equity share Capital 30,00,000 30,000
Profit and Loss Account 20,00,000 40,000
Loan 12,00,000 20,000
Trade Creditors 6,00,000 10,000
Taxation 10,00,000 20,000
78,00,000 1,20,000

A Ltd. acquired 80% of shares in B Incorporated on 1st January 2014, when the Profit & Loss Account
showed a balance of $ 15,000. The Exchange rate prevalent during the relevant dates were:
1st January, 2014 1 $ = `59.46, Average rate 1 $ = `60.25, 31st December, 2014 1 $ = `61.50,
Fixed Asset acquisition rate 1 $ = `58.00.
A Ltd received dividend of $2,400 pertaining to pre acquisition period on 10th March 2014. The exchange
rate prevailing on that date was 1$ = `59.36. Mutual debt payable by A to B is `30,055 equivalent to $500
respectively.
Required: Prepare the consolidated Balance Sheet of A Ltd. and its subsidiary at 31st December, 2014
assuming: Integral and Non–Integral operations; with reference to the provisions of AS-11 Rev.
Solution:
Conversion of Trial Balance ($ → `)[Integral Operations]
ER Dr. (`) Cr. (`)
Fixed Assets NM 58 11,60,000 –
Inventories NM 61.5 * 18,45,000
(Assumed to be out of recent purchase)
Trade Receivable M 61.5 36,90,000
374 Accounting Standards

Cash and Bank M 61.5 6,15,000


Equity Share Capital NM 59.46 – 17,83,800
Profit & Loss A/c NM WN – 24,00,520
Loan M 61.5 – 12,30,000
Trade Creditors M 61.5 – 6,15,000
Taxation M 61.5 – 12,30,000
Forex Gain (balancing fig) – – – 50,680
73,10,000 73,10,000

Working Note – 1
Profit & Loss Balance
$ 40,000

Opening $ 15,000
Dividend $ (3,000)
$ 12,000 $ 28,000 N.P
@ 59.46 @ 60.25
7,13,520 16,87,000 = 24,00,520
Consolidated B/S of A & its subsidiary as on 31/12/2014
Equity
1) Shareholder’s Fund
Share Capital 1 30,00,000
Reserves & Surplus 2 37,87,305
2) Minority Interest 8,47,000
3) Non-Current Liabilities
Loans and Borrowings 24,30,000
4) Current Liabilities
Trade Payables 3 11,84,250
Short –term provisions 3 22,30,000
1,34,79,250
Assets
1) Non-Current Asset
Fixed Assets 3 29,60,000
2) Current Assets
Inventories 3 30,45,000
Trade Receivables 3 60,59,250
Cash /Bank 3 14,15,000
1,34,79,250

Notes to Accounts
1) Share Capital
Authorized –
Issued, Subscribed, paid up
30,000 Equity Share of `10 each fully paid 30,00,000
Extra Practical Problems from Past Year Papers and RTPs of Financial Reporting and Audit 375

2) Reserves & Surplus


Profit & Loss balance 32,46,985
Capital Reserve on Consolidation 5,40,320
3) Consolidated Assets and Liabilities

A Ltd B Inc Adjustment Total


Fixed Assets 18,00,000 1,60,00,000 -- 29,60,000
Inventories 12,00,000 18,45,000 30,45,000
Trade Receivables 24,00,000 36,90,000 30750@ 60,59,250
Cash/Bank 8,00,000 6,15,000 14,15,000
Trade Payable 6,00,000 6,15,000 30750@ 11,84,250
Short Term Provision (Tax) 10,00,000 12,30,000 22,30,000

Cost of Control A/c


Balance b/d 16,00,000 Share of Pre – acquisition net worth 19,97,856
Capital Reserve on cons 5,40,320 Dividend wrongly credited to P/L 1,42,464
($ 2400 × 59.36)

Consolidated P & L
Balance b/d 20,00,000
(+) Share of Post P & L 13,90,144
(-) Stock Reserve –
(-) Dividend declared –
(-) Pre - dividend wrongly credited (1,42,464)
(-) Creditors Revaluation (695)
3,24,6985
Analysis of Net Worth of B Inc.

Pre Post
Share Capital 17,83,800 –
Profit & Loss (WN1) 7,13,520 16,87,000
Forex gain – 50,680
24,97,320 17,37,680
A 80% 19,97,856 13,90,144
Minority 20% 4,99,464 3,47,536
@ Mutual
Owings:
A = 30, 055
B = 500 $

30,055 = 60.11 → spot rate


500
Valuation of monetary item in the books of A Ltd.
Profit & Loss A/c 695
To Creditors 695
$ 500 (61.5 – 50.11)
376 Accounting Standards

Mutual Cancellation
Creditors 30,750
To Debtors 30,750
(30,055 + 695)
OR
($ 500 × 61.5)
#$%@: If this operation is assumed to be Non – Integral then the exchange difference will retained
under the Reserves and Surplus as Foreign Currency Translation Reserve A/C (FCTR) but obvious 80%
of total and the balance will go to Minority Interest.

AS12
Problem 32: Supriya Ltd. received a grant of `2,500 lakhs during the last accounting year 2012-13 from
government for welfare activities to be carried on by the company for its employees. The grant prescribed
conditions for its utilization. However, during the year 2013-14, it was found that the conditions of grants
were not complied with and the grant had to be refunded to the government in full. Elucidate the current
accounting treatment, with reference to the provisions of AS 12.
Solution: As per AS 12 ‘Accounting for Government Grants’, Government grants sometimes become
refundable because certain conditions are not fulfilled. A government grant that becomes refundable is
treated as an extraordinary item as per AS 5 “Net Profit or Loss for the Period, Prior Period Items and Changes
in Accounting Polices”.
The amount refundable in respect of a government grant related to revenue is applied first against any
unamortized deferred credit remaining in respect of the grant. To the extent that the amount refundable
exceeds any such deferred credit, or where no deferred credit exists, the amount is charged immediately to
profit and loss statement. In the present case, the amount of refund of government grant should be shown in
the profit & loss account of the company as an extraordinary item during the year 2009-10.
P roblem 33: On 1st April, 2012. Kodabakkam Ltd. purchased a Fixed Assets worth `125 lakhs for which it
got Government grant of `36.90 lakh. The Salvage Value at the end of useful life of 4 years was estimated at
`51.20 lakhs. Kodabakkam Ltd decides to treat the Grant as a Deferred Income.
You are required to calculate the amount of the Depreciation & Grant to be recognized every year in Profit
and Loss Account during the useful life of the machinery if the company followed (a) Straight Line Method (b)
W.D.V. Method.
Solution: As per AS 12 on Accounting for Government Grants, Deferred Income from grants is recognized
in the Profit and Loss Account on a systematic and rational basis over the useful life of the asset in the
proportion in which depreciation on those assets is charged. Unamortized and Deferred Grant Balance should
be disclosed separately in financial statements.
(i) Statement showing SLM Depreciation & Grant to be recognized in P & L A/c

Year Opening Balance Depreciation Closing Balance Grant to be recognized every year
A B C D=B–C E = 36.9 lakhs × (C/ 73.8 lakhs)
2012-13 125 18.45 106.55 9.225
2013-14 106.55 18.45 88.10 9.225
2014-15 88.10 18.45 69.65 9.225
2015-16 69.65 18.45 51.20 9.225
73.80 36.90

Depreciation p.a. as per Straight Line Method [(125 – 51.20)/4] = 18.45


Extra Practical Problems from Past Year Papers and RTPs of Financial Reporting and Audit 377

(ii) Statement showing WDV Depreciation & Grant to be recognized in P & L A/c

Year Opening Balance Depreciation Closing Balance Grant to be recognized every year
A B C = B × 20% D= B – C E = 36.9 lakhs × (C/ 73.8 lakhs)
2012-13 125 25.0 100 12.5
2013-14 100 20.0 80.0 10.0
2014-15 80 16.0 64.0 8.0
2015-16 64 12.8 51.2 6.4
73.8 36.9

Working note: Calculation of WDV Rate of Depreciation


WDV Depreciation Rate (on 1-4-2009) = 1 – 4√Residual Value/Net cost
= 1 – 4√51.20 lakhs/125 lakhs
= 1 – 4√0.4096 = 1 – 0.8 = 0.2 or 20%

AS13
Problem 34: A manufacturing company purchased shares of another company from stock exchange
on 1st May, 2012 at a cost of `5,00,000. It also purchased Gold of `2,00,000 and Silver of `1,50,000 on
1st April, 2010. How will you treat these investments as per the applicable AS in the books of the company
for the year ended on 31st March, 2013, if the values of these investments are as follows:
`
Shares 2,00,000
Gold 4,00,000
Silver 2,50,000
Solution: As per para 32 of AS 13 on ‘Accounting for Investments’, any investment of long term period is
shown at cost. Hence, the investment in Gold and Silver (purchased on 1st April 2005) shall continue to be
shown at cost i.e., `2,00,000 and `1,50,000 respectively as their value have increased. Also as per AS 13, for
investment in shares - if the investment is for short-term period then the loss of `3,00,000 is to be charged
to profit & loss account for the year ended 31st March, 2008. If investment is of long term period then it will
continue to be shown at cost in the Balance Sheet of the company. However, provision for diminution shall
be made to recognize a decline, other than temporary, in the value of the investments, such reduction being
determined and made for each investment individually.
Problem 35: XYZ & Company has acquired 100% of the equity shares of Company ‘X’ during 2011. Company
‘X’ is a defunct company. The net worth of the Company ‘X’ is represented by land and building it owns. XYZ
& Company acquired the shares of Company ‘X’ only for the land and building owned by it. XYZ & Company
had proposed to start a software development facility at this site at the time of share purchase. The software
development facility has not yet been set up, as Company’s existing facility itself is under-utilized.
In the financials of the year 2012, the above investment was classified as fixed assets under the head ‘land
and building’ to reflect the substance of the transaction, as the intention to acquire the shares was to use the
land and building owned by Company ‘X’.
During the year 2014, the Company has decided to sell these shares and has passed a resolution authorizing
the chairman to negotiate and settle the price. As of March 2014, this investment is reflected as fixed assets in
the books. The property is prime one and is an ideal one for setting up new software units, warehouses etc.
a) Should the above shares be classified under investments or is the current treatment of grouping it
under fixed assets correct?
b) If the current treatment of disclosing the shares as land and building is correct, should we disclose
the same as assets held for disposal, with appropriate classification and disclosures?
378 Accounting Standards

Solution: As per para 20 of AS 13 ‘Accounting for Investments’, the cost of any shares in a company, the
holding of which is directly related to the right to hold the investment property, is added to the carrying
amount of the investment property as long term investment.
In the given case, the acquisition of shares in Company X was with the sole motive of using the land and
buildings, and not for earning income by way of dividends. The substance of the transaction is an investment
in fixed assets, though the “form” may be acquisition of shares.
In view of the foregoing, the accounting treatment so far accorded by the Company, in classifying the
shares as fixed assets is correct.
Since the original intention has undergone a change, therefore, on account of the changed intention, i.e. “to
dispose of the shares”, the accountant should reclassify the holdings in the Company X as investments, and
bring the same in the books as “current investment”.
As per para 23 of the standard when long term investments are reclassified as current investments,
transfers are made at the lower of cost and carrying amount at the date of transfer. Accordingly reclassified
current investment should be shown at cost or carrying amount whichever is less.

AS15
Problem 36: On 01/04/12, a Mehtaab Developers engaged employees on a 6 year period for annual salary
of `6.5 lakh. The annual salary is expected to increase by `50,000 every year. The employee is entitled to
annual pension for 4 years after retirement. The annual pension is calculated as 10% of annual salary for
each of the first 3 years and 20% of annual salary for each next 3 years. Market yield on government bond is
10%. Show Provision for Pension A/c in the books of the company.
Solution:
Calculation of Annual Pension: = Calculation of Current Service Cost
Year Salary × 10%
2012-13 650000 × 10% 65000 380280 x 0.621 2,36,154
2013-14 700000 × 10% 70000 380280 x 0.683 2,59,731
2014-15 750000 × 10% 75000 380280 x 0.751 2,85,590
2015-16 800000 × 20% 160000 380280 x 0.826 3,14,111
2016-17 850000 × 20% 170000 380280 x 0.909 3,45,675
2017-18 900000 × 20% 180000 380280 x 1.000 3,80,280
720000

PV of Annuity of 720000 (for 4 years) as if the whole benefit is payable on 31/3/2018 = 720000 × 3.169
= 22,81,680
Annual amount attributable to each year of service = 2281680/6 = `3,80,280

Pension Liability A/C

2012-13
To balance c/f 236154 By Current Service cost 236154
2013-14
To balance c/f 519500 By balance b/d 236154
By Interest cost 23615
By Current Service cost 259731
Extra Practical Problems from Past Year Papers and RTPs of Financial Reporting and Audit 379

The calculations will continue till 2016-17..............


Closing balance of Obligation = Current Service Cost × n = 345675 × 5 = 1728375
2017-18
To balance c/f 2281493 By balance b/d 1728375
By Interest cost 172838
By Current Service cost 380280
EMPLOYEE RETIRES HERE AND NOW HE WILL GET PENSION
2018-19
To Bank 720000 By balance b/d 2281493
To balance c/f 1789642
By Interest cost 228149
By Current Service cost 0

Similarly pension will continue for next 3 more years.


Problem 37: Vetongi Crystals Ltd engaged 100 workers for a monthly salary `5,500 on April 1, 2012. The
employees are entitled to death-cum-retirement gratuity calculated as 40% of last salary drawn for each
completed year of service. An actuary was appointed to advice the company for measuring its obligations in
respect of the gratuity. The actuary presented the following estimates at the end of 2012-13.
a) Average working life per worker = 26 years (inclusive of 2012-13)
b) Annual growth rate of monthly salary = 5%
c) Expected annual rate of return on plan assets = 10%
d) Annual market yield on government bonds of maturity period of 25 years = 8%
For acquisition of plan assets, the company contributed `1,00,000 on 31/03/13 and `1,10,000 on
31/03/14. Fair value of plan assets on 31/03/2014 was `2,19,000.
Show Gratuity payable A/c and Contribution for Plan Asset A/c in the books of the company for 2012-13
and 2013-14.
Solution:
Calculations for 2012-13 to 2013 - 14
Calculation of Final Salary = 5500 x 1.0525 = 5500 x 3.386 = `18625
Gratuity promised at the end = 18625 x 40% = `7450
Current Service Cost for 2012 – 13 = 7450 payable at the end/(1.08)25 = 7450 x 0.1460 = 1088 for 100
employees = `1,08,800
Current Service Cost for 2013-14 = 7450 payable at the end/(1.08)24 = 7450 x 0.1577 = 1175 for 100
employees = `1,17,500

GRATUITY OBLIGATION ACCOUNT


2012-13
To balance c/f 108800 By Current Service cost 108800
FAIR VALUE OF PLAN ASSETS ACCOUNT
To bank 100000 By balance c/f 100000

GRATUITY OBLIGATION ACCOUNT


2013-14
To balance c/f 235000 By balance b/d 108800
By Interest cost 8700
By Current Service cost 117500
380 Accounting Standards

FAIR VALUE OF PLAN ASSETS ACCOUNT


To balance b/d 100000 By Actuarial g/l 1000
To Interest earned 10000 By balance c/f 219000
To bank 110000

Problem 38: In the above problem, consider the following additional information and hence show Gratuity
Payable A/c and contribution for plan Asset A/c for 2014-15 and 2015-16.
a) Annual market yield on government bonds = 7.2%
b) Average annual growth rate of salary re-estimated to 5.22%.
c) Fair value of Plan Asset on 31/03/15 is `4,60,000 and `6,75,000 on 31/3/16
d) Contribution to fund for acquisition of plan asset is `2,13,900 an 31/3/15 and `1,73,000 on 31/03/16
Solution: Calculations for 2014 – 15 and 2015 – 16.
Revised assumptions:
Calculation of Final Salary = 5500 x 1.0522 25 = 5500 × 3.569 = `19625
Gratuity promised at the end = 19625 × 40% = `7850
Current Service Cost for 2014-15 = 7850 payable at the end/(1.072)23 = 7850 × 0.202 = 1586 for 100
employees = `1,58,600
Current Service Cost for 2015-16 = 7850 payable at the end/(1.072)22 = 7850 × 0.2165 = 1700 for 100
employees = `1,70,000
PV of obligation as on 31/3/2014 (2 years end) = 7850/(1.072)24 = 7850 × 01884 = 1480 for 100 employees
= `1,48,000 × 2 years = `2,96,000
Actuarial G/L = PV of obligation as on 31/3/2014 as per old rates – as per new rates = 7850/(1.072)24 =
7850 × 01884 = 1480 for 100 employees = `1,48,000 × 2 years = `2,96,000 – 2,3,5000 = `61,000

Gratuity Obligation Account


2014-15
To balance c/f (CSC × n) 475800 By balance b/d 235000
By Interest cost (296000 × 7.2%) 21312
By Current Service cost 158600
By Actuarial loss 61000
Fair Value of Plan Assets Account
To balance b/d 219000 By balance c/f 460000
To Interest earned 21900
To bank 213900
To Actuarial gain 5200

Problem 39: Elegance Limited in its 15th year of incorporation. Following is the position of the funded
gratuity plan for a group of employees.
31/03/2014
PV of obligation 37,00,000
Less: FV of Plan assets (25,00,000)
12,00,000
Less: Unrecognized Past Service Cost (70,000)
(2 years remaining for vesting benefits)
Net 11,30,000
Extra Practical Problems from Past Year Papers and RTPs of Financial Reporting and Audit 381

The employees related to the above funds were expected to retire by the end of 2017-18. The concerned
employees had joined the services from 01/04/2007, the retirement plans also started from the same period.
The annual amount attributed towards the services as calculated by the actuary is `6,92,000 each year. The
yield on government bond is expected to be 7% and return on plan assets estimated to be 9.5%. By the
end of 31/3/2015 10% of the employee’s services were curtailed with the settlement of a fixed amount of
`1,40,000. The past service cost will get proportionately reduced. FV of plan assets on account of settlement
remains same. Contributions made to the plan assets were `5,00,000. The FV of the plan assets were valued
at `32,00,000. You are required to compute the following:
1) Current Service Cost, 2) Actuarial gain or loss, 3) Balance Sheet disclosure, 4) Employee Cost charged
to profit and loss a/c.
Solution:
1) Current Service Cost = `5,64,880, 2) Actuarial gain/loss = `32,660 (gain),
3) Balance Sheet Disclosure: PV of obligation 40,67,136
Less: FV of Plan Assets = (32,00,000)
Net : 8,67,136
Less: Unamortized PSC (31,500)
8,35,636
4) Employee Cost charged to P/L = `2,80,316

AS16
Problem 40: As a Statutory Auditor, how would you deal with the following? ABC Ltd. commenced
construction of a flyover in Mumbai in January, 2012 under BOLT scheme. The same was completed in
February, 2013. Due to seasonal heavy rains in July, 2012 in the area, the work on the flyover had to be
suspended for 1 month. The company accordingly suspended borrowing costs of `12.50 lakhs for that month
from capitalization.
Solution: Borrowing costs should be suspended:
i) When active development is interrupted due to avoidable reasons and no work is carried out.
ii) When active development is interrupted but some minor work like security, maintenance etc. may be
carried out.
Borrowing costs should not be suspended: i) When active development is going on without any interruption.
ii) When technical work is carried out but active development may be interrupted. iii) When temporary delay
is necessary for the maturity of the product for its intended use and sale. iv) When suspension is a natural
circumstance like for example Heavy rains during construction of Bridge. Hence in the present case BC can
be continued to be capitalised and should not be suspended.
Problem 41: Parvesh Ltd. had the following borrowings during a year in respect of capital expansion:
Plant Cost of Asset (`) Remarks
Plant P 100 lakhs No specific borrowings
Plant Q 125 lakhs Bank loan of `65 lakhs at 10%
Plant R 175 lakhs 9% Debentures of `125 lakhs were issued.

In addition to the specific borrowings stated above, the Company had obtained term loans from two banks
(1) `100 lakhs at 10% from Corporation Bank and (2) `110 lakhs at 11.50% from State Bank of India, to meet
its capital expansion requirements. Determine the amount of borrowing costs to be capitalized in each of the
above Plants, as per AS-16.
382 Accounting Standards

Solution:
(1) Computation of actual borrowing costs incurred during the year

Sources Loan Amount Interest Rate Interest Amount


(` in lakhs) (` in lakhs)
Bank Loan 65.00 10% 6.50
9% Debentures 125.00 9% 11.25
Term Loan from Corporation Bank 100.00 10% 10.00
Term Loan from State Bank of India 110.00 11.5% 12.65
Total 400.00 40.40
Specific Borrowings included in above 190.00 17.75

(2) Weighted Average Capitalization Rate for General Borrowings


Total Interest–Interest on Specific borrowings
= – Specific borrowings
Total Borrowings
(40.40 –17.75)
= =22.65/210 =10.79% (approx.)
(400 – 190)
(3) Capitalization of Borrowing Costs under AS-16 will be as under:

Plant Borrowing Loan Amount Interest Rate Interest Cost of Assets


Amount
P General 100 10.79% 10.79 110.79
Q Specific 65 10.00% 6.50 71.50
General 60 10.79% 6.47 66.47 137.97
R Specific 125 9.00% 11.25 136.25
General 50 10.79% 5.39 55.39 191.64
Total 400 40.40 440.40

Note: The amount of borrowing costs capitalized should not exceed the actual interest cost.

Problem 42: On 1st April, 2012, Amazing Construction Ltd. obtained a loan of `32 crores to be utilized as
under:
(i) Construction of sea link across two cities:
(work was held up totally for a month during the year due to high water levels) : `25 crores
(ii) Purchase of equipments and machineries : `3 crores
(iii) Working capital : `2 crores
(iv) Purchase of vehicles : `50,00,000
(v) Advance for tools /cranes etc. : `50,00,000
(vi) Purchase of technical know-how : `1 crores
(vii) Total interest charged by the bank for the year ending 31st March, 2013 : `80,00,000.
Show the treatment of interest by Amazing Construction Ltd.
Solution: According to para 3 of AS 16 ‘Borrowing costs’, qualifying asset is an asset that necessarily takes
substantial period of time to get ready for its intended use. As per para 6 of AS 16, borrowing costs that are
directly attributable to the acquisition, construction or production of a qualifying asset should be capitalized
as part of the cost of that asset. Other borrowing costs should be recognized as an expense in the period in
which they are incurred.
The treatment of interest by Amazing Construction Ltd. can be shown as: Capitalization:
Construction of sea-link Yes 62,50,000 (80,00,000*(25/32)
Extra Practical Problems from Past Year Papers and RTPs of Financial Reporting and Audit 383

Purchase of equipments and machineries No 7,50,000 (80,00,000*(3/32)


Working capital No 5,00,000 (80,00,000*(2/32)
Purchase of vehicles No 1,25,000 (80,00,000*(.5/32)
Advance for tools, cranes etc. No 1,25,000 (80,00,000*(.5/32)
Purchase of technical know-how No 2,50,000 (80,00,000*(1/32)
Total Capitalized = `62,50,000 and charged to P/L = `17,50,000
Problem 43: An amount of `20,00,000 was incurred for construction of a building and it was ready for
occupation on 31.12.2012. The construction expenditure was incurred out of working capital facilities
availed from the Bank. Interest payable to it @ 15% p.a. The average working capital loan has never fallen
below `25 lakhs during the construction period.

The details of expenditure incurred are as follows: (`)


July 2012 3,00,000
August, 2012 4,50,000
September, 2012 2,00,000
October, 2012 5,00,000
November, 2012 3,00,000
December, 2012 2,50,000
Total 20,00,000

Calculate the value of the qualifying asset.


Solution:
Month Expenditure on QA Interest Cumulative epxn on QA
July 2012 300000 - 300000
August 2012 450000 3750 753750
October 2012 200000 9422 963172
November 2012 500000 12040 1475212
December 2012 300000 18440 1793652
January 2013 250000 22421 2066073
2000000 66073

The value of the qualifying asset is `20,66,073.


Problem 44:
Amount (`)
Expenditure incurred till 31.03.05 5,00,000
Interest cost capitalised for the financial year 2004-05 @13% 26,000
Amount borrowed till 31.03.05 is 2,00,000
Assets transferred to construction during 2005-06 1,00,000
Cash payment during 2005-06 75,000
Progress payment received 3,50,000
New borrowing during 2005-06 @ 13% 2,00,000
Calculate the amount of borrowing cost to be capitalised.
384 Accounting Standards

AS17
Problem 45: Following details are given for Cheerless Ltd. for the year ended 31st March, 2005:
`(’000) `(’000)
Sales:
Food Products 5,650
Plastic and Packaging 625
Health and Scientific 345
Others 162 6,782
Expenses:
Food Products 3,335
Plastic and Packaging 425
Health and Scientific 222
Others 200 4,182
Other items:
General corporate expenses 562
Income from investments 132
Interest expenses 65
Identifiable assets:
Food Products 7,320
Plastic and Packaging 1,320
Health and Scientific 1,050
Others 665 10,355
General corporate assets 722
Other information:
(a) Inter-segment sales are as below: `(’000)
Food Products 55
Plastic and Packaging 72
Health and Scientific 21
Others 7
(b) Operating profit includes `(’000) 33 on inter-segment sales.
(c) Information about inter-segment expenses are not available.
You are required to prepare a statement showing financial information about Cheerless Ltd.’s operations
in different industry segments.
Solution:
Industry Segments
Item Food Plastic & Health & Others Inter- Consolidated
Products Packaging Scientific segment figs.
Eliminations
`(’000) `(’000) `(’000) `(’000) `(’000) `(’000)
Revenues:
Sales to unaffiliated
Customers 5,595 553 324 155 6,627
Inter-segment sales 55 72 21 7 (155) —
Extra Practical Problems from Past Year Papers and RTPs of Financial Reporting and Audit 385

Total revenue 5,650 625 345 162 (155) 6,627


Segment expenses (3,335) (425) (222) (200) 122 (4,060)
Operating profit 2,315 200 123 (38) (33) 2,567
Other items:
General corporate (562)
expenses
Income from 132
investments
Interest expenses (65)
Income from 2,072
continuing operations
Identifiable assets 7,320 1,320 1,050 665 10,355
General corporate 722
assets
Total assets 11,077

AS18
Problem 46: A Ltd., owns 30% of the equity capital of B Ltd. B Ltd. in turn owns 35% of equity capital C Ltd.,
and 40% of equity capital in D Ltd. You are required to answer the following questions.
(a) Whether B Ltd. is a related party to A Ltd?
(b) Whether C Ltd., is a related party to A Ltd.?
(c) Whether C Ltd. and D Ltd. are related parties?
Solution:
(a) As per AS 18, associates and joint ventures of the reporting enterprise are related parties. Since A
Ltd. holds more than 20% of the voting power in B Ltd. by virtue of this it has substantial interest and
significant influence in B Ltd. Therefore B Ltd. is an associate and is related party to A Ltd.
(b) An associate of an associate is not a related party. Only in the case of a holding company a subsidiary
of a subsidiary (sub-subsidiary) also becomes a related party.
(c) C Ltd. and D Ltd. are co-associates. Co-subsidiaries become related parties because of common
control. In the case of this common control is missing and therefore they are not related parties.
Problem 47: What are your views on the following?
i) A firm of a father and son is received `2 lakhs towards job work done for XYZ Ltd during the year
ended on 31.03.10. The total job work charges paid by XYZ Ltd. during the year are over `50 lakhs.
The father is a Managing Directors of XYZ Ltd having substantial holding. The Managing Director told
the auditor that since he is not involved in the activities of the firm and since the amount paid to it is
insignificant; there is no need to disclose the transaction. He further contended that such a payment
made in the last year was not disclosed. Is Managing Director right in his approach?
ii) Mr. Raj a relative of key Management personnel received remuneration of `2,50,000 for his services
in the company for the period from 1.4.2009 to 30.6.2009. On 1.7.2009 he left the service.
Solution:
(i) The MD is not correct in its view.
The so called individual is a KMP in XYZ Ltd and is a partner in the firm. In accordance with para 3
(e) of AS-18 Enterprises controlled by a common individual referred to in para 3 ( c ) and 3 ( d ), are
related party if they enter the transaction among themselves. Hence disclosure requirements are
386 Accounting Standards

applicable in the books of both the firm and XYZ Ltd. The auditor should qualify the audit report in
accordance with SA 705, SA 550 and Companies Act section 143.
(ii) According to para 10 of AS 18 on Related Party Disclosures, parties are considered to be related if at
any time during the reporting period one party has the ability to control the other party or exercise
significant influence over the other party in making financial and/or operating decisions. Hence, Mr.
Raj, a relative of key management personnel should be identified as relative as at the closing date i.e.
on 31.3.2010.

AS19
Problem 48: X Ltd. sold JCB Machine having WDV of `50 Lakhs to Y Ltd for `60 Lakhs and the same JCB
was leased back by Y Ltd to X Ltd. The lease is operating lease Comment according to relevant Accounting
Standard if:
(i) Sale price of `60 Lakhs is equal to fair value
(ii) Fair Value is `50 Lakhs and sale price is `45 Lakhs.
(iii) Fair value is `55 Lakhs and sale price is`62 lakhs
(iv) Fair value is `45 Lakhs and sale price is `48 Lakhs.
Solution:
(i) X Ltd should immediately record the profit of `10 lakhs.
(ii) Loss of `5 lakhs immediately charged to P/L provided loss is not compensated by future lease rentals.
(iii) Profit of 5 lakhs (55-50) is to be immediately recognized and the profit of `7 lakhs (62-55) is to
deferred over lease period.
(iv) Loss of 5 lakhs (50 -45) is to be immediately recognized and the profit of `3 lakhs (48-45) is to
deferred over lease period.
Problem 49: Lessee Ltd. has taken a machine on lease from Lessor Ltd. The following information is
available to you:
Lease term 3 years
Useful life 3 years
Fair value at the inception of lease `7,00,000
Fixed Lease Rent `3,00,000 payable at the end of each of the 3 years
GRV to be assured by lessee `22,000
IRR @15% p.a.

You are required to work out the following:


(i) Value of Machinery to be recognized by lessee.
(ii) Yearly Finance Charges. (PV factors @15% p.a. = 0.870, 0.756 and 0.658).
(Final C. A. Nov. 2007 – Accounts) (6 Marks)
Solution: Value of machine will be lower of the fair value or present value (PV) of Minimum Lease
Payments (MLP).
Present value (PV) of Minimum Lease Payments (MLP)

Year MLP ` PVat15% PV Amount `


1 3,00,000 0.869 2,60,700
2 3,00,000 0.756 2,26,800
3 3,22,000 (considering residual value) 0.657 2,11,554
6,99,054
Extra Practical Problems from Past Year Papers and RTPs of Financial Reporting and Audit 387

Since PV of MLP `6,99,054 being lower than the fair value `7,00,000, therefore, value of machine
will be taken as `6,99,054.

Calculation of interest (finance charges)

Year Liability Interest at 15% Principal Lease rental


` ` ` `
6,99,054 1,04,858 1,95,142 3,00,000
1st Less Principal 1,95,142 (Rental - Interest)
5,03,912 75,587 2,24413 3,00,000
2nd Less Principal 2,24,413 (Rental - Interest)
2,79,499 41925 2,58,075 3,00,000
3rd Less Principal 2,58,075 (Rental - Interest)
Residual value 21,424*

The difference between this figure and guaranteed residual value (`22,000) is due to approximation in
computing the interest rate implicit in the lease.
Problem 50: AS Ltd. Leased a machine to SB Ltd. on the following terms: (` in Lakhs)
Fair value of the machine 4.00
Lease term 5 years
Lease Rental Per annum 1.00
Guaranteed Residual value 0.20
Expected Residual value 0.40
Internal Rate of Return 15%
Depreciation is provided on straight line method at 10 per cent per annum. Ascertain unearned Financial
Income. Necessary Journal entries in the books of the Lessee in first year may be shown.
(Final C.A. Nov. 2001) (8 Marks), CA final Nov 2009 New Syllabus Similar Qt in Nov 2010)
Solution: As per AS 19 on Leases, unearned finance income is the difference between (a) the gross
investment in the lease and (b) the present value of minimum lease payments under a finance lease from
the standpoint of the lessor; and any unguaranteed residual value accruing to the lessor, at the interest rate
implicit in the lease.
where:
Gross investment in the lease is the aggregate of (i) minimum lease payments from the stand point of the
lessor and (ii) any unguaranteed residual value accruing to the lessor.
Gross investment = Minimum lease payments + Unguaranteed residual value
= [Total lease rent + Guaranteed residual value (GRV)] + Unguaranteed residual value (URV) = [(`1,00,000
× 5 years) + `20,000] + `20,000
= `5,40,000
(a)
1 1,00,000 0.8696 86,960
2 1,00,000 0.7561 75,610
3 1,00,000 0.6575 65,750
4 1,00,000 0.5718 57,180
5 1,00,000 0.4972 49,720
20,000 0.4972 9,944 (grv)
5,20,000 3,45,164
20,000 0.4972 9,944 (ugrv)
5,40,000 3,55,108
388 Accounting Standards

Unearned Finance Income = `5,40,000 – `3,55,108 = `1,84,892


Accounting in the books of SB Ltd (lessee): (only 1 year)
Beginning:
Machine Dr 345164
To AS Ltd a/c. 345164
End:
Depreciation Dr 34516
To Machine 34516
Finance Expenses Dr 51775
To AS Ltd. a/c 51775
AS Ltd a/c Dr 100000
To Bank a/c. 100000
Profit / Loss a/c Dr 86291
To Depn 34516
To Finance exp 51775
Problem 51: The following balances are extracted from the Books of Ram Ltd., a real estate company on
31st March 2011:

Dr. Cr.
(` in ‘000) (` in ‘000)
Leasehold premises 42
Equipment, fixtures and fittings at cost on 01-04-2010 264
Depreciation on equipment fixtures and fittings on 01-04-2010 164

The following additional information are also provided


1. Depreciation on equipment, fittings and fixtures is provided @ 15% on written down value.
2. On 1st October 2010, the company moved to new premises. The premises are on a 12 year lease and
the lease premium paid amounted to ` 42,000. The company used sub-contract labour of ` 40,000
and materials at cost of ` 38,000 in refurbishment of the premises. These are to be considered as part
of the cost of lease hold premises.
You are required to prepare the ‘Notes to accounts’ including significant accounting policies forming part
of the financial statements, for disclosure of the above facts and information provided.
(CA Final Nov 2011) (5 marks)
Solution:
1. Depreciation of Equipment, Fixtures and Fittings:
WDV of Equipment, fixture & fitting as on 1.4.2010 = Cost of Asset –Accumulated Depreciation
= 264 – 164 = ` 100 (` 000)
Depreciation @ 15% of Equipment, fixture & fitting = 100 ×15% = 15 or ` 15,000
Gross Block = 2,64,000
Less: PFD = (1,79,000) → (1,64,000 + 15,000)
85,000
2. Leasehold Premises:
(a) The Total cost incurred = Lease premium + Sub Contract Labour + Cost of Materials. = 42,000 +
40,000 + 38,000 = `1,20,000
(b) Annual Amortisation = Total Cost ` 1,20,000 ÷ Lease Period 12 Years = ` 10,000
Extra Practical Problems from Past Year Papers and RTPs of Financial Reporting and Audit 389

(e) Since, in the given question the leasehold premise was taken only on 1st October, and therefore,
amortization can be done only for 6 Months. Hence, amortization for the current year =`5,000.
Lease hold premises = 120000 Less: Amotz 5000 = 115000.
3. Notes on Accounts:
(a) Fixed Assets: Fixed Assets are carried at historical cost and depreciated at 15% on diminishing
balance method over its useful life.
(b) Leasehold Premises: Cost incurred on the Leasehold Premises are amortized over the Lease
period. Lease hold premises is an Intangible asset.

AS20
Problem 52: Utility Ltd. has 40,000 equity shares of `10 each and 10% 5,000 Convertible Debentures of
`100 each outstanding during 2010-11. Each Debentures got converted into 3 shares on 1/12/2011. Net
profit after tax for the year is `3,45,000. Tax rate 40%. Calculate Basic and Diluted EPS for the year 2011-12.
Solution:
345000
Calculation of Basic EPS = = `7.67
40000+(15000 ×4/12)
345000 + 50000 (1 – 0.40) × 8/12
Calculation of Diluted EPS = = `7.30
40000 + 15000 × 8/12

Problem 53: Calculate the Basic Earning Per Share of x Ltd. for year 2013-14 and restatement for the year
2012-13:

Date Particulars Nos


01.04.13 Balance at the beginning of year - Face value `100 10,000
01.06.13 Issue of Shares for cash 2,400
01.07.13 Issue of Shares to Underwriters 120
01.08.13 Buy – back of Shares 5,385
01.09.13 Issue of Shares against Purchase of Building. (acquisition is recognized on the same day) 600
1.10.13 Issue of Shares in settlement of liability 300
1.11.13 X Ltd. purchased Y Ltd. on the basis of intrinsic value of shares of both the companies. The
intrinsic value of shares of X Ltd. and Y Ltd are `60 and `15 respectively Shares outstanding
on 1st April, 2013 of Y Ltd. are 9600.
01.12.13 Split up of shares into shares of `10 each

Net Profit after tax for the year 2012-13 `34,80,000


Net Profit after tax for the year 2013-14 `36,60,000
Right issue 2 for 5 held
Right issue price `30 per share
Last date to exercise right 31st December, 2013
Fair value of 1 equity share immediately prior to exercise of right on 31st December, `58 per share
2013
Bonus issue on 31st March, 2014 1 equity share for 5 equity shares
outstanding on 31st March, 2014
390 Accounting Standards

Solution:
3660000
Calculation of Basic EPS for 2013 – 14 = = `24.05
152178#
3480000
Calculation of Restated EPS 2012 – 13 = = `23.85
145918##
##
100000 + 16700 + 29218 = 145918
#Calculation of WANES:

Date Particulars Shares Ratio Product


1.4.13 Opening shares 10000 1/1 10000
1.6.13 Cash issue 2400 10/12 2000
1.7.13 Shares issued to underwriters 120 9/12 90
1.8.13 Buy back -5385 8/12 (3590)
1.9.13 Shares against building 600 7/12 350
1.10.13 Shares for settlement of liab 300 6/12 150
1.11.13 Shares issued in Amalgamation 2400 5/12 1000
(9600 × 15/60) = 2400
10435 10000
Impact of Share split 10 12/12 10
Shares after shares split = 104350 100000
31.12.13 Right shares (WN $)
Fresh issue element 25040 3/12 6260
Bonus element 16700 12/12 16700
Shares till right issue = 146090 122960
31.3.14 Bonus 1/5 29218 12/12 29218
175308 152178

(WN $): Right shares:


OS × MP + RI × Subscription Price
Theoretical Ex-right MP =
Total Shares
104350 × 58 + 41740 × 30 7304500
= = = `50
104350 + 41760 146110
Total Right shares 41740
Less: Shares for which proceeds are received 25040 (41740 × 30/50)
Bonus element for which no consideration received 16700

AS22
Problem 54: Comment on the following: T Ltd. an Indian company, is subject to Indian Income Tax Act,
1961, discloses Advances Income-tax paid (Current asset) and Provision for Income Tax (Current tax liability),
separately in Balance Sheet for the year ended 31.3.2013, i.e. do not offset the amount.
Solution: AS-22 provides that an enterprise should offset assets and liabilities representing current tax if
the enterprise: Has a legal enforceable right to set off the recognised amount and intends to settled the asset
and liability on net basis. T Ltd should offset the advance tax paid and provision for tax for the year ended
31/3/2013.
Extra Practical Problems from Past Year Papers and RTPs of Financial Reporting and Audit 391

Problem 55: SRS Ltd. has drawn the financial statement as on 31-03-11 and presented to you alongwith
additional information:

Balance Sheet of SRS Ltd. as on 31-03-11


Liabilities Amount Assets Amount
Share capital 50,00,000 Fixed Assets
Reserves & Surplus Gross block
Profit and Loss A/c 4,00,000 Less: Depreciation 1,00,00,000
Secured Loans 75,00,000 Investments Nil
Current Liabilities and Provisions: Current Assets Loans & Advances:
Creditors for trade 3,00,000 Debtors 25,00,000
Advance received 3,00,000 Advance Paid 10,00,000
1,35,00,000 1,35,00,000

Additional Information:
(a) Entire pre-operative expenses of `7,00,000/- was charged to Profit and .Loss Account whereas for
the purpose of Income Tax, only what is allowable is claimed.
(b) Depreciation as per Books - `35,00,000/-, Depreciation as per Income tax - `50,00,000/-
(c) Losses to be carried forward as per Income Tax Act - `16,00,000/-
(d) Donation disallowed while computing tax - `50,000/-
Considering the additional information, can you certify that the company has complied with the Accounting
Standards and issue an unqualified report?
Solution:
Particulars Comment
Fixed Assets As per the new Schedule III one should draw a detailed Schedule of Fixed Assets in terms of
Gross block, Depn, Net Block.........
Pre operative expenses The company is correct by writing off the entire pre – operative expenses to P/L as per AS-26.
But the company has failed to create DTA on the pre operativ expenses disallowed.
Deferred tax on Depn The company has not provided any DTL for the timing difference on account of depreciation.
Donation It is a permanent difference no DTA / DTL is required.
Losses unabsorbed Assuming company has enough evidence to earn sufficient profits DTA should be provided for
the unabsorbed losses under IT Act.
Other Assets/Liabilities Advance paid / recd/ Crs / Debtors are not presented as per New Schedule III.

The Financial Statements are not in accordance to New Companies Act and AS. Th reports should be
qualified as per SA 705.

AS23
Problem 56: Tarun Ltd. had acquired 25% of the equity share capital of Varun Ltd. at `2,40,000 by 1-7-
2012. It had received `8,000 as dividend for the year 2011-12. Equity share capital of Varun Ltd. is `5,00,000.
Varun Ltd. had not provided for the dividend when the accounts for the year 2011-12 were prepared. Find out
goodwill/capital reserve against investment in Varun Ltd. as well as the value at which investment shall be
reported in consolidated financial statements to be prepared by Tarun Ltd. as on 31-3- 2013, if the balances
in profit and loss account were `84,000 and `1,92,000 respectively at the end of 2011-12 and 2012-13.
392 Accounting Standards

Solution:
Date of Acquisition 1-7-2009
Date of consolidation 31-3-2010
Pre-acquisition period 3 months
Post-acquisition period 9 months
% of Holding 25%

Analysis of Capital and Revenue profits

Profit and loss Account 31-3-2010


1,92,000

Opening 52,000 Current Year


[84,000 – 32,000 1,40,000
(8,000/25%)]

3 months 35,000 9 Months 150000


Capital Profit
87,000 Revenue Profit

Total Tarun Ltd.


100% 25%
(i) Capital Profit 87,000 21,750
(ii) Revenue Profit 1,05,000 26,250
(iii) Equity Share Capital 5,00,000 1,25,000

Computation of Goodwill/Capital Reserve

`
Investments 2,40,000
Less: Pre-Acquisition dividend 8,000
Nominal Value of shares 1,25,000
Share in Capital Profits 21,750 1,54,750
Goodwill 85,250
Investment in associate under equity method
Nominal Value of Investment 1,46,750
Goodwill 85,250
2,32,000
Add: Share of Revenue Profit 26,250
2,58, 250
Extra Practical Problems from Past Year Papers and RTPs of Financial Reporting and Audit 393

Problem 57: Compute the total Stock Reserve and pass journals in respect of stock reserve in each case:
Case 1: H sold goods to A an Associate company at invoice value of `45,000 at 1/8th of cost. In the same year
A sold goods to H at an invoice value of `65,000 at 1/7th of cost. At the year end H could sell 50% of the goods
received from A and A sold 60% of the goods received from H. H holds 25% shares in A.
Solution:
When goods are sold by H to A:
Profit/Loss Dr 2000
To Investments in A 2000
(45000 × 1/9) 40%
When goods are sold by A to H:
Profit/Loss Dr 4063
To Stock Reserve 4063
(65000 × 1/8) 50%
Problem 58: Kadambari invested 2,50,000 in Shetambari an associate company on 1/7/2014 with an
interest of 20%. The Share Capital on that date was `3,00,000, P/L balance `2,00,000. Shetambari declared
dividends for 2013-14 and 2014-15 at the rate 12% and 15% respectively in the month of August. Profits
earned in the 2 years were `1,50,000 and `1,75,000 respectively. Compute the carrying amount of investments
for CFS and SFS as on 1/7/2014, 31/3/2015 and 31/8/2015.
Solution:
Calculation of Carrying Amount as per AS-23
Date 1/7/2014 31/3/2015 31/8/2015
Cost / Carrying amount 250000 250000 265600
Less: Pre Dividend received - (7500) (2250)
(300000 × 12%)20%
(300000 × 15%)20% × 3/12
Add: Share of P/L - 22800 26000
(150000 – 36000)20%
(175000 – 45000)20%
(150000 – 36000)20% -
(175000 – 45000)20%
250000 265600 289350

AS24
Problem 59: A company enters into three sale agreements for disposing off the assets of three separate
components of the business. Can the company report the discontinuing operations on a consolidated basis?
Elucidate.
Solution: As per Para 31 of AS-24, disclosure is to be presented separately for each discontinuing operation.
Hence, reporting on an aggregated basis not permitted.
Problem 60: Hashmi Ltd. has 2 factories producing Shawls, Bed Sheets, Mats, and Curtains. It planned to
discontinue production of Mats. The company does not apply AS 17 segment Reporting since it produces
only one class of goods. The company has no related plan to increase production of other products such
that its sale gets compensated. Mats constitute 15% of the sale of the company. Does this sale qualify for
discontinuing operations.
394 Accounting Standards

Solution: Segmental Reporting as defined in AS-17 normally represents or satisfies separate major line of
business or a geographical area of operation. Even if an enterprise operates in a single business or geographical
segment, a major product or service line may also satisfy this criterion. Production of Mats represents a
separate major line of business. Hence the present case is covered by AS-24.

AS25
Problem 61: Fixed production overheads for the financial year is `19,200. Normal expected production
for the year, after considering planned maintenance and normal breakdown, and the future demand of the
product is 4,800 MT. It is considered that there are no quarterly/seasonal variations. Therefore, the normal
expected production for each quarter is 1200 MT and the fixed production overheads for the quarter are
`4,800.

Actual production achieved


First quarter 1000 MT
Second quarter 1400 MT
Third quarter 800 MT
Fourth quarter 1400 MT

Required:
(i) Presuming that there is no quarterly/seasonal variation, calculate the allocation of fixed production
overheads for all the four quarters as per relevant Accounting Standards.
(ii) In case there are quarterly and seasonal variation how the estimate of normal capacity to be made as
per relevant Accounting Standards.
Solution:
(i) It is considered that there is no quarterly/seasonal variation, therefore normal expected production
for each quarter is 1200 MT and fixed production overheads for the quarter are `4,800.
Fixed production overhead to be allocated per unit of production in every quarter will be `4 per MT
(Fixed overheads/Normal production i.e. `4,800/1200 MT).
First quarter
Actual fixed production overheads = `4,800.
Fixed production overheads based on the allocation rate of `4 per unit allocated to actual production =
`4 × 1000 MT = `4,000.
Unallocated fixed production overheads to be charged as expense as per paragraph 9 of AS-2 and
consequently as per AS-25 = `800.
Second quarter
Actual fixed production overheads as per para 27 of AS 25, on year-to-date basis = `9,600.
Fixed production overheads to be absorbed on year-to-date basis
2400 MT × `4 = `9,600.
`800 was not allocated to production in the first quarter. To give effect to the entire `9,600 to be
allocated in the second quarter, as per paragraph 29(a) of AS-25, `800 is reversed by way of a credit
to the profit and loss account of the second quarter.
Third quarter
Actual production overheads on year-to-date basis = `14,400.
Fixed production overheads to be allocated on year-to-date basis
3,200 MT × `4 = `12,800
Under allocated overheads of `1600 is to be changed as expense as per para 9 of AS 2 and consequently
as per AS-25.
Extra Practical Problems from Past Year Papers and RTPs of Financial Reporting and Audit 395

Fourth quarter/Annual
Actual fixed production overheads on year-to-date basis = `19,200.
Fixed production overheads to be allocated on year-to-date basis
4,600 MT × `4 = `18,400
`1600 was not allocated to production in the third quarter due to less production. However, due
to over production of 200 MT in the fourth quarter, excess allocable fixed overhead in the fourth
quarter is given effect to by reversal of previously charged amount to Profit and Loss account (i.e. in
the third quarter) to the extent of `800 (200 MT x `4). Therefore, `800, over allocable in the fourth
quarter, are to be reversed as per para 29(a) of AS-25 by way of a credit to the profit and loss account
of the fourth quarter.
Unallocated overheads for the year `800 are expensed in the profit and loss account as per para 9 of
AS-2.
The cumulative result of all the quarters would also result in unallocated overheads of `800. Hence,
the requirements of para 27 of AS-25 that the quarterly results should not affect the measurement of
the annual results has been complied with.
(ii) In a case where there are quarterly/seasonal variations, the estimates of normal capacity would
have to be made on the quarterly/seasonal basis. The enterprise will have to estimate its normal
capacity on the basis of the average of the relevant quarters/seasons of past few years, say 3 to 5
years, also considering the demand of the product during the season. (For example, a company is
having seasonal variations say, in the 3rd quarter. The estimate for the 3rd quarter should be based
on the capacity utilisation of the 3rd quarter for the past few years, say 3 to 5 years, also keeping in
mind the future demand of the product during the quarter). Once the normal capacity for a quarter
is determined, as aforesaid, the quarter should be considered for measurement purposes, as per
paragraph 27 of AS-25 on year –to-date basis, i.e. on cumulative basis, which would then be added
up to determine the normal capacity for the year on the basis of which the absorption rate will
be determined. The variations between the seasons would thus be considered normal and treated
accordingly.
If there is an abnormal breakdown during a period, as per AS-2, the amount of fixed production
overheads not allocated to units of production is charged to the profit and loss account. However,
the result of under allocation of overheads or over allocation of overheads should not affect the
measurement of its annual results since interim periods are parts of a financial year.

AS26
Problem 62: Government of Goa has given fishing license to Fishermen Ltd. at a nominal price of `1,000.
The market price of the license is `1,00,000. An expenditure of `2,500 has been incurred by the company in
relation to the fishing license. At what price should the license be recorded in the books of the company?
Solution: Any intangible asset acquired through government grants should be recorded at a concessional
price. But any expenditure incurred on it needs to be capitalised. Accordingly in the present case the cost of
the license fees worth `100000 will be `1000 + 2500 = `3500.
P roblem 63: Rajesh Ltd. has acquired exclusive marketing rights worth `50 lakhs from Ramkumar Ltd. in
exchange for import quotas worth `40 lakhs. Assuming that Rajesh Ltd. has not paid any other consideration
to Ramkumar Ltd, at what price should it initially record the marketing rights?
Solution: As per AS – 10 as well as AS – 26 if any asset is acquired against any other asset, the cost will be
the Fair market value of the asset given up if it is more evident otherwise, FMV of the asset acquired. Here the
cost of marketing rights can be taken as `40 lakhs which FMV of the asset given up.
P roblem 64: Suresh Ltd. purchased a trademark from Trilok Ltd. on 1-5-2012 for a price of `50 lakhs. It
is well established in the industry that the expected useful life of such a trademark is 22 years. Trilok Ltd.
396 Accounting Standards

acquired this trademark two years ago. Suresh Ltd. wants to amortise the trademark over its remaining useful
life of 20 years whereas a consultant has advised that having regard to AS 26, the remaining amortisation
period is 8 years only as the total period allowed for amortisation of intangible assets under this Standard is
10 years only. Comment.
Solution: As per para 63 of AS-26 any intangible asset acquired / generated can be amortized over its
estimated useful life or contractual life. 10 years should be considered only when estimated life cannot be
possible. In the present case the total life for amortization is 22 years but from today it is 20 years. 10 years
should be considered only as a measure of last resort.
P roblem 65: As a statutory auditor, how would you deal with the following: Investopia Pvt. Ltd, started
stock broking activities in 2012. For this purpose is acquires membership of a stock exchange for 200 lacs.
While finalizing the accounts, the company disclosed the above amount under the Fixed Assets schedule as
‘Stock Exchange Membership Rights”. The company also did not write off any amount since the rights would
enable the company to perpetually carry on its business.
Solution: As per para 63 of AS-26 any intangible asset acquired / generated can be amortized over its
estimated useful life or contractual life. 10 years should be considered only when estimated life cannot be
possible. In the present case as the Stock Membership Card carries a perpetual life it is to be amortised over
10 years. If the asset is not amortised then in that case the audit report needs to be qualified under SA 705.
Problem 66: As an auditor, how would you deal with the following: PQR Ltd. had acquired a Brand from
another company for `100 lakhs. PQR Ltd. contended that since the said brand is a very popular and famous
brand, no depreciation or amortization needs to be provided.
Solution: Same as problem 64.
Problem 67: An Enterprise has incurred expense for purchase of Technical know-how for manufacturing
a car. The Enterprise has paid `5 crores for the use of know-how for a period of 4 years. The Enterprise
estimates the production of cars as follows:
Year No. of Mopeds
1 25,000
2 50,000
3 75,000
4 1,00,000
(a) How will the Enterprise amortize the Technical know-how Fees as per AS 26?
(b) Whether this amortization should be directly charged as an expense or should form part of production
cost of the cars?
Solution: Intangible Asset is an identifiable non-monetary asset without physical substance held for use in
the production or supply of goods or services for rental to other or for administrative purposes. Amortisation
is the systematic allocation of the depreciable amount of an intangible asset over its useful life. The depreciable
amount of an intangible asset should be allocated on a systematic basis over the best estimate of its useful life.

Year No. of Cars Amortization p.a.


1 25,000 50 lakhs (500 lakhs x 25000 / 250000)
2 50,000 100 lakhs (500 lakhs x 50000 / 250000)
3 75,000 150 lakhs (500 lakhs x 75000 / 250000)
4 1,00,000 200 lakhs (500 lakhs x 100000 / 250000)

The amortization will form part of production cost


Extra Practical Problems from Past Year Papers and RTPs of Financial Reporting and Audit 397

AS27
Problem 68: Holding company Limited has 45% stake in its JV (JCE). H sold goods worth 32,000 to JV and
even JV sold goods worth `10,000 to H. Profit margin is 25%. Out of the goods sold within the group H sold
50% of the goods to third party and JV could sell 1/3 rd of the goods to third party. Calculate the impact of the
above transaction on the profit and loss account.
Solution:
H sold goods to JCE JCE sold goods to H
Profit / loss Dr 1,800 Profit / loss Dr 750
To Stock Reserve 1,800 To Stock Reserve 750
(32000 x 25%) 0.50 x 45% (10000 x 25%) 2/3 x 45%
Net impact on P/L in the CFS = 2,550 (1800+750)

AS28
Problem 69: A Ltd. is the sole manufacturer of product X. A particular machine is exclusively used for
production of product X. The company had near monopoly of the product. A competitor has recently come
out with a cheaper substitute of product X. The company is anticipating significant fall in demand for its
product and cash flow from the machine used in production of X is also expected to fall. As per the latest
budget estimates, taking the entry of the competitor in consideration, the operating pre-tax cash flows from
the machine expected over next 5 years are `9 lakh, `8 lakh, `6 lakh, `5.5 lakh and `5 lakh respectively. The
expected life of the machine is 10 years. Declining growth rates for future cash flows are estimated from year
6 onwards at 10%, 20%, 30%, 40%, 60% respectively. The disposal value (net of expected cost of disposal)
realisable at the end of year 10 is `1 lakh.
The machine can be disposed off immediately for `25 lakh subject to payment of brokerage 2% on disposal
value. The carrying amount of the machine on the current date is `35 lakh. Taking the risk involved in the use
of the machine for production of X in consideration, a pre-tax rate of return of 10% seems to be appropriate.
Determine impairment loss if any and give the journal entries in the books of A Ltd.
Solution: As per paras 57 and 58 of AS 28 ‘Imapirment of Assets’, if the recoverable amount of an asset is less
than its carrying amount, the carrying amount of the asset should be reduced to its recoverable amount. That
reduction is an impairment loss which should be recognised as an expense in the statement of profit and loss
immediately.
Impairment Loss
Net Selling Price = Disposal value – Brokerage
= `25 lakh – 2% of `25 lakh = `24.5 lakh
Value in use = `32.98 lakh (Refer Working Note)
Recoverable value = `32.98 lakh (Higher of value in use and net selling price)
Carrying amount = `35 lakh
Impairment loss = `35 lakh - `32.98 lakh = `2.02 lakh.
Impairment loss Dr 2.02
To Machine 2.02
P / L a/c. Dr 2.02
To Impairment loss 2.02
398 Accounting Standards

Year Operating CF PVF 10% PV of CF


1 900000 0.909 818100
2 800000 0.826 660800
3 600000 0.751 450600
4 550000 0.683 375650
5 500000 0.621 310500
6 450000 0.564 253800
7 360000 0.513 184680
8 252000 0.467 117684
9 151200 0.424 64109
10 60480 0.386 23345
10 (SV) 100000 0.386 38600
3297870

Problem 70: Light Ltd. is a multi-product manufacturing company, having its corporate office in a big
building. As the area of building is large, Light Ltd. has let out 1/3 area of the building at a monthly rent of
`100 lakhs, the lease agreement with tenant is for next 5 years. Is the building cash-generating unit? If not
what is the cash-generating unit of the building?
Solution: As per para 66 of AS 28 ‘Impairment of Assets, the cash-generating unit is smallest group of asset
that includes the asset and that generates cash inflows from continuing use that are largely independent of
the cash inflows from other assets.
In the instant case the primary purpose of the building is to serve as a corporate office of Light Ltd., which
is engaged in multi product manufacturing activities. Therefore, the building as a whole cannot be considered
to generate cash inflows which are largely independent of the cash flows for the company as a whole. The
building is not held as an investment; therefore, it would be inappropriate to determine ‘value in use’ of the
building based on future market rent.
Hence, building is not a cash generating unit; the cash-generating unit for the building is Light Ltd., the
company as a whole.
AS29
Problem 71: A company is in a dispute involving allegation of infringement of patents by a competitor
company who is seeking damages of a huge sum of `900 lakhs. The directors are of the opinion that the claim
can be successfully resisted by the company. How would you deal the same in the annual accounts of the
company?
Solution: As per para 14 of AS 29, ‘Provisions, Contingent Liabilities and Contingent Assets’, a provision
should be recognised when
(a) an enterprise has a present obligation as a result of a past event;
(b) it is probable that an outflow of resources embodying economic benefits will be required to settle the
obligation; and
(c) a reliable estimate can be made of the amount of the obligation. If these conditions are not met, no
provision should be recognised.
If these conditions are not met, no provision should be recognised.
In the given situation the directors of the company are of the opinion that the claim can be successfully
resisted by the company, therefore there will be no outflow of the resources. The company will disclose the
same as contingent liability by way of the following note:
“Litigation is in process against the company relating to a dispute with a competitor who alleges that
the company has infringed patents and is seeking damages of `900 lakhs. However, the directors are of the
opinion that the claim can be successfully resisted by the company.”
Extra Practical Problems from Past Year Papers and RTPs of Financial Reporting and Audit 399

Problem 72: Shyam Ltd. (a Public Sector Company) provides consultancy and engineering services to its
clients. In the year 2010-11, the Government has set up a commission to decide about the pay revision. The
pay will be revised with respect from 1-1-2006 based on the recommendations of the commission. The
company makes the provision of `680 lakhs for pay revision in the financial year 2010-11 on the estimated
basis as the report of the commission is yet to come. As per the contracts with the client on cost plus job, the
billing is done on the actual payment made to the employees and allocated to jobs based on hours booked by
these employees on each job.
The company discloses through notes to accounts
“Salaries and benefits include the provision of `680 lakhs in respect of pay revision. The amount chargeable
from reimbursable jobs will be billed as per the contract when the actual payment is made”.
The accountant feels that the company should also book/recognise the income by `680 lakhs in Profit
and Loss Account as per the terms of the contract. Otherwise, it will be the violation of matching concept &
understatement of profit.
Solution: As per para 46 of AS-29, ‘Provisions, Contingent Liabilities and Contingent Assets’, where some
or all of the expenditure required to settle a provision is expected to be reimbursed by another party, the
reimbursement should be recognised when, and only when, it is virtually certain that reimbursement will
be received if the enterprise settles the obligation. The reimbursement should be treated as a separate asset.
The amount recognised for the reimbursement should not exceed the amount of the provision.
Accordingly, potential loss to an enterprise may be reduced or avoided because a contingent liability is
matched by a related counter-claim or claim against a third party. In such cases, the amount of the provision
is determined after taking into account the probable recovery under the claim if no significant uncertainty as
to its measurability or collectability exists.
In this case, the provision of salary to employees of `680 lakhs will be ultimately collected from the client,
as per the terms of the contract. Therefore, the liability of `680 lakhs is matched by the counter claim from
the client. Hence, the provision for salary of employees should be made reducing the claim to be made from
the client. It appears that the whole amount of `680 lakhs is recoverable from client and there is no significant
uncertainty about the collection. Hence, the net charge to profit and loss account should be nil.
The opinion of the accountant regarding non-recognition of income of `680 lakhs is not as per AS-29 and AS-9.
However, the concept of prudence will not be followed if `680 lakhs is simultaneously recognized as income. `680
lakhs is not the revenue at present but only reimbursement of claim. However the accountant is correct to the
extent as that non- recognition of `680 lakhs as income will result in the under statement of profit.
P roblem 73: What are your views on the following? Tarkash Ltd was under audit for the year ended
31.3.2012. an appeal filed by A Ltd. against the demand of Excise Duty of `26 crores was pending before
the Supreme Court for which neither provision was made nor was disclosed in the notes to the financial
statements. On 12th July, 2012, the auditor came to know through paper reports that the point involved in
the appeal of A Ltd was adjudicated by the Supreme Court in the case of some other assessee, which is in
favour of the department of Excise Duty. The auditor insisted that provisions is required to be made of `30
crores in the financial statements. The management was of the view that since its own case is still pending, no
provision is called for. It was also of the view that the event does not have any effect on the financial position
of the company on the date of the Balance Sheet. Is the view of the management tenable?
Solution: The management must make the provision for the excise duty of `26 crores as it very much cleared
by the recent SC decision for other assesses. Now the possibility of outflow of resources is probable. The
company should review (visit) the case of provision otherwise the audit report will be qualified for non
compliance of AS29.

AS30
Problem 74: Alif Finance holds 1,000 shares of Ponds Ltd. currently valued at `120 per share. Cost of the
investment per share was `105. The investment is classified as available for sale. In order to protect it self
400 Accounting Standards

from possible fall in the market price of the share, the company acquired a 3 month put at strike price `115
per share `3 share. The company sold the share at market price on expiry of the put.
Show relevant accounting entries in books of Alif Finance if market price per share on maturity is `116.
Solution:
Put ...Dr 3,000
To Bank 3,000
Equity 1000 [120-116] ...Dr 4,000
To Investment 4,000
P & L [Out of Money] ...Dr 3,000
To Put 3,000
Bank [Sales] ...Dr 1,16,000
To Investment 1,16,000
Equity ...Dr 11,000
To P & L [(120-105)1000 - 4000] 11,000
Problem 75: On April 1, 2012 a metal dealer has a firm commitment to buy 1,000 tons of certain metal on
May 31, 2012. The current price of the metal is `60,000 per ton. On April 1, 2012, to cover his short position
in metal market, the dealer took a long position on 1,000 tons of metal futures at strike price `60,000 per ton.
The initial margin was `10 lakh. The settlement prices on different dates were as below:
Date Settlement price/ton (`)
10/04/12 59,500
28/04/12 59,300
08/05/12 59,400
18/05/12 59,800
31/05/12 60,100
Show accounting entries in books of the dealer in respect of above:
Solution: Its a cash flow hedge.
(` in lakhs)
1/4: Initial margins 10
To Bank 10
10/4: Hedging reserve 1,000[- 60,000 + 59,500] 5
To Bank 5
28/4: Hedging reserve 1,000[- 59,500 + 59,300] 2
To Bank 2
8/5: Bank 1
To Hedging reserve 1
18/5: Bank 4
To Hedging reserve 4
31/5: Bank 3
To Hedging reserve 3
31/5: Purchase 601
To Bank 601
31/5: Bank 10
To Initial Margin 10
31/5: Hedging reserve 1
Extra Practical Problems from Past Year Papers and RTPs of Financial Reporting and Audit 401

To P & L 1
Problem 76: Identify the host and embedded derivative in the following cases:
(i) Entity A holds a debenture bond of entity B which is convertible into ordinary shares of entity B at
the option of entity A.
(ii) Sun Ltd. enters into a lease agreement for usage of plant and equipment, rentals in respect of which
lease rentals are payable annually. The lease includes a clause that lease rentals payable will be linked
to changes in Cost Inflation Index announced u/s 48 of Income tax Act, and contingent rentals will be
payable on the basis of Benchmark interest of PLR of SBI.
(iii) Moon Ltd. enters into a lease agreement for usage of plant and equipment, rentals in respect of
which are payable annually. The lease includes a clause that lease rentals payable will be increasing
annually by `1,20,000
(iv) A floating rate debt - (a) with no cap (b) with a cap on interest and (c) a separate agreement for a cap
on interest.
(v) Interest rate on a fixed deposit linked to fluctuation of BSE sensex.
Solution:
(i) The debenture bond is the host contract. The option to convert into shares is the embedded derivative.
(ii) The lease contract is the host contract. The adjustment linked to Cost Inflation Index is the embedded
derivative contract.
(iii) The price adjustment is not a derivative - it does not meet the definition of derivative - there is no
underlying.
(iv)
a. A floating rate debt with no cap - there is no derivative.
b. A floating rate debt with a cap on interest - the cap is the derivative and the debt instrument is
the host contract.
c. Separate agreement for a cap on interest - the cap is a derivative because it is based on the
interest rate of the underlying instrument.
(v) Fixed deposit is the host contract and interest payment linked to BSE sensex is the embedded
derivative.
Problem 77: Basic entries for financial instruments:
Pass the entries in the books of labisha:
Labisha makes a forward sale 200000 of own shares to Mohsina at `400. Delivery of the shares is to be
made after 3 months. On the B/S date the price of the share is `410. On the settlement date the price of the
shares quoted at `433. Assume settlement will be done in own equity shares on net basis. Also calculate how
many shares will be issued to Mohsina on settlement?
Solution:
Initial recognition Nil Nil
Valuation on B/S date Profit/loss Dr 20,00,000 20,00,000
To Financial liability
(410-400) 200000
Valuation on settlement date Profit/loss Dr 46,00,000
To Financial liability 46,00,000
Net settlement (433-410) 200000
Financial liability Dr 66,00,000
To Equity 66,00,000
(20+46)

Labisha will issue its own 15242 variable number of equity shares. 15242 = (6600000/433)
SOLVED QUESTION PAPER—CA FINAL-MAY 2013
(ACCOUNTING STANDARDS)

Part I : Financial Reporting


Question 1: J Ltd. purchased a Machinery from K Ltd on 31.08.2012. Quoted price was `275 Lakhs. The
vendor offers 2% Trade discount. Sales tax on quoted price is 6%. J Ltd spent `60,000 for transportation and
`45,000 for architect’s fees. They borrowed money from HDFC Bank of ` 250 Lakhs for acquisition of asset
@ 15% p.a. They also spent ` 15,000 for Material, `10,000 for Labour and `4,000 as overheads during trial
run of the machine. The machine was ready for use on 15.01.2013 but it was put to use on 15.03.2013.
(5 Marks)
(i)
1. Find out the original cost of the machine.
2. Also suggest the accounting treatment for costs incurred between the date the machine was ready
for use and the date at which it was actually put to use.
Solution:
Particulars ` In lacs
Quoted Price 275
T. Discount (5.5)
269.5
+ S Tax 16.50
Net Price 286.00
Transportation costs 0.60
Architect fees 0.45
Total cost of purchase 287.05
+ Trial run exp (M+L+O) 0.29
Total Cost 287.34

Borrowing costs on loan outstanding for the period of installation can be assumed to be charged to P/L
and not to be capitalized because 2 months cannot be taken as substantial period as per AS-16. Alternatively it
can be taken as substantial period of time which ICAI has taken a view in its solution. If interest is capitalised
then cost of asset will be 287.34 + 14.06 = `301.04 lakhs
(ii) For costs incurred between the date the machine was ready for use and the date at which it was actually
put to use all the expenses will be charged to profit/loss and loss account. If the machine was ready for use
no more capitalisation is permitted.
Question 2: A Ltd. acquires 80% share in B Ltd for ` 15 lakhs. The net assets of B Ltd. on the day are ` 22
lakhs. During the year A LTd. sold the investment for ` 30 lakhs and net assets of B Ltd. on the date of disposal
was ` 35 lakhs. Calculate the profit or loss on disposal of this investment to be recognized in Consolidated
Financial Statements. (5 Marks)
Solved Question Paper—CA Final-May 2013 (Accounting Standards) 403

Solution:
Calculation of profit or loss or disposal (` in Lakhs)
Cost of investments initially 15,00,000
+ Share of post increase in net worth [35-22] × 80% 10,40,000
Carrying amount as on the date of sale (a) 25,40,000
Proceeds from disposal of shares of subsidiary B Ltd. (b) 30,00,000
Loss on disposal in consolidated Profit and Loss A/c (a-b) 4,60,000

Question 3: On 1st January, 2011 Santa Ltd sold equipment for ` 6,14,460. The carrying amount of the
equipment on the date was ` 1,00,000. The sale was part of the package under which Banta Ltd leased the
asset to Santa Ltd for 10 year term. The economic life of the asset is estimated at 10 years. The Minimum
Lease Rents payable by the Lesser has been fixed at ` 1,00,000 payable annually beginning 31st December,
2011. The incremental borrowing Interest Rate of Santa Ltd is estimated at 10% p.a. Calculate the net effect
on the Profit and Loss Account. (5 Marks)
Solution: Profit on sale lease is to be deferred over lease period in the ratio of depreciation (SLM). SP – BV
= 614460 – 100000 = `5,14,460.
Present Value of MLP = 6.1446 × 100000 = `6,14,460, Also leased asset has the FV at 6,14,460. Lessee will
record the asset @ 6,14,460.
As per AS-19 para ....... the profit/loss on sale under the Sale and Lease Back method should be deferred in
the ratio of depreciation.

Net amount charged to P/L is as follows: (Books of Lessee)


Depreciation Dr Interest Charge Dr Deferment Cr Net Charge Dr
61,446 61,446 51,446 71,446
61,446 57,591 51,446 67,591
61,446 53,350 51,446 63,350
61,446 48,685 51,446 58,685
61,446 43,553 51,446 53,553
61,446 37,908 51,446 47,908
61,446 31,699 51,446 41,699
61,446 24,869 51,446 34,869
61,446 17,356 51,446 27,356
61,446 9,092 51,446 19,092

For the calculation of interest please refer interest table from AS-19.
Q uestion 4: X Ltd purchased a Fixed Asset four years ago for ` 150 Lakhs and depreciates it at 10% p.a. on
Straight Line Method. At the end of the 4th year it has revalued the Asset at ` 75 Lakhs and has written off
the loss on revaluation to P&L A/c. However on the date of revaluation, the Market Price is ` 67.50 Lakhs and
expected disposal costs are ` 3 Lakhs. What will be the treatment of Impairment Loss on the basis that fair
value for revaluation purpose is determined by market value and market value in use is estimated at ` 60
Lakhs? (5 Marks)
Solution:
1. Recognition of Loss on Revaluation: (` in lakhs)
1. Original Cost of the Asset Given 150.00
2. Accumulated Depreciation (150.00 × 10% × 4) 60.00
3. Carrying Amount before Revaluation 90.00 (Step1-2)
4. Revalued Amount Given 75.00
5. Loss on downward revaluation (charged to P & L) 15.00
404 Accounting Standards

6. Carrying Amount after Revaluation (AS-10) 75.00


2. Recognition of Impairment Loss: ` Lakhs
1. Net Selling Price = Market Value – Disposal Costs = 67.50 – 3.00 = ` 64.50
2. Value in Use ` 60.00
3. Recoverable Amount = Higher of 1 or 2 as per AS-28 = ` 64.50
4. Carrying Amount after revaluation ` 75.00 as above
5. Impairment Loss = Carrying Amount - Recoverable Amount = 75 – 74.5 = ` 10.50
Question 5: On 01.04.2011, the fair value of Plan Assets were ` 2,50,000 in respect of a pension plan of
Q Ltd. On 30.09.2011, the plan paid out benefits of ` 47,500 and received further contribution of ` 1,22,500.
On 31.03.2012, the fair value of Plan Asset was ` 3,75,000 and the present value of the benefit obligation was
` 3,69,800. Actuarial losses on the obligation for 2011–2012 were ` 1,500. On 01.04.2011, the Company had
made the following estimates:
Interest and Dividend Income after tax payable by the fund 9.25%
Realized and unrealized gain on Plan Asset (after tax) 2.0%
Fund Expenses (1.0%)
Expected Rate of Return 10.25%
Find out the expected and unexpected return on plan assets. (6 Marks)
Solution: Actuarial gain/loss can be calculated by comparing FV of plan asset with Estimated value of asset
and by comparing Expected return with Actual return from the plan assets.
Lets compute expected return on plan assets:
Return on opening balance = ` 2,50,000 × 10.25% = ` 25,625
+ Return on the net cash contributions = (1,22,500 – 47,500) = 75,000 × 10.25 % × ½ = ` 3844.
Total expected return = 25,625+3844 = ` 29,469
Lets compute actual return on plan assets:
Opening FV of plan assets = ` 2,50,000
+ Net cash contributions = 75,000
Expected FV 3,25,000
Less: Actual FV of plan assets 3,75,000
Actual return earned by plan assets 50,000
Comparing with expected return 29,469
Actuarial gain = 20,531
Actuarial loss due to obligation difference = (1,500)
Net actuarial gain = 19,031
Question 6: From the following information relating to W Ltd, calculate diluted Earnings Per Share as per
AS – 20.
1. Net Profit for the Current Year – ` 5,00,00,000
2. Number of Equity Shares outstanding – 1,00,00,000
3. 11% convertible Debentures of ` 100 each (Nos.) – 1,25,000
4. Interest Expenses for current year – ` 13,75,000
5. Tax saving relating to interest expense – 30%
6. Each Debenture is convertible into eight Equity Shares. (4 Marks)
Solution:
Basic EPS = 500 lakhs / 100 lakhs = ` 5/- (Assuming NP is after tax)
Diluted EPS = 500 lakhs + Interest savings after tax / (WANES + Addln shares on conversion)
= (500 + 9.63) / (100+10) = ` 4.63 (Diluted EPS disclosure is mandatory)
Solved Question Paper—CA Final-May 2013 (Accounting Standards) 405

Question 7: W Ltd purchased Machinery for ` 80 Lakhs from X Ltd during the year 2010–2011 and
installed the same immediately. Price includes Excise Duty of ` 8 Lakhs. During the year 2010–2011, the
Company produced Exciseable Goods on which Excise Duty of ` 7.20 Lakhs was charged. Give necessary
entries explaining the treatment of CENVAT Credit. (4 Marks)
Solution: (` in lakhs)
Year of acquisition
Machine Account Dr. 72
CENVAT Credit Receivable Account Dr. 4
CENVAT Credit Deferred Account (note #) Dr. 4
 To Supplier’s Account 80
(Beign machinery purchased by paying excise duty of `8, and thereby recorded as an input refundable
duty)
Excise duty Dr. 7.20
 To CENVAT Credit Receivable Account 4
 To Bank 3.2
(Beign excise duty liability due, adjusted agianst Cenvat credit and balance paid in cash)
Next Year
CENVAT credit receivable Account Dr. 4
 To CENVAT credit deferred Account 4
# CENVAT Credit Deferred Account will be disclosed under the line item “Short term loans and advances” as
per the New Schedule VI.
Q uestion 8: Write short notes on “Disclosure of carrying amounts of Financial Assets and Financial
Liabilities in Balance Sheet”. (4 Marks)
Solution: Financial Assets are: Fair Value through P/L (FVTPL), Available for sale, Loans and receivables,
Held till Maturity
Financial Liabilities are: FVTPL and Others.
Question 9: Vishnu Company has at its financial year ended 31st March, 2013 fifteen law suits outstanding,
none of which has been settled by the time the accounts are approved by the directors.
The directors have estimated that the possible outcomes as below:
Result Probability Amount of loss
For first ten cases:
Win 0.6 —
Lose-low damages 0.3 90,000
Lose-high damages 0.1 1,60,000
For remaining five cases:
Win 0.5 —
Lose-low damages 0.3 60,000
Lose-high damages 0.2 95,000
The directors believe that the outcome of each case is independent of the outcome of all the others.
Estimate the amount of contingent loss and state the accounting treatment of such contingent loss.
(4 Marks)
Solution: As per AS-29 Contingent liability should be disclosed in financial statements if following conditions
are satisfied:—
◘◘ There should be present obligation arising out of past event but not recognised as provision.
◘◘ It is not probable that an outflow of resources embodying economic benefits will be required to settle
the obligation.
◘◘ The possibility of an outflow of resources embodying economic benefits remote.
◘◘ The amount of the obligation cannot be measured with sufficient re” to be recognised as provision.
406 Accounting Standards

In this case, the probability of winning of first 10 cases is 60%and for remaining, five cases 50%. In other
words, the probability of losing is 40%or 50% respectively. As per the AS-29, we make a provision if the
loss is probable. As the loss does not appear to be probable and the possibility of an outflow of resources
embodying economic benefits is not remote rather there is reasonable possibility of loss, therefore disclosure
by way of note should be made. For the purpose of the disclosure of contingent liability by way of note amount
may be calculated as under:
Expected loss in first ten cases = ` 90,000 × 0.3 + ` 1,60,000 × 0.1
= ` 43,000 × 10= ` 4,30,000
Expected loss in remaining five cases = ` 60,000 × 0.3+ ` 95,000 × 02
= ` 37,000 × 5= ` 1,85,000
Total Contingent Liability = ` 4,30,000 + ` 1,85,000 = ` 6,15,000

Part II: Audit May 2013 Paper Solution


Question 1: K Ltd., has 5 Subsidiaries as at 31st March 2013, and the Investments in Subsidiaries are
considered as Long–Term and valued at cost. Two of the Subsidiaries’ Net Worth eroded as at 31st March
2013, and the prospects of their recovery are very bleak and other three Subsidiaries are doing exceptionally
well. The Company did not provide for the decline in the value of investments in Two Subsidiaries,because
the overall investment portfolio in Subsidiaries did not suffer any decline, as the other three Subsidiaries are
doing exceptionally well. Comment. (5 Marks)
Solution: In the stand alone accounts investments made by holding company in its subsidiary is Trade
investments and further classified as Long Term Investments. AS 13 states, “long-term investments should
be carried in the financial statements at cost. However, provision for diminution shall be made to recognise
a decline, other than temporary, in the value of the investments, such reduction being determined and made
for each investment individually”. Therefore provision for loss should be made for individual subsidiaries.
Profit making subsidiary should be carried at Cost. As per revised schedule VI holding company is required
to disclosed provision for losses of its subsidiary company. If the company do not follow AS-13 then the Audit
Reports will be qualified.
Q uestion 2: T Ltd. commenced its manufacturing activities from 1st April 2012. In the course of production,
the Company generated certain by–products. As at 31st March 2013, the Company did not value the by–products
considering the value as insignificant. The Auditor of the Company is of the opinion that the by–products are
inventory of the Company and it should be valued and brought into books of account. Comment.
(5 Marks)
Solution: As per AS-2 the net realisable value of by – product is required to be reduced from the cost of main
product. If it is a joint product then NRV of by – product will be reduced from cost of joint product and then
the balance cost of joint product will be apportioned on a consistent and rational manner.
Q uestion 3: Z Ltd changed its employee remuneration policy from 1st April 2012, to provide for 12%
Contribution to Provident Fund on Leave Encashment also. As per the Leave Encashment Policy, the
Employees can either utilize or encash it. As at 31st March 2013 the Company obtained an actuarial valuation
for the Leave Encashment Liability. However, it did not provide for 12% PF Contribution on it. The Auditor of
the Company wants it to be provided, but the Management replied that as and when the employees availed
leave encashment, the Provident Fund Contribution was made. The Company further contends that this is the
correct treatment, as it is not sure whether the employees will avail leave encashment or utilize it.
(5 Marks)
Solution: The company must provide the 12% PF on leave encashment. As per AS-15 (Rev) any retirement
benefits payable in future is required to be provided on accrual baiss as and when the employees render the
services. Pay – as – you – go theory is abolished i.e. not to record the empoyees cost when they are paid. As
the company is not following AS-15 (Rev) the audit reports are required to be qualified.
SOLVED QUESTION PAPER—CA FINAL-NOVEMBER 2013
(ACCOUNTING STANDARDS)

Part I: Financial Reporting


Question 1: P Ltd. has three business segments which are FMCG, Batteries and Sports Equipment. The
Battery segment has been consistently underperforming and P Ltd. after several discussion with labour unions
have finally decided on closure of this segment. Under the agreement with the labour union the employees
of the Battery Segment will earn no further benefit as the arrangement is a curtailment without settlement
wherein the employees of the discontinued segment will continue to receive benefits for services rendered
when segment was functioning. As a result of the curtailment, the company’s obligation that was arrived on
the basis of actuarial valuations before the curtailment have come down. The following is also furnished:
(i) The value of gross obligations before the curtailment calculated on actuarial basis was ` 4,000 lakhs.
(ii) The value of Unamortized past service costs is ` 100 lakhs.
(iii) The Curtailment will bring down gross obligations by ` 500 lakhs and P Ltd. anticipate a proportional
decline in the value of unamortized past service costs also.
(iv) The fair value of plan assets on date is estimated at `3,250 lakhs.
You are required to calculate the gain from curtailment and also show the liability to be recognized in the
Balance Sheet of P Ltd. after the curtailment. (Marks 5)
Solution: This is a case of Curtailment without Settlement
Particulars Before Curtailment Curtailment gain After Curtailment
PV of obligation 4000 (500) 3500
FV of plan assets (3250) - (3250)
750 (500) 250
Unrecognised Past Service Cost (100) (12.5) (87.5)
Net liability in B/S 650 487.5 162.5

Note: Proportional Decline in Unamortised Past Service Costs = 500 ÷ 4,000 = 12.5%
All the above Rupees in lakhs.
Journal Entry for Curtailment:
PV of obligation a/c Dr 500 lakhs
To Past Service Cost a/c 12.5 lakhs
To Profit and loss a/c 487.5 lakhs
Question 2: To comply with listing requirements and other statutory obligations Quaker Ltd. prepares
interim financial reports at the end of each quarter. The company has brought forward losses of ` 700 lakhs
under Income Tax Law, of which 90% is eligible for set off as per the recent verdict of the Court, that has
attained finality. No deferred Tax Asset has been recognized on such losses in view of the uncertainty over
its eligibility for set off. The company has reported quarterly earnings of `700 lakhs ` 300 lakhs respectively
for the first two quarters of financial year 2013-14 and anticipates a net earning of `800 lakhs in the coming
half year ended March 2014 of which ` 100 lakh will be the loss in the quarter ended Dec, 2013. The tax rate
for the company is 30% with a 10% surcharge. You are required to calculate the amount of Tax Expense to be
reported for each quarter of financial year 2013-14. (Marks 5)
408 Accounting Standards

Solution: As per para 29 (c) – Treatment of Taxes: Income Tax Expense is recognized in each interim period
based on the best estimate of the weighted average annual income tax rate expected for the full financial year.
(a) Estimated Total Income for the year 2013-14: 700 + 300 + 800 = ` 1800 lakhs
(b) Estimated Total Tax Liability: 1800 – 700 x 90% = ` 1170 lakhs
(c) Average Annual Tax = 1170 x 33% = ` 386.1
(d) Average Annual Tax Rate: = ` 386.1 / 1800 = 21.45%
(e) Quarterly Tax Expense: —Q1, Q2, Q3, Q4 = 700× 21.45% = = ` 150.15 Lakhs, 300× 21.45% = ` 64.35
Lakhs (100)× 21.45% = ` (21.45), Lakhs 900× 21.45%= ` 193.05 Lakhs
Question 3: B Ltd., entered into an agreement on 1st March, 2013 to buy computer spares from S Ltd.
at prevailing market price for ` 1200 lakh on which S Ltd made a profit of 20% and received full advance
payment. The transaction was concluded on 15th March,2013. On the same day S Ltd. agrees to buy on
15th Sept, 2013 the same goods from B Ltd. at 20% over cost. The 20% mark up compensates B Ltd for its
inventory holding cost till sale date. You are required to show how both the buyer & seller account for the
above transaction in the year 2012-13 explaining in brief the justification for your treatment and also draft
the Notes on account on disclosure if any required in the annual accounts of year ended 31st March, 2013.
(Marks 5)
Solution: In case of Repo transactions AS-9 treats both the sale and buy back transaction as one and the
same part of the whole package. The first transaction negates the impact of the second one.

IN THE BOOKS OF S LIMITED (` in lakhs)


15.3 Bank Dr. 1200 15.9 Finance Charges Dr. 220
To Advance from B Ltd. 1200 To Advance from B Ltd. 220
31.3 Finance Charges Dr. 20 15.9 P / L Dr. 220
To Advance from B Ltd. 20 To Finance charges 220
31.3 P / L Dr. 20 15.9 Advance from B Ltd. 1440
To Finance charges 20 To Bank 1440

Financial Statement Disclosures:


P/L Statement: Finance Charges is charged to P/L under the head financial expenses.
B/S : Liability side: Advance from B Ltd is disclosed under Short term Loans.
Disclosure: Inventories includes goods costing ` 1000 lakhs sold on repo basis.
Similarly on the other hand B Ltd has to record the Finance income with similar calculations.
Question 4: Vintage Ltd has been in the business of sale of Vintage Wines for the last 12 years and is an
extremely cash rich company. In FY 2011-12 the Board of the company decided to venture into new areas of
business and identified the activity of acquiring Vintage Priorities such as old Bunglows, Heritage buildings
and the like at prime location and after carrying out renovation and refurbishment of the same to let out
these properties on lease to willing parties. The new business was commenced as a separate division of
the company in FY 2012-13 during which the company managed to identify 19 and properties of which
17 were acquired and 9 given on lease. Being the initial year of operations and also since some of the lease
arrangements were entered into at the fag end of the year the income from leasing was only a partly amount.
After the acquisition of the properties as aforesaid very attractive offers for sale of 14 of the properties were
received. Vintage Ltd. after negotiation accepted 12 of the offers and sold these 12 properties making large
profits in the bargain. The accountant of Vintage Ltd. has accounted the acquisition and disposal of properties
as ‘Purchases’ and ‘sales’ in the Profit & Loss account of the Property Division and treated the lease incomes
as part of the other income of the company. The contention of the accountant of vintage Ltd., was that since a
majority of the properties were disposed off within a short span of time, the properties are to be considered
as stock in trade only. Further since the lease income and hence considered as part of other income. You are
Solved Question Paper—CA Final-November 2013 (Accounting Standards) 409

required to examine the correctness of the accountant of Vintage Ltd., considering the relevant Accounting
Standards and provision of Revised Schedule IV of Companies Act, 1956. (Marks 5)
Solution: First of all Vintage Ltd should get ready for disclosure for AS-17 also i.e. (i) Wines; (ii) Leasing
activities. As clearly mentioned that V Ltd has ventured into leasing activities so leasing is the ‘Revenue from
operations’. But fortunately leasing of properties are covered by Indian GAAP. However property leases are
generally considered as operating lease. Such income should be recorded on SLM or any systematic basis.
As far as sale/purchase of Land is considered it is not a stock in trade. It can be other income but not main
operating income.
Question 5: Sea Ltd. has lent a sum of ` 10 lakhs @ 18% per annum for 10 years. The loan had a Fair
Value of ` 12,23,960 at the effective interest rate of 13%. To mitigate prepayment risks but at the same time
retaining control over the loan Sea Ltd. transferred its right to receive the Principal amount of the loan on
its maturity with interest, after retaining right over 10% of principal and 4% interest that carries Fair Value
of ` 29,000 and ` 1,84,620 respectively. The consideration for the transaction was ` 9,90,000. The interest
component retained included a 2% fee towards collection of principal and interest that has a Fair Value of
` 65,160. Defaults if any are deductible to a maximum extent of the company’s claim on principal portion. You
are required to show the journal Entries to record derecognition of the loan. (Marks 6)
Solution:
First lets calculate the FV of Loan:

Int trf Principal trf Int ret Service fees Prin ret
ret
Cash Flows 140000 900000 20000 20000 100000
PVF 13% 5.426306 0.2946 5.426306 5.426306 0.2946
FV 759682.8 265140 108526.1 108526.1 29460 1271335

ret means retained

Present Value of Cash Flows of ` 10 Lakhs at 18% p.a. for 10 years, discounted at 13% Effective Interest
Rate comes to ` 12,71,334. However, the Question states that Fair Value is ` 12,23,960. There seems to be an
error in the problem.
Lets proceed with the data given in the question paper itself.
The company has retained the risk of default with itself, hence it is a case of Continuing Involvement.
Normally Fair Value of each component of Loan is calculated by the above calculations of FV apportionment.
But the problem has already given the FV apportionment which is full of error, we should consider the same.
Total Fair Value of the Loan 12,23,960
Less: Fair Value of Servicing asset (2%) (65,160)
Less: Fair Value of interest strip receivable (2%) (184620-65610) (119460)
Less: Fair Value of Principal portion retained (10%) (29000)
Balancing figure is: Securitized component of loan 10,10,340

Apportionment of Carrying Amount in the ratio of Fair Value:

Fair Value % Carrying amount


Principal and interest transferred 1010340 82.55% 8,25,500
Principal portion retained 29,000 2.369% 23,960
Component of Servicing asset 65,160 5.323% 53,230
Component of interest strip receivable 119460 9.758% 97,310
Total Loan 12,23,960 10,00,000
410 Accounting Standards

Journals:
Bank A/c (Consideration Received) Dr. 9,90,000
To Loan (Asset) A/c (Loan securitized) 8,25,500
To Profit and Loss A/c (Gain on De–Recognition) 1,64,500
Servicing Asset A/c (Carrying Amt of Servicing Asset retained) Dr. 53,230
Interest Asset (Carrying Amt of Interest Retained) Dr. 97,310
To Loan (Asset) A/c 1,50,540
10% New Loan A/c Dr 23,960
To Loan (Asset) A/c 23,960
Financial Asset A/c 1,00,000
To Financial Liability A/c 1,00,000
Note: Maximum Default that may be borne by the Company = its Maximum Principal Claim for 10% of Loan
= `1,00,000. So, an Asset and Liability is recognised to that extent.
Post Debt Securitization the Balance Sheet disclosure is as follows:
Equity and Liabilities: Assets:
Equity: Loan (10%) 23,960
Reserves and Surplus:
Servicing Asset 53,230
Liability: Interest Strip 97,310
Financial Liability 1,00,000 Financial Asset: 1,00,000
(for continued involvement) (for continued involvement)
Question 6: K Ltd. Issued 5,00,000, 6% Convertible Debentures of ` 10 each on the first of April 2010.
The debentures are due for redemption on 31st March, 2014 at a premium of 10% convertible into equity
shares to the extent of 50% and the balance to be settled in cash to the debenture holders. The interest
rate on equivalent debentures without conversion rights was 10%. You are required to separate the debt &
equity components at the time of the issue and show the accounting entry in the company’s book at initial
recognition. (Marks 4)
The following Present Values of ` 1 at 6% and at 10% are supplied to you.

Interest Rate Year 1 Year 2 Year 3 Year 4


6% 0.94 0.89 0.84 0.79
10% 0.91 0.83 0.75 0.68

Solution:
PV of Redemption amount 18,70,000
(5500000 x 50% x 0.68)
PV of interest payments 9,51,000
(300000*3.17)

PV of liability 28,21,000
Less: Cash received 50,00,000
Equity Component 21,79,000
Cash Dr 50,00,000
 To Debt 28,21,000
 To Equity 21,79,000
Solved Question Paper—CA Final-November 2013 (Accounting Standards) 411

Question 7: WIN Ltd. has entered in to three year lease arrangement with Tanya sports club in respect of
Fitness Equipment costing ` 16,99,999.50. The annual lease payments to be made at the end of each year are
structured in such a way that the sum of the Present Value of the lease payments and that of the residual value
together equal the cost of the equipments leased out. The unguaranteed residual value of the equipment at
the expiry of the lease is estimated to be ` 1,33,500. The assets would revert to the lessor at the end of the
lease. Implicit rate of interest is 10%. You are required to compute the amount of the annual lease and the
unearned finance income discounting factor at 10% for years 1,2 and 3 are 0.909, 0.826, 0.751 respectively.
(Marks 4)
Solution:
(1) Computation of annual rent =
PV of Residual value = 1,33,500 x 0.751 = 1,00,259
PV of lease payments = 16,99,999 – 1,00,259 = 15,99,740
Annual payments = 15,99,740 / 2.4869 = ` 6,43,267
(2) Calculation of UFI:
Gross investments = (3 x 6,43,267) +1,33,500 = ` 20,63,300
Less: Cost of property = ` (17,00,000)
Unearned Finance Income = ` 3,63,300
Question 8: Qu Ltd is in the business of manufacture of Passenger Cars and Commercial Vehicles. The
Company is working on a strategic plan to shift from the Passenger Car segment over the coming 5 years.
However, no specific plans have been drawn up for sale of neither the Division nor its Assets. As part of its
plan, it will reduce the production of Passenger Cars by 20% annually. It also plans to commence another New
Factory for the manufacture of Commercial Vehicles and transfer surplus employees in a phased manner:
1. You are required of comment if mere gradual phasing out in itself can be considered as a “Discontinuing
Operation’ within the meaning of AS–24.
2. If the Company passes a Resolution to sell some of the assets in the Passenger Car Division and
also to transfer few other Assets of the Passenger Car Division to the factory, does this trigger the
application of AS–24?
3. Would your answer to the above be different, if the Company resolves to sell the assets of the
Passenger Car Division in a phased but time bound manner? (Marks 4)
Solution:
1. Mere gradual phasing out in itself cannot be considered as a Discontinuing Operation (DO). Gradual
or evolutionary phasing out of a product line or class of service is not DO…….para 9
2. Passing of Resolution to sell some of the assets in the Passenger Car Division and also to transfer few
other Assets of the Passenger Car Division to the factory, does not trigger the application of AS–24. It
is sale in its entirety remember and not some of the assets. Also starting the operations in the New
Factory is altogether a new business and has nothing to do with old business.
3. Resolution to sell assets of the Passenger Car Division in a phased but time bound manner will
lead to identification of a Discontinuing Operation as per AS–24. It means there is a single plan to
discontinue the operation. The manner of discontinuance seems to be piecemeal distribution.
Question 9: Grant Medicare Ltd acquired 5 units of Brain Scan Equipments for US$ 5,00,000 in April 2010,
incurring ` 20,00,000 on Sea Freight and US$ 12,000 per unit towards Transit Insurance, Bank Charges,
etc. The purchase was partly funded out of the Company’s internal accruals and from Government Grant
of ` 94 Lakhs. The prevailing Exchange Rate to the US$ was ` 50. The Company estimated the useful life of
the Equipment at 4 years with an estimated Salvage Value of 13% (approx). The Grant was considered as
Deferred Income up to 2012–2013 and in April 2013, the Company had to return the entire Grant received
due to non–fulfillment of certain conditions. You are required to show the Depreciation and the Grant that is
412 Accounting Standards

to be recognized in the Profit & Loss Accounts for the period commencing 2010–2011 onwards and also draw
up the entry that is passed in April 2013 for the return of the Grant. The Company follows the Written Down
Value Method for depreciating its Assets. (Marks 4)
Solution: (` in lakhs)
The grant is received towards Depreciable Fixed Asset. Deferred approach is adopted by the Company.
Cost of asset (1/4/2010) $500000 × 50 250
Add: Freight and other charges 20 lakhs + $12000 × 50 × 5 50
Cost as per AS-10 300
Less: Depreciation for 2010-2011 40% (120)
Carrying amount as on 1/4/2011 180
Less: Depreciation for 2011-2012 (72)
Carrying amount as on 1/4/2012 108
Less: Depreciation for 2012-2013 (43.2)
Carrying amount as on 1/4/2013 64.8
Less: Depreciation for 2013-2014 (25.92)
38.88

Depreciation as per WDV % = 1 – double sq root of 13/300


Deferment of Grant:
Ratio for deferment will be ratio of depreciation: 120: 72: 43.2: 25.92 = Total of ratio = 261.12

Year Amortz of grants (` in lakhs) %


2010-11 Grants received 94
Less: Deferred to P/L A/C (43.20) 45.96%
2011-12 Balance 50.8
Less: Deferred to P/L A/C (25.92) 27.57%
2012-13 Balance 24.88
Less: Deferred to P/L A/C (15.55) 16.54%
Balance Undeferred Grant 9.33

Journal for refund of grants: (` in lakhs)


April 2013 Deferred Government Grant A/c 9.33
Profit and Loss A/c 84.67
To Bank A/c 94.00
(Being Grant refunded to Govt, and shortage charged to P&L A/c as Expense)
Question 10: Blow Glass Limited manufactures Glass Bottles of various sizes and shapes at its 3
manufacturing facilities in UP, Haryana and MP. The Company follows the WDV Method of Depreciation for all
assets at these units and at its Corporate Office. In 2013 May, it acquired a new unit making Plastic Containers
in Gujarat. The method of depreciation followed in the newly acquired unit was the SLM Method for all its
assets, till the unit was acquired by Blow Glass Ltd. The Chief Accountant of Blow Glass Ltd is of the view that
since the Company has adopted the WDV Method at all its existing assets it is mandatory to follow the WDV
Method in respect of the new unit also, especially since the same class of assets exist at the existing units and
new unit.
Solved Question Paper—CA Final-November 2013 (Accounting Standards) 413

You are requested to comment on the stand of the Chief Accountant. (Marks 4)
Solution: If a company has two class of assets and method of depreciation varies in the two cases then in
such case different method of depreciation is allowed. But such different method is required to be followed
consistently every year. Hence the view of the Chief Accountant that the company cannot follow SLM method
of depreciation at the newly acquired unit is not correct and the company can follow the SLM method of
depreciation at the newly acquired unit.

Part II: Auditing and Professional Ethics Nov. 2013

Queston 1: X Limited, a newly incorporated Company in India commenced its business from 1st April 2012.
The Company purchased Fixed Assets costing ` 4,000 Lakhs on 01–04–2012 and the same was fully financed
by Foreign Currency Loan (U.S. Dollars) payable in three annual equal installments. Exchange Rates were
1 Dollar = ` 40.00 and ` 42.50 as on 01–04–2012 and 31–03–2013 respectively. The Company worked out
Foreign Exchange Loss as per AS 11 at ` 250 Lakhs and expensed the entire amount in the Profit and Loss
Account. The Managing Director of the Company was worried about this heavy revenue loss and asked the
Accountant not to follow AS 11 issued by the ICAI for this particular transaction. The Accountant of the
Company, followed the instruction of the Managing Director and removed Exchange Loss from the Profit
and Loss Account but then he added the entire Exchange Loss to the value of Fixed Asset and computed the
depreciation thereon. As an Auditor of X Limited how you would you deal with this particular transaction?
(Marks 5)
Solution:
1. Treatment as per AS – 11, Foreign Exchange Loss on repayment of a debt in relation to acquisition
of a Fixed Asset shall be written off in the Statement of Profit and Loss. It cannot be capitalized and
amortised over the period of the asset.
2. Special Relaxation to Corporates:
(a) As per MCA Notification dated 29th Dec. 2011, Companies are given an option to capitalize the
above costs and claim depreciation thereon. This Notification is applicable for accounting year
upto 31st March 2020.
If X Ltd follows companies act then it has to disclose the fact that the company has adopted the
option of capitalizing exchange difference towards Cost of Fixed Assets as the option is granted
by the govt.
Question 2: Company- has a scheme for payment of settlement allowance to retiring employees.
Under the scheme, retiring employees are entitled to reimbursement of certain travel expenses for class they
are entitled to as per company rule and to a lump-sum payment to cover expenses on food and stay during the
travel. Alternatively employees can claim a lump-sum amount equal to one month pay last drawn.
The Company’s contentions in this matter are-
(a) Settlement allowance does not depend upon the length of service of employees. It is restricted to
employees’ eligibility under the Travel rule of the Company or where option for lump-sum payment
is exercised, equal to the last pay drawn.
(b) Since it is not related to the length of service of the employees, it is accounted for on claim basis.
State whether the contentions of the Company are correct as per relevant Accounting Standard. Give
reasons in support of your answer. (Marks 4)
Solution: The present case falls under the category of defined benefit scheme under Para 49 of AS 15
(Revised) “Employee Benefits”. The said para encompasses cases where payment promised to be made to an
employee at or near retirement presents significant difficulties in the determination of periodic charge to the
statement of profit and loss. The contention of the Company that the settlement allowance will be accounted
for on claim basis is not correct even if company’s obligation under the scheme is uncertain and requires
414 Accounting Standards

estimation. In estimating the obligation, assumptions may need to be made regarding future conditions and
events, which are largely outside the company’s control.
Thus, (1) Settlement allowance payable by the company is a defined retirement benefit, covered by AS
I5 (Revised). (2) A provision should be made every year in the accounts for the accruing liability on account
of settlement allowance. The amount of provision should be calculated according to actuarial valuation. (3)
where, however, the amount of provision so determined is not material, the company can follow some other
method of accounting for settlement allowances.
Question 3: H Limited, a Company registered with SEBI, has three Subsidiaries and one Associate. While doing
the audit of Consolidated Financial Statements (CFS) of H Limited you have come to know that the Associate
Entity had made a Provision for Proposed Dividend in its Financial Statements. H Limited computed it share
of the results of operations of the Associate after taking into account the Proposed Dividend. Comment.
(Marks 4)
Solution: Proposed dividend by the Associates ASI-16
It is clarified that in case the associate has made a provision for proposed dividend in its financial
statements, the investor’s share of the results of operations of the associate should be computed without
taking into consideration the proposed dividend (also refer practical problem asked in May 2010). Carrying
Amount of Investment is reduced by the distributions (Dividends received) from Associates. However, when
the Associate has made a Provision for Proposed Dividend in its Financial Statements, the Investor’s Share
from the Associate should be computed without taking into consideration the Proposed Dividend. Accounting
treatment given by H Ltd in the above case, is not proper.
Question 4: X Ltd is engaged in the business of Newspaper and Radio Broadcasting. It operates through
different brand names. During Financial Year 2012–2013, it incurred substantial amounts on external trade,
business communication and branding expenses by participation in various Corporate Social Responsibility
initiatives. The Company expects to benefit by this expenditure by attracting new customers over a period of
time and accordingly it has capitalized the same under Brand Development Expenses and intends to amortize
the same over the period in which it expects the benefits to flow. As the Statutory Auditor of the Company, do
you concur? Give reasons. (Marks 4)
Solution:
Assumption 1:
All the Broadcasting Brands are purchased one: Yes the Brands are Intangible assets as per AS-26.
Purchased Brands are intangible assets.
Assumption 2:
All the Broadcasting Brands are internally generated: As per AS-26 internally generated brands, goodwill,
publishing titles and mastheads are not intangible assets…………….. para 50
Now let’s talk about the expenses on promotion and branding development.
Whether the brands are purchased one or internally generated in all such cases expenses on brand building
has to be charged to profit / loss account.
AS-26 has prohibited Selling and Advertisement expenses to be either deferred or capitalized. The
company is incorrect in its contention.
SOLVED QUESTION PAPER—CA FINAL-MAY 2014
(ACCOUNTING STANDARDS)

Part I: Financial Reporting


Question 1 (a): An Employee Roshan has joined a Company XYZ Ltd in the year 2013. The annual emoluments
of Roshan as decided is `14,90,210. The Company also has a policy of giving a lump sum payment of 25% of the
last drawn salary of the Employee for each completed year of service, if the Employee retires after completing
minimum 5 years of service. The salary of Roshan is expected to grow at 10% per annum. The Company has
inducted Roshan in the beginning of the year and it is expected that he will complete the minimum five year
term before retiring. What is the amount the Company should charge in its Profit & Loss A/c every year as
cost for the Defined Benefit Obligation? Also calculate the Current Service Cost and the Interest Cost to be
charged per year, assuming a Discount Rate of 8%. (P.V Factor for 8% – 0.735, 0.794, 0.857, 0.926. 5 Marks
Solution:
Salary increases every year by 10%.
2013 given `14,90,210
2014 `14,90,210 + 10% `16,39,231
2015 `16,39,231 + 10% `18,03,154
2016 `18,03,154 + 10% `19,83,470
2017 `19,83,470 + 10% `21,81,816
Accounting for Ex–gratia: Ex–gratia paid is in relation to service rendered. Hence, the cost of ex–gratia will
be amortised over the period of service.
• Ex–gratia at the end of Year 5 = `21,81,816 × 25% × 5 completed years = `27,27,270.
• Hence cost attributable to every year = 5 years
`27,27,270 = `5,45,454.
Using the PVF at 8% given in the question, the Current Service Cost and Interest Cost are computed as
under – (amts in `)
Lets now build up GRATUITY of obligation as per Projected Unit Credit Method (PUCM):

Year 1 Year 2 Year 3 Year 4 Year 5


Opening obligation b/d - 400908 866071 1402811 2020126
+ Interest Cost 8% - 32073 69286 112225 16161
+ Current Service Cost 400908 433090 467454 505090 545454
Closing obligation 400908 866071 1402811 2020126 2727190

The difference of 2727190 – 2727270 = `80 is due to rounding off.


Calculation of CSC:
2013 - `545,454 × 0.735 = 400,908
2014 - `545,454 × 0.794 = 433,090
2015 - `545,454 × 0.857 = 467,454
2016 - `545,454 × 0.926 = 505,090
2017 - `545,454 × 1.000 = 545,454
Note: The solution given by ICAI is completely wrong. Refer Bare Act where a similar problem is being solved
by taking growth in salary by 4 years and not 5 years. s
416 Accounting Standards

Question 1 (b): Quick Ltd is a Company engaged in the trading of spare parts used in the repair of automobiles.
The Company has been regular in depositing the tax, as such there is no liability of Income Tax, etc. for the Financial
Year 2012–2013. The figures for the year are as under:
Income chargeable to Tax `211.64 Lakhs
Total Income after Adjustments `228.48 Lakhs
Tax thereon `74.13 Lakhs
TDS deducted during the year `30.45 Lakhs
Tax paid for the year `43.68 Lakhs
The Company has prepared its Balance Sheet as per above figures. However, during the assessment proceeding held
before the finalization of the Balance Sheet, the Income Tax Officer has issued demand of `7.52 Lakhs, insisting that this
amount of TDS has not been uploaded online and thus is not acceptable as deduction.
The Company has in reply to the same filed a rectification with the Assessing Officer. The Company is trying to
collect the TDS Certificates, but `2.39 Lakhs deducted by XY Ltd, is not traceable. The rectification is lying pending
with the Assessing Officer. Please suggest the treatment of `2.39 Lakhs and `7.52 Lakhs in Balance Sheet. 5 Marks
Solution: There are 2 things in the problem (1) TDS of `2.39 lakhs which is not traceable, (2) TDS of `5.13 lakhs
which is being traced. However in this case the ITO should accept the TDS evidence: Under the Income Tax Act, the
credit for TDS cannot be denied on the ground that it is not reflected online in Form 26AS. In such cases, the TDS
Credit can be given on the strength of other evidences available with the Assessee.
Accounting treatment:
For TDS of `2.39 lakhs which is not traceable: It may or may not be traceable, in future. This means it is not a real but
possible obligation which means it is a contingent liability. It should be disclosed in the notes to accounts.
For TDS of `5.13 lakhs which is traceable: As the occurrence of the liability is remote no disclosure is required
either as a contingent liability nor it is a provision.
Q uestion 1 (c): Comptech Ltd, having office at Chennai, acquired a sophisticated three dimensional (3D)
Computer Printer having all inclusive MRP (Maximum Retail Price) of `50 Lakhs from a Supplier located at
New Delhi. The terms of the purchase were as under:
(i) The Supplier would buy back the existing unit with Comptech that has carrying amount of
`10.20 Lakhs. Prevailing CST Rate is 2%.
(ii) The Supplier would give a special discount of 10% on MRP to Comptech considering their long
standing relationship.
(iii) A Cash Payment of `38.25 Lakhs would be made by Comptech Ltd, to the Supplier.
(iv) Accessories required to operate the Machine costing `7.60 Lakhs (inclusive of all taxes) will be
purchased by Comptech.
(v) The Supplier will deliver free of cost certain heavy duty cables, etc. having MRP of `5.75 Lakhs, that
are required to run the Machine.
(vi) Transit Insurance Cost will be borne by Comptech at 2% of MRP.
(vii) Freight and other incidentals amounting to `2.30 Lakhs is borne by Comptech.
You are required to arrive at the cost of the new asset and show the Profit/(Loss) incurred by Comptech
on the buy–back arrangement and also draft the Journal Entries to record the above transaction. 5 Marks
Solution:
Note: The question itself is contradictory and contains too many flaws. By the way students have got 5 marks for attempting this question.
No solution can be prepared for such a faulty question. Every attempt one or two questions contains serious errors: ICAI please stop this.

Question 1 (d): Compute Basic and Adjusted EPS from the following information. All workings may be
rounded off to two decimals.
Net Profit for 2012–2013 `22 Lakhs
Net Profit for 2013–2014 `33 Lakhs
No. of Shares before Rights Issue 110,000
Rights Issue Ratio One of Every Four Held
Rights Issue Price `180
Date of Exercising Rights option 31–7–2013 (fully subscribed on this date)
Fair Value of Share before Rights Issue `270 5 Marks
Solved Question Paper—CA Final-May 2014 (Accounting Standards) 417

Solution:
Calculation of WANES:

Particulars No of Shares Weight Product


Original shares 1,10,000 1 1,10,000
Right issue:
Shares issued for consideration 19,643 8/12 13,095
Bonus element 7,857 1 7,857
1,30,952

Original Shares × Cum market price + RI × Subscription Price


(1) Theoretical Ex-right MP =
Total Shares
110000 × 270 + 27500 × 180
= = `252
137500
(2) Total Right shares issued = 27,500
Less: Shares for which proceeds are received (27500 × 180/252) 19643
Bonus element or proceeds not recd 7,857
Calculation of EPS:
Basic EPS 2013-14 (Reported) = Net Profit/WANES = 33,00,000/1,30,952 = `25.20
Adjusted EPS 2012-13 = 22,00,000/1,17,857 = `18.67
Question 7 (c): What are Timing Differences and Permanent Differences as per Accounting Standard – 22?
Explain with example. 4 Marks
Solution:
Timing Differences Permanent Difference
Differences in accounting income and Taxable income which These are the differences in the Accounting income and
arises in one year and is capable of reversal in the subsequent Taxable income which arises in one year and is not capable of
years is known as timing Differences. reversal in the subsequent years.
Deferred taxes are created on TDs No deferred taxes are created on PDs.
Ex: Borrowing Costs, Finance lease, Depreciation, Section 43B Ex: Penalty, Donations disallowed, Agricultural income.
expenses.

Question 7 (d): X Ltd has leased an equipment over its useful life that costs `7,46,55,100 for a three year
lease period. After the lease term, the asset would revert to the Lessor. You are informed that –
(i) The estimated Unguaranteed Residual Value would be `1 Lakh only.
(ii) The Annual Lease Payments have been structured in such a way that the sum of their Present Values
together with that of the Residual Value of the asset will equal the cost thereof.
(iii) Implicit Interest Rate is 10%.
You are required to ascertain the Annual Lease Payment and the Unearned Finance Income. PV Factor
@ 10% for year 1 to 3 are 0.909, 0.826 and 0.751 respectively. 4 Marks
Solution:
Calculation of PV of MLP + UGRV:
Let MLP every year be `X.

Year MLP PVF @ 10% PV (MLP)


0 - - -
1 X 0.909 0.909X
418 Accounting Standards

2 X 0.826 0.826X
3 X+100000 0.751 0.751X+
(Rent + UGRV) 75100
Total 3X + 100000 2.487X + 75100

As per the data given in the problem PV of MLP + RV = Cost of Asset


Therefore, 7,46,55,100 = 2.487X + 75100
X = `2,99,87,937.
i) Annual Lease payment = `2,99,87,937.
ii) UFI = Gross Investments – Net Investments = (3X + 100000) – 74655100 =
3 × 2,99,87,937 + 100000 – 74655100 = 90063811 – 74655100 = `1,54,08,711
Question 7 (e): AQ Ltd, an Investment Company, is finalizing its account for the Financial Year ending 2013
in the month of August 2013.
How will the following incomes to accounted for in the books of AQ Ltd?
(1) X Ltd has declared Interim Dividend which has not been received till 31–03–2013 but received on
25–04–2013.
(2) Y Ltd has declared dividend on 8th May 2013 for the year ending 31–03–2013, which has been
approved by the Shareholders of the Company on 30th June 2013.
(3) Z Ltd, a Subsidiary of AQ Ltd, has declared dividend for the year ended 31–03–2013 on 25th May
2013 the AGM for which is to be held on September 2013. 4 Marks
Solution:
Revenue Recognition Principle:
As per AS–9, Dividend from Investments in Shares are not recognised in the Statement of Profit and Loss
until a right to receive payment is established.
Based on above, the recognition will be as under –
Case 1: Since, Interim Dividend is declared before the financial year ending 31–3–2013.
Case 2: In this case also, AQ Ltd should recognise the Dividend in the financial year 2013–14 and not in the
year ending 31–3–2013, since the right to receive dividend did not exist at the Balance Sheet date, i.e. as at
31–3–2013 but existed only when Annual General Meeting of Y Ltd approved the dividend on 30th June 2013.
Case 3: AQ Ltd will recognise the Dividend in the Financial Year 2013–14, for the same reason mentioned in
Case 2.

Part II: AUDITING


Question 1: ABC Company files a law suit against Unlucky company for `5 crores. The Attorney of Unlucky
feels that the suit is without merit, so unlucky company merely discloses the existence of the law suit in the
notes to accounts accompanying its financial statements. As the Auditor, how would you respond to this?
Marks 5
Hint: As per AS – 29 if the probability of outflow of resources is probable then the provision has to be made.
As it is clear from the data of the problem that the Attorney of Unlucky feels that the suit is without merit the
provision should not be made. But the Auditor has to ascertain the fact regarding the legal suit filed against
the company, still whether provision is required. If provision is required then the company should make
a provision.
SOLVED QUESTION PAPER CA FINAL-NOVEMBER 2014
(ACCOUNTING STANDARDS)

Part I: Financial Reporting


Problem 1: From the given information, you are required to compute the deferred tax assets and deferred
tax liability for Ramanujam Limited as on 31st March 2014. The tax rate applicable is 35%.
(i) The company has charged depreciation of( 7,42,900/- in its books of accounts while as per
income-tax computation, the depreciation available to the company is `8,65,400
(ii) The company has made provision for doubtful debts for `54,300/- during the year.
(iii) The company has debited share issue expenses of `6,23,500/- which will be available for
deduction under the Income-tax Act from the next year.
(iv) The expense of `7,84,5001- has been charged to profit and loss account which are disallowed
under the Income-tax Act.
(v) The company has made donation of `2,00.000/- which has been debited to profit and loss
account and only 50% thereof win be allowed as deduction as per Income-tax law. 5 Marks
Solution:
Particulars Timing Diff Net Deferred tax@35%
Depreciation (865400 - 7,42,900) 1,22,500 42,875
Provision for DD (54300 – 0) 54300 19,005
Share issue exp (623500 – 0) 623500 2,18,225
Net DTA 2,80,105

Note: (1) Item no (iv) `7,84,500 disallowed under IT Act is a permanent difference.
(2) Item no (v) `2,00,000 whereby 50% is allowed by IT Act and 50% is disallowed under IT Act
hence it is a permanent difference.
Problem 2: ABC Limited has three segments viz. A, B and C. The total assets of the company is `15 Crores.
The assets of Segment A is 1.85 crores, Segment B is 6.15 Crores and Segment C is `7.00 Crores. Assets of
each segment include deferred tax assets of `0.50 Crores in A, `0.40 Crores in B .and `0.30 Crores in C. The
accountant of ABC Limited contends that all segments are reportable segments. Based on segment assets
criteria. Determine the veracity of the contention of the accountant. 5 Marks
Solution: According to AS 17 “Segment Reporting”, segment assets do not include income tax assets. Segment
assets should be net off DTA. Segment A holds assets of 1.35 crores (1.85 – 0.5crores); Segment B holds 5.75
crores (6.15 crores – 0.4 crores); and Segment C holds 6.7 crores (7 crores – 0.3 crores). Therefore, the
revised total assets are 13.8 crores [1.35+5.75+6.70)crores.
Segment A holds =9.78% of total segment assets < 10%
Segment B holds =41.67% of total segment assets > 10%
Segment C holds =48.55% of total segment assets > 10%
Only Segment A is not reportable but the other segment viz B and C are reportable. B and C’s assets
exceeds more than 10% of the total assets of all the segments.
420 Accounting Standards

Problem 3: Sunny Limited is developing a new production process. During the financialst year ended 31st
March 2013, the company has incurred total expenditure of `40 lacs on the process. On 1 December, 2012,
the process has met the norms to be recognized as ‘intangible assets’ and the expenditure incurred till that
date is `16 lacs. During the financial year ending on 3l SI March 2014, the company has further incurred f
70 lacs. The recoverable amount as on 31st March 20 14 of the process is estimated to be `62 lacs. You are
required to work out:
(i) Expenditure to be charged to profit and loss account for the financial year ending on 31st March
2013 and 31st March 2014. (ignore depreciation)
(ii) Carrying amount of the ‘Intangible asset’ as at 31st March. 20 13 and 31st March 2014 5 Marks
Solution: Expenditure charged to P&L A/c for 2012-13: `16 lakhs will be recognized as an expense because
the recognition criteria were not met until 1st December 2012. This expenditure will not form part of the
cost of the production process recognized in the balance sheet. Carrying value of intangible as on 31.03.2013
will be `24 lakhs.
(i) Expenditure charged to P&L A/c for 2012-13 will be `16 lakhs.
Carrying amount as on 31/3/2013 = 24 lakhs
(ii) Next year:

Particulars ` lakhs
Development cost as on 31/3/2014 94
(24 lakhs + 70 lakhs) 62
Less: recoverable amount
Impairment loss to be charged to P&L A/c 32

Carrying value of intangibles as on 31.03.2014: 62 lakhs and the Amount charged to P/L will be 32 lakhs
SUMMARY:

31/3/2013 31/3/2014
Amount charged to P/L a/c 16,00,000 32,00,000
Carrying amount of asset 24,00,000 62,00,000

Problem 4: XYZ Limited is having the following Fixed Deposit Receipts.


Date of FDR Maturity Date Amount `
Axis Bank Limited 01.Jan-14 30-Apr-15 10,00,000
Punjab National Bank 01.Jan-14 30-Jun-15 15,00,000
State Bank of India 28.Feb-14 30-May-15 10,00,000
ICICI Bank 31.Jan-13 31-Jan-15 10,00,000

Prepare ‘Notes’ accounts showing the above deposits in accordance with the requirements of Revised
Schedule VI. 5 Marks
Solution:
Assumption: Year ending: 31/3/2014
As per the revised Schedule VI disclosures are as follows:
Current Assets:
(a) Cash & Cash Equivalents: (maturity < 3 months)
Punjab National Bank FD `15,00,000
State Bank of India FD `10,00,000
Solved Question Paper—CA Final-November 2014 (Accounting Standards) 421

(b) Other Current Assetss: (maturity > 3 months but < 12 months)
ICICI Bank FD `10,00,000
Non – Current Assets:
Other Non – Current Assets: (maturity > 12 months)
Axis Bank FD `10,00,000

Problem 5: Adventure Limited issued 20,000, 9% convertible debentures of `. 100 each at par at the
beginning of the year. The debentures are of 6 years term. The interest will be paid half yearly. The debenture-
holders have the option to get 50% of the debentures converted into 2 ordinary shares at the end of 3rd year.
The debenture holders who do not opt for conversion will be paid 50% of their face value at the end of year
3.The balance non-convertible portion will be repaid at 10% premium at the end of term of the debenture.
At the time of issue, the prevailing market interest rate for similar debt without convertibility option is 10%.
Present Value of annuity is as under
Period 1-3 4-6 7-12
Annuity factor @ 10% 2.487 1.868 2.459
Annuity factor @ 5% 2.723 2.353 3.787

Present value of `1 at the end of 3 years at 10% and 5% is 0.565 and 0.747 respectively. Present value
of `1 at the end of 6 years at 1% and 5 is 0.317 and 0.557 respectively.
Computer journal entries recognizing the issue of debentures. 4 Marks
Solution: Calculation of PV of Cash Flows:
Yr CF PVF Product Yr CF PVF Product
1-6 900001 5.076 4,56,840 6th 5000002 0.747 3,73,500
7-12 450003 3.787 1,70,415 12th 11000004 0.557 6,12,700
627255 986200
• 627255+986200 = `16,13,455

Note:
1) 20000 × 100 × 9% × ½ = 90000 ; 2) 20000 × 100 × 50% × 50% = 45000
3) 20000 × 100 × 50% × 9% × ½ = 45000 ; 4) 20000 × 100 × 50% × 110% = 1100000
5) Interest is taken as 5% for 12 years (6 × 2 multiples).

Split up of Equity and Debt component Journal entry


Total proceeds recd 20,00,000 Bank A/C Dr 20,00,000
Debentures element (16,13,455) To 9% Debentures 16,13,455
Equity (bal fig) 3,86,545 To Equity 3,86,545
(Being Equity and Debt initially recognised)

Problem 6 : Answer any FOUR of the following:


(a) JVR Limited has made investments of `97.84 Crores in equity shares of QSR Limited in pursuance
of Joint Venture agreement in 2001-02. The investment has been made at par. QSR Limited has been
m continuous losses for the last 2 years. JVR Limited is willing to re-assess the carrying amount of
its investment in QSR Limited’ and wish to provide for diminution in value of investments. However,
QSR Limited has a futuristic and profitable business plans and projection for the coming years.
422 Accounting Standards

Discuss whether the contention of JVR Limited to bring down the carrying amount of investment in
QSR Limited is in accordance with Accounting Standard. 4 Marks
(b) What is a derivative? Define with reference to AS-30, Financial Instruments: Recognition and
Measurement. 4 Marks
(c) Finished Goods costing `10 lacs were damaged due to flood in July 2013. These goods were included
in Closing Stock as on March 31, 2014 at an estimated realizable value of `4.00 lacs.
These goods could be ultimately sold for `3 lacs only in August 2014. The difference of f 1 lac was
debited as prior period expenditure in financial year 2014-15. As an auditor please comments in the
light of provisions of accounting standards. 4 Marks
(d) XYZ Limited acquired a bank Joan of `40 lacs on interest rate of 20% per annum on 1 July 2013. The
said loan was utilized by the company for three transactions as under:
(i) Construction of factory shed `10,00,000
(ii) Purchase of Plant and Machinery `25,00,000
(iii) Balance loan was unallocated and used generally for the purpose of business.
The accountant of the company has charged the total interest to Profit and Loss account. Comment
in view of provisions of AS-16. 4 Marks
(e) Samrat Limited has set up its business in a designated backward area which entitles ‘the company
for subsidy of 25% of the total investment from Govenunent of India. The company has invested
`80 Crores in the eligible investments. The company is eligible for the subsidy and has received `20
crores from the government in February 2014. The company wants to recognize the, said subsidy as
its income to improve the bottom line of the company. Do you approve the action of the company in
accordance with the, Accounting Standard? 4 Marks
Solution (a): Valuation of Investments: AS 13 on “Accounting for Investments” requires investments to be
classified as long term and current investments distinctly in its financial statements. The investments in
shares in a Joint Venture can very well be considered as trade investments since they would not be intended
to be liquidated within a period of one year from its acquisition. Hence they would be classified as long term
investments.
AS 13 states, “long-term investments should be carried in the financial statements at cost. However,
provision for diminution shall be made to recognize a decline, other than temporary, in the value of the
investments, such reduction being determined and made for each investment individually”. In the instant
case, the decline in the value of investment seems to be temporary and the company has profitable plans in
the coming years. Also investments in LTI are never valued at MV but even cash flows are considered.
Conclusion: No diminution required.
Solution (b): Derivatives: (AS-30)
Derivatives are contracts such as options, forwards, futures, and swaps. Because they are often entered
into at no cost, many times derivatives were not recognized in financial statements prior to IAS 39 / AS-30.
But now the accounting is done as soon as the contract for derivative is entered into. Even MTM valuation is
done on year ending.
For an instrument to be eligible for derivative all the combined conditions should be satisfied:
(1) Value of contract is based on current value of underlying asset. Ex: Interest rate in case of IRS, forward
contract may be based upon the forex rate etc.
(2) It requires low or no initial investments. In case of Swaps / forward Zero investments are required,
in case of options premium is required. The initial investments are smaller as compared to other
contract to have similar response to change in market factor.
(3) Settlement takes place at a future date.
Solved Question Paper—CA Final-November 2014 (Accounting Standards) 423

Solution (c): The difference of `100000 cannot be a prior period item as no error took place in the preparation
of financial statement of past period. The estimated write off of the inventory `6,00,000 (10-4) is revised to
`7,00,000. This is change in Accounting Estimates as per AS-5. Many items in the financial statement cannot
be exactly calculated but can only be estimated. Estimation changes because of change in circumstances,
more development in the case. In the given case the company is required to just disclose the change in a/c
estimate in the financial statement.
Solution (d): Construction of factory shed is a qualifying asset because factory shed in being constructed. P & M
is just installed which is not eligible for Q.A. Assuming that plant installation takes less period of time to get ready.
This means only `10,00,000 of loan is used for QA. Accordingly Interest of `1,50,000 (1000000 x 9/12 x 20%)
needs to be capitalised. The remaining interest will be charged to profit / loss a/c i.e. `450000. (6-1.5).
Assumption: Year ending is March, Plant installation does not tak substantial period of time for installation.
If we assume that the installation of plant is a part of expansion programme or installation takes substantial
period of time then in such case the Borrowing costs capitalised will be: `5,25,000
Solution (e): As per para 10 of AS 12 ‘Accounting for Government Grants’, where the government grants
are in the nature of promoters’ contribution, i.e. they are given with reference to the total investment in
an undertaking or by way of contribution towards its total capital outlay (for example, central investment
subsidy scheme) and no repayment is ordinarily expected in respect thereof, the grants are treated as capital
reserve which can be neither distributed as dividend nor considered as deferred income.
In the given case of Samrat Ltd, the subsidy received is neither in relation to specific fixed asset nor in
relation to revenue. Thus it is inappropriate to recognise government grants in the profit and loss statement,
since they are not earned but represent an incentive provided by government without related costs. The
correct treatment is to credit the subsidy to capital reserve. Therefore, the accounting treatment followed by
the company is not proper. The Auditor is required to issued a qualified report under section 143 and SA 705
of the Companies Act.
• Under the new schedule VI (because the paper is based on Sch VI) such a grant is disclosed under the
head EQUITY.

Part II: Auditing (AS based questions)


Problem 1: In the Notes to Accounts of Z Co. Ltd as on 31–03–2015, Note No. 10 states that certain machinery
items are lying at customs warehouses and that the Company has paid `500 lakhs up to 30–06–2014 as
Detention Charges, out of which a sum of `220 Lakhs is written back during the year 2014–2015 based
on settlement with the concerned authorities in respect of a major spares of machinery. For the remaining
machinery items, negotiations are pending and a provision of `48 Lakhs has been made. As such a total
amount of `328 Lakhs paid / provided on account of Detention Charges have been capitalized and included in
the Fixed Assets / Capital Work in Progress. The Management is of the view that these expenses are directly
attributable to the acquisition of the related Fixed Assets. As the Auditor, how would you respond to this?
Marks 5
Hint: Detention charges are not to be capitalised as per AS-10. Such charges are avoidable expenses. They are
abnormal expenses.
Problem 2: Big and Small Ltd received a show cause notice in December 2014 from the Central Excise
department intending to levy a sum of `25 Lakhs. The Company replied to the above Notice in January 2015
contending that it is not liable for the proposed levy. No further action was initiated by the Central Excise
Department up to the finalization of the audit for the year ended on 31st March 2015. As the Auditor of the
Company, how would you deal with this matter in your Report? Marks 3
Hint: As per AS – 29 it can be shown as a Contingent liability because the company does not accept the
liability. The probability of outflow is subject to the further legal action taken by the company.
424 Accounting Standards

Problem 3: In the course of your audit you have come across a Related Party Transaction which prima facie
appears to be biased. How would you deal with this? Marks 4
Hint: Auditor has to understand the possible related party relationship defined under para 3. As per AS-18
he is required to check whether the total disclosure is made as per the Standard. Also refer SA – 550 Auditors
Practice on AS-18.
Problem 4: Short Note on : Intangible Asset Vs Intangible Item. Marks 4
Solution:
What is an Intangible asset: As per AS – 26 intangible assets is one which satisfies all the following conditions:
i) It should be an asset, ii) It should be a non – monetary asset, iii) Identifiable, iv) Without physical substance, v) Held for
used in production, administrative purposes, fetching rent etc.

What is an Intangible item: It is not covered by AS – 26. It reflects an insignificant value of an item which is
used in common parlance.
SOLVED QUESTION PAPER—CA FINAL-MAY 2015
(ACCOUNTING STANDARDS)

FINANCIAL REPORTING
Question 1:
From the following information, value the Inventories as on 31st March, 2015: Raw Material has
been purchased at ` 125 per kg. Prices of Raw Material are on the decline. The Finished Goods being
manufactured with the Raw Material is also being sold at below Cost. The Stock of Raw Material is of
15,000 kg and the Replacement Cost of Raw Material is ` 100 per kg. Cost of Finished Goods per kg
is as under: Particulars ` per kg
Material Cost 125
Direct Labour Cost 20
Direct Variable Production Overhead 10
Fixed Production Overhead for the year for a normal capacity of 1,00,000 kgs of production is `
10 Lakhs. At the year end, there were 2,000 kgs of Finished Goods in stock. Net Realisable Value of
Finished Goods is ` 140 per kg.
Solution:
Valuation of Finished goods: Cost or NRV whichever is less:
Cost = 125+20+10+10 = 165. Also NRV = 140
Valuation of Finished goods = 140 × 2000 = Rs 280000
If the price of raw material is on decline and the finished goods are sold below cost then the materials
are valued at replacement cost.
Valuation of raw materials = 15000 × 100 = Rs 1500000
Total Value of inventory = 280000+1500000 = 17,80,000
Question 2:
SMC Limited is having a Plant (an Asset) whose Carrying Amount as on 01–10–2012 is ` 38,000
Lakhs and the Plant was having a useful life till 31–03–2020. The estimated Residual Value is ` 900
Lakhs. The Selling Price on 31st March 2015 is expected to be ` 20,000 Lakhs and the Cost of Disposal
is expected to be ` 100 Lakhs.
The Expected Cash Flows from the Plant are as under –
Financial Year 2015–2016 2016–2017 2017–2018 2018–2019 2019–2020
Cash Flow ` Lakhs 4,100 5,900 6,000 7,800 4,500
The Company expects the Discount Rate of 10%. Discount Factor at 10% for 1, 2, 3, 4 and 5 years are
0.909, 0.826, 0.751, 0.683 and 0.621 respectively. The Company provides depreciation on SLM basis.
You are required to determine as at 31st March 2015:
(i) Value in Use of the Plant
(ii) Impairment Loss, if any, to be recognized for the year.
(iii) Revised Carrying Amount for the Financial Year ending 31st March 2015.
Solution: Original Cost 38,000.00 Less: Depreciation for 2012–2013, 2013–2014 and 2014–2015 (12366.68)
= Carrying Amount on 31.03.2015 before Impairment = Rs 25,633.33 lakhs.
426 Accounting Standards

Recoverable Amount (Net Selling Price 19,900 [or] Value in Use 21,787.10, whichever is higher) = Rs
21,787.10
Impairment Loss = Carrying Amount Less Recoverable Amount =
Rs 25,633.33 – Rs 21,787.10 = Rs. 3,846.23 lakhs.
WN: 1) Depreciation = (38000 – 900) / 7.5 = Rs 4946.67. Depn for 2 ½ years = 4946.67 x 2.5 = Rs. 12366.68
WN: 2) Value in Use = 4100 x 0.909 + 5900 x 0.826 + 6000 x 0.751 + 7800 x 0.683 + (4500 + 900)x 0.621 =
Rs 21787.10 lakhs.
(i) VIU = Rs 21787.10lakhs
(ii) Impairment loss = Rs 3846.23 lakhs
(iii) Revised Carrying amount on 31/3/2015 = Rs 21,787.10 lakhs.
Question 3: A Company sells the goods with right to return. The following pattern has been observed:
Time frame of Return from date of purchase % of Cumulative Sales
Within 10 days 5%
Between 11 days and 20 days 7%
Between 21 days and 30 days 8%
Between 31 days and 45 days 9%
The Company has made Sales of ` 30 Lakhs in the month of February 2015 and of ` 36 Lakhs in the
month of March 2015. The Total Sales for the Financial Year have been ` 450 Lakhs and the Cost of
Sales was ` 360 Lakhs. Determine the amount of Provision to be made and Revenue to be recognized
in accordance with AS–9. A year may be considered of 360 days.
Solution: Accounting treatment for recognition of revenue
Revenue in respect of sale of goods is recognised fully at the time of sale itself assumed that the
company has complied with the conditions stated in AS 9 relating to recognition of revenue in the
case of sale of goods. AS 9 also provides that in case of retail sales offering a guarantee of ‘money
back, if not completely satisfied, it may be appropriate to recognize the sale but to make a suitable
provisions for returns based on previous experiences.
Therefore, sale of ` 30,00,000 and ` 36,00,000 made in the month of February and March, 2015 will
be recognized at full value.
Amount of provision
The goods are sold with a right to return. The existence of such right gives rise to a present obligation
on the company as per AS 29, ‘Provisions, Contingent Liabilities and Contingent Assets’. According
to the standard, a provision should be created on the Balance sheet date, for sales returns after the
Balance Sheet date, at the best estimate of the loss expected, along with any estimated incremental
cost that would be necessary to resell the goods expected to be returned.
It is assumed that the sales has been made evenly throughout the month and every month is of 30
days based on the information given in the question i.e. a year is of 360 days.
Sales during Sales value Sales value Likely Likely Provision @
(` in lacs) (cumulative) returns returns 20% (` in lacs)
(` in lacs) (%) (` in lacs) (Refer W.N.)
Last 10 days of March 36/3 or 12 12 5% 0.600 0.120
Previous 10 days of March 36/3 or 12 24 7% 1.680 0.336
Previous 10 days of March 36/3 or 12 36 8% 2.880 0.576
Last 15 days of February 30/2 or 15 51 9% 4.590 0.918
Total 9.75 1.950
Solved Question Paper—CA Final-May 2015 (Accounting Standards) 427

Working Note:
Calculation of Profit % on sales
(` in lacs)
Sales for the year 450
Less: Cost of sales (360)
Profit 90
Profit mark up on sales (90/450) x 100 = 20%

Note: The above solution is on the basis of the facts given in the question.
The heading of Column 1 is “the time frame of sales return from the date of purchase”. However, the
time ranges given in three out of four rows in column 1 is not reckoned from the date of purchase
but from the date after the close of the previous range. Further, the heading of column 2 is “% of
cumulative sales”. However, they are standalone ranges commencing from the date after the close of
the previous range. The two sets of data which are not exactly in line with each other may give rise
to alternative computation of amount of provision.
Alternatively, AS 9 provides that Revenue should not be recognized until the goods have formally
been accepted by the buyer or the buyer has done an act adopting the transaction or the time period
for rejection has elapsed or where no time has been fixed, a reasonable time has been elapsed. Based
on this, an alternative view can be taken whereby the revenue shall not be recognized in full. In such
a case, the revised sales will be as follows:
(` in lacs)
Revised Sales when estimated sales return is 9.75 lacs 450 – 9.75 440.25
Revised Cost of Sales 440.25 x 80% 352.20
Revised Gross Profit 88.05
Given Gross Profit 90
Reduction in Gross Profit 1.95
Reduction in receivables and sales 9.75
Inventory will stand increased by 7.80

Question 4: Saurav Limited reported a Profit before Tax of ` 8.00 Lakhs for the 2nd quarter ending on 30th
September 2014. On enquiry, following issues were noticed:
(i) Property Tax of ` 60,000 paid during the quarter for the full year has been recognized in full.
(ii) 1/5th of ` 15 Lakhs being Marketing Promotional Expenses incurred on 23rd September, 2014 has
been recognized based on past experience of higher sales in the last quarter of the year.
(iii) 50% of the Loss of ` 2 Lakhs incurred on disposal of a Business Segment has been allocated to this
quarter.
(iv) Cumulative Loss of ` 3 Lakhs resulting from the change in the method of Valuation of Inventory was
recognised in the 2nd quarter, which included ` 2 Lakhs related to earlier quarters.
(v) Gain of ` 15 Lakhs from Sale of Investments sold in the 1st quarter was apportioned equally over the
full year.
You are required to give proper treatment as required by AS–25 on Interim Financial Reporting and
to recast the adjusted Profit before Tax for the 2nd quarter.
428 Accounting Standards

Solution:
Particulars ` lakhs
Profits as reported 8,00,000
Add: Property taxes recorded on period basis 45,000
(60000 – 60000 × 3/12)
Less: Sales promotion expenses balance 4/5 th recorded (12,00,000)
(1500000 × 4/5)
Less: Loss on disposal of segment (1,00,000)
(200000 × 0.5) à full loss should be recognised
Add: Loss of previous quarters on change in stock valuation policy (Change in a/c policy should be 2,00,000
apportioned)
Less: Gain on sale of investments adjusted (gain recognised in this quarter has to be reversed) (3,75,000)
Adjusted PBT for the quarter ended 30/9/2014 = (6,30,000)
AS -25 follows a discrete approach for recognising revenue and costs.
Question 5: Lovely Limited has advanced Staff Loan of ` 50 Lakhs to its Employees on 1st July 2014 at a
concessional rate of 6% per annum, to be repaid in 5 semi–annual instalments along with interest thereon.
The prevailing rate is 8% per annum. Find out the value at which the Loan should initially be recognsied
and its amortization till closure thereof. Also give necessary journal entries with appropriate narration for
financial year 2014–2015.
Solution:
Present value of loan = CF1 × 0.9615 + CF2 × 0.9246 + CF3 × 0.8890 + CF4 × 0.8548 + CF5 × 0.8219 =
1150000 × 0.9615 + 1120000 × 0.9246 + 1090000 × 0.8890 + 1060000 × 0.8548
+ 1030000 × 0.8219 = ` 48,62,932
Note: Yield rate for semi annual = 4%.
CF1 = 1000000 + 5000000 × 3% = ` 11,50,000, similarly for CF2, CF3, CF4, CF5.
Amortization table:
Year Effective int 4% Cash int recd Principle cash flow Closing balance
3% recd 48,62,932
A B C D = Opening + A –
B–C
01/07/2014 till 1,94,517 1,50,000 10,00,000 39,07,449
31/12/2014
01/01/2015 till 1,56,298 1,20,000 10,00,000 29,43,747
30/6/2015
01/07/2015 till 1,17,750 90,000 10,00,000 19,71,497
31/12/2015
01/01/2016 till 78,860 60,000 10,00,000 9,90,357
30/6/2016
01/07/2016 till 39,643 30,000 10,00,000 R-off
31/12/2016
Solved Question Paper—CA Final-May 2015 (Accounting Standards) 429

Amortization table:
Date Particulars Dr Cr
01/07/2014 Staff Loan A/C Dr 48,62,932
Staff Cost A/C Dr. 1,37,068
To Cash 50,00,000
(Staff cost recorded @FV)
31/12/2014 Staff Loan A/C Dr 1,94,517
To Interest A/C. 1,94,517
(Interest income accrued)
Cash A/C Dr 11,50,000
To Staff Loan A/C 11,50,000
(First instalment recd)
Profit / loss A/C Dr 1,37,068
To Staff Cost A/C 1,37,068
Interest A/C Dr 1,94,517
To Profit / loss A/C 1,94,517
(Transfer to P/L A/C)
31/3/2015 Staff Loan A/C Dr (156298 x ½) 78,149
To Interest A/C. 78,149
(Interest income accrued but not due)
Interest A/C Dr 78,149
To Profit / loss A/C 78,149
(Transfer to P/L A/C)

Question 6: Give major differences between IFRS and AS (applicable in India) with respect to Property,
Plant and Equipment.
Fixed Assets Plant, Property, Equipment
AS-10 and AS-6 Single standard IAS – 16 covers both FA and
Depreciation
Dismantling charges are ignored while recognising Dismantling charges are recognised based on present
the asset. value when the asset is recognised.
Change in the method of depreciation is change in Change in the method of depreciation is change in
A/C Policy A/C Estimates
Revaluation balance is transferred to profit / loss a/c Revaluation balance is transferred to equity.
Depreciation is charged as per Schedule II of the Depreciation is based on management estimates.
Companies Act
Residual value once calculated in not updated. Residual value once calculated requires regular
updates.

Question 7: AB Limited acquired at the start of the financial year, Fixed Assets from USA at a price of US$
1,25,000 and made a down payment of US$ 25,000. The Exchange Rate was ` 61.50 per Dollar at the date
of transaction. The balance amount was payable in 4 equal half–yearly installments with interest @ 8% per
annum. The Exchange Rate on the due dates of installment has been ` 61.60, ` 61.80, ` 61.90 and ` 62.10. The
Asset was under construction during the period of six months from its acquisition. Ascertain the amount to
be capitalized and the Gain or Loss to be recognised in each of the years.
Solution:
Assumption: 1) MCA circular is not applicable. Hence no capitalisation.
2) Asset is not a qualifying asset, as the construction period is 6 months only.
430 Accounting Standards

This means purely AS-11 is applicable where no capitalisation is permitted.


Amount charged to P/L :
Installment Interest cost Exchange G/L Net (P/L)
1 (6 month) 2,46,400 10,000 2,56,400
100000 x 4% x 61.60 100000 (61.50-61.60)
2 1,85,400 15,000 2,00400
75000 x 4% x 61.80 75000 (61.60-61.80)
3 1,23,800 5,000 1,28,800
50000 x 4% x 61.90 50000 (61.80-61.90)
4 62,100 5,000 67,100
25000 x 4% x 62.10 25000 (61.90-62.10)
Amount charged to P/L:
Year 1: 256400+200400 = 4,56,800 : Year 2: 128800+67100 = 195,900
Question 8: HS Limited manufactures goods and caters to both national and international markets. As on
31st March 2015, it has the following Stocks in its Warehouse at Factory:
◘◘ Goods meant for National Market – Sale Value of ` 100 Lakhs
◘◘ Goods meant for International Market – Export Value of ` 50 Lakhs
The Company has a policy to mark up the products for national markets at one–third of cost while
those for exports are marked up at 150% of its cost. Excise Duty on goods is payable @ 12.36%. The
Management is of the opinion that Excise is payable only on clearance of goods from Factory and as
such the same should not be a part of Cost of Inventory.
You are required to guide the Company in the light of relevant Guidance Note.
Solution: Yes Excise duty has to be provided on local goods once they are manufactured. (For details please
refer same problem AS – 2 Problem No: 11)

Question 9: Krishna sold goods to Madhav for ` 100 Crores against an export order of Madhav. Subsequent
to the sale by Krishna, the export order of Madhav was cancelled for unavoidable reasons. Madhav decided to
sell the good in local market, provided a Price Discount is allowed by Krishna. Krishna acceded to the request
of Madhav. Advise how the discount given shall be dealt in the books of accounts of Krishna.
Solution: Treat the discount as an expense but not as a deduction from Sales.
(For details please refer same problem AS – 9 Problem No: 10)
Question 10: A Company desires to make provision in respect of its non–moving or slow moving items of
stock. The following information is available: (amounts ` in Lakhs)
Particulars Current Year Previous Year
Value of Closing Stock 169 105
Provision based on No. of issues during the year 4.50 4.00
Provision based on products technicality 5.50 4.25
The Company has been making provision based on number of issues. However, from this year, the
Management has decided to make provision based on technical evaluation. Explain whether such
change will amount to change in ‘Accounting Policy’. Also draw a suitable Note, if in your view the
proposed change requires the same to be given in the Financial Statement of the Current Year.
Solution:
It is a change in accounting policy. (For details please refer same problem AS – 5 Problem No: 3 (vi) )
Solved Question Paper—CA Final-May 2015 (Accounting Standards) 431

Question 11: Lucky P Limited has been assessed to Income –Tax, in which a demand of ` 10 lakhs has been
made. The Company has gone in appeal. The Company has deposited ` 6.00 Lakhs against the demand, on
being pursued by the Department. The Company has been advised by its Counsel that there is 80% chance
of losing in respect of one of the ground which may end up confirming the demand of ` 4.00 Lakhs, while
on other ground, there is fair chance of winning the appeal. How the Company should treat the same while
preparing the Final Accounts for the year ending 31st March 2015?
Solution:
An enterprise should recognise a provision only when all of the following conditions are met:
(i) there is a present obligation as a result of a past event;
(ii) it is probable that an outflow of resources embodying economic benefits will be required to settle the
obligation; and
(iii) reliable estimate can be made of the amount of the obligation.
Accounting treatment:
As there is 80% chance of losing the case it is highly probable that outflow of resources will be spent
to settle the obligation. Hence Lucky Ltd should make a provision of Rs. 4,00,000. For the balance
demand the company should make a disclosure as a Contingent Liability as there is a fair chance of
winning the appeal.
Regarding Rs. 6,00,000 which is not a settlement of the Liability, it should be disclosed under the
head “Non – Current Assets” indicating that it is paid in relation to an appeal pending.

SUGGESTED SOLUTION OF AUDIT PAPER - ACCOUNTING STANDARD


RELATED QUESTIONS MAY 2015
Question 1: Parent Ltd acquired 51% Shares of Child Ltd, during the year ending 31–03–2014. During the
Financial Year 2014–2015 the 20% Shares of Child Ltd, were sold by Parent Ltd. Parent Ltd, while preparing
the Financial Statements for the year ending 31–03–2014 and 31–03–2015 did not consider the Financial
Statements of Child Ltd, for consolidation. As a Statutory Auditor how would you deal with it?

Solution:
Parent Ltd has to comply with the CFS for both the years viz: 2013–2014 and 2014–2015. At the year end the
then subsidiary company will now become the Associate company. This means last year full consolidation is
applied and in the year 2014-15 Equity Consolidated will be applied. The Auditor of the Company should also
see the Compliance of AS–21,23.
SOLVED QUESTION PAPER CA FINAL-NOVEMBER 2015
(ACCOUNTING STANDARDS)

Financial Reporting
Question 1:
(a)
From the fol1owing information, you are required to compute the basic and adjusted earnings per
share:
Net profit for 2013-14 ` 11 Lakh
Net profit for 2014-15 ` 15 Lakh
No. of shares issued before rights issue ` 5 Lakh
Right issue One for every 5 held
Right issue price ` 15 per share
Last date of exercising right option 01-06-2014
Fair value of shares before right issue ` 21 per share
Solution: Similar to AS- 20 Question No.6: Answer: EPS 2014-15 ₹ 2.55 ; Restated EPS for 2013-14 ₹ 2.10.
(b)
Fine Ltd. acquired a machine on lst April 2009 for ` 14 crore that had an estimated useful life of 7
years. The machine is depreciated on straight line basis and does not carry any residual value. On 1st
April, 2013, the carrying value of the machine was reassessed at ` 10.20 crore and the surplus arising
out of the revaluation being credited to revaluation reserve. For the year ended 31st March 2013,
conditions indicating an impairment of the machine exited and the amount recoverable ascertained
to be only ` 140 lakhs.
You are required to calculate the loss on impairment of the machine and show how this loss is to be
treated in the books of Fine Ltd.
Fine Ltd. had followed the policy of writing down the revaluation surplus by the increased charge of
depreciation resulting from the revaluation.
Solution: Similar to AS – 28 Question No. 6: Answer : Impairment loss in total ₹ 2.00 crore, out of it charged
to P/L ₹ 0.60 crores.
(c) Sagar Future is ,a non-banking finance company. It makes available to you the costs and market price
of various investments held by it as on 3 1-03-2015 as under:

Scripts: Cost Market Price (in ` Lakh)


A. Equity Shares-
A 60.00 61.20
B 31.50 24.00
C 60.00 36.00
D 60.00 120.00
E 90.00 105.00
F 75.00 90.00
G 30.00 6.000
Solved Question Paper—CA Final-November 2014 (Accounting Standards) 433

B. Mutual Funds-
MF-1 39.00 24.00
MF-2 30.00 21.00
MF-3 6.00 9.00
C. Government Securities-
GV-1 60.00 66.00
GV-2 75.00 72.00

(i) Can the company adjust depreciation of a particular item of investment within a category ?
(ii) What should be the value of investments as on 31.03.2015?
(iii) Is it possible to off-set depreciation in investment in mutual fund against appreciation of the value of
investment in equity shares and government securities?
Solution:
(i) Depreciation adjustment within the category is allowed as per the prudential norms. It means
Investments are to be valued at Cost of Market Value, whichever is lower, category wise.
(ii) Valuation of investments as on 31/3/2015: Cost of MV whichever is less:
Cost MP Valuation
Equity shares 406.50 442.20 406.50
Mutual Funds 75.00 54.00 54.00
G – Securities 135.00 138.00 135.00
595.50
(iii) Inter Category adjustment is not allowed as per prudential norms.
(d) As on 1st April, 2014, the fair value of planned assets was ` 1,00,00.0 in respect of a pension plan of
Zeieous Ltd. On 30th September, 2014, the plan paid out benefits of ` 19,000 and received inward
contribution of ` 49,000. On 3151 March, 2015, the fair value of .plan assets was ` 1,50,000 and
present value of the defined benefit obligation was ` 1,47,920. Actuarial losses on the obligations for
the year 2014-15 were ` 600.
On 1st April, 2014, the company made the following estimates, based on its market studies,
understanding and prevailing prices:
%
Interest & dividend income, after tax payable by the fund . 9.25
Realised and unrealised gains on plan assets (after tax) 2.00
Fund administrative costs -1.00
Expected rate of return 10.25
You are required to find the expected and actual returns on plan assets.
Solution: Similar to AS – 15 Question No. 10: Answer : Expected return ₹ 11,750 and Actual return ₹ 20,000.
You are required to 8
Question 4:
(b) You are required to:
(i) Identify Equity and Liability Components.
(ii) Compute bond liability at the end of each year. and
(iii) Pass necessary journal entries from the following information:
434 Accounting Standards

Number of convertible bonds : 5000 bonds issued at the at beginning of year 1


Value of Bonds : ` 500 per bond
Period of Bonds : 3 years validity
Interest Rateon the bond : 9% p.a. payable annually
Proceeds Received : ` 25 Lacs
Conversion : At the bond holders’ discretion, conversion into 125
ordinary shares for each bond of ` 500.
Prevailing Market Rate : 11% p.a., for bond issued without conversion option
Present value factor for 11% : 0.900, 0.812, 0.731 (for 1 year, 2 years, 3 years
respectively
Solution: Similar to Ind AS – 109 Qt No. 22, 23, 27, 28:
1. Computation of Fair Value of Liability Component
This is measured using the Interest Rate of 11% for Non–Convertible Debt as the Benchmark Discount
Rate, as under –
Fair Value of Liability = PV of Annual Interest + PV of Principal repayment
= 2,25,000 x 2.443 (0.900 + 0.812 + 0.731) + 25,00,000 x 0.731
= ₹ 5,49,675 + ₹ 18,27,500 = 23,77,175
2. Initial Recognition of Compound Financial Instrument :
Fair Value of the Compound Instrument as a whole = ₹ 25,00,000
Less: Fair Value of Liability = ₹ 23,77,175
Equity Component = ₹ 1,22,825
3. Bond Liability at the end of each year
Amortization table:
Year Effective int 11% Cash int recd 9% Principle cash flow recd Closing balance
23,77,175
A B C D = Opening + A – B – C
Year 1 2,61,489 2,25,000 Nil 24,13,664
Year 2 2,65,503 2,25,000 Nil 24,54,167
Year 3 2,70,833# 2,25,000 25,00,000 Nil
# Balancing figure.
Journal Entries:
Date Particulars Dr Cr

Year 1 Cash A/C Dr 25,00,000


Beginning To Conv Debenture A/C 23,77,175
To Equity A/C. 1,22,825
Year 1 Finance expense A/C Dr 2,61,489
End To Cash A/c. 2,25,000
To Conv Debenture A/C. 36,489
P/L A/C Dr 2,61,489
To Finance Expenses 2,61,489
Solved Question Paper—CA Final-November 2014 (Accounting Standards) 435

Question 6:
(b) During the financial year 2014-15, Power Ltd. had the following 8 transactions:
(i) On 01.04.2014, Power Ltd. purchased a new asset of Dark Ltd. for `11,40,000. The fair value of Dark
Ltd.’s identifiable net assets was ` 8,50,000. Power Ltd. is of the view that due to popularity of Dark
Ltd. ‘s product, the life of goodwill is 10 years.
(ii) On 01.05.2014, Power Ltd. purchased a franchise to operate transport service from the Government
for ` 12,00,000 and at a annual fees of 4% of transport revenues. The franchise expires after 5 years.
Transport Revenue were ` 1.20,000 for Financial Year 2014-15. Power Ltd. projects future revenue
of ` 2,40.000 in 2015-16 and ` 3,50,000 p.a. for 3 years thereafter.
(iii) On 05.07.2014, Power Ltd. was granted a patent that had been applied for by Dark Ltd. During 2014-
15. Power Ltd. incurred legal cost of ` 1,10,000 to register the patent- and an additional ` 3,00,000 to
successfully prosecute a patent infringement suit against a competitor. Power Ltd. expects the patent’s
economic life to be 10 years. Power Ltd. follows an accounting Policy to amortize all intangibles on
SLM basis over a maximum period permitted by Accounting Standard taking a full year amortization
in the year of acquisition.
Prepare:
a. A Schedule showing’ the intangible section in Power Ltd. balance sheet at 31st March, 2015.
b. A Schedule showing the related expenses that would appear in the Statement of Profit and Loss of
Power Ltd. for 2014;.15.
Solution:
(i) The excess of consideration paid over the Fair Value of identifiable Net Assets is recognized as
Goodwill. Hence, Goodwill = 11,40,000 – 8,50,000 = ₹ 2,90,000.
As per AS-14 Goodwill is written off over 5 years. But a higher period is allowed only when it can be
evidenced by robust proof.
Amortisation p.a. = 290000 / 10 = ₹ 29,000.
(ii) Franchise fee ₹12,00,000 would be recognized as Intangible Asset. The Annual Amortization should
be allocated over the assets useful life, on a systematic basis i.e. SLM basis or Ratio of Revenues to
be earned. As per AS-26 it is more appropriate to compute amortization on the basis of production /
cash flow ratio and not SLM.
In this case cash flow ratio = 120:240:350:350:350
Therefore the Amortisation will be: Year (1-5) = ₹1,02,128, ₹ 2,04,256, ₹2,97,872 each of the next 3
years.
Alternatively, Amortisation p.a. under SLM = 12,00,000 ÷ 5 = 2,40,000.
(iii) Cost of Patent as per AS – 26 = Registration + Directly attributable Cost = 1,10,000 + 3,00,000 = ₹
4,10,000. Amortisation p.a. = 4,10,000 ÷ 10 years = 41,000.
Balance Sheet as on 31/3/2015 (Extract)
Assets:
Non – Current Assets:
Goodwill (290000 – 29000) 2,61,000
Transport license (1200000 – 102128) 10,97,872
Patents (410000 – 41000) 3,69,000
17,27,872
436 Accounting Standards

Profit / loss Statement for 31/3/2015 (Extract)


Amortization:
Goodwill 29,000
Transport license 1,02,128
Patents 41,000
1,72,128
Question 7:
(a)
XYZ Company is engaged in the business of financial services and is undergoing tight liquidity
position, since most of the assets of the company are locked in various claims/petition in a special
court. XYZ has accepted inter corporate deposits (lCDs) and it is making its best efforts to settle the
dues. There were claims at varied rates of interest, from lenders, from the due date of ICDs .to the
date of repayment. The company has provided interest, as per the terms of the contract till the due
date and a note for non-provision of interest on the due date to date of repayment was accepted in
the financial statements. On account of uncertainties existing regarding the determination of the
amount and in the absence of any specific legal obligation at present as per the terms of the contract,
the company considers that these claims are in the nature of “claims against the company not
acknowledged as debt”, and the same has been disclosed by way of a note in the accounts instead of
making a provision in the Profit and Loss Account. State whether the treatment done by the company
is correct or not.
(b)
Company owns ‘an office building. Company enters into a put option with an investor that permits
the company to put the building to the investor for ` 150 million. The current value of the building is
f 175 million. The option expires in 5 years. The option, if exercised, may be settled through physical
delivery or net cash, at company’s option. How do the Company and the investor account for the
option.
(c)
Aakash Limited is the owner of a CGU (Cash Generating Unit) block of assets whose current carrying
cost is `1000 lakhs. Current carrying cost of the CGU block of assets as per Accounting and Tax
records are after charging depreciation of the current year. The company, after a detailed study by
its technical team, has assessed the present value recoverable amount of this CGU block of assets
at `550 lakhs. The value of the block of assets as per the Income Tax records is `800 lakhs. The
Approving Authority of the company have issued a signed statement confirming that the impairment
in the value of CGU is only a temporary phenomenon which is reversible in subsequent periods and
also assuring virtual certainty of taxable incomes in the foreseeable future. You are required to show
Deferred Tax workings as per Accounting Standards in force, given the tax rate of 30% plus 10%
surcharge thereon. The depreciation rate for tax purposes is 15% and that as per books is 13.91%
(d)
A Company enters into a fixed price forward contract to purchase one million kilograms requirements.
The of copper in accordance with its expected usage contract permits the company to take physical
delivery of the copper at the end of 12 ‘months or to payor receive a net settlement ‘in cash, based on
the change in fair· value’ of copper. Is the contract accounted for as a derivative? Explain
Solution:
(a)
AS 1 “Disclosure of Accounting Policies” recognises prudence as one of the major considerations
governing the selection and application of accounting policies. In view of the uncertainty attached to
future events, profits are not anticipated but recognized only when realized though not necessarily
in cash. Provision is made for all known liabilities and losses even though the amount cannot be
determined with certainty and represents only a best estimate in the light of available information.
Further, ‘accrual’ is one of the fundamental accounting assumptions as per AS - 1. Irrespective of the
terms of the contract, so long as the principal amount of a loan is not repaid, the lender cannot be
Solved Question Paper—CA Final-November 2014 (Accounting Standards) 437

placed in a disadvantageous position for non-payment of interest in respect of claim for interest from
the due date to date of repayment of loan.
From the aforesaid, it is apparent that the company should provide for the liability (since it
is not waived by the lenders) at an amount estimated or on reasonable basis based on facts and
circumstances of each case. Non-provision of the interest from the due date to the date of repayment
of loan amounts to violation of accrual basis of accounting as well as AS-29 Provisions, Contingent
liabilities and Contingent assets.
(b)
Similar to Q. 10 of AS-30…………….already asked 2 times in CA Final.
Answer / Hint: It is a Put option but not a financial instrument.
(c)
Calculation of Deferred Tax Asset (₹ in lakhs)
Depreciation as per Accounting Books for the current year 161.58
[1000 / (100%- 13.91%)] x 13.91%
Depreciation as per Income Tax Records for the current year 141.18
[800 / (100%-15%)] x 15%
Timing Difference due to Depreciation (161.58-141.18)……..(a) 20.40

Impairment Loss = (Carrying amount – Recoverable Value) = 450


= (1000 – 550) (The entire Impairment Loss will be disallowed for the Income tax purpose)
Timing Difference due to Impairment Loss ……………(b) 450

Total Timing Difference ……………..(a + b) 470.40

Tax effect of the above timing difference at 33% (Deferred Tax Asset) 155.23

(d)
First lets prove that it is derivative instrument:
Condition 1: The contract should derive its value from Underlying Asset:
(a) Yes. Metal Prices are the underlying Asset
(b) Requires negligible Investment Yes. Only a Contract is entered.
(c) Can be settled on Net basis Yes. (Given)
(d) Volatility in underlying asset Metal Prices are subject to fluctuation
Condition 1: The contract should derive its value from an Yes. Metal Prices are the underlying Asset
underlying asset:
Condition 2: Either low investment or low investments Here No Investments.
Condition 3: Settled on a later date Will be settled after 12 months.
Yes it is a Derivative Contract as all conditions are satisfied.

Now let’s see whether it is a Financial Instrument:
If the contract is settled on Gross Delivery basis then it is not financial instrument but an Executory
Contract.
But if the contract is settled on net cash basis or delivery of goods will be immediately sold at a profit
further to another party then it is a financial instrument.

You might also like